Π‘ΠΎΠ΄Π΅Ρ€ΠΆΠ°Π½ΠΈΠ΅

Как ΠΎΠΏΡ€Π΅Π΄Π΅Π»ΠΈΡ‚ΡŒ Π²Π½ΡƒΡ‚Ρ€Π΅Π½Π½Π΅Π΅ сопротивлСниС. ЭлСктродвиТущая сила

Π­Π”Π‘ ΠΈ напряТСниС. Π’Π½ΡƒΡ‚Ρ€Π΅Π½Π½Π΅Π΅ сопротивлСниС источников питания.
Π›ΠΈΠΊΠ±Π΅Π· Ρ‚Π°ΠΊ Π»ΠΈΠΊΠ±Π΅Π·!
Π—Π°ΠΊΠΎΠ½ Ома. Π’ΠΎΡ‚ я ΠΎ Ρ‡Π΅ΠΌ.
О Π·Π°ΠΊΠΎΠ½Π΅ Ома ΠΌΡ‹ ΡƒΠΆΠ΅ Π³ΠΎΠ²ΠΎΡ€ΠΈΠ»ΠΈ. ΠŸΠΎΠ³ΠΎΠ²ΠΎΡ€ΠΈΠΌ Π΅Ρ‰Π΅ Ρ€Π°Π· — с нСсколько ΠΈΠ½ΠΎΠΉ стороны. НС вдаваясь Π² физичСскиС подробности ΠΈ Π²Ρ‹Ρ€Π°ΠΆΠ°ΡΡΡŒ простым ΠΊΠΎΡˆΠ°Ρ‡ΡŒΠΈΠΌ языком, Π·Π°ΠΊΠΎΠ½ Ома гласит: Ρ‡Π΅ΠΌ большС э.Π΄.с. (элСктродвиТущая сила), Ρ‚Π΅ΠΌ большС Ρ‚ΠΎΠΊ, Ρ‡Π΅ΠΌ большС сопротивлСниС, Ρ‚Π΅ΠΌ мСньшС Ρ‚ΠΎΠΊ.
ΠŸΠ΅Ρ€Π΅Π²Π΅Π΄Ρ сиС Π·Π°ΠΊΠ»ΠΈΠ½Π°Π½ΠΈΠ΅ Π½Π° язык сухих Ρ„ΠΎΡ€ΠΌΡƒΠ» ΠΏΠΎΠ»ΡƒΡ‡Π°Π΅ΠΌ:

I=E/R

Π³Π΄Π΅:I — сила Ρ‚ΠΎΠΊΠ°,E — Π­.Π”.Π‘. — элСктродвиТущая силаR — сопротивлСниС
Π’ΠΎΠΊ измСряСтся Π² Π°ΠΌΠΏΠ΅Ρ€Π°Ρ…, э.Π΄.с. — Π² Π²ΠΎΠ»ΡŒΡ‚Π°Ρ…, Π° сопротивлСниС носит Π³ΠΎΡ€Π΄ΠΎΠ΅ имя Ρ‚ΠΎΠ²Π°Ρ€ΠΈΡ‰Π° Ома.Π­.Π΄.с. — это Π΅ΡΡ‚ΡŒ характСристика идСального Π³Π΅Π½Π΅Ρ€Π°Ρ‚ΠΎΡ€Π°, Π²Π½ΡƒΡ‚Ρ€Π΅Π½Π½Π΅ сопротивлСниС ΠΊΠΎΡ‚ΠΎΡ€ΠΎΠ³ΠΎ принято ΡΡ‡ΠΈΡ‚Π°Ρ‚ΡŒ бСсконСчно ΠΌΠ°Π»Ρ‹ΠΌ. Π’ Ρ€Π΅Π°Π»ΡŒΠ½ΠΎΠΉ ΠΆΠΈΠ·Π½ΠΈ Ρ‚Π°ΠΊΠΎΠ΅ Π±Ρ‹Π²Π°Π΅Ρ‚ Ρ€Π΅Π΄ΠΊΠΎ, поэтому Π² силу вступаСт Π·Π°ΠΊΠΎΠ½ Ома для ΠΏΠΎΡΠ»Π΅Π΄ΠΎΠ²Π°Ρ‚Π΅Π»ΡŒΠ½ΠΎΠΉ Ρ†Π΅ΠΏΠΈ (Π±ΠΎΠ»Π΅Π΅ Π·Π½Π°ΠΊΠΎΠΌΡ‹ΠΉ Π½Π°ΠΌ):

I=U/R

Π³Π΄Π΅:U — напряТСниС источника нСпосрСдствСнно Π½Π° Π΅Π³ΠΎ ΠΊΠ»Π΅ΠΌΠΌΠ°Ρ….


Рассмотрим простой ΠΏΡ€ΠΈΠΌΠ΅Ρ€.
ΠŸΡ€Π΅Π΄ΡΡ‚Π°Π²ΠΈΠΌ сСбС ΠΎΠ±Ρ‹Ρ‡Π½ΡƒΡŽ Π±Π°Ρ‚Π°Ρ€Π΅ΠΉΠΊΡƒ Π² Π²ΠΈΠ΄Π΅ источника э.Π΄.с. ΠΈ Π²ΠΊΠ»ΡŽΡ‡Π΅Π½Π½ΠΎΠ³ΠΎ ΠΏΠΎΡΠ»Π΅Π΄ΠΎΠ²Π°Ρ‚Π΅Π»ΡŒΠ½ΠΎ с Π½ΠΈΠΌ Π½Π΅ΠΊΠΎΠ΅Π³ΠΎ рСзистора, ΠΊΠΎΡ‚ΠΎΡ€Ρ‹ΠΉ Π±ΡƒΠ΄Π΅Ρ‚ ΠΎΠ»ΠΈΡ†Π΅Ρ‚Π²ΠΎΡ€ΡΡ‚ΡŒ собой Π²Π½ΡƒΡ‚Ρ€Π΅Π½Π½Π΅Π΅ сопротивлСниС Π±Π°Ρ‚Π°Ρ€Π΅ΠΉΠΊΠΈ. ΠŸΠΎΠ΄ΠΊΠ»ΡŽΡ‡ΠΈΠΌ ΠΏΠ°Ρ€Π°Π»Π»Π΅Π»ΡŒΠ½ΠΎ Π±Π°Ρ‚Π°Ρ€Π΅ΠΉΠΊΠ΅ Π²ΠΎΠ»ΡŒΡ‚ΠΌΠ΅Ρ‚Ρ€. Π•Π³ΠΎ Π²Ρ…ΠΎΠ΄Π½ΠΎΠ΅ сопротивлСниС Π·Π½Π°Ρ‡ΠΈΡ‚Π΅Π»ΡŒΠ½ΠΎ большС Π²Π½ΡƒΡ‚Ρ€Π΅Π½Π½Π΅Π³ΠΎ сопротивлСния Π±Π°Ρ‚Π°Ρ€Π΅ΠΉΠΊΠΈ, Π½ΠΎ Π½Π΅ бСсконСчно большоС — Ρ‚ΠΎ Π΅ΡΡ‚ΡŒ, Ρ‡Π΅Ρ€Π΅Π· Π½Π΅Π³ΠΎ ΠΏΠΎΡ‚Π΅Ρ‡Π΅Ρ‚ Ρ‚ΠΎΠΊ. Π’Π΅Π»ΠΈΡ‡ΠΈΠ½Π° напряТСния, ΠΊΠΎΡ‚ΠΎΡ€ΡƒΡŽ ΠΏΠΎΠΊΠ°ΠΆΠ΅Ρ‚ Π²ΠΎΠ»ΡŒΡ‚ΠΌΠ΅Ρ‚Ρ€ Π±ΡƒΠ΄Π΅Ρ‚ мСньшС Π²Π΅Π»ΠΈΡ‡ΠΈΠ½Ρ‹ э.Π΄.с. ΠΊΠ°ΠΊ Ρ€Π°Π· Π½Π° Π²Π΅Π»ΠΈΡ‡ΠΈΠ½Ρƒ падСния напряТСния Π½Π° Π²Π½ΡƒΡ‚Ρ€Π΅Π½Π½Π΅ΠΌ Π²ΠΎΠΎΠ±Ρ€Π°ΠΆΠ°Π΅ΠΌΠΎΠΌ рСзисторС ΠΏΡ€ΠΈ Π΄Π°Π½Π½ΠΎΠΌ Ρ‚ΠΎΠΊΠ΅.Но, Ρ‚Π΅ΠΌ Π½Π΅ ΠΌΠ΅Π½Π΅Π΅ ΠΈΠΌΠ΅Π½Π½ΠΎ эта Π²Π΅Π»ΠΈΡ‡ΠΈΠ½Π° ΠΈ принимаСтся Π·Π° напряТСниС Π±Π°Ρ‚Π°Ρ€Π΅ΠΉΠΊΠΈ.
Π€ΠΎΡ€ΠΌΡƒΠ»Π° ΠΊΠΎΠ½Π΅Ρ‡Π½ΠΎΠ³ΠΎ напряТСния ΠΏΡ€ΠΈ этом Π±ΡƒΠ΄Π΅Ρ‚ ΠΈΠΌΠ΅Ρ‚ΡŒ ΡΠ»Π΅Π΄ΡƒΡŽΡ‰ΠΈΠΉ Π²ΠΈΠ΄:

U(Π±Π°Ρ‚)=E-U(Π²Π½ΡƒΡ‚Ρ€)

Π’Π°ΠΊ ΠΊΠ°ΠΊ со Π²Ρ€Π΅ΠΌΠ΅Π½Π΅ΠΌ Ρƒ всСх элСмСнтов питания Π²Π½ΡƒΡ‚Ρ€Π΅Π½Π½Π΅Π΅ сопротивлСниС увСличиваСтся, Ρ‚ΠΎ ΠΈ ΠΏΠ°Π΄Π΅Π½ΠΈΠ΅ напряТСния Π½Π° Π²Π½ΡƒΡ‚Ρ€Π΅Π½Π½Π΅ΠΌ сопротивлСнии Ρ‚ΠΎΠΆΠ΅ увСличиваСтся. ΠŸΡ€ΠΈ этом напряТСниС Π½Π° ΠΊΠ»Π΅ΠΌΠΌΠ°Ρ… Π±Π°Ρ‚Π°Ρ€Π΅ΠΉΠΊΠΈ ΡƒΠΌΠ΅Π½ΡŒΡˆΠ°Π΅Ρ‚ΡΡ. ΠœΡΡƒ!
Π Π°Π·ΠΎΠ±Ρ€Π°Π»ΠΈΡΡŒ!
Π§Ρ‚ΠΎ ΠΆΠ΅ происходит, Ссли вмСсто Π²ΠΎΠ»ΡŒΡ‚ΠΌΠ΅Ρ‚Ρ€Π° ΠΊ Π±Π°Ρ‚Π°Ρ€Π΅ΠΉΠΊΠ΅ ΠΏΠΎΠ΄ΠΊΠ»ΡŽΡ‡ΠΈΡ‚ΡŒ Π°ΠΌΠΏΠ΅Ρ€ΠΌΠ΅Ρ‚Ρ€? Π’Π°ΠΊ ΠΊΠ°ΠΊ собствСнноС сопротивлСниС Π°ΠΌΠΏΠ΅Ρ€ΠΌΠ΅Ρ‚Ρ€Π° стрСмится ΠΊ Π½ΡƒΠ»ΡŽ, ΠΌΡ‹ фактичСски Π±ΡƒΠ΄Π΅ΠΌ ΠΈΠ·ΠΌΠ΅Ρ€ΡΡ‚ΡŒ Ρ‚ΠΎΠΊ, ΠΏΡ€ΠΎΡ‚Π΅ΠΊΠ°ΡŽΡ‰ΠΈΠΉ Ρ‡Π΅Ρ€Π΅Π· Π²Π½ΡƒΡ‚Ρ€Π΅Π½Π½Π΅Π΅ сопротивлСниС Π±Π°Ρ‚Π°Ρ€Π΅ΠΉΠΊΠΈ. Π’Π°ΠΊ ΠΊΠ°ΠΊ Π²Π½ΡƒΡ‚Ρ€Π΅Π½Π½Π΅ сопротивлСниС источника ΠΎΡ‡Π΅Π½ΡŒ нСбольшоС, ΠΈΠ·ΠΌΠ΅Ρ€Π΅Π½Π½Ρ‹ΠΉ ΠΏΡ€ΠΈ этом Ρ‚ΠΎΠΊ ΠΌΠΎΠΆΠ΅Ρ‚ Π΄ΠΎΡΡ‚ΠΈΠ³Π°Ρ‚ΡŒ Π½ Π΅ΡΠΊΠΎΠ»ΡŒΠΊΠΈΡ… Π°ΠΌΠΏΠ΅Ρ€.

Однако слСдуСт Π·Π°ΠΌΠ΅Ρ‚ΠΈΡ‚ΡŒ, Ρ‡Ρ‚ΠΎ Π²Π½ΡƒΡ‚Ρ€Π΅Π½Π½Π΅Π΅ сопротивлСниС источника являСтся Ρ‚Π°ΠΊΠΈΠΌ ΠΆΠ΅ элСмСнтом Ρ†Π΅ΠΏΠΈ, ΠΊΠ°ΠΊ ΠΈ всС ΠΎΡΡ‚Π°Π»ΡŒΠ½Ρ‹Π΅. ΠŸΠΎΡΡ‚ΠΎΠΌΡƒ ΠΏΡ€ΠΈ ΡƒΠ²Π΅Π»ΠΈΡ‡Π΅Π½ΠΈΠΈ Ρ‚ΠΎΠΊΠ° Π½Π°Π³Ρ€ΡƒΠ·ΠΊΠΈ ΠΏΠ°Π΄Π΅Π½ΠΈΠ΅ напряТСния Π½Π° Π²Π½ΡƒΡ‚Ρ€Π΅Π½Π½Π΅ΠΌ сопротивлСнии Ρ‚Π°ΠΊΠΆΠ΅ увСличится, Ρ‡Ρ‚ΠΎ ΠΏΡ€ΠΈΠ²ΠΎΠ΄ΠΈΡ‚ ΠΊ ΡƒΠΌΠ΅Π½ΡŒΡˆΠ΅Π½ΠΈΡŽ напряТСния Π½Π° Π½Π°Π³Ρ€ΡƒΠ·ΠΊΠ΅. Или ΠΊΠ°ΠΊ ΠΌΡ‹, Ρ€Π°Π΄ΠΈΠΎΠΊΠΎΡ‚Ρ‹, любим Π²Ρ‹Ρ€Π°ΠΆΠ°Ρ‚ΡŒΡΡ — ΠΊ просадкС Π½Π°ΠΏΡ€ΡƒΠ³ΠΈ.
Π§Ρ‚ΠΎΠ±Ρ‹ ΠΈΠ·ΠΌΠ΅Π½Π΅Π½ΠΈΠ΅ Π½Π°Π³Ρ€ΡƒΠ·ΠΊΠΈ ΠΊΠ°ΠΊ ΠΌΠΎΠΆΠ½ΠΎ мСньшС влияло Π½Π° Π²Ρ‹Ρ…ΠΎΠ΄Π½ΠΎΠ΅ напряТСниС источника Π΅Π³ΠΎ Π²Π½ΡƒΡ‚Ρ€Π΅Π½Π½Π΅Π΅ сопротивлСниС ΡΡ‚Π°Ρ€Π°ΡŽΡ‚ΡΡ свСсти ΠΊ ΠΌΠΈΠ½ΠΈΠΌΡƒΠΌΡƒ.
МоТно Ρ‚Π°ΠΊ ΠΏΠΎΠ΄ΠΎΠ±Ρ€Π°Ρ‚ΡŒ элСмСнты ΠΏΠΎΡΠ»Π΅Π΄ΠΎΠ²Π°Ρ‚Π΅Π»ΡŒΠ½ΠΎΠΉ Ρ†Π΅ΠΏΠΈ, Ρ‡Ρ‚ΠΎΠ±Ρ‹ Π½Π° ΠΊΠ°ΠΊΠΎΠΌ-Π½ΠΈΠ±ΡƒΠ΄ΡŒ ΠΈΠ· Π½ΠΈΡ… ΠΏΠΎΠ»ΡƒΡ‡ΠΈΡ‚ΡŒ напряТСниС, ΡƒΠΌΠ΅Π½ΡŒΡˆΠ΅Π½Π½ΠΎΠ΅, ΠΏΠΎ ΡΡ€Π°Π²Π½Π΅Π½ΠΈΡŽ с исходным, Π²ΠΎ сколько ΡƒΠ³ΠΎΠ΄Π½ΠΎ Ρ€Π°Π·.

Π”Π²ΡƒΡ…ΠΏΠΎΠ»ΡŽΡΠ½ΠΈΠΊ ΠΈ Π΅Π³ΠΎ эквивалСнтная схСма

Вну́трСннСС сопротивлС́ниС Π΄Π²ΡƒΡ…ΠΏΠΎΠ»ΡŽΡΠ½ΠΈΠΊΠ° — импСданс Π² эквивалСнтной схСмС Π΄Π²ΡƒΡ…ΠΏΠΎΠ»ΡŽΡΠ½ΠΈΠΊΠ°, состоящСй ΠΈΠ· ΠΏΠΎΡΠ»Π΅Π΄ΠΎΠ²Π°Ρ‚Π΅Π»ΡŒΠ½ΠΎ Π²ΠΊΠ»ΡŽΡ‡Π΅Π½Π½Ρ‹Ρ… Π³Π΅Π½Π΅Ρ€Π°Ρ‚ΠΎΡ€Π° напряТСния ΠΈ импСданса (см. рисунок). ΠŸΠΎΠ½ΡΡ‚ΠΈΠ΅ примСняСтся Π² Ρ‚Π΅ΠΎΡ€ΠΈΠΈ Ρ†Π΅ΠΏΠ΅ΠΉ ΠΏΡ€ΠΈ Π·Π°ΠΌΠ΅Π½Π΅ Ρ€Π΅Π°Π»ΡŒΠ½ΠΎΠ³ΠΎ источника ΠΈΠ΄Π΅Π°Π»ΡŒΠ½Ρ‹ΠΌΠΈ элСмСнтами, Ρ‚ΠΎ Π΅ΡΡ‚ΡŒ ΠΏΡ€ΠΈ ΠΏΠ΅Ρ€Π΅Ρ…ΠΎΠ΄Π΅ ΠΊ эквивалСнтной схСмС.

Π’Π²Π΅Π΄Π΅Π½ΠΈΠ΅

Рассмотрим ΠΏΡ€ΠΈΠΌΠ΅Ρ€. Π’ Π»Π΅Π³ΠΊΠΎΠ²ΠΎΠΌ Π°Π²Ρ‚ΠΎΠΌΠΎΠ±ΠΈΠ»Π΅ Π·Π°ΠΏΠΈΡ‚Π°Π΅ΠΌ Π±ΠΎΡ€Ρ‚ΠΎΠ²ΡƒΡŽ ΡΠ΅Ρ‚ΡŒ Π½Π΅ ΠΎΡ‚ ΡˆΡ‚Π°Ρ‚Π½ΠΎΠ³ΠΎ свинцово-кислотного аккумулятора напряТСниСм 12 Π²ΠΎΠ»ΡŒΡ‚ ΠΈ Ρ‘ΠΌΠΊΠΎΡΡ‚ΡŒΡŽ 55 А·ч, Π° ΠΎΡ‚ ΠΏΠΎΡΠ»Π΅Π΄ΠΎΠ²Π°Ρ‚Π΅Π»ΡŒΠ½ΠΎ соСдинённых восьми Π±Π°Ρ‚Π°Ρ€Π΅Π΅ΠΊ (Π½Π°ΠΏΡ€ΠΈΠΌΠ΅Ρ€, Ρ‚ΠΈΠΏΠΎΡ€Π°Π·ΠΌΠ΅Ρ€Π° АА, Ρ‘ΠΌΠΊΠΎΡΡ‚ΡŒΡŽ ΠΎΠΊΠΎΠ»ΠΎ 1 А·ч). ΠŸΠΎΠΏΡ€ΠΎΠ±ΡƒΠ΅ΠΌ завСсти Π΄Π²ΠΈΠ³Π°Ρ‚Π΅Π»ΡŒ. ΠžΠΏΡ‹Ρ‚ ΠΏΠΎΠΊΠ°Π·Ρ‹Π²Π°Π΅Ρ‚, Ρ‡Ρ‚ΠΎ ΠΏΡ€ΠΈ ΠΏΠΈΡ‚Π°Π½ΠΈΠΈ ΠΎΡ‚ Π±Π°Ρ‚Π°Ρ€Π΅Π΅ΠΊ Π²Π°Π» стартСра Π½Π΅ повСрнётся Π½ΠΈ Π½Π° градус. Π‘ΠΎΠ»Π΅Π΅ Ρ‚ΠΎΠ³ΠΎ, Π½Π΅ сработаСт Π΄Π°ΠΆΠ΅ Π²Ρ‚ΡΠ³ΠΈΠ²Π°ΡŽΡ‰Π΅Π΅ Ρ€Π΅Π»Π΅.

Π˜Π½Ρ‚ΡƒΠΈΡ‚ΠΈΠ²Π½ΠΎ понятно, Ρ‡Ρ‚ΠΎ Π±Π°Ρ‚Π°Ρ€Π΅ΠΉΠΊΠ° «нСдостаточно мощная» для ΠΏΠΎΠ΄ΠΎΠ±Π½ΠΎΠ³ΠΎ примСнСния, ΠΎΠ΄Π½Π°ΠΊΠΎ рассмотрСниС Π΅Ρ‘ заявлСнных элСктричСских характСристик — напряТСния ΠΈ заряда (ёмкости) — Π½Π΅ Π΄Π°Ρ‘Ρ‚ количСствСнного описания Π΄Π°Π½Π½ΠΎΠ³ΠΎ явлСния. НапряТСниС Π² ΠΎΠ±ΠΎΠΈΡ… случаях ΠΎΠ΄ΠΈΠ½Π°ΠΊΠΎΠ²ΠΎ:

Аккумулятор: 12 Π²ΠΎΠ»ΡŒΡ‚

Π“Π°Π»ΡŒΠ²Π°Π½ΠΈΡ‡Π΅ΡΠΊΠΈΠ΅ элСмСнты: 8Β·1,5 Π²ΠΎΠ»ΡŒΡ‚ = 12 Π²ΠΎΠ»ΡŒΡ‚

Ёмкости Ρ‚Π°ΠΊΠΆΠ΅ Π²ΠΏΠΎΠ»Π½Π΅ достаточно: ΠΎΠ΄Π½ΠΎΠ³ΠΎ ампСр·часа Π² Π±Π°Ρ‚Π°Ρ€Π΅ΠΉΠΊΠ΅ Π΄ΠΎΠ»ΠΆΠ½ΠΎ Ρ…Π²Π°Ρ‚ΠΈΡ‚ΡŒ, Ρ‡Ρ‚ΠΎΠ±Ρ‹ Π²Ρ€Π°Ρ‰Π°Ρ‚ΡŒ стартСр Π² Ρ‚Π΅Ρ‡Π΅Π½ΠΈΠ΅ 14 сСкунд (ΠΏΡ€ΠΈ Ρ‚ΠΎΠΊΠ΅ 250 Π°ΠΌΠΏΠ΅Ρ€).

Казалось Π±Ρ‹, Π² соотвСтствии с Π·Π°ΠΊΠΎΠ½ΠΎΠΌ Ома Ρ‚ΠΎΠΊ Π² ΠΎΠ΄ΠΈΠ½Π°ΠΊΠΎΠ²ΠΎΠΉ Π½Π°Π³Ρ€ΡƒΠ·ΠΊΠ΅ ΠΏΡ€ΠΈ элСктричСски ΠΎΠ΄ΠΈΠ½Π°ΠΊΠΎΠ²Ρ‹Ρ… источниках Ρ‚Π°ΠΊΠΆΠ΅ Π΄ΠΎΠ»ΠΆΠ΅Π½ Π±Ρ‹Ρ‚ΡŒ ΠΎΠ΄ΠΈΠ½Π°ΠΊΠΎΠ²Ρ‹ΠΌ. Однако Π² Π΄Π΅ΠΉΡΡ‚Π²ΠΈΡ‚Π΅Π»ΡŒΠ½ΠΎΡΡ‚ΠΈ это Π½Π΅ совсСм Ρ‚Π°ΠΊ. Π˜ΡΡ‚ΠΎΡ‡Π½ΠΈΠΊΠΈ Π²Π΅Π»ΠΈ Π±Ρ‹ сСбя ΠΎΠ΄ΠΈΠ½Π°ΠΊΠΎΠ²ΠΎ, Ссли Π±Ρ‹ Π±Ρ‹Π»ΠΈ ΠΈΠ΄Π΅Π°Π»ΡŒΠ½Ρ‹ΠΌΠΈ Π³Π΅Π½Π΅Ρ€Π°Ρ‚ΠΎΡ€Π°ΠΌΠΈ напряТСния. Для описания стСпСни отличия Ρ€Π΅Π°Π»ΡŒΠ½Ρ‹Ρ… источников ΠΎΡ‚ ΠΈΠ΄Π΅Π°Π»ΡŒΠ½Ρ‹Ρ… Π³Π΅Π½Π΅Ρ€Π°Ρ‚ΠΎΡ€ΠΎΠ² ΠΈ примСняСтся понятиС Π²Π½ΡƒΡ‚Ρ€Π΅Π½Π½Π΅Π΅ сопротивлСниС.

Π‘ΠΎΠΏΡ€ΠΎΡ‚ΠΈΠ²Π»Π΅Π½ΠΈΠ΅ ΠΈ Π²Π½ΡƒΡ‚Ρ€Π΅Π½Π½Π΅Π΅ сопротивлСниС

Основной характСристикой Π΄Π²ΡƒΡ…ΠΏΠΎΠ»ΡŽΡΠ½ΠΈΠΊΠ° являСтся Π΅Π³ΠΎ сопротивлСниС (ΠΈΠ»ΠΈ импСданс). Однако Ρ…Π°Ρ€Π°ΠΊΡ‚Π΅Ρ€ΠΈΠ·ΠΎΠ²Π°Ρ‚ΡŒ Π΄Π²ΡƒΡ…ΠΏΠΎΠ»ΡŽΡΠ½ΠΈΠΊ ΠΎΠ΄Π½ΠΈΠΌ Ρ‚ΠΎΠ»ΡŒΠΊΠΎ сопротивлСниСм Π½Π΅ всСгда Π²ΠΎΠ·ΠΌΠΎΠΆΠ½ΠΎ. Π”Π΅Π»ΠΎ Π² Ρ‚ΠΎΠΌ, Ρ‡Ρ‚ΠΎ Ρ‚Π΅Ρ€ΠΌΠΈΠ½ сопротивлСниС примСни́м Ρ‚ΠΎΠ»ΡŒΠΊΠΎ для чисто пассивных элСмСнтов, Ρ‚ΠΎ Π΅ΡΡ‚ΡŒ Π½Π΅ содСрТащих Π² сСбС источников энСргии. Если Π΄Π²ΡƒΡ…ΠΏΠΎΠ»ΡŽΡΠ½ΠΈΠΊ содСрТит источник энСргии, Ρ‚ΠΎ понятиС «сопротивлСниС» ΠΊ Π½Π΅ΠΌΡƒ просто Π½Π΅ ΠΏΡ€ΠΈΠΌΠ΅Π½ΠΈΠΌΠΎ, ΠΏΠΎΡΠΊΠΎΠ»ΡŒΠΊΡƒ Π·Π°ΠΊΠΎΠ½ Ома Π² Ρ„ΠΎΡ€ΠΌΡƒΠ»ΠΈΡ€ΠΎΠ²ΠΊΠ΅ U=Ir Π½Π΅ выполняСтся.

Π’Π°ΠΊΠΈΠΌ ΠΎΠ±Ρ€Π°Π·ΠΎΠΌ, для Π΄Π²ΡƒΡ…ΠΏΠΎΠ»ΡŽΡΠ½ΠΈΠΊΠΎΠ², содСрТащих источники (Ρ‚ΠΎ Π΅ΡΡ‚ΡŒ Π³Π΅Π½Π΅Ρ€Π°Ρ‚ΠΎΡ€Ρ‹ напряТСния ΠΈ Π³Π΅Π½Π΅Ρ€Π°Ρ‚ΠΎΡ€Ρ‹ Ρ‚ΠΎΠΊΠ°) Π½Π΅ΠΎΠ±Ρ…ΠΎΠ΄ΠΈΠΌΠΎ Π³ΠΎΠ²ΠΎΡ€ΠΈΡ‚ΡŒ ΠΈΠΌΠ΅Π½Π½ΠΎ ΠΎ Π²Π½ΡƒΡ‚Ρ€Π΅Π½Π½Π΅ΠΌ сопротивлСнии (ΠΈΠ»ΠΈ импСдансС). Если ΠΆΠ΅ Π΄Π²ΡƒΡ…ΠΏΠΎΠ»ΡŽΡΠ½ΠΈΠΊ Π½Π΅ содСрТит источников, Ρ‚ΠΎ Β«Π²Π½ΡƒΡ‚Ρ€Π΅Π½Π½Π΅Π΅ сопротивлСниС» для Ρ‚Π°ΠΊΠΎΠ³ΠΎ Π΄Π²ΡƒΡ…ΠΏΠΎΠ»ΡŽΡΠ½ΠΈΠΊΠ° ΠΎΠ·Π½Π°Ρ‡Π°Π΅Ρ‚ Ρ‚ΠΎ ΠΆΠ΅ самоС, Ρ‡Ρ‚ΠΎ ΠΈ просто «сопротивлСниС».

РодствСнныС Ρ‚Π΅Ρ€ΠΌΠΈΠ½Ρ‹

Если Π² ΠΊΠ°ΠΊΠΎΠΉ-Π»ΠΈΠ±ΠΎ систСмС ΠΌΠΎΠΆΠ½ΠΎ Π²Ρ‹Π΄Π΅Π»ΠΈΡ‚ΡŒ Π²Ρ…ΠΎΠ΄ ΠΈ/ΠΈΠ»ΠΈ Π²Ρ‹Ρ…ΠΎΠ΄, Ρ‚ΠΎ часто ΡƒΠΏΠΎΡ‚Ρ€Π΅Π±Π»ΡΡŽΡ‚ΡΡ ΡΠ»Π΅Π΄ΡƒΡŽΡ‰ΠΈΠ΅ Ρ‚Π΅Ρ€ΠΌΠΈΠ½Ρ‹:

Π’Ρ…ΠΎΠ΄Π½ΠΎΠ΅ сопротивлСниС — Π²Π½ΡƒΡ‚Ρ€Π΅Π½Π½Π΅Π΅ сопротивлСниС Π΄Π²ΡƒΡ…ΠΏΠΎΠ»ΡŽΡΠ½ΠΈΠΊΠ°, ΠΊΠΎΡ‚ΠΎΡ€Ρ‹ΠΌ являСтся Π²Ρ…ΠΎΠ΄ систСмы.

Π’Ρ‹Ρ…ΠΎΠ΄Π½ΠΎΠ΅ сопротивлСниС — Π²Π½ΡƒΡ‚Ρ€Π΅Π½Π½Π΅Π΅ сопротивлСниС Π΄Π²ΡƒΡ…ΠΏΠΎΠ»ΡŽΡΠ½ΠΈΠΊΠ°, ΠΊΠΎΡ‚ΠΎΡ€Ρ‹ΠΌ являСтся Π²Ρ‹Ρ…ΠΎΠ΄ систСмы.

ЀизичСскиС ΠΏΡ€ΠΈΠ½Ρ†ΠΈΠΏΡ‹

НСсмотря Π½Π° Ρ‚ΠΎ, Ρ‡Ρ‚ΠΎ Π½Π° эквивалСнтной схСмС Π²Π½ΡƒΡ‚Ρ€Π΅Π½Π½Π΅Π΅ сопротивлСниС прСдставлСно ΠΊΠ°ΠΊ ΠΎΠ΄ΠΈΠ½ пассивный элСмСнт (ΠΏΡ€ΠΈΡ‡Π΅ΠΌ Π°ΠΊΡ‚ΠΈΠ²Π½ΠΎΠ΅ сопротивлСниС, Ρ‚ΠΎ Π΅ΡΡ‚ΡŒ рСзистор Π² Π½Π΅ΠΌ присутствуСт ΠΎΠ±ΡΠ·Π°Ρ‚Π΅Π»ΡŒΠ½ΠΎ), Π²Π½ΡƒΡ‚Ρ€Π΅Π½Π½Π΅Π΅ сопротивлСниС Π½Π΅ сосрСдоточСно Π² ΠΊΠ°ΠΊΠΎΠΌ-Π»ΠΈΠ±ΠΎ ΠΎΠ΄Π½ΠΎΠΌ элСмСнтС. Π”Π²ΡƒΡ…ΠΏΠΎΠ»ΡŽΡΠ½ΠΈΠΊ лишь внСшнС Π²Π΅Π΄Π΅Ρ‚ сСбя Ρ‚Π°ΠΊ, словно Π² Π½Π΅ΠΌ имССтся сосрСдоточСнный Π²Π½ΡƒΡ‚Ρ€Π΅Π½Π½ΠΈΠΉ импСданс ΠΈ Π³Π΅Π½Π΅Ρ€Π°Ρ‚ΠΎΡ€ напряТСния. Π’ Π΄Π΅ΠΉΡΡ‚Π²ΠΈΡ‚Π΅Π»ΡŒΠ½ΠΎΡΡ‚ΠΈ Π²Π½ΡƒΡ‚Ρ€Π΅Π½Π½Π΅Π΅ сопротивлСниС являСтся внСшним проявлСниСм совокупности физичСских эффСктов:

Если Π² Π΄Π²ΡƒΡ…ΠΏΠΎΠ»ΡŽΡΠ½ΠΈΠΊΠ΅ имССтся Ρ‚ΠΎΠ»ΡŒΠΊΠΎ источник энСргии Π±Π΅Π· ΠΊΠ°ΠΊΠΎΠΉ-Π»ΠΈΠ±ΠΎ элСктричСской схСмы (Π½Π°ΠΏΡ€ΠΈΠΌΠ΅Ρ€, Π³Π°Π»ΡŒΠ²Π°Π½ΠΈΡ‡Π΅ΡΠΊΠΈΠΉ элСмСнт), Ρ‚ΠΎ Π²Π½ΡƒΡ‚Ρ€Π΅Π½Π½Π΅Π΅ сопротивлСниС чисто Π°ΠΊΡ‚ΠΈΠ²Π½ΠΎΠ΅, ΠΎΠ½ΠΎ обусловлСно физичСскими эффСктами, ΠΊΠΎΡ‚ΠΎΡ€Ρ‹Π΅ Π½Π΅ ΠΏΠΎΠ·Π²ΠΎΠ»ΡΡŽΡ‚ мощности, ΠΎΡ‚Π΄Π°Π²Π°Π΅ΠΌΠΎΠΉ этим источником Π² Π½Π°Π³Ρ€ΡƒΠ·ΠΊΡƒ, ΠΏΡ€Π΅Π²Ρ‹ΡΠΈΡ‚ΡŒ ΠΎΠΏΡ€Π΅Π΄Π΅Π»Π΅Π½Π½Ρ‹ΠΉ ΠΏΡ€Π΅Π΄Π΅Π». НаиболСС простой ΠΏΡ€ΠΈΠΌΠ΅Ρ€ Ρ‚Π°ΠΊΠΎΠ³ΠΎ эффСкта — Π½Π΅Π½ΡƒΠ»Π΅Π²ΠΎΠ΅ сопротивлСниС ΠΏΡ€ΠΎΠ²ΠΎΠ΄Π½ΠΈΠΊΠΎΠ² элСктричСской Ρ†Π΅ΠΏΠΈ. Но, ΠΊΠ°ΠΊ ΠΏΡ€Π°Π²ΠΈΠ»ΠΎ, наибольший Π²ΠΊΠ»Π°Π΄ Π² ΠΎΠ³Ρ€Π°Π½ΠΈΡ‡Π΅Π½ΠΈΠ΅ мощности вносят эффСкты нСэлСктричСской ΠΏΡ€ΠΈΡ€ΠΎΠ΄Ρ‹. Π’Π°ΠΊ, Π½Π°ΠΏΡ€ΠΈΠΌΠ΅Ρ€, Π² химичСском источникС ΠΌΠΎΡ‰Π½ΠΎΡΡ‚ΡŒ ΠΌΠΎΠΆΠ΅Ρ‚ Π±Ρ‹Ρ‚ΡŒ ΠΎΠ³Ρ€Π°Π½ΠΈΡ‡Π΅Π½Π° ΠΏΠ»ΠΎΡ‰Π°Π΄ΡŒΡŽ соприкосновСния ΡƒΡ‡Π°ΡΡ‚Π²ΡƒΡŽΡ‰ΠΈΡ… Π² Ρ€Π΅Π°ΠΊΡ†ΠΈΠΈ вСщСств, Π² Π³Π΅Π½Π΅Ρ€Π°Ρ‚ΠΎΡ€Π΅ гидроэлСктростанции — ΠΎΠ³Ρ€Π°Π½ΠΈΡ‡Π΅Π½Π½Ρ‹ΠΌ Π½Π°ΠΏΠΎΡ€ΠΎΠΌ Π²ΠΎΠ΄Ρ‹ ΠΈ Ρ‚. Π΄.

Π’ случаС Π΄Π²ΡƒΡ…ΠΏΠΎΠ»ΡŽΡΠ½ΠΈΠΊΠ°, содСрТащСго Π²Π½ΡƒΡ‚Ρ€ΠΈ ΡΠ»Π΅ΠΊΡ‚Ρ€ΠΈΡ‡Π΅ΡΠΊΡƒΡŽ схСму, Π²Π½ΡƒΡ‚Ρ€Π΅Π½Π½Π΅Π΅ сопротивлСниС «рассрСдоточСно» Π² элСмСнтах схСмы (Π² Π΄ΠΎΠΏΠΎΠ»Π½Π΅Π½ΠΈΠ΅ ΠΊ пСрСчислСнным Π²Ρ‹ΡˆΠ΅ ΠΌΠ΅Ρ…Π°Π½ΠΈΠ·ΠΌΠ°ΠΌ Π² источникС).

ΠžΡ‚ΡΡŽΠ΄Π° Ρ‚Π°ΠΊΠΆΠ΅ ΡΠ»Π΅Π΄ΡƒΡŽΡ‚ Π½Π΅ΠΊΠΎΡ‚ΠΎΡ€Ρ‹Π΅ особСнности Π²Π½ΡƒΡ‚Ρ€Π΅Π½Π½Π΅Π³ΠΎ сопротивлСния:

Π’Π½ΡƒΡ‚Ρ€Π΅Π½Π½Π΅Π΅ сопротивлСниС Π½Π΅Π²ΠΎΠ·ΠΌΠΎΠΆΠ½ΠΎ ΡƒΠ±Ρ€Π°Ρ‚ΡŒ ΠΈΠ· Π΄Π²ΡƒΡ…ΠΏΠΎΠ»ΡŽΡΠ½ΠΈΠΊΠ°

Π’Π½ΡƒΡ‚Ρ€Π΅Π½Π½Π΅Π΅ сопротивлСниС Π½Π΅ являСтся ΡΡ‚Π°Π±ΠΈΠ»ΡŒΠ½ΠΎΠΉ Π²Π΅Π»ΠΈΡ‡ΠΈΠ½ΠΎΠΉ: ΠΎΠ½ΠΎ ΠΌΠΎΠΆΠ΅Ρ‚ ΠΈΠ·ΠΌΠ΅Π½ΡΡ‚ΡŒΡΡ ΠΏΡ€ΠΈ ΠΈΠ·ΠΌΠ΅Π½Π΅Π½ΠΈΠΈ ΠΊΠ°ΠΊΠΈΡ…-Π»ΠΈΠ±ΠΎ Π²Π½Π΅ΡˆΠ½ΠΈΡ… условий.

ВлияниС Π²Π½ΡƒΡ‚Ρ€Π΅Π½Π½Π΅Π³ΠΎ сопротивлСния Π½Π° свойства Π΄Π²ΡƒΡ…ΠΏΠΎΠ»ΡŽΡΠ½ΠΈΠΊΠ°

Π­Ρ„Ρ„Π΅ΠΊΡ‚ Π²Π½ΡƒΡ‚Ρ€Π΅Π½Π½Π΅Π³ΠΎ сопротивлСния являСтся Π½Π΅ΠΎΡ‚ΡŠΠ΅ΠΌΠ»Π΅ΠΌΡ‹ΠΌ свойством любого Π΄Π²ΡƒΡ…ΠΏΠΎΠ»ΡŽΡΠ½ΠΈΠΊΠ°.

Основной Ρ€Π΅Π·ΡƒΠ»ΡŒΡ‚Π°Ρ‚ наличия Π²Π½ΡƒΡ‚Ρ€Π΅Π½Π½Π΅Π³ΠΎ сопротивлСния — это ΠΎΠ³Ρ€Π°Π½ΠΈΡ‡Π΅Π½ΠΈΠ΅ элСктричСской мощности, ΠΊΠΎΡ‚ΠΎΡ€ΡƒΡŽ ΠΌΠΎΠΆΠ½ΠΎ ΠΏΠΎΠ»ΡƒΡ‡ΠΈΡ‚ΡŒ Π² Π½Π°Π³Ρ€ΡƒΠ·ΠΊΠ΅, ΠΏΠΈΡ‚Π°Π΅ΠΌΠΎΠΉ ΠΎΡ‚ этого Π΄Π²ΡƒΡ…ΠΏΠΎΠ»ΡŽΡΠ½ΠΈΠΊΠ°.

Если ΠΊ источнику с Π­Π”Π‘ Π³Π΅Π½Π΅Ρ€Π°Ρ‚ΠΎΡ€Π° напряТСния E ΠΈ Π°ΠΊΡ‚ΠΈΠ²Π½Ρ‹ΠΌ Π²Π½ΡƒΡ‚Ρ€Π΅Π½Π½ΠΈΠΌ сопротивлСниСм r ΠΏΠΎΠ΄ΠΊΠ»ΡŽΡ‡Π΅Π½Π° Π½Π°Π³Ρ€ΡƒΠ·ΠΊΠ° с сопротивлСниСм R, Ρ‚ΠΎ Ρ‚ΠΎΠΊ, напряТСниС ΠΈ ΠΌΠΎΡ‰Π½ΠΎΡΡ‚ΡŒ Π² Π½Π°Π³Ρ€ΡƒΠ·ΠΊΠ΅ Π²Ρ‹Ρ€Π°ΠΆΠ°ΡŽΡ‚ΡΡ ΡΠ»Π΅Π΄ΡƒΡŽΡ‰ΠΈΠΌ ΠΎΠ±Ρ€Π°Π·ΠΎΠΌ.

Расчёт

ΠŸΠΎΠ½ΡΡ‚ΠΈΠ΅ расчёт ΠΏΡ€ΠΈΠΌΠ΅Π½ΠΈΠΌΠΎ ΠΊ схСмС (Π½ΠΎ Π½Π΅ ΠΊ Ρ€Π΅Π°Π»ΡŒΠ½ΠΎΠΌΡƒ устройству). Расчёт ΠΏΡ€ΠΈΠ²Π΅Π΄Ρ‘Π½ для случая чисто Π°ΠΊΡ‚ΠΈΠ²Π½ΠΎΠ³ΠΎ Π²Π½ΡƒΡ‚Ρ€Π΅Π½Π½Π΅Π³ΠΎ сопротивлСния (отличия Ρ€Π΅Π°ΠΊΡ‚ΠΈΠ²Π½ΠΎΠ³ΠΎ сопротивлСния Π±ΡƒΠ΄ΡƒΡ‚ рассмотрСны Π΄Π°Π»Π΅Π΅).

ΠŸΡƒΡΡ‚ΡŒ, имССтся Π΄Π²ΡƒΡ…ΠΏΠΎΠ»ΡŽΡΠ½ΠΈΠΊ, ΠΊΠΎΡ‚ΠΎΡ€Ρ‹ΠΉ ΠΌΠΎΠΆΠ΅Ρ‚ Π±Ρ‹Ρ‚ΡŒ описан ΠΏΡ€ΠΈΠ²Π΅Π΄Π΅Π½Π½ΠΎΠΉ Π²Ρ‹ΡˆΠ΅ эквивалСнтной схСмой. Π”Π²ΡƒΡ…ΠΏΠΎΠ»ΡŽΡΠ½ΠΈΠΊ ΠΎΠ±Π»Π°Π΄Π°Π΅Ρ‚ двумя нСизвСстными ΠΏΠ°Ρ€Π°ΠΌΠ΅Ρ‚Ρ€Π°ΠΌΠΈ, ΠΊΠΎΡ‚ΠΎΡ€Ρ‹Π΅ Π½Π΅ΠΎΠ±Ρ…ΠΎΠ΄ΠΈΠΌΠΎ Π½Π°ΠΉΡ‚ΠΈ:

Π­Π”Π‘ Π³Π΅Π½Π΅Ρ€Π°Ρ‚ΠΎΡ€Π° напряТСния U

Π’Π½ΡƒΡ‚Ρ€Π΅Π½Π½Π΅Π΅ сопротивлСниС r

Π’ ΠΎΠ±Ρ‰Π΅ΠΌ случаС, для опрСдСлСния Π΄Π²ΡƒΡ… нСизвСстных Π½Π΅ΠΎΠ±Ρ…ΠΎΠ΄ΠΈΠΌΠΎ ΡΠ΄Π΅Π»Π°Ρ‚ΡŒ Π΄Π²Π° измСрСния: ΠΈΠ·ΠΌΠ΅Ρ€ΠΈΡ‚ΡŒ напряТСниС Π½Π° Π²Ρ‹Ρ…ΠΎΠ΄Π΅ Π΄Π²ΡƒΡ…ΠΏΠΎΠ»ΡŽΡΠ½ΠΈΠΊΠ° (Ρ‚ΠΎ Π΅ΡΡ‚ΡŒ Ρ€Π°Π·Π½ΠΎΡΡ‚ΡŒ ΠΏΠΎΡ‚Π΅Π½Ρ†ΠΈΠ°Π»ΠΎΠ² Uout = Ο†2 βˆ’ Ο†1) ΠΏΡ€ΠΈ Π΄Π²ΡƒΡ… Ρ€Π°Π·Π»ΠΈΡ‡Π½Ρ‹Ρ… Ρ‚ΠΎΠΊΠ°Ρ… Π½Π°Π³Ρ€ΡƒΠ·ΠΊΠΈ. Π’ΠΎΠ³Π΄Π° нСизвСстныС ΠΏΠ°Ρ€Π°ΠΌΠ΅Ρ‚Ρ€Ρ‹ ΠΌΠΎΠΆΠ½ΠΎ Π½Π°ΠΉΡ‚ΠΈ ΠΈΠ· систСмы ΡƒΡ€Π°Π²Π½Π΅Π½ΠΈΠΉ:

Π³Π΄Π΅ Uout1 — Π²Ρ‹Ρ…ΠΎΠ΄Π½ΠΎΠ΅ напряТСниС ΠΏΡ€ΠΈ Ρ‚ΠΎΠΊΠ΅ I1, Uout2 — Π²Ρ‹Ρ…ΠΎΠ΄Π½ΠΎΠ΅ напряТСниС ΠΏΡ€ΠΈ Ρ‚ΠΎΠΊΠ΅ I2. РСшая систСму ΡƒΡ€Π°Π²Π½Π΅Π½ΠΈΠΉ, Π½Π°Ρ…ΠΎΠ΄ΠΈΠΌ искомыС нСизвСстныС:

ΠžΠ±Ρ‹Ρ‡Π½ΠΎ для вычислСния Π²Π½ΡƒΡ‚Ρ€Π΅Π½Π½Π΅Π³ΠΎ сопротивлСния ΠΈΡΠΏΠΎΠ»ΡŒΠ·ΡƒΠ΅Ρ‚ΡΡ Π±ΠΎΠ»Π΅Π΅ простая ΠΌΠ΅Ρ‚ΠΎΠ΄ΠΈΠΊΠ°: находится напряТСниС Π² Ρ€Π΅ΠΆΠΈΠΌΠ΅ холостого Ρ…ΠΎΠ΄Π° ΠΈ Ρ‚ΠΎΠΊ Π² Ρ€Π΅ΠΆΠΈΠΌΠ΅ ΠΊΠΎΡ€ΠΎΡ‚ΠΊΠΎΠ³ΠΎ замыкания Π΄Π²ΡƒΡ…ΠΏΠΎΠ»ΡŽΡΠ½ΠΈΠΊΠ°. Π’ этом случаС систСма (1) записываСтся ΡΠ»Π΅Π΄ΡƒΡŽΡ‰ΠΈΠΌ ΠΎΠ±Ρ€Π°Π·ΠΎΠΌ:

Π³Π΄Π΅ Uoc — Π²Ρ‹Ρ…ΠΎΠ΄Π½ΠΎΠ΅ напряТСниС Π² Ρ€Π΅ΠΆΠΈΠΌΠ΅ холостого Ρ…ΠΎΠ΄Π° (Π°Π½Π³Π». open circuit), Ρ‚ΠΎ Π΅ΡΡ‚ΡŒ ΠΏΡ€ΠΈ Π½ΡƒΠ»Π΅Π²ΠΎΠΌ Ρ‚ΠΎΠΊΠ΅ Π½Π°Π³Ρ€ΡƒΠ·ΠΊΠΈ; Isc — Ρ‚ΠΎΠΊ Π½Π°Π³Ρ€ΡƒΠ·ΠΊΠΈ Π² Ρ€Π΅ΠΆΠΈΠΌΠ΅ ΠΊΠΎΡ€ΠΎΡ‚ΠΊΠΎΠ³ΠΎ замыкания (Π°Π½Π³Π». short circuit), Ρ‚ΠΎ Π΅ΡΡ‚ΡŒ ΠΏΡ€ΠΈ Π½Π°Π³Ρ€ΡƒΠ·ΠΊΠ΅ с Π½ΡƒΠ»Π΅Π²Ρ‹ΠΌ сопротивлСниСм. Π—Π΄Π΅ΡΡŒ ΡƒΡ‡Ρ‚Π΅Π½ΠΎ, Ρ‡Ρ‚ΠΎ Π²Ρ‹Ρ…ΠΎΠ΄Π½ΠΎΠΉ Ρ‚ΠΎΠΊ Π² Ρ€Π΅ΠΆΠΈΠΌΠ΅ холостого Ρ…ΠΎΠ΄Π° ΠΈ Π²Ρ‹Ρ…ΠΎΠ΄Π½ΠΎΠ΅ напряТСниС Π² Ρ€Π΅ΠΆΠΈΠΌΠ΅ ΠΊΠΎΡ€ΠΎΡ‚ΠΊΠΎΠ³ΠΎ замыкания Ρ€Π°Π²Π½Ρ‹ Π½ΡƒΠ»ΡŽ. Из послСдних ΡƒΡ€Π°Π²Π½Π΅Π½ΠΈΠΉ сразу ΠΆΠ΅ ΠΏΠΎΠ»ΡƒΡ‡Π°Π΅ΠΌ:

Π˜Π·ΠΌΠ΅Ρ€Π΅Π½ΠΈΠ΅

ΠŸΠΎΠ½ΡΡ‚ΠΈΠ΅ ΠΈΠ·ΠΌΠ΅Ρ€Π΅Π½ΠΈΠ΅ ΠΏΡ€ΠΈΠΌΠ΅Π½ΠΈΠΌΠΎ ΠΊ Ρ€Π΅Π°Π»ΡŒΠ½ΠΎΠΌΡƒ устройству (Π½ΠΎ Π½Π΅ ΠΊ схСмС). НСпосрСдствСнноС ΠΈΠ·ΠΌΠ΅Ρ€Π΅Π½ΠΈΠ΅ ΠΎΠΌΠΌΠ΅Ρ‚Ρ€ΠΎΠΌ Π½Π΅Π²ΠΎΠ·ΠΌΠΎΠΆΠ½ΠΎ, ΠΏΠΎΡΠΊΠΎΠ»ΡŒΠΊΡƒ нСльзя ΠΏΠΎΠ΄ΠΊΠ»ΡŽΡ‡ΠΈΡ‚ΡŒ Ρ‰ΡƒΠΏΡ‹ ΠΏΡ€ΠΈΠ±ΠΎΡ€Π° ΠΊ Π²Ρ‹Π²ΠΎΠ΄Π°ΠΌ Π²Π½ΡƒΡ‚Ρ€Π΅Π½Π½Π΅Π³ΠΎ сопротивлСния. ΠŸΠΎΡΡ‚ΠΎΠΌΡƒ Π½Π΅ΠΎΠ±Ρ…ΠΎΠ΄ΠΈΠΌΠΎ косвСнноС ΠΈΠ·ΠΌΠ΅Ρ€Π΅Π½ΠΈΠ΅, ΠΊΠΎΡ‚ΠΎΡ€ΠΎΠ΅ ΠΏΡ€ΠΈΠ½Ρ†ΠΈΠΏΠΈΠ°Π»ΡŒΠ½ΠΎ Π½Π΅ отличаСтся ΠΎΡ‚ расчСта — Ρ‚Π°ΠΊΠΆΠ΅ Π½Π΅ΠΎΠ±Ρ…ΠΎΠ΄ΠΈΠΌΡ‹ напряТСния Π½Π° Π½Π°Π³Ρ€ΡƒΠ·ΠΊΠ΅ ΠΏΡ€ΠΈ Π΄Π²ΡƒΡ… Ρ€Π°Π·Π»ΠΈΡ‡Π½Ρ‹Ρ… значСниях Ρ‚ΠΎΠΊΠ°. Однако Π²ΠΎΡΠΏΠΎΠ»ΡŒΠ·ΠΎΠ²Π°Ρ‚ΡŒΡΡ ΡƒΠΏΡ€ΠΎΡ‰Π΅Π½Π½ΠΎΠΉ Ρ„ΠΎΡ€ΠΌΡƒΠ»ΠΎΠΉ (2) Π½Π΅ всСгда Π²ΠΎΠ·ΠΌΠΎΠΆΠ½ΠΎ, ΠΏΠΎΡΠΊΠΎΠ»ΡŒΠΊΡƒ Π½Π΅ ΠΊΠ°ΠΆΠ΄Ρ‹ΠΉ Ρ€Π΅Π°Π»ΡŒΠ½Ρ‹ΠΉ Π΄Π²ΡƒΡ…ΠΏΠΎΠ»ΡŽΡΠ½ΠΈΠΊ допускаСт Ρ€Π°Π±ΠΎΡ‚Ρƒ Π² Ρ€Π΅ΠΆΠΈΠΌΠ΅ ΠΊΠΎΡ€ΠΎΡ‚ΠΊΠΎΠ³ΠΎ замыкания.

Часто примСняСтся ΡΠ»Π΅Π΄ΡƒΡŽΡ‰ΠΈΠΉ простой способ измСрСния, Π½Π΅ Ρ‚Ρ€Π΅Π±ΡƒΡŽΡ‰ΠΈΠΉ вычислСний:

Π˜Π·ΠΌΠ΅Ρ€ΡΠ΅Ρ‚ΡΡ напряТСниС холостого Ρ…ΠΎΠ΄Π°

Π’ качСствС Π½Π°Π³Ρ€ΡƒΠ·ΠΊΠΈ ΠΏΠΎΠ΄ΠΊΠ»ΡŽΡ‡Π°Π΅Ρ‚ΡΡ ΠΏΠ΅Ρ€Π΅ΠΌΠ΅Π½Π½Ρ‹ΠΉ рСзистор ΠΈ Π΅Π³ΠΎ сопротивлСниС подбираСтся Ρ‚Π°ΠΊΠΈΠΌ ΠΎΠ±Ρ€Π°Π·ΠΎΠΌ, Ρ‡Ρ‚ΠΎΠ±Ρ‹ напряТСниС Π½Π° Π½Π΅ΠΌ составило ΠΏΠΎΠ»ΠΎΠ²ΠΈΠ½Ρƒ ΠΎΡ‚ напряТСния холостого Ρ…ΠΎΠ΄Π°.

ПослС описанных ΠΏΡ€ΠΎΡ†Π΅Π΄ΡƒΡ€ сопротивлСниС рСзистора Π½Π°Π³Ρ€ΡƒΠ·ΠΊΠΈ Π½Π΅ΠΎΠ±Ρ…ΠΎΠ΄ΠΈΠΌΠΎ ΠΈΠ·ΠΌΠ΅Ρ€ΠΈΡ‚ΡŒ ΠΎΠΌΠΌΠ΅Ρ‚Ρ€ΠΎΠΌ — ΠΎΠ½ΠΎ Π±ΡƒΠ΄Π΅Ρ‚ Ρ€Π°Π²Π½ΠΎ Π²Π½ΡƒΡ‚Ρ€Π΅Π½Π½Π΅ΠΌΡƒ ΡΠΎΠΏΡ€ΠΎΡ‚ΠΈΠ²Π»Π΅Π½ΠΈΡŽ Π΄Π²ΡƒΡ…ΠΏΠΎΠ»ΡŽΡΠ½ΠΈΠΊΠ°.

Какой Π±Ρ‹ способ измСрСния Π½ΠΈ использовался, слСдуСт ΠΎΠΏΠ°ΡΠ°Ρ‚ΡŒΡΡ ΠΏΠ΅Ρ€Π΅Π³Ρ€ΡƒΠ·ΠΊΠΈ Π΄Π²ΡƒΡ…ΠΏΠΎΠ»ΡŽΡΠ½ΠΈΠΊΠ° Ρ‡Ρ€Π΅Π·ΠΌΠ΅Ρ€Π½Ρ‹ΠΌ Ρ‚ΠΎΠΊΠΎΠΌ, Ρ‚ΠΎ Π΅ΡΡ‚ΡŒ Ρ‚ΠΎΠΊ Π½Π΅ Π΄ΠΎΠ»ΠΆΠ΅Π½ ΠΏΡ€Π΅Π²Ρ‹ΡˆΠ°Ρ‚ΡŒ максимально допустимого значСниях для Π΄Π°Π½Π½ΠΎΠ³ΠΎ Π΄Π²ΡƒΡ…ΠΏΠΎΠ»ΡŽΡΠ½ΠΈΠΊΠ°.

Π Π΅Π°ΠΊΡ‚ΠΈΠ²Π½ΠΎΠ΅ Π²Π½ΡƒΡ‚Ρ€Π΅Π½Π½Π΅Π΅ сопротивлСниС

Если эквивалСнтная схСма Π΄Π²ΡƒΡ…ΠΏΠΎΠ»ΡŽΡΠ½ΠΈΠΊΠ° содСрТит Ρ€Π΅Π°ΠΊΡ‚ΠΈΠ²Π½Ρ‹Π΅ элСмСнты — кондСнсаторы ΠΈ/ΠΈΠ»ΠΈ ΠΊΠ°Ρ‚ΡƒΡˆΠΊΠΈ индуктивности, Ρ‚ΠΎ расчСт Ρ€Π΅Π°ΠΊΡ‚ΠΈΠ²Π½ΠΎΠ³ΠΎ Π²Π½ΡƒΡ‚Ρ€Π΅Π½Π½Π΅Π³ΠΎ сопротивлСния выполняСтся Ρ‚Π°ΠΊΠΆΠ΅, ΠΊΠ°ΠΊ ΠΈ Π°ΠΊΡ‚ΠΈΠ²Π½ΠΎΠ³ΠΎ, Π½ΠΎ вмСсто сопротивлСний рСзисторов бСрутся комплСксныС импСдансы элСмСнтов, входящих Π² схСму, Π° вмСсто напряТСний ΠΈ Ρ‚ΠΎΠΊΠΎΠ² — ΠΈΡ… комплСксныС Π°ΠΌΠΏΠ»ΠΈΡ‚ΡƒΠ΄Ρ‹, Ρ‚ΠΎ Π΅ΡΡ‚ΡŒ расчСт производится ΠΌΠ΅Ρ‚ΠΎΠ΄ΠΎΠΌ комплСксных Π°ΠΌΠΏΠ»ΠΈΡ‚ΡƒΠ΄.

Π˜Π·ΠΌΠ΅Ρ€Π΅Π½ΠΈΠ΅ Ρ€Π΅Π°ΠΊΡ‚ΠΈΠ²Π½ΠΎΠ³ΠΎ Π²Π½ΡƒΡ‚Ρ€Π΅Π½Π½Π΅Π³ΠΎ сопротивлСния ΠΈΠΌΠ΅Π΅Ρ‚ Π½Π΅ΠΊΠΎΡ‚ΠΎΡ€Ρ‹Π΅ особСнности, ΠΏΠΎΡΠΊΠΎΠ»ΡŒΠΊΡƒ ΠΎΠ½ΠΎ являСтся комплСкснозначной Ρ„ΡƒΠ½ΠΊΡ†ΠΈΠ΅ΠΉ, Π° Π½Π΅ скалярным Π·Π½Π°Ρ‡Π΅Π½ΠΈΠ΅ΠΌ:

МоТно ΠΈΡΠΊΠ°Ρ‚ΡŒ Ρ€Π°Π·Π»ΠΈΡ‡Π½Ρ‹Π΅ ΠΏΠ°Ρ€Π°ΠΌΠ΅Ρ‚Ρ€Ρ‹ комплСксного значСния: ΠΌΠΎΠ΄ΡƒΠ»ΡŒ, Π°Ρ€Π³ΡƒΠΌΠ΅Π½Ρ‚, Ρ‚ΠΎΠ»ΡŒΠΊΠΎ Π²Π΅Ρ‰Π΅ΡΡ‚Π²Π΅Π½Π½ΡƒΡŽ ΠΈΠ»ΠΈ ΠΌΠ½ΠΈΠΌΡƒΡŽ Ρ‡Π°ΡΡ‚ΡŒ, Π° Ρ‚Π°ΠΊΠΆΠ΅ комплСксноС число ΠΏΠΎΠ»Π½ΠΎΡΡ‚ΡŒΡŽ. БоотвСтствСнно, ΠΌΠ΅Ρ‚ΠΎΠ΄ΠΈΠΊΠ° ΠΈΠ·ΠΌΠ΅Ρ€Π΅Π½ΠΈΠΉ Π±ΡƒΠ΄Π΅Ρ‚ Π·Π°Π²ΠΈΡΠ΅Ρ‚ΡŒ ΠΎΡ‚ Ρ‚ΠΎΠ³ΠΎ, Ρ‡Ρ‚ΠΎ Ρ…ΠΎΡ‚ΠΈΠΌ ΠΏΠΎΠ»ΡƒΡ‡ΠΈΡ‚ΡŒ.

Допустим, Π΅ΡΡ‚ΡŒ ΠΏΡ€ΠΎΡΡ‚Π΅ΠΉΡˆΠ°Ρ элСктричСская замкнутая Ρ†Π΅ΠΏΡŒ, Π²ΠΊΠ»ΡŽΡ‡Π°ΡŽΡ‰Π°Ρ Π² сСбя источник Ρ‚ΠΎΠΊΠ°, Π½Π°ΠΏΡ€ΠΈΠΌΠ΅Ρ€ Π³Π΅Π½Π΅Ρ€Π°Ρ‚ΠΎΡ€, Π³Π°Π»ΡŒΠ²Π°Π½ΠΈΡ‡Π΅ΡΠΊΠΈΠΉ элСмСнт ΠΈΠ»ΠΈ аккумулятор, ΠΈ рСзистор, ΠΎΠ±Π»Π°Π΄Π°ΡŽΡ‰ΠΈΠΉ сопротивлСниСм R. ΠŸΠΎΡΠΊΠΎΠ»ΡŒΠΊΡƒ Ρ‚ΠΎΠΊ Π² Ρ†Π΅ΠΏΠΈ Π½ΠΈΠ³Π΄Π΅ Π½Π΅ прСрываСтся, Ρ‚ΠΎ ΠΈ Π²Π½ΡƒΡ‚Ρ€ΠΈ источника ΠΎΠ½ Ρ‚Π΅Ρ‡Π΅Ρ‚.

Π’ Ρ‚Π°ΠΊΠΎΠΉ ситуации ΠΌΠΎΠΆΠ½ΠΎ ΡΠΊΠ°Π·Π°Ρ‚ΡŒ, Ρ‡Ρ‚ΠΎ любой источник ΠΎΠ±Π»Π°Π΄Π°Π΅Ρ‚ Π½Π΅ΠΊΠΎΡ‚ΠΎΡ€Ρ‹ΠΌ Π²Π½ΡƒΡ‚Ρ€Π΅Π½Π½ΠΈΠΌ сопротивлСниСм, ΠΏΡ€Π΅ΠΏΡΡ‚ΡΡ‚Π²ΡƒΡŽΡ‰ΠΈΠΌ Ρ‚ΠΎΠΊΡƒ. Π­Ρ‚ΠΎ Π²Π½ΡƒΡ‚Ρ€Π΅Π½Π½Π΅Π΅ сопротивлСниС Ρ…Π°Ρ€Π°ΠΊΡ‚Π΅Ρ€ΠΈΠ·ΡƒΠ΅Ρ‚ источник Ρ‚ΠΎΠΊΠ° ΠΈ обозначаСтся Π±ΡƒΠΊΠ²ΠΎΠΉ r. Для ΠΈΠ»ΠΈ аккумулятора Π²Π½ΡƒΡ‚Ρ€Π΅Π½Π½Π΅Π΅ сопротивлСниС — это сопротивлСниС раствора элСктролита ΠΈ элСктродов, для Π³Π΅Π½Π΅Ρ€Π°Ρ‚ΠΎΡ€Π° — сопротивлСниС ΠΎΠ±ΠΌΠΎΡ‚ΠΎΠΊ статора ΠΈ Ρ‚. Π΄.

Π’Π°ΠΊΠΈΠΌ ΠΎΠ±Ρ€Π°Π·ΠΎΠΌ, источник Ρ‚ΠΎΠΊΠ° характСризуСтся ΠΊΠ°ΠΊ Π²Π΅Π»ΠΈΡ‡ΠΈΠ½ΠΎΠΉ Π­Π”Π‘, Ρ‚Π°ΠΊ ΠΈ Π²Π΅Π»ΠΈΡ‡ΠΈΠ½ΠΎΠΉ собствСнного Π²Π½ΡƒΡ‚Ρ€Π΅Π½Π½Π΅Π³ΠΎ сопротивлСния r – ΠΎΠ±Π΅ эти характСристики ΡΠ²ΠΈΠ΄Π΅Ρ‚Π΅Π»ΡŒΡΡ‚Π²ΡƒΡŽΡ‚ ΠΎ качСствС источника.

ЭлСктростатичСскиС Π²Ρ‹ΡΠΎΠΊΠΎΠ²ΠΎΠ»ΡŒΡ‚Π½Ρ‹Π΅ Π³Π΅Π½Π΅Ρ€Π°Ρ‚ΠΎΡ€Ρ‹ (ΠΊΠ°ΠΊ Π³Π΅Π½Π΅Ρ€Π°Ρ‚ΠΎΡ€ Π’Π°Π½ Π΄Π΅ Π“Ρ€Π°Π°Ρ„Π° ΠΈΠ»ΠΈ Π³Π΅Π½Π΅Ρ€Π°Ρ‚ΠΎΡ€ Π£ΠΈΠΌΡˆΡƒΡ€ΡΡ‚Π°), ΠΊ ΠΏΡ€ΠΈΠΌΠ΅Ρ€Ρƒ, ΠΎΡ‚Π»ΠΈΡ‡Π°ΡŽΡ‚ΡΡ ΠΎΠ³Ρ€ΠΎΠΌΠ½ΠΎΠΉ Π­Π”Π‘ измСряСмой ΠΌΠΈΠ»Π»ΠΈΠΎΠ½Π°ΠΌΠΈ Π²ΠΎΠ»ΡŒΡ‚, ΠΏΡ€ΠΈ этом ΠΈΡ… Π²Π½ΡƒΡ‚Ρ€Π΅Π½Π½Π΅Π΅ сопротивлСниС измСряСтся сотнями ΠΌΠ΅Π³Π°ΠΎΠΌ, ΠΏΠΎΡ‚ΠΎΠΌΡƒ ΠΎΠ½ΠΈ ΠΈ Π½Π΅ΠΏΡ€ΠΈΠ³ΠΎΠ΄Π½Ρ‹ для получСния Π±ΠΎΠ»ΡŒΡˆΠΈΡ… Ρ‚ΠΎΠΊΠΎΠ².


Π“Π°Π»ΡŒΠ²Π°Π½ΠΈΡ‡Π΅ΡΠΊΠΈΠ΅ элСмСнты (Ρ‚Π°ΠΊΠΈΠ΅ ΠΊΠ°ΠΊ Π±Π°Ρ‚Π°Ρ€Π΅ΠΉΠΊΠ°) — Π½Π°ΠΏΡ€ΠΎΡ‚ΠΈΠ² — ΠΈΠΌΠ΅ΡŽΡ‚ Π­Π”Π‘ порядка 1 Π²ΠΎΠ»ΡŒΡ‚Π°, хотя Π²Π½ΡƒΡ‚Ρ€Π΅Π½Π½Π΅Π΅ сопротивлСниС Ρƒ Π½ΠΈΡ… порядка Π΄ΠΎΠ»Π΅ΠΉ ΠΈΠ»ΠΈ максимум — дСсятка Ом, ΠΈ ΠΎΡ‚ Π³Π°Π»ΡŒΠ²Π°Π½ΠΈΡ‡Π΅ΡΠΊΠΈΡ… элСмСнтов поэтому ΠΌΠΎΠΆΠ½ΠΎ ΠΏΠΎΠ»ΡƒΡ‡Π°Ρ‚ΡŒ Ρ‚ΠΎΠΊΠΈ Π² Π΅Π΄ΠΈΠ½ΠΈΡ†Ρ‹ ΠΈ дСсятки Π°ΠΌΠΏΠ΅Ρ€.

На Π΄Π°Π½Π½ΠΎΠΉ схСмС ΠΏΠΎΠΊΠ°Π·Π°Π½ Ρ€Π΅Π°Π»ΡŒΠ½Ρ‹ΠΉ источник с присоСдинСнной Π½Π°Π³Ρ€ΡƒΠ·ΠΊΠΎΠΉ. Π—Π΄Π΅ΡΡŒ ΠΎΠ±ΠΎΠ·Π½Π°Ρ‡Π΅Π½Ρ‹ , Π΅Π³ΠΎ Π²Π½ΡƒΡ‚Ρ€Π΅Π½Π½Π΅Π΅ сопротивлСниС, Π° Ρ‚Π°ΠΊΠΆΠ΅ сопротивлСниС Π½Π°Π³Ρ€ΡƒΠ·ΠΊΠΈ. Богласно , Ρ‚ΠΎΠΊ Π² Π΄Π°Π½Π½ΠΎΠΉ Ρ†Π΅ΠΏΠΈ Π±ΡƒΠ΄Π΅Ρ‚ Ρ€Π°Π²Π΅Π½:

ΠŸΠΎΡΠΊΠΎΠ»ΡŒΠΊΡƒ участок внСшнСй Ρ†Π΅ΠΏΠΈ ΠΎΠ΄Π½ΠΎΡ€ΠΎΠ΄Π΅Π½, Ρ‚ΠΎ ΠΈΠ· Π·Π°ΠΊΠΎΠ½Π° Ома ΠΌΠΎΠΆΠ½ΠΎ Π½Π°ΠΉΡ‚ΠΈ напряТСниС Π½Π° Π½Π°Π³Ρ€ΡƒΠ·ΠΊΠ΅:

Π’Ρ‹Ρ€Π°Π·ΠΈΠ² ΠΈΠ· ΠΏΠ΅Ρ€Π²ΠΎΠ³ΠΎ уравнСния сопротивлСниС Π½Π°Π³Ρ€ΡƒΠ·ΠΊΠΈ, ΠΈ подставив Π΅Π³ΠΎ Π·Π½Π°Ρ‡Π΅Π½ΠΈΠ΅ Π²ΠΎ Π²Ρ‚ΠΎΡ€ΠΎΠ΅ ΡƒΡ€Π°Π²Π½Π΅Π½ΠΈΠ΅, ΠΏΠΎΠ»ΡƒΡ‡ΠΈΠΌ Π·Π°Π²ΠΈΡΠΈΠΌΠΎΡΡ‚ΡŒ напряТСния Π½Π° Π½Π°Π³Ρ€ΡƒΠ·ΠΊΠ΅ ΠΎΡ‚ Ρ‚ΠΎΠΊΠ° Π² Π·Π°ΠΌΠΊΠ½ΡƒΡ‚ΠΎΠΉ Ρ†Π΅ΠΏΠΈ:

Π’ Π·Π°ΠΌΠΊΠ½ΡƒΡ‚ΠΎΠΌ ΠΊΠΎΠ½Ρ‚ΡƒΡ€Π΅ Π­Π”Π‘ Ρ€Π°Π²Π½Π° суммС ΠΏΠ°Π΄Π΅Π½ΠΈΠΉ напряТСний Π½Π° элСмСнтах внСшнСй Ρ†Π΅ΠΏΠΈ ΠΈ Π½Π° Π²Π½ΡƒΡ‚Ρ€Π΅Π½Π½Π΅ΠΌ сопротивлСнии самого источника. Π—Π°Π²ΠΈΡΠΈΠΌΠΎΡΡ‚ΡŒ напряТСния Π½Π° Π½Π°Π³Ρ€ΡƒΠ·ΠΊΠ΅ ΠΎΡ‚ Ρ‚ΠΎΠΊΠ° Π½Π°Π³Ρ€ΡƒΠ·ΠΊΠΈ Π² идСальном случаС Π»ΠΈΠ½Π΅ΠΉΠ½Π°.

Π“Ρ€Π°Ρ„ΠΈΠΊ это ΠΏΠΎΠΊΠ°Π·Ρ‹Π²Π°Π΅Ρ‚, Π½ΠΎ ΡΠΊΡΠΏΠ΅Ρ€ΠΈΠΌΠ΅Π½Ρ‚Π°Π»ΡŒΠ½Ρ‹Π΅ Π΄Π°Π½Π½Ρ‹Π΅ Π½Π° Ρ€Π΅Π°Π»ΡŒΠ½ΠΎΠΌ рСзисторС (крСстики Π²ΠΎΠ·Π»Π΅ Π³Ρ€Π°Ρ„ΠΈΠΊΠ°) всСгда ΠΎΡ‚Π»ΠΈΡ‡Π°ΡŽΡ‚ΡΡ ΠΎΡ‚ ΠΈΠ΄Π΅Π°Π»Π°:


ЭкспСримСнты ΠΈ Π»ΠΎΠ³ΠΈΠΊΠ° ΠΏΠΎΠΊΠ°Π·Ρ‹Π²Π°ΡŽΡ‚, Ρ‡Ρ‚ΠΎ ΠΏΡ€ΠΈ Π½ΡƒΠ»Π΅Π²ΠΎΠΌ Ρ‚ΠΎΠΊΠ΅ Π½Π°Π³Ρ€ΡƒΠ·ΠΊΠΈ напряТСниС Π½Π° внСшнСй Ρ†Π΅ΠΏΠΈ Ρ€Π°Π²Π½ΠΎ Π­Π”Π‘ источника, Π° ΠΏΡ€ΠΈ Π½ΡƒΠ»Π΅Π²ΠΎΠΌ напряТСнии Π½Π° Π½Π°Π³Ρ€ΡƒΠ·ΠΊΠ΅ Ρ‚ΠΎΠΊ Π² Ρ†Π΅ΠΏΠΈ Ρ€Π°Π²Π΅Π½ . Π­Ρ‚ΠΎ свойство Ρ€Π΅Π°Π»ΡŒΠ½Ρ‹Ρ… Ρ†Π΅ΠΏΠ΅ΠΉ ΠΏΠΎΠΌΠΎΠ³Π°Π΅Ρ‚ ΡΠΊΡΠΏΠ΅Ρ€ΠΈΠΌΠ΅Π½Ρ‚Π°Π»ΡŒΠ½ΠΎ Π½Π°Ρ…ΠΎΠ΄ΠΈΡ‚ΡŒ Π­Π”Π‘ ΠΈ Π²Π½ΡƒΡ‚Ρ€Π΅Π½Π½Π΅Π΅ сопротивлСниС Ρ€Π΅Π°Π»ΡŒΠ½Ρ‹Ρ… источников.

Π­ΠΊΡΠΏΠ΅Ρ€ΠΈΠΌΠ΅Π½Ρ‚Π°Π»ΡŒΠ½ΠΎΠ΅ Π½Π°Ρ…ΠΎΠΆΠ΄Π΅Π½ΠΈΠ΅ Π²Π½ΡƒΡ‚Ρ€Π΅Π½Π½Π΅Π³ΠΎ сопротивлСния

Π§Ρ‚ΠΎΠ±Ρ‹ ΡΠΊΡΠΏΠ΅Ρ€ΠΈΠΌΠ΅Π½Ρ‚Π°Π»ΡŒΠ½ΠΎ ΠΎΠΏΡ€Π΅Π΄Π΅Π»ΠΈΡ‚ΡŒ Π΄Π°Π½Π½Ρ‹Π΅ характСристики, строят Π³Ρ€Π°Ρ„ΠΈΠΊ зависимости напряТСния Π½Π° Π½Π°Π³Ρ€ΡƒΠ·ΠΊΠ΅ ΠΎΡ‚ Π²Π΅Π»ΠΈΡ‡ΠΈΠ½Ρ‹ Ρ‚ΠΎΠΊΠ°, Π·Π°Ρ‚Π΅ΠΌ ΡΠΊΡΡ‚Ρ€Π°ΠΏΠΎΠ»ΠΈΡ€ΡƒΡŽΡ‚ Π΅Π³ΠΎ Π΄ΠΎ пСрСсСчСния с осями.

Π’ Ρ‚ΠΎΡ‡ΠΊΠ΅ пСрСсСчСния Π³Ρ€Π°Ρ„ΠΈΠΊΠ° с ΠΎΡΡ‚ΡŒΡŽ напряТСния находится Π·Π½Π°Ρ‡Π΅Π½ΠΈΠ΅ Π­Π”Π‘ источника, Π° Π² Ρ‚ΠΎΡ‡ΠΊΠ΅ пСрСсСчСния с осью Ρ‚ΠΎΠΊΠ° находится Π²Π΅Π»ΠΈΡ‡ΠΈΠ½Π° Ρ‚ΠΎΠΊΠ° ΠΊΠΎΡ€ΠΎΡ‚ΠΊΠΎΠ³ΠΎ замыкания. Π’ ΠΈΡ‚ΠΎΠ³Π΅ Π²Π½ΡƒΡ‚Ρ€Π΅Π½Π½Π΅Π΅ сопротивлСниС находится ΠΏΠΎ Ρ„ΠΎΡ€ΠΌΡƒΠ»Π΅:

РазвиваСмая источником полСзная ΠΌΠΎΡ‰Π½ΠΎΡΡ‚ΡŒ выдСляСтся Π½Π° Π½Π°Π³Ρ€ΡƒΠ·ΠΊΠ΅. Π“Ρ€Π°Ρ„ΠΈΠΊ зависимости этой мощности ΠΎΡ‚ сопротивлСния Π½Π°Π³Ρ€ΡƒΠ·ΠΊΠΈ ΠΏΡ€ΠΈΠ²Π΅Π΄Π΅Π½ Π½Π° рисункС. Π­Ρ‚Π° кривая начинаСтся ΠΎΡ‚ пСрСсСчСния осСй ΠΊΠΎΠΎΡ€Π΄ΠΈΠ½Π°Ρ‚ Π² Π½ΡƒΠ»Π΅Π²ΠΎΠΉ Ρ‚ΠΎΡ‡ΠΊΠ΅, Π·Π°Ρ‚Π΅ΠΌ возрастаСт Π΄ΠΎ максимального значСния мощности, послС Ρ‡Π΅Π³ΠΎ спадаСт Π΄ΠΎ нуля ΠΏΡ€ΠΈ сопротивлСнии Π½Π°Π³Ρ€ΡƒΠ·ΠΊΠΈ Ρ€Π°Π²Π½ΠΎΠΌ бСсконСчности.


Π§Ρ‚ΠΎΠ±Ρ‹ Π½Π°ΠΉΡ‚ΠΈ максимальноС сопротивлСниС Π½Π°Π³Ρ€ΡƒΠ·ΠΊΠΈ, ΠΏΡ€ΠΈ ΠΊΠΎΡ‚ΠΎΡ€ΠΎΠΌ тСорСтичСски Ρ€Π°Π·ΠΎΠ²ΡŒΠ΅Ρ‚ΡΡ максимальная ΠΌΠΎΡ‰Π½ΠΎΡΡ‚ΡŒ ΠΏΡ€ΠΈ Π΄Π°Π½Π½ΠΎΠΌ источникС, бСрСтся производная ΠΎΡ‚ Ρ„ΠΎΡ€ΠΌΡƒΠ»Ρ‹ мощности ΠΏΠΎ R ΠΈ приравниваСтся ΠΊ Π½ΡƒΠ»ΡŽ. Максимальная ΠΌΠΎΡ‰Π½ΠΎΡΡ‚ΡŒ Ρ€Π°Π·ΠΎΠ²ΡŒΠ΅Ρ‚ΡΡ ΠΏΡ€ΠΈ сопротивлСнии внСшнСй Ρ†Π΅ΠΏΠΈ, Ρ€Π°Π²Π½ΠΎΠΌ Π²Π½ΡƒΡ‚Ρ€Π΅Π½Π½Π΅ΠΌΡƒ ΡΠΎΠΏΡ€ΠΎΡ‚ΠΈΠ²Π»Π΅Π½ΠΈΡŽ источника:

Π­Ρ‚ΠΎ ΠΏΠΎΠ»ΠΎΠΆΠ΅Π½ΠΈΠ΅ ΠΎ максимальной мощности ΠΏΡ€ΠΈ R = r, позволяСт ΡΠΊΡΠΏΠ΅Ρ€ΠΈΠΌΠ΅Π½Ρ‚Π°Π»ΡŒΠ½ΠΎ Π½Π°ΠΉΡ‚ΠΈ Π²Π½ΡƒΡ‚Ρ€Π΅Π½Π½Π΅Π΅ сопротивлСниС источника, построив Π·Π°Π²ΠΈΡΠΈΠΌΠΎΡΡ‚ΡŒ мощности, выдСляСмой Π½Π° Π½Π°Π³Ρ€ΡƒΠ·ΠΊΠ΅, ΠΎΡ‚ Π²Π΅Π»ΠΈΡ‡ΠΈΠ½Ρ‹ сопротивлСния Π½Π°Π³Ρ€ΡƒΠ·ΠΊΠΈ. Найдя Ρ€Π΅Π°Π»ΡŒΠ½ΠΎΠ΅, Π° Π½Π΅ тСорСтичСскоС, сопротивлСниС Π½Π°Π³Ρ€ΡƒΠ·ΠΊΠΈ, ΠΎΠ±Π΅ΡΠΏΠ΅Ρ‡ΠΈΠ²Π°ΡŽΡ‰Π΅Π΅ ΠΌΠ°ΠΊΡΠΈΠΌΠ°Π»ΡŒΠ½ΡƒΡŽ ΠΌΠΎΡ‰Π½ΠΎΡΡ‚ΡŒ, ΠΎΠΏΡ€Π΅Π΄Π΅Π»ΡΡŽΡ‚ Ρ€Π΅Π°Π»ΡŒΠ½ΠΎΠ΅ Π²Π½ΡƒΡ‚Ρ€Π΅Π½Π½Π΅Π΅ сопротивлСниС источника питания.

ΠšΠŸΠ” источника Ρ‚ΠΎΠΊΠ° ΠΏΠΎΠΊΠ°Π·Ρ‹Π²Π°Π΅Ρ‚ ΠΎΡ‚Π½ΠΎΡˆΠ΅Π½ΠΈΠ΅ максимальной выдСляСмой Π½Π° Π½Π°Π³Ρ€ΡƒΠ·ΠΊΠ΅ мощности ΠΊ ΠΏΠΎΠ»Π½ΠΎΠΉ мощности, ΠΊΠΎΡ‚ΠΎΡ€ΡƒΡŽ Π² Π΄Π°Π½Π½Ρ‹ΠΉ ΠΌΠΎΠΌΠ΅Π½Ρ‚ Ρ€Π°Π·Π²ΠΈΠ²Π°Π΅Ρ‚

ΠΠ΅ΠΎΠ±Ρ…ΠΎΠ΄ΠΈΠΌΠΎΡΡ‚ΡŒ ввСдСния Ρ‚Π΅Ρ€ΠΌΠΈΠ½Π° ΠΌΠΎΠΆΠ½ΠΎ ΠΏΡ€ΠΎΠΈΠ»Π»ΡŽΡΡ‚Ρ€ΠΈΡ€ΠΎΠ²Π°Ρ‚ΡŒ ΡΠ»Π΅Π΄ΡƒΡŽΡ‰ΠΈΠΌ ΠΏΡ€ΠΈΠΌΠ΅Ρ€ΠΎΠΌ. Π‘Ρ€Π°Π²Π½ΠΈΠΌ Π΄Π²Π° химичСских источника постоянного Ρ‚ΠΎΠΊΠ° с ΠΎΠ΄ΠΈΠ½Π°ΠΊΠΎΠ²Ρ‹ΠΌ напряТСниСм:

  • ΠΠ²Ρ‚ΠΎΠΌΠΎΠ±ΠΈΠ»ΡŒΠ½Ρ‹ΠΉ свинцово-кислотный аккумулятор напряТСниСм 12 Π²ΠΎΠ»ΡŒΡ‚ ΠΈ Ρ‘ΠΌΠΊΠΎΡΡ‚ΡŒΡŽ 55 А·ч
  • Π’ΠΎΡΠ΅ΠΌΡŒ Π±Π°Ρ‚Π°Ρ€Π΅Π΅ΠΊ Ρ‚ΠΈΠΏΠΎΡ€Π°Π·ΠΌΠ΅Ρ€Π° АА, соСдинСнных ΠΏΠΎΡΠ»Π΅Π΄ΠΎΠ²Π°Ρ‚Π΅Π»ΡŒΠ½ΠΎ. Π‘ΡƒΠΌΠΌΠ°Ρ€Π½ΠΎΠ΅ напряТСниС Ρ‚Π°ΠΊΠΎΠΉ Π±Π°Ρ‚Π°Ρ€Π΅ΠΈ Ρ‚Π°ΠΊΠΆΠ΅ 12 Π²ΠΎΠ»ΡŒΡ‚, Ρ‘ΠΌΠΊΠΎΡΡ‚ΡŒ Π·Π½Π°Ρ‡ΠΈΡ‚Π΅Π»ΡŒΠ½ΠΎ мСньшС — ΠΏΡ€ΠΈΠΌΠ΅Ρ€Π½ΠΎ 1 А·ч

НСсмотря Π½Π° ΠΎΠ΄ΠΈΠ½Π°ΠΊΠΎΠ²ΠΎΠ΅ напряТСниС, эти источники Π·Π½Π°Ρ‡ΠΈΡ‚Π΅Π»ΡŒΠ½ΠΎ ΠΎΡ‚Π»ΠΈΡ‡Π°ΡŽΡ‚ΡΡ ΠΏΡ€ΠΈ Ρ€Π°Π±ΠΎΡ‚Π΅ Π½Π° ΠΎΠ΄ΠΈΠ½Π°ΠΊΠΎΠ²ΡƒΡŽ Π½Π°Π³Ρ€ΡƒΠ·ΠΊΡƒ. Π’Π°ΠΊ, Π°Π²Ρ‚ΠΎΠΌΠΎΠ±ΠΈΠ»ΡŒΠ½Ρ‹ΠΉ аккумулятор способСн ΠΎΡ‚Π΄Π°Ρ‚ΡŒ Π² Π½Π°Π³Ρ€ΡƒΠ·ΠΊΡƒ большой Ρ‚ΠΎΠΊ (ΠΎΡ‚ аккумулятора заводится Π΄Π²ΠΈΠ³Π°Ρ‚Π΅Π»ΡŒ автомобиля, ΠΏΡ€ΠΈ этом стартСр потрСбляСт Ρ‚ΠΎΠΊ 250 Π°ΠΌΠΏΠ΅Ρ€), Π° ΠΎΡ‚ Ρ†Π΅ΠΏΠΎΡ‡ΠΊΠΈ Π±Π°Ρ‚Π°Ρ€Π΅Π΅ΠΊ стартСр Π²ΠΎΠΎΠ±Ρ‰Π΅ Π½Π΅ вращаСтся. ΠžΡ‚Π½ΠΎΡΠΈΡ‚Π΅Π»ΡŒΠ½ΠΎ нСбольшая Π΅ΠΌΠΊΠΎΡΡ‚ΡŒ Π±Π°Ρ‚Π°Ρ€Π΅Π΅ΠΊ Π½Π΅ являСтся ΠΏΡ€ΠΈΡ‡ΠΈΠ½ΠΎΠΉ: ΠΎΠ΄Π½ΠΎΠ³ΠΎ Π°ΠΌΠΏΠ΅Ρ€-часа Π² Π±Π°Ρ‚Π°Ρ€Π΅ΠΉΠΊΠ°Ρ… Ρ…Π²Π°Ρ‚ΠΈΠ»ΠΎ Π±Ρ‹ для Ρ‚ΠΎΠ³ΠΎ, Ρ‡Ρ‚ΠΎΠ±Ρ‹ Π²Ρ€Π°Ρ‰Π°Ρ‚ΡŒ стартСр Π² Ρ‚Π΅Ρ‡Π΅Π½ΠΈΠ΅ 14 сСкунд (ΠΏΡ€ΠΈ Ρ‚ΠΎΠΊΠ΅ 250 Π°ΠΌΠΏΠ΅Ρ€).

Π’Π°ΠΊΠΈΠΌ ΠΎΠ±Ρ€Π°Π·ΠΎΠΌ, для Π΄Π²ΡƒΡ…ΠΏΠΎΠ»ΡŽΡΠ½ΠΈΠΊΠΎΠ², содСрТащих источники (Ρ‚ΠΎ Π΅ΡΡ‚ΡŒ Π³Π΅Π½Π΅Ρ€Π°Ρ‚ΠΎΡ€Ρ‹ напряТСния ΠΈ Π³Π΅Π½Π΅Ρ€Π°Ρ‚ΠΎΡ€Ρ‹ Ρ‚ΠΎΠΊΠ°) Π½Π΅ΠΎΠ±Ρ…ΠΎΠ΄ΠΈΠΌΠΎ Π³ΠΎΠ²ΠΎΡ€ΠΈΡ‚ΡŒ ΠΈΠΌΠ΅Π½Π½ΠΎ ΠΎ Π²Π½ΡƒΡ‚Ρ€Π΅Π½Π½Π΅ΠΌ сопротивлСнии (ΠΈΠ»ΠΈ импСдансС). Если ΠΆΠ΅ Π΄Π²ΡƒΡ…ΠΏΠΎΠ»ΡŽΡΠ½ΠΈΠΊ Π½Π΅ содСрТит источников, Ρ‚ΠΎ Β«Π²Π½ΡƒΡ‚Ρ€Π΅Π½Π½Π΅Π΅ сопротивлСниС» для Ρ‚Π°ΠΊΠΎΠ³ΠΎ Π΄Π²ΡƒΡ…ΠΏΠΎΠ»ΡŽΡΠ½ΠΈΠΊΠ° ΠΎΠ·Π½Π°Ρ‡Π°Π΅Ρ‚ Ρ‚ΠΎ ΠΆΠ΅ самоС, Ρ‡Ρ‚ΠΎ ΠΈ просто «сопротивлСниС».

РодствСнныС Ρ‚Π΅Ρ€ΠΌΠΈΠ½Ρ‹

Если Π² ΠΊΠ°ΠΊΠΎΠΉ-Π»ΠΈΠ±ΠΎ систСмС ΠΌΠΎΠΆΠ½ΠΎ Π²Ρ‹Π΄Π΅Π»ΠΈΡ‚ΡŒ Π²Ρ…ΠΎΠ΄ ΠΈ/ΠΈΠ»ΠΈ Π²Ρ‹Ρ…ΠΎΠ΄, Ρ‚ΠΎ часто ΡƒΠΏΠΎΡ‚Ρ€Π΅Π±Π»ΡΡŽΡ‚ΡΡ ΡΠ»Π΅Π΄ΡƒΡŽΡ‰ΠΈΠ΅ Ρ‚Π΅Ρ€ΠΌΠΈΠ½Ρ‹:

ЀизичСскиС ΠΏΡ€ΠΈΠ½Ρ†ΠΈΠΏΡ‹

НСсмотря Π½Π° Ρ‚ΠΎ, Ρ‡Ρ‚ΠΎ Π½Π° эквивалСнтной схСмС Π²Π½ΡƒΡ‚Ρ€Π΅Π½Π½Π΅Π΅ сопротивлСниС прСдставлСно ΠΊΠ°ΠΊ ΠΎΠ΄ΠΈΠ½ пассивный элСмСнт (ΠΏΡ€ΠΈΡ‡Ρ‘ΠΌ Π°ΠΊΡ‚ΠΈΠ²Π½ΠΎΠ΅ сопротивлСниС , Ρ‚ΠΎ Π΅ΡΡ‚ΡŒ рСзистор Π² Π½Ρ‘ΠΌ присутствуСт ΠΎΠ±ΡΠ·Π°Ρ‚Π΅Π»ΡŒΠ½ΠΎ), Π²Π½ΡƒΡ‚Ρ€Π΅Π½Π½Π΅Π΅ сопротивлСниС Π½Π΅ сосрСдоточСно Π² ΠΊΠ°ΠΊΠΎΠΌ-Π»ΠΈΠ±ΠΎ ΠΎΠ΄Π½ΠΎΠΌ элСмСнтС. Π”Π²ΡƒΡ…ΠΏΠΎΠ»ΡŽΡΠ½ΠΈΠΊ лишь внСшнС Π²Π΅Π΄Ρ‘Ρ‚ сСбя Ρ‚Π°ΠΊ, словно Π² Π½Ρ‘ΠΌ имССтся сосрСдоточСнный Π²Π½ΡƒΡ‚Ρ€Π΅Π½Π½ΠΈΠΉ импСданс ΠΈ Π³Π΅Π½Π΅Ρ€Π°Ρ‚ΠΎΡ€ напряТСния. Π’ Π΄Π΅ΠΉΡΡ‚Π²ΠΈΡ‚Π΅Π»ΡŒΠ½ΠΎΡΡ‚ΠΈ Π²Π½ΡƒΡ‚Ρ€Π΅Π½Π½Π΅Π΅ сопротивлСниС являСтся внСшним проявлСниСм совокупности физичСских эффСктов:

  • Если Π² Π΄Π²ΡƒΡ…ΠΏΠΎΠ»ΡŽΡΠ½ΠΈΠΊΠ΅ имССтся Ρ‚ΠΎΠ»ΡŒΠΊΠΎ источник энСргии Π±Π΅Π· ΠΊΠ°ΠΊΠΎΠΉ-Π»ΠΈΠ±ΠΎ элСктричСской схСмы (Π½Π°ΠΏΡ€ΠΈΠΌΠ΅Ρ€, Π³Π°Π»ΡŒΠ²Π°Π½ΠΈΡ‡Π΅ΡΠΊΠΈΠΉ элСмСнт), Ρ‚ΠΎ Π²Π½ΡƒΡ‚Ρ€Π΅Π½Π½Π΅Π΅ сопротивлСниС практичСски чисто Π°ΠΊΡ‚ΠΈΠ²Π½ΠΎΠ΅ (Ссли Ρ‚ΠΎΠ»ΡŒΠΊΠΎ Ρ€Π΅Ρ‡ΡŒ Π½Π΅ ΠΈΠ΄Π΅Ρ‚ ΠΎΠ± ΠΎΡ‡Π΅Π½ΡŒ высоких частотах), ΠΎΠ½ΠΎ обусловлСно физичСскими эффСктами, ΠΊΠΎΡ‚ΠΎΡ€Ρ‹Π΅ Π½Π΅ ΠΏΠΎΠ·Π²ΠΎΠ»ΡΡŽΡ‚ мощности , ΠΎΡ‚Π΄Π°Π²Π°Π΅ΠΌΠΎΠΉ этим источником Π² Π½Π°Π³Ρ€ΡƒΠ·ΠΊΡƒ, ΠΏΡ€Π΅Π²Ρ‹ΡΠΈΡ‚ΡŒ ΠΎΠΏΡ€Π΅Π΄Π΅Π»Ρ‘Π½Π½Ρ‹ΠΉ ΠΏΡ€Π΅Π΄Π΅Π». НаиболСС простой ΠΏΡ€ΠΈΠΌΠ΅Ρ€ Ρ‚Π°ΠΊΠΎΠ³ΠΎ эффСкта — Π½Π΅Π½ΡƒΠ»Π΅Π²ΠΎΠ΅ сопротивлСниС ΠΏΡ€ΠΎΠ²ΠΎΠ΄Π½ΠΈΠΊΠΎΠ² элСктричСской Ρ†Π΅ΠΏΠΈ. Но, ΠΊΠ°ΠΊ ΠΏΡ€Π°Π²ΠΈΠ»ΠΎ, наибольший Π²ΠΊΠ»Π°Π΄ Π² ΠΎΠ³Ρ€Π°Π½ΠΈΡ‡Π΅Π½ΠΈΠ΅ мощности вносят эффСкты нСэлСктричСской ΠΏΡ€ΠΈΡ€ΠΎΠ΄Ρ‹. Π’Π°ΠΊ, Π½Π°ΠΏΡ€ΠΈΠΌΠ΅Ρ€, Π² ΠΌΠΎΡ‰Π½ΠΎΡΡ‚ΡŒ ΠΌΠΎΠΆΠ΅Ρ‚ Π±Ρ‹Ρ‚ΡŒ ΠΎΠ³Ρ€Π°Π½ΠΈΡ‡Π΅Π½Π° ΠΏΠ»ΠΎΡ‰Π°Π΄ΡŒΡŽ соприкосновСния ΡƒΡ‡Π°ΡΡ‚Π²ΡƒΡŽΡ‰ΠΈΡ… Π² Ρ€Π΅Π°ΠΊΡ†ΠΈΠΈ вСщСств, Π² Π³Π΅Π½Π΅Ρ€Π°Ρ‚ΠΎΡ€Π΅ гидроэлСктростанции — ΠΎΠ³Ρ€Π°Π½ΠΈΡ‡Π΅Π½Π½Ρ‹ΠΌ Π½Π°ΠΏΠΎΡ€ΠΎΠΌ Π²ΠΎΠ΄Ρ‹ ΠΈ Ρ‚. Π΄.
  • Π’ случаС Π΄Π²ΡƒΡ…ΠΏΠΎΠ»ΡŽΡΠ½ΠΈΠΊΠ°, содСрТащСго Π²Π½ΡƒΡ‚Ρ€ΠΈ ΡΠ»Π΅ΠΊΡ‚Ρ€ΠΈΡ‡Π΅ΡΠΊΡƒΡŽ схСму , Π²Π½ΡƒΡ‚Ρ€Π΅Π½Π½Π΅Π΅ сопротивлСниС «рассрСдоточСно» Π² элСмСнтах схСмы (Π² Π΄ΠΎΠΏΠΎΠ»Π½Π΅Π½ΠΈΠ΅ ΠΊ пСрСчислСнным Π²Ρ‹ΡˆΠ΅ ΠΌΠ΅Ρ…Π°Π½ΠΈΠ·ΠΌΠ°ΠΌ Π² источникС).

ΠžΡ‚ΡΡŽΠ΄Π° Ρ‚Π°ΠΊΠΆΠ΅ ΡΠ»Π΅Π΄ΡƒΡŽΡ‚ Π½Π΅ΠΊΠΎΡ‚ΠΎΡ€Ρ‹Π΅ особСнности Π²Π½ΡƒΡ‚Ρ€Π΅Π½Π½Π΅Π³ΠΎ сопротивлСния:

ВлияниС Π²Π½ΡƒΡ‚Ρ€Π΅Π½Π½Π΅Π³ΠΎ сопротивлСния Π½Π° свойства Π΄Π²ΡƒΡ…ΠΏΠΎΠ»ΡŽΡΠ½ΠΈΠΊΠ°

Π­Ρ„Ρ„Π΅ΠΊΡ‚ Π²Π½ΡƒΡ‚Ρ€Π΅Π½Π½Π΅Π³ΠΎ сопротивлСния являСтся Π½Π΅ΠΎΡ‚ΡŠΠ΅ΠΌΠ»Π΅ΠΌΡ‹ΠΌ свойством любого Π°ΠΊΡ‚ΠΈΠ²Π½ΠΎΠ³ΠΎ Π΄Π²ΡƒΡ…ΠΏΠΎΠ»ΡŽΡΠ½ΠΈΠΊΠ°. Основной Ρ€Π΅Π·ΡƒΠ»ΡŒΡ‚Π°Ρ‚ наличия Π²Π½ΡƒΡ‚Ρ€Π΅Π½Π½Π΅Π³ΠΎ сопротивлСния — это ΠΎΠ³Ρ€Π°Π½ΠΈΡ‡Π΅Π½ΠΈΠ΅ элСктричСской мощности, ΠΊΠΎΡ‚ΠΎΡ€ΡƒΡŽ ΠΌΠΎΠΆΠ½ΠΎ ΠΏΠΎΠ»ΡƒΡ‡ΠΈΡ‚ΡŒ Π² Π½Π°Π³Ρ€ΡƒΠ·ΠΊΠ΅, ΠΏΠΈΡ‚Π°Π΅ΠΌΠΎΠΉ ΠΎΡ‚ этого Π΄Π²ΡƒΡ…ΠΏΠΎΠ»ΡŽΡΠ½ΠΈΠΊΠ°.

ΠŸΡƒΡΡ‚ΡŒ, имССтся Π΄Π²ΡƒΡ…ΠΏΠΎΠ»ΡŽΡΠ½ΠΈΠΊ, ΠΊΠΎΡ‚ΠΎΡ€Ρ‹ΠΉ ΠΌΠΎΠΆΠ΅Ρ‚ Π±Ρ‹Ρ‚ΡŒ описан ΠΏΡ€ΠΈΠ²Π΅Π΄Ρ‘Π½Π½ΠΎΠΉ Π²Ρ‹ΡˆΠ΅ эквивалСнтной схСмой. Π”Π²ΡƒΡ…ΠΏΠΎΠ»ΡŽΡΠ½ΠΈΠΊ ΠΎΠ±Π»Π°Π΄Π°Π΅Ρ‚ двумя нСизвСстными ΠΏΠ°Ρ€Π°ΠΌΠ΅Ρ‚Ρ€Π°ΠΌΠΈ, ΠΊΠΎΡ‚ΠΎΡ€Ρ‹Π΅ Π½Π΅ΠΎΠ±Ρ…ΠΎΠ΄ΠΈΠΌΠΎ Π½Π°ΠΉΡ‚ΠΈ:

  • Π­Π”Π‘ Π³Π΅Π½Π΅Ρ€Π°Ρ‚ΠΎΡ€Π° напряТСния U
  • Π’Π½ΡƒΡ‚Ρ€Π΅Π½Π½Π΅Π΅ сопротивлСниС r

Π’ ΠΎΠ±Ρ‰Π΅ΠΌ случаС, для опрСдСлСния Π΄Π²ΡƒΡ… нСизвСстных Π½Π΅ΠΎΠ±Ρ…ΠΎΠ΄ΠΈΠΌΠΎ ΡΠ΄Π΅Π»Π°Ρ‚ΡŒ Π΄Π²Π° измСрСния: ΠΈΠ·ΠΌΠ΅Ρ€ΠΈΡ‚ΡŒ напряТСниС Π½Π° Π²Ρ‹Ρ…ΠΎΠ΄Π΅ Π΄Π²ΡƒΡ…ΠΏΠΎΠ»ΡŽΡΠ½ΠΈΠΊΠ° (Ρ‚ΠΎ Π΅ΡΡ‚ΡŒ Ρ€Π°Π·Π½ΠΎΡΡ‚ΡŒ ΠΏΠΎΡ‚Π΅Π½Ρ†ΠΈΠ°Π»ΠΎΠ² U out = Ο† 2 βˆ’ Ο† 1 ) ΠΏΡ€ΠΈ Π΄Π²ΡƒΡ… Ρ€Π°Π·Π»ΠΈΡ‡Π½Ρ‹Ρ… Ρ‚ΠΎΠΊΠ°Ρ… Π½Π°Π³Ρ€ΡƒΠ·ΠΊΠΈ. Π’ΠΎΠ³Π΄Π° нСизвСстныС ΠΏΠ°Ρ€Π°ΠΌΠ΅Ρ‚Ρ€Ρ‹ ΠΌΠΎΠΆΠ½ΠΎ Π½Π°ΠΉΡ‚ΠΈ ΠΈΠ· систСмы ΡƒΡ€Π°Π²Π½Π΅Π½ΠΈΠΉ:

Π³Π΄Π΅ U out1 I 1 , U out2 — Π²Ρ‹Ρ…ΠΎΠ΄Π½ΠΎΠ΅ напряТСниС ΠΏΡ€ΠΈ Ρ‚ΠΎΠΊΠ΅ I 2 . РСшая систСму ΡƒΡ€Π°Π²Π½Π΅Π½ΠΈΠΉ, Π½Π°Ρ…ΠΎΠ΄ΠΈΠΌ искомыС нСизвСстныС:

ΠžΠ±Ρ‹Ρ‡Π½ΠΎ для вычислСния Π²Π½ΡƒΡ‚Ρ€Π΅Π½Π½Π΅Π³ΠΎ сопротивлСния ΠΈΡΠΏΠΎΠ»ΡŒΠ·ΡƒΠ΅Ρ‚ΡΡ Π±ΠΎΠ»Π΅Π΅ простая ΠΌΠ΅Ρ‚ΠΎΠ΄ΠΈΠΊΠ°: находится напряТСниС Π² Ρ€Π΅ΠΆΠΈΠΌΠ΅ холостого Ρ…ΠΎΠ΄Π° ΠΈ Ρ‚ΠΎΠΊ Π² Ρ€Π΅ΠΆΠΈΠΌΠ΅ ΠΊΠΎΡ€ΠΎΡ‚ΠΊΠΎΠ³ΠΎ замыкания Π΄Π²ΡƒΡ…ΠΏΠΎΠ»ΡŽΡΠ½ΠΈΠΊΠ°. Π’ этом случаС систСма () записываСтся ΡΠ»Π΅Π΄ΡƒΡŽΡ‰ΠΈΠΌ ΠΎΠ±Ρ€Π°Π·ΠΎΠΌ:

Π³Π΄Π΅ U oc — Π²Ρ‹Ρ…ΠΎΠ΄Π½ΠΎΠ΅ напряТСниС Π² Ρ€Π΅ΠΆΠΈΠΌΠ΅ холостого Ρ…ΠΎΠ΄Π° (Π°Π½Π³Π». open circuit ), Ρ‚ΠΎ Π΅ΡΡ‚ΡŒ ΠΏΡ€ΠΈ Π½ΡƒΠ»Π΅Π²ΠΎΠΌ Ρ‚ΠΎΠΊΠ΅ Π½Π°Π³Ρ€ΡƒΠ·ΠΊΠΈ; I sc — Ρ‚ΠΎΠΊ Π½Π°Π³Ρ€ΡƒΠ·ΠΊΠΈ Π² Ρ€Π΅ΠΆΠΈΠΌΠ΅ ΠΊΠΎΡ€ΠΎΡ‚ΠΊΠΎΠ³ΠΎ замыкания (Π°Π½Π³Π». short circuit ), Ρ‚ΠΎ Π΅ΡΡ‚ΡŒ ΠΏΡ€ΠΈ Π½Π°Π³Ρ€ΡƒΠ·ΠΊΠ΅ с Π½ΡƒΠ»Π΅Π²Ρ‹ΠΌ сопротивлСниСм. Π—Π΄Π΅ΡΡŒ ΡƒΡ‡Ρ‚Π΅Π½ΠΎ, Ρ‡Ρ‚ΠΎ Π²Ρ‹Ρ…ΠΎΠ΄Π½ΠΎΠΉ Ρ‚ΠΎΠΊ Π² Ρ€Π΅ΠΆΠΈΠΌΠ΅ холостого Ρ…ΠΎΠ΄Π° ΠΈ Π²Ρ‹Ρ…ΠΎΠ΄Π½ΠΎΠ΅ напряТСниС Π² Ρ€Π΅ΠΆΠΈΠΌΠ΅ ΠΊΠΎΡ€ΠΎΡ‚ΠΊΠΎΠ³ΠΎ замыкания Ρ€Π°Π²Π½Ρ‹ Π½ΡƒΠ»ΡŽ. Из послСдних ΡƒΡ€Π°Π²Π½Π΅Π½ΠΈΠΉ сразу ΠΆΠ΅ ΠΏΠΎΠ»ΡƒΡ‡Π°Π΅ΠΌ:

Π˜Π·ΠΌΠ΅Ρ€Π΅Π½ΠΈΠ΅

ΠŸΠΎΠ½ΡΡ‚ΠΈΠ΅ ΠΈΠ·ΠΌΠ΅Ρ€Π΅Π½ΠΈΠ΅ ΠΏΡ€ΠΈΠΌΠ΅Π½ΠΈΠΌΠΎ ΠΊ Ρ€Π΅Π°Π»ΡŒΠ½ΠΎΠΌΡƒ устройству (Π½ΠΎ Π½Π΅ ΠΊ схСмС). НСпосрСдствСнноС ΠΈΠ·ΠΌΠ΅Ρ€Π΅Π½ΠΈΠ΅ ΠΎΠΌΠΌΠ΅Ρ‚Ρ€ΠΎΠΌ Π½Π΅Π²ΠΎΠ·ΠΌΠΎΠΆΠ½ΠΎ, ΠΏΠΎΡΠΊΠΎΠ»ΡŒΠΊΡƒ нСльзя ΠΏΠΎΠ΄ΠΊΠ»ΡŽΡ‡ΠΈΡ‚ΡŒ Ρ‰ΡƒΠΏΡ‹ ΠΏΡ€ΠΈΠ±ΠΎΡ€Π° ΠΊ Π²Ρ‹Π²ΠΎΠ΄Π°ΠΌ Π²Π½ΡƒΡ‚Ρ€Π΅Π½Π½Π΅Π³ΠΎ сопротивлСния. ΠŸΠΎΡΡ‚ΠΎΠΌΡƒ Π½Π΅ΠΎΠ±Ρ…ΠΎΠ΄ΠΈΠΌΠΎ косвСнноС ΠΈΠ·ΠΌΠ΅Ρ€Π΅Π½ΠΈΠ΅ , ΠΊΠΎΡ‚ΠΎΡ€ΠΎΠ΅ ΠΏΡ€ΠΈΠ½Ρ†ΠΈΠΏΠΈΠ°Π»ΡŒΠ½ΠΎ Π½Π΅ отличаСтся ΠΎΡ‚ расчёта — Ρ‚Π°ΠΊΠΆΠ΅ Π½Π΅ΠΎΠ±Ρ…ΠΎΠ΄ΠΈΠΌΡ‹ напряТСния Π½Π° Π½Π°Π³Ρ€ΡƒΠ·ΠΊΠ΅ ΠΏΡ€ΠΈ Π΄Π²ΡƒΡ… Ρ€Π°Π·Π»ΠΈΡ‡Π½Ρ‹Ρ… значСниях Ρ‚ΠΎΠΊΠ°. Однако Π²ΠΎΡΠΏΠΎΠ»ΡŒΠ·ΠΎΠ²Π°Ρ‚ΡŒΡΡ ΡƒΠΏΡ€ΠΎΡ‰Ρ‘Π½Π½ΠΎΠΉ Ρ„ΠΎΡ€ΠΌΡƒΠ»ΠΎΠΉ (2) Π½Π΅ всСгда Π²ΠΎΠ·ΠΌΠΎΠΆΠ½ΠΎ, ΠΏΠΎΡΠΊΠΎΠ»ΡŒΠΊΡƒ Π½Π΅ ΠΊΠ°ΠΆΠ΄Ρ‹ΠΉ Ρ€Π΅Π°Π»ΡŒΠ½Ρ‹ΠΉ Π΄Π²ΡƒΡ…ΠΏΠΎΠ»ΡŽΡΠ½ΠΈΠΊ допускаСт Ρ€Π°Π±ΠΎΡ‚Ρƒ Π² Ρ€Π΅ΠΆΠΈΠΌΠ΅ ΠΊΠΎΡ€ΠΎΡ‚ΠΊΠΎΠ³ΠΎ замыкания.

Иногда примСняСтся ΡΠ»Π΅Π΄ΡƒΡŽΡ‰ΠΈΠΉ простой способ измСрСния, Π½Π΅ Ρ‚Ρ€Π΅Π±ΡƒΡŽΡ‰ΠΈΠΉ вычислСний:

  • Π˜Π·ΠΌΠ΅Ρ€ΡΠ΅Ρ‚ΡΡ напряТСниС холостого Ρ…ΠΎΠ΄Π°
  • Π’ качСствС Π½Π°Π³Ρ€ΡƒΠ·ΠΊΠΈ ΠΏΠΎΠ΄ΠΊΠ»ΡŽΡ‡Π°Π΅Ρ‚ΡΡ ΠΏΠ΅Ρ€Π΅ΠΌΠ΅Π½Π½Ρ‹ΠΉ рСзистор ΠΈ Π΅Π³ΠΎ сопротивлСниС подбираСтся Ρ‚Π°ΠΊΠΈΠΌ ΠΎΠ±Ρ€Π°Π·ΠΎΠΌ, Ρ‡Ρ‚ΠΎΠ±Ρ‹ напряТСниС Π½Π° Π½Ρ‘ΠΌ составило ΠΏΠΎΠ»ΠΎΠ²ΠΈΠ½Ρƒ ΠΎΡ‚ напряТСния холостого Ρ…ΠΎΠ΄Π°.

ПослС описанных ΠΏΡ€ΠΎΡ†Π΅Π΄ΡƒΡ€ сопротивлСниС рСзистора Π½Π°Π³Ρ€ΡƒΠ·ΠΊΠΈ Π½Π΅ΠΎΠ±Ρ…ΠΎΠ΄ΠΈΠΌΠΎ ΠΈΠ·ΠΌΠ΅Ρ€ΠΈΡ‚ΡŒ ΠΎΠΌΠΌΠ΅Ρ‚Ρ€ΠΎΠΌ — ΠΎΠ½ΠΎ Π±ΡƒΠ΄Π΅Ρ‚ Ρ€Π°Π²Π½ΠΎ Π²Π½ΡƒΡ‚Ρ€Π΅Π½Π½Π΅ΠΌΡƒ ΡΠΎΠΏΡ€ΠΎΡ‚ΠΈΠ²Π»Π΅Π½ΠΈΡŽ Π΄Π²ΡƒΡ…ΠΏΠΎΠ»ΡŽΡΠ½ΠΈΠΊΠ°.

Какой Π±Ρ‹ способ измСрСния Π½ΠΈ использовался, слСдуСт ΠΎΠΏΠ°ΡΠ°Ρ‚ΡŒΡΡ ΠΏΠ΅Ρ€Π΅Π³Ρ€ΡƒΠ·ΠΊΠΈ Π΄Π²ΡƒΡ…ΠΏΠΎΠ»ΡŽΡΠ½ΠΈΠΊΠ° Ρ‡Ρ€Π΅Π·ΠΌΠ΅Ρ€Π½Ρ‹ΠΌ Ρ‚ΠΎΠΊΠΎΠΌ, Ρ‚ΠΎ Π΅ΡΡ‚ΡŒ Ρ‚ΠΎΠΊ Π½Π΅ Π΄ΠΎΠ»ΠΆΠ΅Π½ ΠΏΡ€Π΅Π²Ρ‹ΡˆΠ°Ρ‚ΡŒ максимально допустимого значСниях для Π΄Π°Π½Π½ΠΎΠ³ΠΎ Π΄Π²ΡƒΡ…ΠΏΠΎΠ»ΡŽΡΠ½ΠΈΠΊΠ°.

Π Π΅Π°ΠΊΡ‚ΠΈΠ²Π½ΠΎΠ΅ Π²Π½ΡƒΡ‚Ρ€Π΅Π½Π½Π΅Π΅ сопротивлСниС

Если эквивалСнтная схСма Π΄Π²ΡƒΡ…ΠΏΠΎΠ»ΡŽΡΠ½ΠΈΠΊΠ° содСрТит Ρ€Π΅Π°ΠΊΡ‚ΠΈΠ²Π½Ρ‹Π΅ элСмСнты — кондСнсаторы ΠΈ/ΠΈΠ»ΠΈ ΠΊΠ°Ρ‚ΡƒΡˆΠΊΠΈ индуктивности , Ρ‚ΠΎ расчСт Ρ€Π΅Π°ΠΊΡ‚ΠΈΠ²Π½ΠΎΠ³ΠΎ Π²Π½ΡƒΡ‚Ρ€Π΅Π½Π½Π΅Π³ΠΎ сопротивлСния выполняСтся Ρ‚Π°ΠΊΠΆΠ΅, ΠΊΠ°ΠΊ ΠΈ Π°ΠΊΡ‚ΠΈΠ²Π½ΠΎΠ³ΠΎ, Π½ΠΎ вмСсто сопротивлСний рСзисторов бСрутся комплСксныС импСдансы элСмСнтов, входящих Π² схСму, Π° вмСсто напряТСний ΠΈ Ρ‚ΠΎΠΊΠΎΠ² — ΠΈΡ… комплСксныС Π°ΠΌΠΏΠ»ΠΈΡ‚ΡƒΠ΄Ρ‹ , Ρ‚ΠΎ Π΅ΡΡ‚ΡŒ расчСт производится ΠΌΠ΅Ρ‚ΠΎΠ΄ΠΎΠΌ комплСксных Π°ΠΌΠΏΠ»ΠΈΡ‚ΡƒΠ΄ .

Π˜Π·ΠΌΠ΅Ρ€Π΅Π½ΠΈΠ΅ Ρ€Π΅Π°ΠΊΡ‚ΠΈΠ²Π½ΠΎΠ³ΠΎ Π²Π½ΡƒΡ‚Ρ€Π΅Π½Π½Π΅Π³ΠΎ сопротивлСния ΠΈΠΌΠ΅Π΅Ρ‚ Π½Π΅ΠΊΠΎΡ‚ΠΎΡ€Ρ‹Π΅ особСнности, ΠΏΠΎΡΠΊΠΎΠ»ΡŒΠΊΡƒ ΠΎΠ½ΠΎ являСтся комплСкснозначной Ρ„ΡƒΠ½ΠΊΡ†ΠΈΠ΅ΠΉ , Π° Π½Π΅ скалярным Π·Π½Π°Ρ‡Π΅Π½ΠΈΠ΅ΠΌ:

  • МоТно ΠΈΡΠΊΠ°Ρ‚ΡŒ Ρ€Π°Π·Π»ΠΈΡ‡Π½Ρ‹Π΅ ΠΏΠ°Ρ€Π°ΠΌΠ΅Ρ‚Ρ€Ρ‹ комплСксного значСния: ΠΌΠΎΠ΄ΡƒΠ»ΡŒ , Π°Ρ€Π³ΡƒΠΌΠ΅Π½Ρ‚ , Ρ‚ΠΎΠ»ΡŒΠΊΠΎ Π²Π΅Ρ‰Π΅ΡΡ‚Π²Π΅Π½Π½ΡƒΡŽ ΠΈΠ»ΠΈ ΠΌΠ½ΠΈΠΌΡƒΡŽ Ρ‡Π°ΡΡ‚ΡŒ, Π° Ρ‚Π°ΠΊΠΆΠ΅ комплСксноС число ΠΏΠΎΠ»Π½ΠΎΡΡ‚ΡŒΡŽ. БоотвСтствСнно, ΠΌΠ΅Ρ‚ΠΎΠ΄ΠΈΠΊΠ° ΠΈΠ·ΠΌΠ΅Ρ€Π΅Π½ΠΈΠΉ Π±ΡƒΠ΄Π΅Ρ‚ Π·Π°Π²ΠΈΡΠ΅Ρ‚ΡŒ ΠΎΡ‚ Ρ‚ΠΎΠ³ΠΎ, Ρ‡Ρ‚ΠΎ Ρ…ΠΎΡ‚ΠΈΠΌ ΠΏΠΎΠ»ΡƒΡ‡ΠΈΡ‚ΡŒ.
  • Π›ΡŽΠ±ΠΎΠΉ ΠΈΠ· пСрСчислСнных ΠΏΠ°Ρ€Π°ΠΌΠ΅Ρ‚Ρ€ΠΎΠ² зависит ΠΎΡ‚ частоты. ВСорСтичСски, Ρ‡Ρ‚ΠΎΠ±Ρ‹ ΠΏΠΎΠ»ΡƒΡ‡ΠΈΡ‚ΡŒ ΠΏΡƒΡ‚Π΅ΠΌ измСрСния ΠΏΠΎΠ»Π½ΡƒΡŽ ΠΈΠ½Ρ„ΠΎΡ€ΠΌΠ°Ρ†ΠΈΡŽ ΠΎ Ρ€Π΅Π°ΠΊΡ‚ΠΈΠ²Π½ΠΎΠΌ Π²Π½ΡƒΡ‚Ρ€Π΅Π½Π½Π΅ΠΌ сопротивлСнии, Π½Π΅ΠΎΠ±Ρ…ΠΎΠ΄ΠΈΠΌΠΎ ΡΠ½ΡΡ‚ΡŒ Π·Π°Π²ΠΈΡΠΈΠΌΠΎΡΡ‚ΡŒ ΠΎΡ‚ частоты, Ρ‚ΠΎ Π΅ΡΡ‚ΡŒ провСсти измСрСния Π½Π° всСх частотах, ΠΊΠΎΡ‚ΠΎΡ€Ρ‹Π΅ ΠΌΠΎΠΆΠ΅Ρ‚ Π³Π΅Π½Π΅Ρ€ΠΈΡ€ΠΎΠ²Π°Ρ‚ΡŒ источник Π΄Π°Π½Π½ΠΎΠ³ΠΎ Π΄Π²ΡƒΡ…ΠΏΠΎΠ»ΡŽΡΠ½ΠΈΠΊΠ°.

ΠŸΡ€ΠΈΠΌΠ΅Π½Π΅Π½ΠΈΠ΅

Π’ Π±ΠΎΠ»ΡŒΡˆΠΈΠ½ΡΡ‚Π²Π΅ случаСв слСдуСт Π³ΠΎΠ²ΠΎΡ€ΠΈΡ‚ΡŒ Π½Π΅ ΠΎ ΠΏΡ€ΠΈΠΌΠ΅Π½Π΅Π½ΠΈΠΈ Π²Π½ΡƒΡ‚Ρ€Π΅Π½Π½Π΅Π³ΠΎ сопротивлСния, Π° ΠΎΠ± ΡƒΡ‡Π΅Ρ‚Π΅ Π΅Π³ΠΎ Π½Π΅Π³Π°Ρ‚ΠΈΠ²Π½ΠΎΠ³ΠΎ влияния, ΠΏΠΎΡΠΊΠΎΠ»ΡŒΠΊΡƒ Π²Π½ΡƒΡ‚Ρ€Π΅Π½Π½Π΅Π΅ сопротивлСниС являСтся скорСС Π½Π΅Π³Π°Ρ‚ΠΈΠ²Π½Ρ‹ΠΌ эффСктом. Π’Π΅ΠΌ Π½Π΅ ΠΌΠ΅Π½Π΅Π΅, Π² Π½Π΅ΠΊΠΎΡ‚ΠΎΡ€Ρ‹Ρ… систСмах Π½Π°Π»ΠΈΡ‡ΠΈΠ΅ Π²Π½ΡƒΡ‚Ρ€Π΅Π½Π½Π΅Π³ΠΎ сопротивлСния с Π½ΠΎΠΌΠΈΠ½Π°Π»ΡŒΠ½Ρ‹ΠΌ Π·Π½Π°Ρ‡Π΅Π½ΠΈΠ΅ΠΌ являСтся просто Π½Π΅ΠΎΠ±Ρ…ΠΎΠ΄ΠΈΠΌΡ‹ΠΌ.

Π£ΠΏΡ€ΠΎΡ‰Π΅Π½ΠΈΠ΅ эквивалСнтных схСм

ΠŸΡ€Π΅Π΄ΡΡ‚Π°Π²Π»Π΅Π½ΠΈΠ΅ Π΄Π²ΡƒΡ…ΠΏΠΎΠ»ΡŽΡΠ½ΠΈΠΊΠ° ΠΊΠ°ΠΊ ΡΠΎΠ²ΠΎΠΊΡƒΠΏΠ½ΠΎΡΡ‚ΡŒ Π³Π΅Π½Π΅Ρ€Π°Ρ‚ΠΎΡ€Π° напряТСния ΠΈ Π²Π½ΡƒΡ‚Ρ€Π΅Π½Π½Π΅Π³ΠΎ сопротивлСния являСтся Π½Π°ΠΈΠ±ΠΎΠ»Π΅Π΅ простой ΠΈ часто ΠΈΡΠΏΠΎΠ»ΡŒΠ·ΡƒΠ΅ΠΌΠΎΠΉ эквивалСнтной схСмой Π΄Π²ΡƒΡ…ΠΏΠΎΠ»ΡŽΡΠ½ΠΈΠΊΠ°.

БогласованиС источника ΠΈ Π½Π°Π³Ρ€ΡƒΠ·ΠΊΠΈ

БогласованиС источника ΠΈ Π½Π°Π³Ρ€ΡƒΠ·ΠΊΠΈ — это Π²Ρ‹Π±ΠΎΡ€ ΡΠΎΠΎΡ‚Π½ΠΎΡˆΠ΅Π½ΠΈΡ сопротивлСния Π½Π°Π³Ρ€ΡƒΠ·ΠΊΠΈ ΠΈ Π²Π½ΡƒΡ‚Ρ€Π΅Π½Π½Π΅Π³ΠΎ сопротивлСния источника с Ρ†Π΅Π»ΡŒΡŽ достиТСния Π·Π°Π΄Π°Π½Π½Ρ‹Ρ… свойств ΠΏΠΎΠ»ΡƒΡ‡Π΅Π½Π½ΠΎΠΉ систСмы (ΠΊΠ°ΠΊ ΠΏΡ€Π°Π²ΠΈΠ»ΠΎ, ΡΡ‚Π°Ρ€Π°ΡŽΡ‚ΡΡ Π΄ΠΎΡΡ‚ΠΈΡ‡ΡŒ максимального значСния ΠΊΠ°ΠΊΠΎΠ³ΠΎ-Π»ΠΈΠ±ΠΎ ΠΏΠ°Ρ€Π°ΠΌΠ΅Ρ‚Ρ€Π° для Π΄Π°Π½Π½ΠΎΠ³ΠΎ источника). НаиболСС часто ΠΈΡΠΏΠΎΠ»ΡŒΠ·ΡƒΡŽΡ‚ΡΡ ΡΠ»Π΅Π΄ΡƒΡŽΡ‰ΠΈΠ΅ Ρ‚ΠΈΠΏΡ‹ согласования:

БогласованиС ΠΏΠΎ Ρ‚ΠΎΠΊΡƒ ΠΈ мощности слСдуСт ΠΈΡΠΏΠΎΠ»ΡŒΠ·ΠΎΠ²Π°Ρ‚ΡŒ с ΠΎΡΡ‚ΠΎΡ€ΠΎΠΆΠ½ΠΎΡΡ‚ΡŒΡŽ, Ρ‚Π°ΠΊ ΠΊΠ°ΠΊ Π΅ΡΡ‚ΡŒ ΠΎΠΏΠ°ΡΠ½ΠΎΡΡ‚ΡŒ ΠΏΠ΅Ρ€Π΅Π³Ρ€ΡƒΠ·ΠΈΡ‚ΡŒ источник.

ПониТСниС высоких напряТСний

Иногда ΠΊ источнику искусствСнно Π΄ΠΎΠ±Π°Π²Π»ΡΡŽΡ‚ большоС сопротивлСниС (ΠΎΠ½ΠΎ добавляСтся ΠΊ Π²Π½ΡƒΡ‚Ρ€Π΅Π½Π½Π΅ΠΌΡƒ ΡΠΎΠΏΡ€ΠΎΡ‚ΠΈΠ²Π»Π΅Π½ΠΈΡŽ источника) для Ρ‚ΠΎΠ³ΠΎ, Ρ‡Ρ‚ΠΎΠ±Ρ‹ Π·Π½Π°Ρ‡ΠΈΡ‚Π΅Π»ΡŒΠ½ΠΎ ΠΏΠΎΠ½ΠΈΠ·ΠΈΡ‚ΡŒ ΠΏΠΎΠ»ΡƒΡ‡Π°Π΅ΠΌΠΎΠ΅ ΠΎΡ‚ Π½Π΅Π³ΠΎ напряТСниС. Однако Π΄ΠΎΠ±Π°Π²Π»Π΅Π½ΠΈΠ΅ рСзистора Π² качСствС Π΄ΠΎΠΏΠΎΠ»Π½ΠΈΡ‚Π΅Π»ΡŒΠ½ΠΎΠ³ΠΎ сопротивлСния (Ρ‚Π°ΠΊ Π½Π°Π·Ρ‹Π²Π°Π΅ΠΌΡ‹ΠΉ гасящий рСзистор) Π²Π΅Π΄Ρ‘Ρ‚ ΠΊ бСсполСзному Π²Ρ‹Π΄Π΅Π»Π΅Π½ΠΈΡŽ мощности Π½Π° Π½Ρ‘ΠΌ. Π§Ρ‚ΠΎΠ±Ρ‹ Π½Π΅ Ρ€Π°ΡΡ…ΠΎΠ΄ΠΎΠ²Π°Ρ‚ΡŒ ΡΠ½Π΅Ρ€Π³ΠΈΡŽ Π²ΠΏΡƒΡΡ‚ΡƒΡŽ, Π² систСмах ΠΏΠ΅Ρ€Π΅ΠΌΠ΅Π½Π½ΠΎΠ³ΠΎ Ρ‚ΠΎΠΊΠ° ΠΈΡΠΏΠΎΠ»ΡŒΠ·ΡƒΡŽΡ‚ Ρ€Π΅Π°ΠΊΡ‚ΠΈΠ²Π½Ρ‹Π΅ гасящиС импСдансы, Ρ‡Π°Ρ‰Π΅ всСго кондСнсаторы . Π’Π°ΠΊΠΈΠΌ ΠΎΠ±Ρ€Π°Π·ΠΎΠΌ строятся кондСнсаторныС Π±Π»ΠΎΠΊΠΈ питания. Аналогично, ΠΏΡ€ΠΈ ΠΏΠΎΠΌΠΎΡ‰ΠΈ ёмкостного ΠΎΡ‚Π²ΠΎΠ΄Π° ΠΎΡ‚ Π²Ρ‹ΡΠΎΠΊΠΎΠ²ΠΎΠ»ΡŒΡ‚Π½ΠΎΠΉ Π›Π­ΠŸ ΠΌΠΎΠΆΠ½ΠΎ ΠΏΠΎΠ»ΡƒΡ‡ΠΈΡ‚ΡŒ нСбольшиС напряТСния для питания ΠΊΠ°ΠΊΠΈΡ…-Π»ΠΈΠ±ΠΎ Π°Π²Ρ‚ΠΎΠ½ΠΎΠΌΠ½Ρ‹Ρ… устройств.

ΠœΠΈΠ½ΠΈΠΌΠΈΠ·Π°Ρ†ΠΈΡ ΡˆΡƒΠΌΠ°

ΠŸΡ€ΠΈ усилСнии слабых сигналов часто Π²ΠΎΠ·Π½ΠΈΠΊΠ°Π΅Ρ‚ Π·Π°Π΄Π°Ρ‡Π° ΠΌΠΈΠ½ΠΈΠΌΠΈΠ·Π°Ρ†ΠΈΠΈ ΡˆΡƒΠΌΠ°, вносимого усилитСлСм Π² сигнал. Для этого ΠΈΡΠΏΠΎΠ»ΡŒΠ·ΡƒΡŽΡ‚ΡΡ ΡΠΏΠ΅Ρ†ΠΈΠ°Π»ΡŒΠ½Ρ‹Π΅ ΠΌΠ°Π»ΠΎΡˆΡƒΠΌΡΡ‰ΠΈΠ΅ усилитСли , ΠΎΠ΄Π½Π°ΠΊΠΎ ΠΎΠ½ΠΈ спроСктированы Ρ‚Π°ΠΊΠΈΠΌ ΠΎΠ±Ρ€Π°Π·ΠΎΠΌ, Ρ‡Ρ‚ΠΎ наимСньший коэффициСнт ΡˆΡƒΠΌΠ° достигаСтся лишь Π² ΠΎΠΏΡ€Π΅Π΄Π΅Π»Π΅Π½Π½ΠΎΠΌ Π΄ΠΈΠ°ΠΏΠ°Π·ΠΎΠ½Π΅ Π²Ρ‹Ρ…ΠΎΠ΄Π½ΠΎΠ³ΠΎ сопротивлСния источника сигнала. НапримСр, ΠΌΠ°Π»ΠΎΡˆΡƒΠΌΡΡ‰ΠΈΠΉ ΡƒΡΠΈΠ»ΠΈΡ‚Π΅Π»ΡŒ обСспСчиваСт ΠΌΠΈΠ½ΠΈΠΌΠ°Π»ΡŒΠ½Ρ‹ΠΉ ΡˆΡƒΠΌ Ρ‚ΠΎΠ»ΡŒΠΊΠΎ Π² Π΄ΠΈΠ°ΠΏΠ°Π·ΠΎΠ½Π΅ Π²Ρ‹Ρ…ΠΎΠ΄Π½Ρ‹Ρ… сопротивлСний источника ΠΎΡ‚ 1 кОм Π΄ΠΎ 10 кОм; Ссли источник сигнала ΠΎΠ±Π»Π°Π΄Π°Π΅Ρ‚ мСньшим Π²Ρ‹Ρ…ΠΎΠ΄Π½Ρ‹ΠΌ сопротивлСниСм (Π½Π°ΠΏΡ€ΠΈΠΌΠ΅Ρ€, ΠΌΠΈΠΊΡ€ΠΎΡ„ΠΎΠ½ с Π²Ρ‹Ρ…ΠΎΠ΄Π½Ρ‹ΠΌ сопротивлСниСм 30 Ом), Ρ‚ΠΎ слСдуСт ΠΏΡ€ΠΈΠΌΠ΅Π½ΠΈΡ‚ΡŒ ΠΌΠ΅ΠΆΠ΄Ρƒ источником ΠΈ усилитСлСм ΠΏΠΎΠ²Ρ‹ΡˆΠ°ΡŽΡ‰ΠΈΠΉ трансформатор , ΠΊΠΎΡ‚ΠΎΡ€Ρ‹ΠΉ повысит Π²Ρ‹Ρ…ΠΎΠ΄Π½ΠΎΠ΅ сопротивлСниС (Π° Ρ‚Π°ΠΊΠΆΠ΅ напряТСниС сигнала) Π΄ΠΎ Π½Π΅ΠΎΠ±Ρ…ΠΎΠ΄ΠΈΠΌΠΎΠ³ΠΎ значСния.

ΠžΠ³Ρ€Π°Π½ΠΈΡ‡Π΅Π½ΠΈΡ

ΠŸΠΎΠ½ΡΡ‚ΠΈΠ΅ Π²Π½ΡƒΡ‚Ρ€Π΅Π½Π½Π΅Π³ΠΎ сопротивлСния вводится Ρ‡Π΅Ρ€Π΅Π· ΡΠΊΠ²ΠΈΠ²Π°Π»Π΅Π½Ρ‚Π½ΡƒΡŽ схСму, поэтому ΠΈΠΌΠ΅ΡŽΡ‚ силу Ρ‚Π΅ ΠΆΠ΅ ограничСния , Ρ‡Ρ‚ΠΎ ΠΈ для примСнимости эквивалСнтных схСм.

ΠŸΡ€ΠΈΠΌΠ΅Ρ€Ρ‹

ЗначСния Π²Π½ΡƒΡ‚Ρ€Π΅Π½Π½Π΅Π³ΠΎ сопротивлСния ΠΎΡ‚Π½ΠΎΡΠΈΡ‚Π΅Π»ΡŒΠ½Ρ‹: Ρ‚ΠΎ, Ρ‡Ρ‚ΠΎ считаСтся ΠΌΠ°Π»Ρ‹ΠΌ, Π½Π°ΠΏΡ€ΠΈΠΌΠ΅Ρ€, для Π³Π°Π»ΡŒΠ²Π°Π½ΠΈΡ‡Π΅ΡΠΊΠΎΠ³ΠΎ элСмСнта, являСтся ΠΎΡ‡Π΅Π½ΡŒ большим для ΠΌΠΎΡ‰Π½ΠΎΠ³ΠΎ аккумулятора. НиТС ΠΏΡ€ΠΈΠ²Π΅Π΄Π΅Π½Ρ‹ ΠΏΡ€ΠΈΠΌΠ΅Ρ€Ρ‹ Π΄Π²ΡƒΡ…ΠΏΠΎΠ»ΡŽΡΠ½ΠΈΠΊΠΎΠ² ΠΈ значСния ΠΈΡ… Π²Π½ΡƒΡ‚Ρ€Π΅Π½Π½Π΅Π³ΠΎ сопротивлСния r . Π’Ρ€ΠΈΠ²ΠΈΠ°Π»ΡŒΠ½Ρ‹Π΅ случаи Π΄Π²ΡƒΡ…ΠΏΠΎΠ»ΡŽΡΠ½ΠΈΠΊΠΎΠ² Π±Π΅Π· источников ΠΎΠ³ΠΎΠ²ΠΎΡ€Π΅Π½Ρ‹ особо.

МалоС Π²Π½ΡƒΡ‚Ρ€Π΅Π½Π½Π΅Π΅ сопротивлСниС

Π‘ΠΎΠ»ΡŒΡˆΠΎΠ΅ Π²Π½ΡƒΡ‚Ρ€Π΅Π½Π½Π΅Π΅ сопротивлСниС

ΠžΡ‚Ρ€ΠΈΡ†Π°Ρ‚Π΅Π»ΡŒΠ½ΠΎΠ΅ Π²Π½ΡƒΡ‚Ρ€Π΅Π½Π½Π΅Π΅ сопротивлСниС

Π‘ΡƒΡ‰Π΅ΡΡ‚Π²ΡƒΡŽΡ‚ Π΄Π²ΡƒΡ…ΠΏΠΎΠ»ΡŽΡΠ½ΠΈΠΊΠΈ, Π²Π½ΡƒΡ‚Ρ€Π΅Π½Π½Π΅Π΅ сопротивлСниС ΠΊΠΎΡ‚ΠΎΡ€Ρ‹Ρ… ΠΈΠΌΠ΅Π΅Ρ‚ ΠΎΡ‚Ρ€ΠΈΡ†Π°Ρ‚Π΅Π»ΡŒΠ½ΠΎΠ΅ Π·Π½Π°Ρ‡Π΅Π½ΠΈΠ΅. Π’ ΠΎΠ±Ρ‹Ρ‡Π½ΠΎΠΌ Π°ΠΊΡ‚ΠΈΠ²Π½ΠΎΠΌ сопротивлСнии происходит диссипация энСргии, Π² Ρ€Π΅Π°ΠΊΡ‚ΠΈΠ²Π½ΠΎΠΌ сопротивлСнии энСргия запасаСтся, Π° Π·Π°Ρ‚Π΅ΠΌ выдСляСтся ΠΎΠ±Ρ€Π°Ρ‚Π½ΠΎ Π² источник. ΠžΡΠΎΠ±Π΅Π½Π½ΠΎΡΡ‚ΡŒ ΠΎΡ‚Ρ€ΠΈΡ†Π°Ρ‚Π΅Π»ΡŒΠ½ΠΎΠ³ΠΎ сопротивлСния Π² Ρ‚ΠΎΠΌ, Ρ‡Ρ‚ΠΎ ΠΎΠ½ΠΎ само являСтся источником энСргии. ΠŸΠΎΡΡ‚ΠΎΠΌΡƒ ΠΎΡ‚Ρ€ΠΈΡ†Π°Ρ‚Π΅Π»ΡŒΠ½ΠΎΠ΅ сопротивлСниС Π² чистом Π²ΠΈΠ΄Π΅ Π½Π΅ встрСчаСтся, ΠΎΠ½ΠΎ ΠΌΠΎΠΆΠ΅Ρ‚ Π±Ρ‹Ρ‚ΡŒ Ρ‚ΠΎΠ»ΡŒΠΊΠΎ ΠΈΠΌΠΈΡ‚ΠΈΡ€ΠΎΠ²Π°Π½ΠΎ элСктронной схСмой, которая ΠΎΠ±ΡΠ·Π°Ρ‚Π΅Π»ΡŒΠ½ΠΎ содСрТит источник энСргии. ΠžΡ‚Ρ€ΠΈΡ†Π°Ρ‚Π΅Π»ΡŒΠ½ΠΎΠ΅ Π²Π½ΡƒΡ‚Ρ€Π΅Π½Π½Π΅Π΅ сопротивлСниС ΠΌΠΎΠΆΠ΅Ρ‚ Π±Ρ‹Ρ‚ΡŒ ΠΏΠΎΠ»ΡƒΡ‡Π΅Π½ΠΎ Π² схСмах ΠΏΡƒΡ‚Ρ‘ΠΌ использования:

  • элСмСнтов с ΠΎΡ‚Ρ€ΠΈΡ†Π°Ρ‚Π΅Π»ΡŒΠ½Ρ‹ΠΌ Π΄ΠΈΡ„Ρ„Π΅Ρ€Π΅Π½Ρ†ΠΈΠ°Π»ΡŒΠ½Ρ‹ΠΌ сопротивлСниСм , Π½Π°ΠΏΡ€ΠΈΠΌΠ΅Ρ€, Ρ‚ΡƒΠ½Π½Π΅Π»ΡŒΠ½Ρ‹Ρ… Π΄ΠΈΠΎΠ΄ΠΎΠ²

БистСмы с ΠΎΡ‚Ρ€ΠΈΡ†Π°Ρ‚Π΅Π»ΡŒΠ½Ρ‹ΠΌ сопротивлСниСм ΠΏΠΎΡ‚Π΅Π½Ρ†ΠΈΠ°Π»ΡŒΠ½ΠΎ нСустойчивы ΠΈ поэтому ΠΌΠΎΠ³ΡƒΡ‚ Π±Ρ‹Ρ‚ΡŒ ΠΈΡΠΏΠΎΠ»ΡŒΠ·ΠΎΠ²Π°Π½Ρ‹ для построСния Π°Π²Ρ‚ΠΎΠ³Π΅Π½Π΅Ρ€Π°Ρ‚ΠΎΡ€ΠΎΠ² .

Π‘ΠΌ. Ρ‚Π°ΠΊΠΆΠ΅

Бсылки

Π›ΠΈΡ‚Π΅Ρ€Π°Ρ‚ΡƒΡ€Π°

  • Π—Π΅Ρ€Π½ΠΎΠ² Н. Π’., ΠšΠ°Ρ€ΠΏΠΎΠ² Π’.Π“. ВСория радиотСхничСских Ρ†Π΅ΠΏΠ΅ΠΉ. — М. — Π›.: ЭнСргия, 1965. — 892 с.
  • ДТонс М. Π₯. Π­Π»Π΅ΠΊΡ‚Ρ€ΠΎΠ½ΠΈΠΊΠ° — практичСский курс. — М.: ВСхносфСра, 2006. — 512 с.

Как Π½Π°ΠΉΡ‚ΠΈ ΠΎΠ±Ρ‰Π΅Π΅ Π²Π½ΡƒΡ‚Ρ€Π΅Π½Π½Π΅Π΅ сопротивлСниС

Π’Π΅Π»ΠΈΡ‡ΠΈΠ½Π°, Ρ…Π°Ρ€Π°ΠΊΡ‚Π΅Ρ€ΠΈΠ·ΡƒΡŽΡ‰Π°Ρ количСство энСргСтичСских ΠΏΠΎΡ‚Π΅Ρ€ΡŒ, Π²ΠΎΠ·Π½ΠΈΠΊΠ°ΡŽΡ‰ΠΈΡ… ΠΏΡ€ΠΈ ΠΏΡ€ΠΎΡ‚Π΅ΠΊΠ°Π½ΠΈΠΈ Ρ‚ΠΎΠΊΠ° Ρ‡Π΅Ρ€Π΅Π· Π΅Π³ΠΎ источник, опрСдСляСтся ΠΊΠ°ΠΊ Π²Π½ΡƒΡ‚Ρ€Π΅Π½Π½Π΅Π΅ сопротивлСниС источника Ρ‚ΠΎΠΊΠ°. Как ΠΈ ΠΎΠ±Ρ‹Ρ‡Π½ΠΎΠ΅ сопротивлСниС, ΠΈΠΌΠ΅Π΅Ρ‚ Π΅Π΄ΠΈΠ½ΠΈΡ†Ρƒ измСрСния, Ρ€Π°Π²Π½ΡƒΡŽ 1 Ом. Π’ΠΎΠΊ, двигаясь Ρ‡Π΅Ρ€Π΅Π· источник, тСряСт Ρ‡Π°ΡΡ‚ΡŒ своСй энСргии, которая ΠΏΠ΅Ρ€Π΅Ρ…ΠΎΠ΄ΠΈΡ‚ Π² Ρ‚Π΅ΠΏΠ»ΠΎ, Ρ‚ΠΎΡ‡Π½ΠΎ Ρ‚Π°ΠΊ ΠΆΠ΅, ΠΊΠ°ΠΊ Π½Π° любом Π½Π°Π³Ρ€ΡƒΠ·ΠΎΡ‡Π½ΠΎΠΌ сопротивлСнии. Π­Ρ‚ΠΎ Π·Π½Π°Ρ‡ΠΈΡ‚, Ρ‡Ρ‚ΠΎ Π²Π΅Π»ΠΈΡ‡ΠΈΠ½Π° напряТСния Π½Π° Π²Ρ‹Π²ΠΎΠ΄Π°Ρ… источника зависит ΠΎΡ‚ Π²Π΅Π»ΠΈΡ‡ΠΈΠ½Ρ‹ Ρ‚ΠΎΠΊΠ°, Π° Π½Π΅ ΠΎΡ‚ Π­Π”Π‘.

Если Ρ€Π°ΡΡΠΌΠΎΡ‚Ρ€Π΅Ρ‚ΡŒ Π·Π°ΠΌΠΊΠ½ΡƒΡ‚ΡƒΡŽ ΡΠ»Π΅ΠΊΡ‚Ρ€ΠΈΡ‡Π΅ΡΠΊΡƒΡŽ Ρ†Π΅ΠΏΡŒ, Π² ΠΊΠΎΡ‚ΠΎΡ€ΡƒΡŽ Π²ΠΊΠ»ΡŽΡ‡Ρ‘Π½ источник Ρ‚ΠΎΠΊΠ° (Π±Π°Ρ‚Π°Ρ€Π΅ΠΉΠΊΠ°, аккумулятор ΠΈΠ»ΠΈ Π³Π΅Π½Π΅Ρ€Π°Ρ‚ΠΎΡ€), ΠΈ Π½Π°Π³Ρ€ΡƒΠ·ΠΊΡƒ R, Ρ‚ΠΎ Ρ‚ΠΎΠΊ Ρ‚Π΅Ρ‡Ρ‘Ρ‚ ΠΈ Π²Π½ΡƒΡ‚Ρ€ΠΈ источника. Π’Π½ΡƒΡ‚Ρ€Π΅Π½Π½Π΅Π΅ сопротивлСниС источника, ΠΎΠ±ΠΎΠ·Π½Π°Ρ‡Π°Π΅ΠΌΠΎΠ΅ Π±ΡƒΠΊΠ²ΠΎΠΉ r, Π΅ΠΌΡƒ прСпятствуСт.

Π£ Π³Π΅Π½Π΅Ρ€Π°Ρ‚ΠΎΡ€Π° r – это Π²Π½ΡƒΡ‚Ρ€Π΅Π½Π½Π΅Π΅ сопротивлСниС ΠΎΠ±ΠΌΠΎΡ‚ΠΎΠΊ статора, Ρƒ аккумулятора – сопротивлСниС элСктролита.

Π˜Π·ΠΌΠ΅Ρ€Π΅Π½ΠΈΠ΅ сопротивлСния ΠΏΠ΅Ρ‚Π»ΠΈ Ρ„Π°Π·Π°-Π½ΡƒΠ»ΡŒ

ΠŸΠ΅Ρ‚Π»Ρ Β«Ρ„Π°Π·Π° – Π½ΡƒΠ»ΡŒΒ» – это элСктричСская Ρ†Π΅ΠΏΡŒ ΠΏΠ΅Ρ€Π΅ΠΌΠ΅Π½Π½ΠΎΠ³ΠΎ Ρ‚ΠΎΠΊΠ°, которая ΠΌΠΎΠΆΠ΅Ρ‚ Π²ΠΎΠ·Π½ΠΈΠΊΠ½ΡƒΡ‚ΡŒ Π² Ρ€Π΅Π·ΡƒΠ»ΡŒΡ‚Π°Ρ‚Π΅ ΠΊΠΎΡ€ΠΎΡ‚ΠΊΠΎΠ³ΠΎ замыкания ΠΌΠ΅ΠΆΠ΄Ρƒ ΠΏΡ€ΠΎΠ²ΠΎΠ΄Π°ΠΌΠΈ: Β«Ρ„Π°Π·Π°Β» ΠΈ «ноль» ΠΈΠ»ΠΈ Β«Ρ„Π°Π·Π°Β» ΠΈ Β«Ρ„Π°Π·Π°Β». Π Π°Π·Ρ€ΡƒΡˆΠ΅Π½ΠΈΠ΅ изоляции, мСханичСскиС поврСТдСния ΠΈΠ»ΠΈ случайноС соСдинСниС ΠΎΠ³ΠΎΠ»Ρ‘Π½Π½Ρ‹Ρ… участков кабСля ΠΌΠ΅ΠΆΠ΄Ρƒ собой ΠΌΠΎΠ³ΡƒΡ‚ ΡΡ‚Π°Ρ‚ΡŒ этому ΠΏΡ€ΠΈΡ‡ΠΈΠ½ΠΎΠΉ. Π’ установках с Π³Π»ΡƒΡ…ΠΎ Π·Π°Π·Π΅ΠΌΠ»Ρ‘Π½Π½ΠΎΠΉ Π½Π΅ΠΉΡ‚Ρ€Π°Π»ΡŒΡŽ Π½ΡƒΠ»Π΅Π²ΠΎΠΉ ΠΏΡ€ΠΎΠ²ΠΎΠ΄Π½ΠΈΠΊ физичСски связан с Π½Π΅ΠΉΡ‚Ρ€Π°Π»ΡŒΡŽ трансформатора, ΠΎΠ½Π° ΠΏΠΎΠ΄ΠΊΠ»ΡŽΡ‡Π΅Π½Π° ΠΊ ΠΊΠΎΠ½Ρ‚ΡƒΡ€Ρƒ зазСмлСния. ΠŸΡ€ΠΈ Π·Π°ΠΌΡ‹ΠΊΠ°Π½ΠΈΠΈ Π½Π° корпус ΠΈΠ»ΠΈ соСдинСнии Ρ„Π°Π· ΠΌΠ΅ΠΆΠ΄Ρƒ собой образуСтся Ρ†Π΅ΠΏΡŒ (пСтля).

Главная Π·Π°Π΄Π°Ρ‡Π° ΠΏΡ€ΠΎΠ²ΠΎΠ΄ΠΈΠΌΡ‹Ρ… ΠΈΠ·ΠΌΠ΅Ρ€Π΅Π½ΠΈΠΉ – ΡƒΠ·Π½Π°Π²Π°Ρ‚ΡŒ, ΠΊΠ°ΠΊΠΈΠΌ Π±ΡƒΠ΄Π΅Ρ‚ Π²Π΅Π»ΠΈΡ‡ΠΈΠ½Π° Ρ‚ΠΎΠΊΠ° Ρ‡Π΅Ρ€Π΅Π· ΠΏΠ΅Ρ‚Π»ΡŽ ΠΏΡ€ΠΈ ΠšΠ—. Π­Ρ‚ΠΎ ΠΎΠ±ΡΠ·Π°Ρ‚Π΅Π»ΡŒΠ½ΠΎ для расчёта ΠΈ ΠΏΠΎΠ΄Π±ΠΎΡ€Π° Π·Π°Ρ‰ΠΈΡ‚Π½ΠΎΠ³ΠΎ оборудования. Π₯ΠΎΡ€ΠΎΡˆΠΈΠΌ Ρ€Π΅Π·ΡƒΠ»ΡŒΡ‚Π°Ρ‚ΠΎΠΌ Π±ΡƒΠ΄Π΅Ρ‚ малСнькоС сопротивлСниС ΠΏΠ΅Ρ‚Π»ΠΈ, Ρ‚ΠΎΠ³Π΄Π° Ρ‚ΠΎΠΊ IΠΊ. Π·. Π±ΡƒΠ΄Π΅Ρ‚ наибольшим. ΠžΡ‚ Π΅Π³ΠΎ Π²Π΅Π»ΠΈΡ‡ΠΈΠ½Ρ‹ зависит, ΠΊΠ°ΠΊ быстро сработаСт Π·Π°Ρ‰ΠΈΡ‚Π½Ρ‹ΠΉ автоматичСский Π²Ρ‹ΠΊΠ»ΡŽΡ‡Π°Ρ‚Π΅Π»ΡŒ.

Π§Π΅ΠΌ мСньшС Π²Ρ€Π΅ΠΌΠ΅Π½ΠΈ Π±ΡƒΠ΄Π΅Ρ‚ Π·Π°Ρ‚Ρ€Π°Ρ‡Π΅Π½ΠΎ Π½Π° ΠΎΡ‚ΠΊΠ»ΡŽΡ‡Π΅Π½ΠΈΠ΅ ΠΏΠΎΠ²Ρ€Π΅ΠΆΠ΄Ρ‘Π½Π½ΠΎΠΉ ΠΈΠ»ΠΈ Π·Π°ΠΊΠΎΡ€ΠΎΡ‡Π΅Π½Π½ΠΎΠΉ Ρ†Π΅ΠΏΠΈ, Ρ‚Π΅ΠΌ большС шансов ΠΏΡ€Π΅Π΄ΠΎΡ‚Π²Ρ€Π°Ρ‚ΠΈΡ‚ΡŒ ΠΏΠΎΠΆΠ°Ρ€ ΠΎΡ‚ возгорания кабСльной сСти. ΠŸΡ€ΠΈ ΠΏΠΎΠΏΠ°Π΄Π°Π½ΠΈΠΈ Ρ‡Π΅Π»ΠΎΠ²Π΅ΠΊΠ° ΠΏΠΎΠ΄ ΡƒΠ΄Π°Ρ€ элСктричСского Ρ‚ΠΎΠΊΠ° Π² Ρ€Π΅Π·ΡƒΠ»ΡŒΡ‚Π°Ρ‚Π΅ прикосновСния ΠΈΠ»ΠΈ ΠΊΠΎΡ€ΠΎΡ‚ΠΊΠΎΠ³ΠΎ замыкания автоматичСскоС снятиС напряТСния спасёт Π΅ΠΌΡƒ Тизнь.

На прСдприятиях Π΅ΠΆΠ΅Π³ΠΎΠ΄Π½ΠΎ проводится комплСкс ΠΈΠ·ΠΌΠ΅Ρ€Π΅Π½ΠΈΠΉ Π·Π°Ρ‰ΠΈΡ‚Π½ΠΎΠ³ΠΎ зазСмлСния ΠΈ сопротивлСния ΠΏΠ΅Ρ‚Π»ΠΈ Ρ„Π°Π·Π° – ноль. ΠŸΡ€ΠΈ Π½Π΅ΡƒΠ΄ΠΎΠ²Π»Π΅Ρ‚Π²ΠΎΡ€ΠΈΡ‚Π΅Π»ΡŒΠ½Ρ‹Ρ… Ρ€Π΅Π·ΡƒΠ»ΡŒΡ‚Π°Ρ‚Π°Ρ… проводится ряд мСроприятий:

  • Π·Π°ΠΌΠ΅Π½ΡΡŽΡ‚ΡΡ участки ΠΏΡ€ΠΎΠ²ΠΎΠ΄Π°, Π½Π΅ ΠΎΡ‚Π²Π΅Ρ‡Π°ΡŽΡ‰ΠΈΠ΅ трСбованиям ΠΏΠΎ Π΄ΠΈΠ°ΠΌΠ΅Ρ‚Ρ€Ρƒ сСчСния;
  • ΠΏΠ΅Ρ€Π΅ΠΊΡ€ΡƒΡ‡ΠΈΠ²Π°ΡŽΡ‚ΡΡ Π±ΠΎΠ»Ρ‚ΠΎΠ²Ρ‹Π΅ соСдинСния с ΠΎΠ±ΡΠ·Π°Ρ‚Π΅Π»ΡŒΠ½ΠΎΠΉ установкой Π²Ρ€Π΅Π·Π½Ρ‹Ρ… шайб;
  • Π²ΡΠΊΡ€Ρ‹Π²Π°ΡŽΡ‚ΡΡ ΠΊΠΎΠ½Ρ‚ΡƒΡ€Ρ‹ Π·Π°Ρ‰ΠΈΡ‚Π½Ρ‹Ρ… Π·Π°Π·Π΅ΠΌΠ»Π΅Π½ΠΈΠΉ ΠΈ ΠΎΡΠΌΠ°Ρ‚Ρ€ΠΈΠ²Π°ΡŽΡ‚ΡΡ Π½Π° ΠΏΡ€Π΅Π΄ΠΌΠ΅Ρ‚ цСлостности сварных соСдинСний ΠΈ состояния элСмСнтов зазСмлСния;
  • ΠΏΡ€ΠΈ нСобходимости Π² ΠΊΠΎΠ½Ρ‚ΡƒΡ€ Π·Π°Ρ‰ΠΈΡ‚Π½ΠΎΠ³ΠΎ зазСмлСния Π΄ΠΎΠ±Π°Π²Π»ΡΡŽΡ‚ΡΡ Π΄ΠΎΠΏΠΎΠ»Π½ΠΈΡ‚Π΅Π»ΡŒΠ½Ρ‹Π΅ элСмСнты;
  • ΠΈΡΠΊΠ»ΡŽΡ‡Π°Π΅Ρ‚ΡΡ ΠΏΠΎΡΠ»Π΅Π΄ΠΎΠ²Π°Ρ‚Π΅Π»ΡŒΠ½ΠΎΠ΅ ΠΏΠΎΠ΄ΠΊΠ»ΡŽΡ‡Π΅Π½ΠΈΠ΅ корпусов устройств ΠΊ ΠΎΠ±Ρ‰Π΅ΠΉ шинС зазСмлСния.

ПослС выполнСния комплСкса мСроприятий измСрСния проводятся ΠΏΠΎΠ²Ρ‚ΠΎΡ€Π½ΠΎ.

НахоТдСниС Π²Π½ΡƒΡ‚Ρ€Π΅Π½Π½Π΅Π³ΠΎ сопротивлСния

Π•Π³ΠΎ ΠΌΠΎΠΆΠ½ΠΎ Π½Π°Ρ…ΠΎΠ΄ΠΈΡ‚ΡŒ двумя путями: Ρ€Π°ΡΡΡ‡ΠΈΡ‚Π°Ρ‚ΡŒ ΠΈΠ»ΠΈ ΠΈΠ·ΠΌΠ΅Ρ€ΠΈΡ‚ΡŒ. ΠŸΠ΅Ρ€Π²Ρ‹ΠΌ ΠΏΡƒΡ‚Ρ‘ΠΌ ΠΈΠ΄ΡƒΡ‚ ΠΏΡ€ΠΈ Ρ€Π°Π±ΠΎΡ‚Π΅ с элСктричСскими схСмами, Π²Ρ‚ΠΎΡ€ΠΎΠΉ – Π²Ρ‹Π±ΠΈΡ€Π°ΡŽΡ‚, занимаясь с Ρ€Π΅Π°Π»ΡŒΠ½Ρ‹ΠΌΠΈ устройствами.

ΠŸΡ€ΠΎΡΡ‚ΠΎΠΉ расчёт производится с использованиСм Ρ„ΠΎΡ€ΠΌΡƒΠ»Ρ‹ Π—Π°ΠΊΠΎΠ½Π° Ома для участка ΠΏΠΎΠ»Π½ΠΎΠΉ Ρ†Π΅ΠΏΠΈ:

Π§Ρ‚ΠΎΠ±Ρ‹ ΡƒΠ·Π½Π°Ρ‚ΡŒ силу Ρ‚ΠΎΠΊΠ°, Π½ΡƒΠΆΠ½ΠΎ напряТСниС Π­Π”Π‘ Π΄Π΅Π»ΠΈΡ‚ΡŒ Π½Π° сумму сопротивлСний.

Π’Ρ‹Ρ€Π°Π·ΠΈΠ² ΠΎΡ‚ΡΡŽΠ΄Π° r, ΠΏΠΎΠ»ΡƒΡ‡Π°ΡŽΡ‚ Ρ„ΠΎΡ€ΠΌΡƒΠ»Ρƒ для Π΅Π³ΠΎ вычислСния:

Π³Π΄Π΅:

  • r – Π²Π½ΡƒΡ‚Ρ€Π΅Π½Π½Π΅Π΅ сопротивлСниС источника;
  • Ξ΅ – Π­Π”Π‘ источника;
  • I – сила Ρ‚ΠΎΠΊΠ° Π² ΠΏΠΎΠ»Π½ΠΎΠΉ Ρ†Π΅ΠΏΠΈ;
  • R – сопротивлСниС Π² ΠΏΠΎΠ»Π½ΠΎΠΉ Ρ†Π΅ΠΏΠΈ.

КомплСкс ΠΈΠ·ΠΌΠ΅Ρ€Π΅Π½ΠΈΠΉ этого ΠΏΠ°Ρ€Π°ΠΌΠ΅Ρ‚Ρ€Π° Ρƒ настоящСго устройства Π½Π΅ ΠΏΠΎΠ΄Ρ€Π°Π·ΡƒΠΌΠ΅Π²Π°Π΅Ρ‚ нСпосрСдствСнных Π·Π°ΠΌΠ΅Ρ€ΠΎΠ². Π’Π΅ΡΡ‚ΠΈΡ€ΡƒΡŽΡ‚ΡΡ напряТСния Π½Π° Π½Π°Π³Ρ€ΡƒΠ·ΠΎΡ‡Π½ΠΎΠΌ сопротивлСнии Π² Π΄Π²ΡƒΡ… Ρ€Π΅ΠΆΠΈΠΌΠ°Ρ… Ρ‚ΠΎΠΊΠ°: холостом ΠΈ ΠšΠ—.

Π’Π°ΠΊ ΠΊΠ°ΠΊ Π½Π΅ любой источник ΠΌΠΎΠΆΠ΅Ρ‚ Π²Ρ‹Π΄Π΅Ρ€ΠΆΠ°Ρ‚ΡŒ Π΄Π°ΠΆΠ΅ ΠΊΡ€Π°Ρ‚ΠΊΠΎΠ²Ρ€Π΅ΠΌΠ΅Π½Π½Ρ‹ΠΉ Ρ€Π΅ΠΆΠΈΠΌ замыкания, бСрётся ΠΌΠ΅Ρ‚ΠΎΠ΄ измСрСния Π±Π΅Π· вычислСний.

Π’ схСму Π²ΠΊΠ»ΡŽΡ‡Π°Π΅Ρ‚ΡΡ внСшнСС сопротивлСниС Π½Π°Π³Ρ€ΡƒΠ·ΠΊΠΈ Π² Π²ΠΈΠ΄Π΅ подстроСчного рСзистора RΠ½. ВыставляСтся Ρ‚Π°ΠΊΠΎΠ΅ Π·Π½Π°Ρ‡Π΅Π½ΠΈΠ΅, ΠΏΡ€ΠΈ ΠΊΠΎΡ‚ΠΎΡ€ΠΎΠΌ ΠΏΠ°Π΄Π΅Π½ΠΈΠ΅ напряТСния Π½Π° рСзисторС Ρ€Π°Π²Π½ΡΠ»ΠΎΡΡŒ Π±Ρ‹ 1/2 U холостого Ρ…ΠΎΠ΄Π°. Π’ΠΎΠ³Π΄Π° ΠΈΠ·ΠΌΠ΅Ρ€Π΅Π½Π½ΠΎΠ΅ ΠΎΠΌΠΌΠ΅Ρ‚Ρ€ΠΎΠΌ RΠ½ Π±ΡƒΠ΄Π΅Ρ‚ ΡΠΎΠΎΡ‚Π²Π΅Ρ‚ΡΡ‚Π²ΠΎΠ²Π°Ρ‚ΡŒ Π²Π½ΡƒΡ‚Ρ€Π΅Π½Π½Π΅ΠΌΡƒ ΡΠΎΠΏΡ€ΠΎΡ‚ΠΈΠ²Π»Π΅Π½ΠΈΡŽ источника.

МалоС Π²Π½ΡƒΡ‚Ρ€Π΅Π½Π½Π΅Π΅ сопротивлСниС

Малой Π²Π΅Π»ΠΈΡ‡ΠΈΠ½Ρ‹ Π²Π½ΡƒΡ‚Ρ€Π΅Π½Π½Π΅Π³ΠΎ сопротивлСния Π΄ΠΎΠ±ΠΈΠ²Π°ΡŽΡ‚ΡΡ ΠΏΡ€ΠΈΠΌΠ΅Π½Π΅Π½ΠΈΠ΅ΠΌ ΠΎΠ±Ρ€Π°Ρ‚Π½ΠΎΠΉ связи Π² схСмах, ΠΊΡƒΠ΄Π° Π²ΠΊΠ»ΡŽΡ‡Ρ‘Π½ Π΄Π²ΡƒΡ…ΠΏΠΎΠ»ΡŽΡΠ½ΠΈΠΊ. Π’ стабилизаторах напряТСния r достигаСт Π·Π½Π°Ρ‡Π΅Π½ΠΈΠΉ ΠΌΠ΅Π½Π΅Π΅ 9*10-4 Ом. ΠΠ²Ρ‚ΠΎΠΌΠΎΠ±ΠΈΠ»ΡŒΠ½Π°Ρ ΠΠšΠ‘ 6Π‘Π’-60 ΠΎΠ±Π»Π°Π΄Π°Π΅Ρ‚ сопротивлСниСм ΠΎΠΊΠΎΠ»ΠΎ 0,01 Ом. Если произвСсти измСрСния ΠΏΠ΅Ρ‚Π»ΠΈ Ρ„Π°Π·Π°-ноль Π±Ρ‹Ρ‚ΠΎΠ²ΠΎΠΉ сСти, Ρ‚ΠΎ Π½ΠΎΡ€ΠΌΠ° значСния Π»Π΅ΠΆΠΈΡ‚ Π² ΠΏΡ€Π΅Π΄Π΅Π»Π°Ρ… 0,05-1 Ом.

Π Π΅Π°ΠΊΡ‚ΠΈΠ²Π½ΠΎΠ΅ Π²Π½ΡƒΡ‚Ρ€Π΅Π½Π½Π΅Π΅ сопротивлСниС

ΠšΡ€ΠΎΠΌΠ΅ Π³Π°Π»ΡŒΠ²Π°Π½ΠΈΡ‡Π΅ΡΠΊΠΈΡ… ΠΈ элСктролитичСских Π΄Π²ΡƒΡ…ΠΏΠΎΠ»ΡŽΡΠ½ΠΈΠΊΠΎΠ², ΡΡƒΡ‰Π΅ΡΡ‚Π²ΡƒΡŽΡ‚ источники питания, схСмы ΠΊΠΎΡ‚ΠΎΡ€Ρ‹Ρ… Π²ΠΊΠ»ΡŽΡ‡Π°ΡŽΡ‚ Π² сСбя Ρ€Π΅Π°ΠΊΡ‚ΠΈΠ²Π½Ρ‹Π΅ элСмСнты. ΠŸΡ€ΠΈ ΠΎΠΏΡ€Π΅Π΄Π΅Π»Π΅Π½ΠΈΠΈ ΠΈΡ… Π²Π½ΡƒΡ‚Ρ€Π΅Π½Π½Π΅Π³ΠΎ сопротивлСния ΠΈΡΠΏΠΎΠ»ΡŒΠ·ΡƒΡŽΡ‚ ΠΌΠ΅Ρ‚ΠΎΠ΄ комплСксных Π°ΠΌΠΏΠ»ΠΈΡ‚ΡƒΠ΄. Он ΠΏΠΎΠ΄Ρ€Π°Π·ΡƒΠΌΠ΅Π²Π°Π΅Ρ‚ ΠΈΡΠΏΠΎΠ»ΡŒΠ·ΠΎΠ²Π°Ρ‚ΡŒ ΠΏΡ€ΠΈ расчётах комплСксныС сопротивлСния элСмСнтов, Π²ΠΊΠ»ΡŽΡ‡Ρ‘Π½Π½Ρ‹Ρ… Π² схСму. Π’Π΅Π»ΠΈΡ‡ΠΈΠ½Ρ‹ Ρ‚ΠΎΠΊΠΎΠ² ΠΈ напряТСний Π·Π°ΠΌΠ΅Π½ΡΡŽΡ‚ΡΡ значСниями ΠΈΡ… комплСксных Π°ΠΌΠΏΠ»ΠΈΡ‚ΡƒΠ΄. Π‘Π°ΠΌ Π°Π»Π³ΠΎΡ€ΠΈΡ‚ΠΌ вычислСния Ρ‚Π°ΠΊΠΎΠΉ ΠΆΠ΅, ΠΊΠ°ΠΊ ΠΏΡ€ΠΈ расчётС Π°ΠΊΡ‚ΠΈΠ²Π½ΠΎΠ³ΠΎ сопротивлСния.

ΠŸΡ€ΠΎΡ†Π΅ΡΡ ΠΈΠ·ΠΌΠ΅Ρ€Π΅Π½ΠΈΠΉ r-Ρ€Π΅Π°ΠΊΡ‚ΠΈΠ²Π½ΠΎΠ³ΠΎ Π½Π΅ΠΌΠ½ΠΎΠ³ΠΎ отличаСтся ΠΎΡ‚ измСрСния Π°ΠΊΡ‚ΠΈΠ²Π½ΠΎΠΉ ΡΠΎΡΡ‚Π°Π²Π»ΡΡŽΡ‰Π΅ΠΉ сопротивлСния. ΠœΠ΅Ρ‚ΠΎΠ΄Ρ‹ зависят ΠΎΡ‚ Ρ‚ΠΎΠ³ΠΎ, ΠΊΠ°ΠΊΠΈΠ΅ ΠΏΠ°Ρ€Π°ΠΌΠ΅Ρ‚Ρ€Ρ‹ этой комплСксной Ρ„ΡƒΠ½ΠΊΡ†ΠΈΠΈ Π½ΡƒΠΆΠ½ΠΎ ΡƒΠ·Π½Π°Ρ‚ΡŒ: ΠΎΡ‚Π΄Π΅Π»ΡŒΠ½Ρ‹Π΅ ΡΠΎΡΡ‚Π°Π²Π»ΡΡŽΡ‰ΠΈΠ΅ ΠΈΠ»ΠΈ комплСксноС число.

На эти ΠΏΠ°Ρ€Π°ΠΌΠ΅Ρ‚Ρ€Ρ‹ влияСт частота, поэтому, Ρ‡Ρ‚ΠΎΠ±Ρ‹ ΠΏΡ€ΠΈ тСстировании Π΄ΠΎΠ±ΠΈΡ‚ΡŒΡΡ ΠΈΠ½Ρ„ΠΎΡ€ΠΌΠ°Ρ†ΠΈΠΈ ΠΎ Π²Π½ΡƒΡ‚Ρ€Π΅Π½Π½Π΅ΠΌ Ρ€Π΅Π°ΠΊΡ‚ΠΈΠ²Π½ΠΎΠΌ Π·Π½Π°Ρ‡Π΅Π½ΠΈΠΈ r, Π½ΡƒΠΆΠ½ΠΎ ΡƒΠ±Ρ€Π°Ρ‚ΡŒ Ρ‡Π°ΡΡ‚ΠΎΡ‚Π½ΡƒΡŽ Π·Π°Π²ΠΈΡΠΈΠΌΠΎΡΡ‚ΡŒ. Π­Ρ‚ΠΎ достигаСтся комплСксом Π·Π°ΠΌΠ΅Ρ€ΠΎΠ² Π½Π° всём Π΄ΠΈΠ°ΠΏΠ°Π·ΠΎΠ½Π΅ частот, Π³Π΅Π½Π΅Ρ€ΠΈΡ€ΡƒΠ΅ΠΌΡ‹Ρ… Ρ‚Π°ΠΊΠΈΠΌ Π΄Π²ΡƒΡ…ΠΏΠΎΠ»ΡŽΡΠ½ΠΈΠΊΠΎΠΌ.

Π‘ΠΎΠ»ΡŒΡˆΠΎΠ΅ Π²Π½ΡƒΡ‚Ρ€Π΅Π½Π½Π΅Π΅ сопротивлСниС

ΠŸΡŒΠ΅Π·ΠΎΡΠ»Π΅ΠΊΡ‚Ρ€ΠΈΡ‡Π΅ΡΠΊΠΈΠ΅ Π΄Π°Ρ‚Ρ‡ΠΈΠΊΠΈ, кондСнсаторныС ΠΌΠΈΠΊΡ€ΠΎΡ„ΠΎΠ½Ρ‹ ΠΈ Π΄Ρ€ΡƒΠ³ΠΈΠ΅ источники ΠΈΠΌΠΏΡƒΠ»ΡŒΡΠΎΠ² ΠΎΠ±Π»Π°Π΄Π°ΡŽΡ‚ ΠΏΠΎΠ²Ρ‹ΡˆΠ΅Π½Π½Ρ‹ΠΌ Π²Π½ΡƒΡ‚Ρ€Π΅Π½Π½ΠΈΠΌ импСдансом. Π§Ρ‚ΠΎΠ±Ρ‹ эффСктивно ΠΈΡΠΏΠΎΠ»ΡŒΠ·ΠΎΠ²Π°Ρ‚ΡŒ Ρ‚Π°ΠΊΠΈΠ΅ устройства, Π½ΡƒΠΆΠ½ΠΎ ΠΏΡ€Π°Π²ΠΈΠ»ΡŒΠ½ΠΎ ΡΠΎΠ³Π»Π°ΡΠΎΠ²Π°Ρ‚ΡŒ схСму считывания сигнала. ΠŸΡ€ΠΈ Π½Π΅ΡƒΠ΄Π°Ρ‡Π½ΠΎΠΌ согласовании Π½Π΅ΠΈΠ·Π±Π΅ΠΆΠ½Ρ‹ ΠΏΠΎΡ‚Π΅Ρ€ΠΈ.

Π’Π°ΠΆΠ½ΠΎ! Π£Π΄Π°Ρ‡Π½ΠΎΠ΅ согласованиС ΠΏΠΎ Π½Π°ΠΏΡ€ΡΠΆΠ΅Π½ΠΈΡŽ получаСтся ΠΏΡ€ΠΈ использовании для снятия сигнала устройства, с большим Π²Ρ…ΠΎΠ΄Π½Ρ‹ΠΌ сопротивлСниСм, Ρ‡Π΅ΠΌ Ρƒ источника сигнала. Π’ случаС высокоомного источника для считывания сигнала примСняСтся Π±ΡƒΡ„Π΅Ρ€Π½Ρ‹ΠΉ ΡƒΡΠΈΠ»ΠΈΡ‚Π΅Π»ΡŒ.

Π’Π½ΡƒΡ‚Ρ€Π΅Π½Π½Π΅Π΅ сопротивлСниС ΠΈ импСданс

ИмпСданс – ΠΏΠΎΠ»Π½ΠΎΠ΅ (комплСксноС) Π²Π½ΡƒΡ‚Ρ€Π΅Π½Π½Π΅Π΅ сопротивлСниС эквивалСнтного Π΄Π²ΡƒΡ…ΠΏΠΎΠ»ΡŽΡΠ½ΠΈΠΊΠ° ΠΏΠ΅Ρ€Π΅ΠΌΠ΅Π½Π½ΠΎΠΌΡƒ Ρ‚ΠΎΠΊΡƒ. ΠžΠ±ΠΎΠ·Π½Π°Ρ‡Π°Π΅Ρ‚ΡΡ Π±ΡƒΠΊΠ²ΠΎΠΉ Z ΠΈ Ρ‚Π°ΠΊ ΠΆΠ΅ измСряСтся Π² ΠžΠΌΠ°Ρ….

Π”Π²ΡƒΡ…ΠΏΠΎΠ»ΡŽΡΠ½ΠΈΠΊ ΠΈ Π΅Π³ΠΎ эквивалСнтная схСма

Π”Π²ΡƒΡ…ΠΏΠΎΠ»ΡŽΡΠ½ΠΈΠΊ прСдставляСт собой ΡΠ»Π΅ΠΊΡ‚Ρ€ΠΈΡ‡Π΅ΡΠΊΡƒΡŽ Ρ†Π΅ΠΏΡŒ, ΡΠΎΠ΄Π΅Ρ€ΠΆΠ°Ρ‰ΡƒΡŽ Π΄Π²Π΅ Ρ‚ΠΎΡ‡ΠΊΠΈ присоСдинСния ΠΊ Π΄Ρ€ΡƒΠ³ΠΈΠΌ цСпям. Π‘Ρ‹Π²Π°Π΅Ρ‚ Π΄Π²Π° Π²ΠΈΠ΄Π° элСктричСских Ρ†Π΅ΠΏΠ΅ΠΉ:

  • Ρ†Π΅ΠΏΠΈ, содСрТащиС источник Ρ‚ΠΎΠΊΠ° ΠΈΠ»ΠΈ напряТСния;
  • Π΄Π²ΡƒΡ…ΠΏΠΎΠ»ΡŽΡΠ½ΠΈΠΊΠΈ, Π½Π΅ ΡΠ²Π»ΡΡŽΡ‰ΠΈΠ΅ΡΡ источниками.

ΠŸΠ΅Ρ€Π²Ρ‹Π΅ Ρ…Π°Ρ€Π°ΠΊΡ‚Π΅Ρ€ΠΈΠ·ΡƒΡŽΡ‚ΡΡ элСктричСскими ΠΏΠ°Ρ€Π°ΠΌΠ΅Ρ‚Ρ€Π°ΠΌΠΈ: силой Ρ‚ΠΎΠΊΠ°, напряТСниСм ΠΈ импСдансом. Для расчёта ΠΏΠ°Ρ€Π°ΠΌΠ΅Ρ‚Ρ€ΠΎΠ² Ρ‚Π°ΠΊΠΈΡ… Π΄Π²ΡƒΡ…ΠΏΠΎΠ»ΡŽΡΠ½ΠΈΠΊΠΎΠ² ΠΏΡ€Π΅Π΄Π²Π°Ρ€ΠΈΡ‚Π΅Π»ΡŒΠ½ΠΎ производят Π·Π°ΠΌΠ΅Π½Ρƒ Ρ€Π΅Π°Π»ΡŒΠ½Ρ‹Ρ… элСмСнтов Ρ†Π΅ΠΏΠΈ Π½Π° ΠΈΠ΄Π΅Π°Π»ΡŒΠ½Ρ‹Π΅ элСмСнты. ΠšΠΎΠΌΠ±ΠΈΠ½Π°Ρ†ΠΈΡ, которая получаСтся Π² Ρ€Π΅Π·ΡƒΠ»ΡŒΡ‚Π°Ρ‚Π΅ ΠΏΠΎΠ΄ΠΎΠ±Π½ΠΎΠΉ Π·Π°ΠΌΠ΅Π½Ρ‹, называСтся эквивалСнтной схСмой.

Π’Π½ΠΈΠΌΠ°Π½ΠΈΠ΅! ΠŸΡ€ΠΈ Ρ€Π°Π±ΠΎΡ‚Π΅ со слоТными элСктричСскими схСмами с ΡƒΡ‡Ρ‘Ρ‚ΠΎΠΌ Ρ‚ΠΎΠ³ΠΎ, Ρ‡Ρ‚ΠΎ устройство Ρ€Π°Π±ΠΎΡ‚Π°Π΅Ρ‚ Π½Π° ΠΎΠ΄Π½ΠΎΠΉ частотС, допустимо ΠΏΡ€Π΅ΠΎΠ±Ρ€Π°Π·ΠΎΠ²Ρ‹Π²Π°Ρ‚ΡŒ ΠΏΠΎΡΠ»Π΅Π΄ΠΎΠ²Π°Ρ‚Π΅Π»ΡŒΠ½Ρ‹Π΅ ΠΈ ΠΏΠ°Ρ€Π°Π»Π»Π΅Π»ΡŒΠ½Ρ‹Π΅ Π²Π΅Ρ‚Π²ΠΈ Π΄ΠΎ получСния простой схСмы, доступной для расчёта ΠΏΠ°Ρ€Π°ΠΌΠ΅Ρ‚Ρ€ΠΎΠ².

Π’Ρ‚ΠΎΡ€ΠΎΠΉ Π²ΠΈΠ΄ Π΄Π²ΡƒΡ…ΠΏΠΎΠ»ΡŽΡΠ½ΠΈΠΊΠΎΠ² ΠΌΠΎΠΆΠ½ΠΎ ΠΎΡ…Π°Ρ€Π°ΠΊΡ‚Π΅Ρ€ΠΈΠ·ΠΎΠ²Π°Ρ‚ΡŒ Ρ‚ΠΎΠ»ΡŒΠΊΠΎ Π²Π΅Π»ΠΈΡ‡ΠΈΠ½ΠΎΠΉ Π²Π½ΡƒΡ‚Ρ€Π΅Π½Π½Π΅Π³ΠΎ сопротивлСния.

ВлияниС Π²Π½ΡƒΡ‚Ρ€Π΅Π½Π½Π΅Π³ΠΎ сопротивлСния Π½Π° свойства Π΄Π²ΡƒΡ…ΠΏΠΎΠ»ΡŽΡΠ½ΠΈΠΊΠ°

Π§Π΅ΠΌ ΠΎΠ½ΠΎ Π²Ρ‹ΡˆΠ΅, Ρ‚Π΅ΠΌ ΠΌΠ΅Π½ΡŒΡˆΡƒΡŽ ΠΌΠΎΡ‰Π½ΠΎΡΡ‚ΡŒ Π²Ρ‹Π΄Π°Ρ‘Ρ‚ источник ΠΏΡ€ΠΈ ΠΏΠΎΠ΄ΠΊΠ»ΡŽΡ‡Π΅Π½ΠΈΠΈ Π½Π°Π³Ρ€ΡƒΠ·ΠΊΠΈ. ΠžΠΏΡ€Π΅Π΄Π΅Π»ΠΈΡ‚ΡŒ ΠΌΠΎΡ‰Π½ΠΎΡΡ‚ΡŒ Π² Π½Π°Π³Ρ€ΡƒΠ·ΠΊΠ΅ ΠΌΠΎΠΆΠ½ΠΎ ΠΏΠΎ Ρ„ΠΎΡ€ΠΌΡƒΠ»Π΅:

Π³Π΄Π΅:

  • E – напряТСниС Π­Π”Π‘;
  • R – сопротивлСниС Π½Π°Π³Ρ€ΡƒΠ·ΠΊΠΈ;
  • r – Π°ΠΊΡ‚ΠΈΠ²Π½ΠΎΠ΅ Π²Π½ΡƒΡ‚Ρ€Π΅Π½Π½Π΅Π΅ сопротивлСниС Π΄Π²ΡƒΡ…ΠΏΠΎΠ»ΡŽΡΠ½ΠΈΠΊΠ°.

Π€ΠΎΡ€ΠΌΡƒΠ»Π° ΠΏΡ€ΠΈΠΌΠ΅Π½ΠΈΠΌΠ° ΠΊ Π΄Π²ΡƒΡ…ΠΏΠΎΠ»ΡŽΡΠ½ΠΈΠΊΠ°ΠΌ, Π½Π΅ ΠΎΡ‚Π΄Π°ΡŽΡ‰ΠΈΠΌ ΡΠ½Π΅Ρ€Π³ΠΈΡŽ.

К свСдСнию. Когда Π²Π΅Π»ΠΈΡ‡ΠΈΠ½Π° Π²Π½ΡƒΡ‚Ρ€Π΅Π½Π½Π΅Π³ΠΎ сопротивлСния Π΄Π²ΡƒΡ…ΠΏΠΎΠ»ΡŽΡΠ½ΠΈΠΊΠ° приблиТаСтся ΠΏΠΎ своСму Π·Π½Π°Ρ‡Π΅Π½ΠΈΡŽ ΠΊ ΡΠΎΠΏΡ€ΠΎΡ‚ΠΈΠ²Π»Π΅Π½ΠΈΡŽ Π½Π°Π³Ρ€ΡƒΠ·ΠΊΠΈ, ΠΏΠ΅Ρ€Π΅Π΄Π°Ρ‡Π° мощности достигаСт максимума.

Разрядная Π΅ΠΌΠΊΠΎΡΡ‚ΡŒ источника

Π’Π΅Π»ΠΈΡ‡ΠΈΠ½Π°, зависящая ΠΎΡ‚ силы Ρ‚ΠΎΠΊΠ° разряда, называСтся разрядной Ρ‘ΠΌΠΊΠΎΡΡ‚ΡŒΡŽ источника. Π­Ρ‚ΠΎ элСктричСский заряд, ΠΊΠΎΡ‚ΠΎΡ€Ρ‹ΠΉ ΠΎΡ‚Π΄Π°Ρ‘Ρ‚ источник Π² процСссС эксплуатации Π² зависимости ΠΎΡ‚ Ρ‚ΠΎΠΊΠ° Π½Π°Π³Ρ€ΡƒΠ·ΠΊΠΈ. Π­Ρ‚Ρƒ Π²Π΅Π»ΠΈΡ‡ΠΈΠ½Ρƒ ΠΌΠΎΠΆΠ½ΠΎ ΡΡ‡ΠΈΡ‚Π°Ρ‚ΡŒ постоянной условно. Π’Π°ΠΊ, стартСрный аккумулятор, ΠΈΠΌΠ΅ΡŽΡ‰ΠΈΠΉ Ρ€Π°Π·Ρ€ΡΠ΄Π½ΡƒΡŽ Ρ‘ΠΌΠΊΠΎΡΡ‚ΡŒ Π‘ = 55 А*Ρ‡, ΠΏΡ€ΠΈ Ρ‚ΠΎΠΊΠ΅ разряда 5,5 А ΠΏΡ€ΠΎΡ€Π°Π±ΠΎΡ‚Π°Π΅Ρ‚ 10 часов. ΠŸΡ€ΠΈ запусках Ρ…ΠΎΠ»ΠΎΠ΄Π½ΠΎΠ³ΠΎ ΠΈΠ»ΠΈ ΠΈΠΌΠ΅ΡŽΡ‰Π΅Π³ΠΎ Π½Π΅ΠΈΡΠΏΡ€Π°Π²Π½ΠΎΡΡ‚ΡŒ автомобиля аккумулятор ΠΌΠΎΠΆΠ½ΠΎ Ρ€Π°Π·Ρ€ΡΠ΄ΠΈΡ‚ΡŒ Π·Π° нСсколько ΠΌΠΈΠ½ΡƒΡ‚.

Для Ρ‚ΠΎΠ³ΠΎ Ρ‡Ρ‚ΠΎΠ±Ρ‹ Π½Π°ΠΉΡ‚ΠΈ ΠΎΡΡ‚Π°Ρ‚ΠΎΡ‡Π½ΡƒΡŽ Ρ€Π°Π·Ρ€ΡΠ΄Π½ΡƒΡŽ Ρ‘ΠΌΠΊΠΎΡΡ‚ΡŒ, производят Ρ†ΠΈΠΊΠ»Ρ‹ «заряд – разряд». Они Π²Ρ‹ΠΏΠΎΠ»Π½ΡΡŽΡ‚ΡΡ ΠΏΡ€ΠΈ ΠΏΠΎΠΌΠΎΡ‰ΠΈ Π½Π°Π³Ρ€ΡƒΠ·ΠΎΡ‡Π½Ρ‹Ρ… сопротивлСний. Разряд Π½Π° Π½Π°Π³Ρ€ΡƒΠ·ΠΎΡ‡Π½ΠΎΠ΅ сопротивлСниС производят Π΄ΠΎ минимально допустимых Π·Π½Π°Ρ‡Π΅Π½ΠΈΠΉ плотности элСктролита. ΠŸΡ€ΠΈ этом замСряСтся врСмя Ρ€Π°Π±ΠΎΡ‚Ρ‹ ΠΏΠΎΠ΄ Π½Π°Π³Ρ€ΡƒΠ·ΠΊΠΎΠΉ. Π­Ρ‚ΠΎ Π°ΠΊΡ‚ΡƒΠ°Π»ΡŒΠ½ΠΎ ΠΏΡ€ΠΈ сСзонном обслуТивании аккумуляторов для выявлСния процСссов саморазряда.

Π’Π½ΡƒΡ‚Ρ€Π΅Π½Π½Π΅Π΅ сопротивлСниС источников Ρ‚ΠΎΠΊΠ° – ваТная Π²Π΅Π»ΠΈΡ‡ΠΈΠ½Π°. ΠœΠ΅Ρ‚ΠΎΠ΄Ρ‹, примСняСмыС для Π΅Ρ‘ сниТСния, ΡΠ²Π»ΡΡŽΡ‚ΡΡ прямыми путями увСличСния ΠΎΡ‚Π΄Π°Π²Π°Π΅ΠΌΠΎΠΉ мощности источника, Π·Π½Π°Ρ‡ΠΈΡ‚, ΠΏΠΎΠ²Ρ‹ΡˆΠ΅Π½ΠΈΡ ΠΏΡ€ΠΎΠΈΠ·Π²ΠΎΠ΄ΠΈΡ‚Π΅Π»ΡŒΠ½ΠΎΡΡ‚ΠΈ Π΄Π²ΡƒΡ…ΠΏΠΎΠ»ΡŽΡΠ½ΠΈΠΊΠΎΠ². ΠŸΡ€Π°Π²ΠΈΠ»ΡŒΠ½ΠΎΠ΅ ΠΈΠ·ΠΌΠ΅Ρ€Π΅Π½ΠΈΠ΅ ΠΈ вычислСниС импСданса эквивалСнтных схСм ΠΏΠΎΠ·Π²ΠΎΠ»ΡΡŽΡ‚ ΠΏΡ€ΠΈΠ±Π»ΠΈΠ·ΠΈΡ‚ΡŒ Π΄Π²ΡƒΡ…ΠΏΠΎΠ»ΡŽΡΠ½ΠΈΠΊ ΠΊ ΠΈΠ΄Π΅Π°Π»ΡŒΠ½ΠΎΠΌΡƒ источнику.

Π’ΠΈΠ΄Π΅ΠΎ

Π’Π΅Π»ΠΈΡ‡ΠΈΠ½Π°, Ρ…Π°Ρ€Π°ΠΊΡ‚Π΅Ρ€ΠΈΠ·ΡƒΡŽΡ‰Π°Ρ количСство энСргСтичСских ΠΏΠΎΡ‚Π΅Ρ€ΡŒ, Π²ΠΎΠ·Π½ΠΈΠΊΠ°ΡŽΡ‰ΠΈΡ… ΠΏΡ€ΠΈ ΠΏΡ€ΠΎΡ‚Π΅ΠΊΠ°Π½ΠΈΠΈ Ρ‚ΠΎΠΊΠ° Ρ‡Π΅Ρ€Π΅Π· Π΅Π³ΠΎ источник, опрСдСляСтся ΠΊΠ°ΠΊ Π²Π½ΡƒΡ‚Ρ€Π΅Π½Π½Π΅Π΅ сопротивлСниС источника Ρ‚ΠΎΠΊΠ°. Как ΠΈ ΠΎΠ±Ρ‹Ρ‡Π½ΠΎΠ΅ сопротивлСниС, ΠΈΠΌΠ΅Π΅Ρ‚ Π΅Π΄ΠΈΠ½ΠΈΡ†Ρƒ измСрСния, Ρ€Π°Π²Π½ΡƒΡŽ 1 Ом. Π’ΠΎΠΊ, двигаясь Ρ‡Π΅Ρ€Π΅Π· источник, тСряСт Ρ‡Π°ΡΡ‚ΡŒ своСй энСргии, которая ΠΏΠ΅Ρ€Π΅Ρ…ΠΎΠ΄ΠΈΡ‚ Π² Ρ‚Π΅ΠΏΠ»ΠΎ, Ρ‚ΠΎΡ‡Π½ΠΎ Ρ‚Π°ΠΊ ΠΆΠ΅, ΠΊΠ°ΠΊ Π½Π° любом Π½Π°Π³Ρ€ΡƒΠ·ΠΎΡ‡Π½ΠΎΠΌ сопротивлСнии. Π­Ρ‚ΠΎ Π·Π½Π°Ρ‡ΠΈΡ‚, Ρ‡Ρ‚ΠΎ Π²Π΅Π»ΠΈΡ‡ΠΈΠ½Π° напряТСния Π½Π° Π²Ρ‹Π²ΠΎΠ΄Π°Ρ… источника зависит ΠΎΡ‚ Π²Π΅Π»ΠΈΡ‡ΠΈΠ½Ρ‹ Ρ‚ΠΎΠΊΠ°, Π° Π½Π΅ ΠΎΡ‚ Π­Π”Π‘.

Если Ρ€Π°ΡΡΠΌΠΎΡ‚Ρ€Π΅Ρ‚ΡŒ Π·Π°ΠΌΠΊΠ½ΡƒΡ‚ΡƒΡŽ ΡΠ»Π΅ΠΊΡ‚Ρ€ΠΈΡ‡Π΅ΡΠΊΡƒΡŽ Ρ†Π΅ΠΏΡŒ, Π² ΠΊΠΎΡ‚ΠΎΡ€ΡƒΡŽ Π²ΠΊΠ»ΡŽΡ‡Ρ‘Π½ источник Ρ‚ΠΎΠΊΠ° (Π±Π°Ρ‚Π°Ρ€Π΅ΠΉΠΊΠ°, аккумулятор ΠΈΠ»ΠΈ Π³Π΅Π½Π΅Ρ€Π°Ρ‚ΠΎΡ€), ΠΈ Π½Π°Π³Ρ€ΡƒΠ·ΠΊΡƒ R, Ρ‚ΠΎ Ρ‚ΠΎΠΊ Ρ‚Π΅Ρ‡Ρ‘Ρ‚ ΠΈ Π²Π½ΡƒΡ‚Ρ€ΠΈ источника. Π’Π½ΡƒΡ‚Ρ€Π΅Π½Π½Π΅Π΅ сопротивлСниС источника, ΠΎΠ±ΠΎΠ·Π½Π°Ρ‡Π°Π΅ΠΌΠΎΠ΅ Π±ΡƒΠΊΠ²ΠΎΠΉ r, Π΅ΠΌΡƒ прСпятствуСт.

Π£ Π³Π΅Π½Π΅Ρ€Π°Ρ‚ΠΎΡ€Π° r – это Π²Π½ΡƒΡ‚Ρ€Π΅Π½Π½Π΅Π΅ сопротивлСниС ΠΎΠ±ΠΌΠΎΡ‚ΠΎΠΊ статора, Ρƒ аккумулятора – сопротивлСниС элСктролита.

Π˜Π·ΠΌΠ΅Ρ€Π΅Π½ΠΈΠ΅ сопротивлСния ΠΏΠ΅Ρ‚Π»ΠΈ Ρ„Π°Π·Π°-Π½ΡƒΠ»ΡŒ

ΠŸΠ΅Ρ‚Π»Ρ Β«Ρ„Π°Π·Π° – Π½ΡƒΠ»ΡŒΒ» – это элСктричСская Ρ†Π΅ΠΏΡŒ ΠΏΠ΅Ρ€Π΅ΠΌΠ΅Π½Π½ΠΎΠ³ΠΎ Ρ‚ΠΎΠΊΠ°, которая ΠΌΠΎΠΆΠ΅Ρ‚ Π²ΠΎΠ·Π½ΠΈΠΊΠ½ΡƒΡ‚ΡŒ Π² Ρ€Π΅Π·ΡƒΠ»ΡŒΡ‚Π°Ρ‚Π΅ ΠΊΠΎΡ€ΠΎΡ‚ΠΊΠΎΠ³ΠΎ замыкания ΠΌΠ΅ΠΆΠ΄Ρƒ ΠΏΡ€ΠΎΠ²ΠΎΠ΄Π°ΠΌΠΈ: Β«Ρ„Π°Π·Π°Β» ΠΈ «ноль» ΠΈΠ»ΠΈ Β«Ρ„Π°Π·Π°Β» ΠΈ Β«Ρ„Π°Π·Π°Β». Π Π°Π·Ρ€ΡƒΡˆΠ΅Π½ΠΈΠ΅ изоляции, мСханичСскиС поврСТдСния ΠΈΠ»ΠΈ случайноС соСдинСниС ΠΎΠ³ΠΎΠ»Ρ‘Π½Π½Ρ‹Ρ… участков кабСля ΠΌΠ΅ΠΆΠ΄Ρƒ собой ΠΌΠΎΠ³ΡƒΡ‚ ΡΡ‚Π°Ρ‚ΡŒ этому ΠΏΡ€ΠΈΡ‡ΠΈΠ½ΠΎΠΉ. Π’ установках с Π³Π»ΡƒΡ…ΠΎ Π·Π°Π·Π΅ΠΌΠ»Ρ‘Π½Π½ΠΎΠΉ Π½Π΅ΠΉΡ‚Ρ€Π°Π»ΡŒΡŽ Π½ΡƒΠ»Π΅Π²ΠΎΠΉ ΠΏΡ€ΠΎΠ²ΠΎΠ΄Π½ΠΈΠΊ физичСски связан с Π½Π΅ΠΉΡ‚Ρ€Π°Π»ΡŒΡŽ трансформатора, ΠΎΠ½Π° ΠΏΠΎΠ΄ΠΊΠ»ΡŽΡ‡Π΅Π½Π° ΠΊ ΠΊΠΎΠ½Ρ‚ΡƒΡ€Ρƒ зазСмлСния. ΠŸΡ€ΠΈ Π·Π°ΠΌΡ‹ΠΊΠ°Π½ΠΈΠΈ Π½Π° корпус ΠΈΠ»ΠΈ соСдинСнии Ρ„Π°Π· ΠΌΠ΅ΠΆΠ΄Ρƒ собой образуСтся Ρ†Π΅ΠΏΡŒ (пСтля).

Главная Π·Π°Π΄Π°Ρ‡Π° ΠΏΡ€ΠΎΠ²ΠΎΠ΄ΠΈΠΌΡ‹Ρ… ΠΈΠ·ΠΌΠ΅Ρ€Π΅Π½ΠΈΠΉ – ΡƒΠ·Π½Π°Π²Π°Ρ‚ΡŒ, ΠΊΠ°ΠΊΠΈΠΌ Π±ΡƒΠ΄Π΅Ρ‚ Π²Π΅Π»ΠΈΡ‡ΠΈΠ½Π° Ρ‚ΠΎΠΊΠ° Ρ‡Π΅Ρ€Π΅Π· ΠΏΠ΅Ρ‚Π»ΡŽ ΠΏΡ€ΠΈ ΠšΠ—. Π­Ρ‚ΠΎ ΠΎΠ±ΡΠ·Π°Ρ‚Π΅Π»ΡŒΠ½ΠΎ для расчёта ΠΈ ΠΏΠΎΠ΄Π±ΠΎΡ€Π° Π·Π°Ρ‰ΠΈΡ‚Π½ΠΎΠ³ΠΎ оборудования. Π₯ΠΎΡ€ΠΎΡˆΠΈΠΌ Ρ€Π΅Π·ΡƒΠ»ΡŒΡ‚Π°Ρ‚ΠΎΠΌ Π±ΡƒΠ΄Π΅Ρ‚ малСнькоС сопротивлСниС ΠΏΠ΅Ρ‚Π»ΠΈ, Ρ‚ΠΎΠ³Π΄Π° Ρ‚ΠΎΠΊ IΠΊ.Π·. Π±ΡƒΠ΄Π΅Ρ‚ наибольшим. ΠžΡ‚ Π΅Π³ΠΎ Π²Π΅Π»ΠΈΡ‡ΠΈΠ½Ρ‹ зависит, ΠΊΠ°ΠΊ быстро сработаСт Π·Π°Ρ‰ΠΈΡ‚Π½Ρ‹ΠΉ автоматичСский Π²Ρ‹ΠΊΠ»ΡŽΡ‡Π°Ρ‚Π΅Π»ΡŒ.

Π§Π΅ΠΌ мСньшС Π²Ρ€Π΅ΠΌΠ΅Π½ΠΈ Π±ΡƒΠ΄Π΅Ρ‚ Π·Π°Ρ‚Ρ€Π°Ρ‡Π΅Π½ΠΎ Π½Π° ΠΎΡ‚ΠΊΠ»ΡŽΡ‡Π΅Π½ΠΈΠ΅ ΠΏΠΎΠ²Ρ€Π΅ΠΆΠ΄Ρ‘Π½Π½ΠΎΠΉ ΠΈΠ»ΠΈ Π·Π°ΠΊΠΎΡ€ΠΎΡ‡Π΅Π½Π½ΠΎΠΉ Ρ†Π΅ΠΏΠΈ, Ρ‚Π΅ΠΌ большС шансов ΠΏΡ€Π΅Π΄ΠΎΡ‚Π²Ρ€Π°Ρ‚ΠΈΡ‚ΡŒ ΠΏΠΎΠΆΠ°Ρ€ ΠΎΡ‚ возгорания кабСльной сСти. ΠŸΡ€ΠΈ ΠΏΠΎΠΏΠ°Π΄Π°Π½ΠΈΠΈ Ρ‡Π΅Π»ΠΎΠ²Π΅ΠΊΠ° ΠΏΠΎΠ΄ ΡƒΠ΄Π°Ρ€ элСктричСского Ρ‚ΠΎΠΊΠ° Π² Ρ€Π΅Π·ΡƒΠ»ΡŒΡ‚Π°Ρ‚Π΅ прикосновСния ΠΈΠ»ΠΈ ΠΊΠΎΡ€ΠΎΡ‚ΠΊΠΎΠ³ΠΎ замыкания автоматичСскоС снятиС напряТСния спасёт Π΅ΠΌΡƒ Тизнь.

На прСдприятиях Π΅ΠΆΠ΅Π³ΠΎΠ΄Π½ΠΎ проводится комплСкс ΠΈΠ·ΠΌΠ΅Ρ€Π΅Π½ΠΈΠΉ Π·Π°Ρ‰ΠΈΡ‚Π½ΠΎΠ³ΠΎ зазСмлСния ΠΈ сопротивлСния ΠΏΠ΅Ρ‚Π»ΠΈ Ρ„Π°Π·Π° – ноль. ΠŸΡ€ΠΈ Π½Π΅ΡƒΠ΄ΠΎΠ²Π»Π΅Ρ‚Π²ΠΎΡ€ΠΈΡ‚Π΅Π»ΡŒΠ½Ρ‹Ρ… Ρ€Π΅Π·ΡƒΠ»ΡŒΡ‚Π°Ρ‚Π°Ρ… проводится ряд мСроприятий:

  • Π·Π°ΠΌΠ΅Π½ΡΡŽΡ‚ΡΡ участки ΠΏΡ€ΠΎΠ²ΠΎΠ΄Π°, Π½Π΅ ΠΎΡ‚Π²Π΅Ρ‡Π°ΡŽΡ‰ΠΈΠ΅ трСбованиям ΠΏΠΎ Π΄ΠΈΠ°ΠΌΠ΅Ρ‚Ρ€Ρƒ сСчСния;
  • ΠΏΠ΅Ρ€Π΅ΠΊΡ€ΡƒΡ‡ΠΈΠ²Π°ΡŽΡ‚ΡΡ Π±ΠΎΠ»Ρ‚ΠΎΠ²Ρ‹Π΅ соСдинСния с ΠΎΠ±ΡΠ·Π°Ρ‚Π΅Π»ΡŒΠ½ΠΎΠΉ установкой Π²Ρ€Π΅Π·Π½Ρ‹Ρ… шайб;
  • Π²ΡΠΊΡ€Ρ‹Π²Π°ΡŽΡ‚ΡΡ ΠΊΠΎΠ½Ρ‚ΡƒΡ€Ρ‹ Π·Π°Ρ‰ΠΈΡ‚Π½Ρ‹Ρ… Π·Π°Π·Π΅ΠΌΠ»Π΅Π½ΠΈΠΉ ΠΈ ΠΎΡΠΌΠ°Ρ‚Ρ€ΠΈΠ²Π°ΡŽΡ‚ΡΡ Π½Π° ΠΏΡ€Π΅Π΄ΠΌΠ΅Ρ‚ цСлостности сварных соСдинСний ΠΈ состояния элСмСнтов зазСмлСния;
  • ΠΏΡ€ΠΈ нСобходимости Π² ΠΊΠΎΠ½Ρ‚ΡƒΡ€ Π·Π°Ρ‰ΠΈΡ‚Π½ΠΎΠ³ΠΎ зазСмлСния Π΄ΠΎΠ±Π°Π²Π»ΡΡŽΡ‚ΡΡ Π΄ΠΎΠΏΠΎΠ»Π½ΠΈΡ‚Π΅Π»ΡŒΠ½Ρ‹Π΅ элСмСнты;
  • ΠΈΡΠΊΠ»ΡŽΡ‡Π°Π΅Ρ‚ΡΡ ΠΏΠΎΡΠ»Π΅Π΄ΠΎΠ²Π°Ρ‚Π΅Π»ΡŒΠ½ΠΎΠ΅ ΠΏΠΎΠ΄ΠΊΠ»ΡŽΡ‡Π΅Π½ΠΈΠ΅ корпусов устройств ΠΊ ΠΎΠ±Ρ‰Π΅ΠΉ шинС зазСмлСния.

ПослС выполнСния комплСкса мСроприятий измСрСния проводятся ΠΏΠΎΠ²Ρ‚ΠΎΡ€Π½ΠΎ.

НахоТдСниС Π²Π½ΡƒΡ‚Ρ€Π΅Π½Π½Π΅Π³ΠΎ сопротивлСния

Π•Π³ΠΎ ΠΌΠΎΠΆΠ½ΠΎ Π½Π°Ρ…ΠΎΠ΄ΠΈΡ‚ΡŒ двумя путями: Ρ€Π°ΡΡΡ‡ΠΈΡ‚Π°Ρ‚ΡŒ ΠΈΠ»ΠΈ ΠΈΠ·ΠΌΠ΅Ρ€ΠΈΡ‚ΡŒ. ΠŸΠ΅Ρ€Π²Ρ‹ΠΌ ΠΏΡƒΡ‚Ρ‘ΠΌ ΠΈΠ΄ΡƒΡ‚ ΠΏΡ€ΠΈ Ρ€Π°Π±ΠΎΡ‚Π΅ с элСктричСскими схСмами, Π²Ρ‚ΠΎΡ€ΠΎΠΉ – Π²Ρ‹Π±ΠΈΡ€Π°ΡŽΡ‚, занимаясь с Ρ€Π΅Π°Π»ΡŒΠ½Ρ‹ΠΌΠΈ устройствами.

ΠŸΡ€ΠΎΡΡ‚ΠΎΠΉ расчёт производится с использованиСм Ρ„ΠΎΡ€ΠΌΡƒΠ»Ρ‹ Π—Π°ΠΊΠΎΠ½Π° Ома для участка ΠΏΠΎΠ»Π½ΠΎΠΉ Ρ†Π΅ΠΏΠΈ:

Π§Ρ‚ΠΎΠ±Ρ‹ ΡƒΠ·Π½Π°Ρ‚ΡŒ силу Ρ‚ΠΎΠΊΠ°, Π½ΡƒΠΆΠ½ΠΎ напряТСниС Π­Π”Π‘ Π΄Π΅Π»ΠΈΡ‚ΡŒ Π½Π° сумму сопротивлСний.

Π’Ρ‹Ρ€Π°Π·ΠΈΠ² ΠΎΡ‚ΡΡŽΠ΄Π° r, ΠΏΠΎΠ»ΡƒΡ‡Π°ΡŽΡ‚ Ρ„ΠΎΡ€ΠΌΡƒΠ»Ρƒ для Π΅Π³ΠΎ вычислСния:

Π³Π΄Π΅:

  • r – Π²Π½ΡƒΡ‚Ρ€Π΅Π½Π½Π΅Π΅ сопротивлСниС источника;
  • Ξ΅ – Π­Π”Π‘ источника;
  • I – сила Ρ‚ΠΎΠΊΠ° Π² ΠΏΠΎΠ»Π½ΠΎΠΉ Ρ†Π΅ΠΏΠΈ;
  • R – сопротивлСниС Π² ΠΏΠΎΠ»Π½ΠΎΠΉ Ρ†Π΅ΠΏΠΈ.

КомплСкс ΠΈΠ·ΠΌΠ΅Ρ€Π΅Π½ΠΈΠΉ этого ΠΏΠ°Ρ€Π°ΠΌΠ΅Ρ‚Ρ€Π° Ρƒ настоящСго устройства Π½Π΅ ΠΏΠΎΠ΄Ρ€Π°Π·ΡƒΠΌΠ΅Π²Π°Π΅Ρ‚ нСпосрСдствСнных Π·Π°ΠΌΠ΅Ρ€ΠΎΠ². Π’Π΅ΡΡ‚ΠΈΡ€ΡƒΡŽΡ‚ΡΡ напряТСния Π½Π° Π½Π°Π³Ρ€ΡƒΠ·ΠΎΡ‡Π½ΠΎΠΌ сопротивлСнии Π² Π΄Π²ΡƒΡ… Ρ€Π΅ΠΆΠΈΠΌΠ°Ρ… Ρ‚ΠΎΠΊΠ°: холостом ΠΈ ΠšΠ—.

Π’Π°ΠΊ ΠΊΠ°ΠΊ Π½Π΅ любой источник ΠΌΠΎΠΆΠ΅Ρ‚ Π²Ρ‹Π΄Π΅Ρ€ΠΆΠ°Ρ‚ΡŒ Π΄Π°ΠΆΠ΅ ΠΊΡ€Π°Ρ‚ΠΊΠΎΠ²Ρ€Π΅ΠΌΠ΅Π½Π½Ρ‹ΠΉ Ρ€Π΅ΠΆΠΈΠΌ замыкания, бСрётся ΠΌΠ΅Ρ‚ΠΎΠ΄ измСрСния Π±Π΅Π· вычислСний.

Π’ схСму Π²ΠΊΠ»ΡŽΡ‡Π°Π΅Ρ‚ΡΡ внСшнСС сопротивлСниС Π½Π°Π³Ρ€ΡƒΠ·ΠΊΠΈ Π² Π²ΠΈΠ΄Π΅ подстроСчного рСзистора RΠ½. ВыставляСтся Ρ‚Π°ΠΊΠΎΠ΅ Π·Π½Π°Ρ‡Π΅Π½ΠΈΠ΅, ΠΏΡ€ΠΈ ΠΊΠΎΡ‚ΠΎΡ€ΠΎΠΌ ΠΏΠ°Π΄Π΅Π½ΠΈΠ΅ напряТСния Π½Π° рСзисторС Ρ€Π°Π²Π½ΡΠ»ΠΎΡΡŒ Π±Ρ‹ 1/2 U холостого Ρ…ΠΎΠ΄Π°. Π’ΠΎΠ³Π΄Π° ΠΈΠ·ΠΌΠ΅Ρ€Π΅Π½Π½ΠΎΠ΅ ΠΎΠΌΠΌΠ΅Ρ‚Ρ€ΠΎΠΌ RΠ½ Π±ΡƒΠ΄Π΅Ρ‚ ΡΠΎΠΎΡ‚Π²Π΅Ρ‚ΡΡ‚Π²ΠΎΠ²Π°Ρ‚ΡŒ Π²Π½ΡƒΡ‚Ρ€Π΅Π½Π½Π΅ΠΌΡƒ ΡΠΎΠΏΡ€ΠΎΡ‚ΠΈΠ²Π»Π΅Π½ΠΈΡŽ источника.

МалоС Π²Π½ΡƒΡ‚Ρ€Π΅Π½Π½Π΅Π΅ сопротивлСниС

Малой Π²Π΅Π»ΠΈΡ‡ΠΈΠ½Ρ‹ Π²Π½ΡƒΡ‚Ρ€Π΅Π½Π½Π΅Π³ΠΎ сопротивлСния Π΄ΠΎΠ±ΠΈΠ²Π°ΡŽΡ‚ΡΡ ΠΏΡ€ΠΈΠΌΠ΅Π½Π΅Π½ΠΈΠ΅ΠΌ ΠΎΠ±Ρ€Π°Ρ‚Π½ΠΎΠΉ связи Π² схСмах, ΠΊΡƒΠ΄Π° Π²ΠΊΠ»ΡŽΡ‡Ρ‘Π½ Π΄Π²ΡƒΡ…ΠΏΠΎΠ»ΡŽΡΠ½ΠΈΠΊ. Π’ стабилизаторах напряТСния r достигаСт Π·Π½Π°Ρ‡Π΅Π½ΠΈΠΉ ΠΌΠ΅Π½Π΅Π΅ 9*10-4 Ом. ΠΠ²Ρ‚ΠΎΠΌΠΎΠ±ΠΈΠ»ΡŒΠ½Π°Ρ ΠΠšΠ‘ 6Π‘Π’-60 ΠΎΠ±Π»Π°Π΄Π°Π΅Ρ‚ сопротивлСниСм ΠΎΠΊΠΎΠ»ΠΎ 0,01 Ом. Если произвСсти измСрСния ΠΏΠ΅Ρ‚Π»ΠΈ Ρ„Π°Π·Π°-ноль Π±Ρ‹Ρ‚ΠΎΠ²ΠΎΠΉ сСти, Ρ‚ΠΎ Π½ΠΎΡ€ΠΌΠ° значСния Π»Π΅ΠΆΠΈΡ‚ Π² ΠΏΡ€Π΅Π΄Π΅Π»Π°Ρ… 0,05-1 Ом.

Π Π΅Π°ΠΊΡ‚ΠΈΠ²Π½ΠΎΠ΅ Π²Π½ΡƒΡ‚Ρ€Π΅Π½Π½Π΅Π΅ сопротивлСниС

ΠšΡ€ΠΎΠΌΠ΅ Π³Π°Π»ΡŒΠ²Π°Π½ΠΈΡ‡Π΅ΡΠΊΠΈΡ… ΠΈ элСктролитичСских Π΄Π²ΡƒΡ…ΠΏΠΎΠ»ΡŽΡΠ½ΠΈΠΊΠΎΠ², ΡΡƒΡ‰Π΅ΡΡ‚Π²ΡƒΡŽΡ‚ источники питания, схСмы ΠΊΠΎΡ‚ΠΎΡ€Ρ‹Ρ… Π²ΠΊΠ»ΡŽΡ‡Π°ΡŽΡ‚ Π² сСбя Ρ€Π΅Π°ΠΊΡ‚ΠΈΠ²Π½Ρ‹Π΅ элСмСнты. ΠŸΡ€ΠΈ ΠΎΠΏΡ€Π΅Π΄Π΅Π»Π΅Π½ΠΈΠΈ ΠΈΡ… Π²Π½ΡƒΡ‚Ρ€Π΅Π½Π½Π΅Π³ΠΎ сопротивлСния ΠΈΡΠΏΠΎΠ»ΡŒΠ·ΡƒΡŽΡ‚ ΠΌΠ΅Ρ‚ΠΎΠ΄ комплСксных Π°ΠΌΠΏΠ»ΠΈΡ‚ΡƒΠ΄. Он ΠΏΠΎΠ΄Ρ€Π°Π·ΡƒΠΌΠ΅Π²Π°Π΅Ρ‚ ΠΈΡΠΏΠΎΠ»ΡŒΠ·ΠΎΠ²Π°Ρ‚ΡŒ ΠΏΡ€ΠΈ расчётах комплСксныС сопротивлСния элСмСнтов, Π²ΠΊΠ»ΡŽΡ‡Ρ‘Π½Π½Ρ‹Ρ… Π² схСму. Π’Π΅Π»ΠΈΡ‡ΠΈΠ½Ρ‹ Ρ‚ΠΎΠΊΠΎΠ² ΠΈ напряТСний Π·Π°ΠΌΠ΅Π½ΡΡŽΡ‚ΡΡ значСниями ΠΈΡ… комплСксных Π°ΠΌΠΏΠ»ΠΈΡ‚ΡƒΠ΄. Π‘Π°ΠΌ Π°Π»Π³ΠΎΡ€ΠΈΡ‚ΠΌ вычислСния Ρ‚Π°ΠΊΠΎΠΉ ΠΆΠ΅, ΠΊΠ°ΠΊ ΠΏΡ€ΠΈ расчётС Π°ΠΊΡ‚ΠΈΠ²Π½ΠΎΠ³ΠΎ сопротивлСния.

ΠŸΡ€ΠΎΡ†Π΅ΡΡ ΠΈΠ·ΠΌΠ΅Ρ€Π΅Π½ΠΈΠΉ r-Ρ€Π΅Π°ΠΊΡ‚ΠΈΠ²Π½ΠΎΠ³ΠΎ Π½Π΅ΠΌΠ½ΠΎΠ³ΠΎ отличаСтся ΠΎΡ‚ измСрСния Π°ΠΊΡ‚ΠΈΠ²Π½ΠΎΠΉ ΡΠΎΡΡ‚Π°Π²Π»ΡΡŽΡ‰Π΅ΠΉ сопротивлСния. ΠœΠ΅Ρ‚ΠΎΠ΄Ρ‹ зависят ΠΎΡ‚ Ρ‚ΠΎΠ³ΠΎ, ΠΊΠ°ΠΊΠΈΠ΅ ΠΏΠ°Ρ€Π°ΠΌΠ΅Ρ‚Ρ€Ρ‹ этой комплСксной Ρ„ΡƒΠ½ΠΊΡ†ΠΈΠΈ Π½ΡƒΠΆΠ½ΠΎ ΡƒΠ·Π½Π°Ρ‚ΡŒ: ΠΎΡ‚Π΄Π΅Π»ΡŒΠ½Ρ‹Π΅ ΡΠΎΡΡ‚Π°Π²Π»ΡΡŽΡ‰ΠΈΠ΅ ΠΈΠ»ΠΈ комплСксноС число.

На эти ΠΏΠ°Ρ€Π°ΠΌΠ΅Ρ‚Ρ€Ρ‹ влияСт частота, поэтому, Ρ‡Ρ‚ΠΎΠ±Ρ‹ ΠΏΡ€ΠΈ тСстировании Π΄ΠΎΠ±ΠΈΡ‚ΡŒΡΡ ΠΈΠ½Ρ„ΠΎΡ€ΠΌΠ°Ρ†ΠΈΠΈ ΠΎ Π²Π½ΡƒΡ‚Ρ€Π΅Π½Π½Π΅ΠΌ Ρ€Π΅Π°ΠΊΡ‚ΠΈΠ²Π½ΠΎΠΌ Π·Π½Π°Ρ‡Π΅Π½ΠΈΠΈ r, Π½ΡƒΠΆΠ½ΠΎ ΡƒΠ±Ρ€Π°Ρ‚ΡŒ Ρ‡Π°ΡΡ‚ΠΎΡ‚Π½ΡƒΡŽ Π·Π°Π²ΠΈΡΠΈΠΌΠΎΡΡ‚ΡŒ. Π­Ρ‚ΠΎ достигаСтся комплСксом Π·Π°ΠΌΠ΅Ρ€ΠΎΠ² Π½Π° всём Π΄ΠΈΠ°ΠΏΠ°Π·ΠΎΠ½Π΅ частот, Π³Π΅Π½Π΅Ρ€ΠΈΡ€ΡƒΠ΅ΠΌΡ‹Ρ… Ρ‚Π°ΠΊΠΈΠΌ Π΄Π²ΡƒΡ…ΠΏΠΎΠ»ΡŽΡΠ½ΠΈΠΊΠΎΠΌ.

Π‘ΠΎΠ»ΡŒΡˆΠΎΠ΅ Π²Π½ΡƒΡ‚Ρ€Π΅Π½Π½Π΅Π΅ сопротивлСниС

ΠŸΡŒΠ΅Π·ΠΎΡΠ»Π΅ΠΊΡ‚Ρ€ΠΈΡ‡Π΅ΡΠΊΠΈΠ΅ Π΄Π°Ρ‚Ρ‡ΠΈΠΊΠΈ, кондСнсаторныС ΠΌΠΈΠΊΡ€ΠΎΡ„ΠΎΠ½Ρ‹ ΠΈ Π΄Ρ€ΡƒΠ³ΠΈΠ΅ источники ΠΈΠΌΠΏΡƒΠ»ΡŒΡΠΎΠ² ΠΎΠ±Π»Π°Π΄Π°ΡŽΡ‚ ΠΏΠΎΠ²Ρ‹ΡˆΠ΅Π½Π½Ρ‹ΠΌ Π²Π½ΡƒΡ‚Ρ€Π΅Π½Π½ΠΈΠΌ импСдансом. Π§Ρ‚ΠΎΠ±Ρ‹ эффСктивно ΠΈΡΠΏΠΎΠ»ΡŒΠ·ΠΎΠ²Π°Ρ‚ΡŒ Ρ‚Π°ΠΊΠΈΠ΅ устройства, Π½ΡƒΠΆΠ½ΠΎ ΠΏΡ€Π°Π²ΠΈΠ»ΡŒΠ½ΠΎ ΡΠΎΠ³Π»Π°ΡΠΎΠ²Π°Ρ‚ΡŒ схСму считывания сигнала. ΠŸΡ€ΠΈ Π½Π΅ΡƒΠ΄Π°Ρ‡Π½ΠΎΠΌ согласовании Π½Π΅ΠΈΠ·Π±Π΅ΠΆΠ½Ρ‹ ΠΏΠΎΡ‚Π΅Ρ€ΠΈ.

Π’Π°ΠΆΠ½ΠΎ! Π£Π΄Π°Ρ‡Π½ΠΎΠ΅ согласованиС ΠΏΠΎ Π½Π°ΠΏΡ€ΡΠΆΠ΅Π½ΠΈΡŽ получаСтся ΠΏΡ€ΠΈ использовании для снятия сигнала устройства, с большим Π²Ρ…ΠΎΠ΄Π½Ρ‹ΠΌ сопротивлСниСм, Ρ‡Π΅ΠΌ Ρƒ источника сигнала. Π’ случаС высокоомного источника для считывания сигнала примСняСтся Π±ΡƒΡ„Π΅Ρ€Π½Ρ‹ΠΉ ΡƒΡΠΈΠ»ΠΈΡ‚Π΅Π»ΡŒ.

Π’Π½ΡƒΡ‚Ρ€Π΅Π½Π½Π΅Π΅ сопротивлСниС ΠΈ импСданс

ИмпСданс – ΠΏΠΎΠ»Π½ΠΎΠ΅ (комплСксноС) Π²Π½ΡƒΡ‚Ρ€Π΅Π½Π½Π΅Π΅ сопротивлСниС эквивалСнтного Π΄Π²ΡƒΡ…ΠΏΠΎΠ»ΡŽΡΠ½ΠΈΠΊΠ° ΠΏΠ΅Ρ€Π΅ΠΌΠ΅Π½Π½ΠΎΠΌΡƒ Ρ‚ΠΎΠΊΡƒ. ΠžΠ±ΠΎΠ·Π½Π°Ρ‡Π°Π΅Ρ‚ΡΡ Π±ΡƒΠΊΠ²ΠΎΠΉ Z ΠΈ Ρ‚Π°ΠΊ ΠΆΠ΅ измСряСтся Π² ΠžΠΌΠ°Ρ….

Π”Π²ΡƒΡ…ΠΏΠΎΠ»ΡŽΡΠ½ΠΈΠΊ ΠΈ Π΅Π³ΠΎ эквивалСнтная схСма

Π”Π²ΡƒΡ…ΠΏΠΎΠ»ΡŽΡΠ½ΠΈΠΊ прСдставляСт собой ΡΠ»Π΅ΠΊΡ‚Ρ€ΠΈΡ‡Π΅ΡΠΊΡƒΡŽ Ρ†Π΅ΠΏΡŒ, ΡΠΎΠ΄Π΅Ρ€ΠΆΠ°Ρ‰ΡƒΡŽ Π΄Π²Π΅ Ρ‚ΠΎΡ‡ΠΊΠΈ присоСдинСния ΠΊ Π΄Ρ€ΡƒΠ³ΠΈΠΌ цСпям. Π‘Ρ‹Π²Π°Π΅Ρ‚ Π΄Π²Π° Π²ΠΈΠ΄Π° элСктричСских Ρ†Π΅ΠΏΠ΅ΠΉ:

  • Ρ†Π΅ΠΏΠΈ, содСрТащиС источник Ρ‚ΠΎΠΊΠ° ΠΈΠ»ΠΈ напряТСния;
  • Π΄Π²ΡƒΡ…ΠΏΠΎΠ»ΡŽΡΠ½ΠΈΠΊΠΈ, Π½Π΅ ΡΠ²Π»ΡΡŽΡ‰ΠΈΠ΅ΡΡ источниками.

ΠŸΠ΅Ρ€Π²Ρ‹Π΅ Ρ…Π°Ρ€Π°ΠΊΡ‚Π΅Ρ€ΠΈΠ·ΡƒΡŽΡ‚ΡΡ элСктричСскими ΠΏΠ°Ρ€Π°ΠΌΠ΅Ρ‚Ρ€Π°ΠΌΠΈ: силой Ρ‚ΠΎΠΊΠ°, напряТСниСм ΠΈ импСдансом. Для расчёта ΠΏΠ°Ρ€Π°ΠΌΠ΅Ρ‚Ρ€ΠΎΠ² Ρ‚Π°ΠΊΠΈΡ… Π΄Π²ΡƒΡ…ΠΏΠΎΠ»ΡŽΡΠ½ΠΈΠΊΠΎΠ² ΠΏΡ€Π΅Π΄Π²Π°Ρ€ΠΈΡ‚Π΅Π»ΡŒΠ½ΠΎ производят Π·Π°ΠΌΠ΅Π½Ρƒ Ρ€Π΅Π°Π»ΡŒΠ½Ρ‹Ρ… элСмСнтов Ρ†Π΅ΠΏΠΈ Π½Π° ΠΈΠ΄Π΅Π°Π»ΡŒΠ½Ρ‹Π΅ элСмСнты. ΠšΠΎΠΌΠ±ΠΈΠ½Π°Ρ†ΠΈΡ, которая получаСтся Π² Ρ€Π΅Π·ΡƒΠ»ΡŒΡ‚Π°Ρ‚Π΅ ΠΏΠΎΠ΄ΠΎΠ±Π½ΠΎΠΉ Π·Π°ΠΌΠ΅Π½Ρ‹, называСтся эквивалСнтной схСмой.

Π’Π½ΠΈΠΌΠ°Π½ΠΈΠ΅! ΠŸΡ€ΠΈ Ρ€Π°Π±ΠΎΡ‚Π΅ со слоТными элСктричСскими схСмами с ΡƒΡ‡Ρ‘Ρ‚ΠΎΠΌ Ρ‚ΠΎΠ³ΠΎ, Ρ‡Ρ‚ΠΎ устройство Ρ€Π°Π±ΠΎΡ‚Π°Π΅Ρ‚ Π½Π° ΠΎΠ΄Π½ΠΎΠΉ частотС, допустимо ΠΏΡ€Π΅ΠΎΠ±Ρ€Π°Π·ΠΎΠ²Ρ‹Π²Π°Ρ‚ΡŒ ΠΏΠΎΡΠ»Π΅Π΄ΠΎΠ²Π°Ρ‚Π΅Π»ΡŒΠ½Ρ‹Π΅ ΠΈ ΠΏΠ°Ρ€Π°Π»Π»Π΅Π»ΡŒΠ½Ρ‹Π΅ Π²Π΅Ρ‚Π²ΠΈ Π΄ΠΎ получСния простой схСмы, доступной для расчёта ΠΏΠ°Ρ€Π°ΠΌΠ΅Ρ‚Ρ€ΠΎΠ².

Π’Ρ‚ΠΎΡ€ΠΎΠΉ Π²ΠΈΠ΄ Π΄Π²ΡƒΡ…ΠΏΠΎΠ»ΡŽΡΠ½ΠΈΠΊΠΎΠ² ΠΌΠΎΠΆΠ½ΠΎ ΠΎΡ…Π°Ρ€Π°ΠΊΡ‚Π΅Ρ€ΠΈΠ·ΠΎΠ²Π°Ρ‚ΡŒ Ρ‚ΠΎΠ»ΡŒΠΊΠΎ Π²Π΅Π»ΠΈΡ‡ΠΈΠ½ΠΎΠΉ Π²Π½ΡƒΡ‚Ρ€Π΅Π½Π½Π΅Π³ΠΎ сопротивлСния.

ВлияниС Π²Π½ΡƒΡ‚Ρ€Π΅Π½Π½Π΅Π³ΠΎ сопротивлСния Π½Π° свойства Π΄Π²ΡƒΡ…ΠΏΠΎΠ»ΡŽΡΠ½ΠΈΠΊΠ°

Π§Π΅ΠΌ ΠΎΠ½ΠΎ Π²Ρ‹ΡˆΠ΅, Ρ‚Π΅ΠΌ ΠΌΠ΅Π½ΡŒΡˆΡƒΡŽ ΠΌΠΎΡ‰Π½ΠΎΡΡ‚ΡŒ Π²Ρ‹Π΄Π°Ρ‘Ρ‚ источник ΠΏΡ€ΠΈ ΠΏΠΎΠ΄ΠΊΠ»ΡŽΡ‡Π΅Π½ΠΈΠΈ Π½Π°Π³Ρ€ΡƒΠ·ΠΊΠΈ. ΠžΠΏΡ€Π΅Π΄Π΅Π»ΠΈΡ‚ΡŒ ΠΌΠΎΡ‰Π½ΠΎΡΡ‚ΡŒ Π² Π½Π°Π³Ρ€ΡƒΠ·ΠΊΠ΅ ΠΌΠΎΠΆΠ½ΠΎ ΠΏΠΎ Ρ„ΠΎΡ€ΠΌΡƒΠ»Π΅:

Π³Π΄Π΅:

  • E – напряТСниС Π­Π”Π‘;
  • R – сопротивлСниС Π½Π°Π³Ρ€ΡƒΠ·ΠΊΠΈ;
  • r – Π°ΠΊΡ‚ΠΈΠ²Π½ΠΎΠ΅ Π²Π½ΡƒΡ‚Ρ€Π΅Π½Π½Π΅Π΅ сопротивлСниС Π΄Π²ΡƒΡ…ΠΏΠΎΠ»ΡŽΡΠ½ΠΈΠΊΠ°.

Π€ΠΎΡ€ΠΌΡƒΠ»Π° ΠΏΡ€ΠΈΠΌΠ΅Π½ΠΈΠΌΠ° ΠΊ Π΄Π²ΡƒΡ…ΠΏΠΎΠ»ΡŽΡΠ½ΠΈΠΊΠ°ΠΌ, Π½Π΅ ΠΎΡ‚Π΄Π°ΡŽΡ‰ΠΈΠΌ ΡΠ½Π΅Ρ€Π³ΠΈΡŽ.

К свСдСнию. Когда Π²Π΅Π»ΠΈΡ‡ΠΈΠ½Π° Π²Π½ΡƒΡ‚Ρ€Π΅Π½Π½Π΅Π³ΠΎ сопротивлСния Π΄Π²ΡƒΡ…ΠΏΠΎΠ»ΡŽΡΠ½ΠΈΠΊΠ° приблиТаСтся ΠΏΠΎ своСму Π·Π½Π°Ρ‡Π΅Π½ΠΈΡŽ ΠΊ ΡΠΎΠΏΡ€ΠΎΡ‚ΠΈΠ²Π»Π΅Π½ΠΈΡŽ Π½Π°Π³Ρ€ΡƒΠ·ΠΊΠΈ, ΠΏΠ΅Ρ€Π΅Π΄Π°Ρ‡Π° мощности достигаСт максимума.

Разрядная Π΅ΠΌΠΊΠΎΡΡ‚ΡŒ источника

Π’Π΅Π»ΠΈΡ‡ΠΈΠ½Π°, зависящая ΠΎΡ‚ силы Ρ‚ΠΎΠΊΠ° разряда, называСтся разрядной Ρ‘ΠΌΠΊΠΎΡΡ‚ΡŒΡŽ источника. Π­Ρ‚ΠΎ элСктричСский заряд, ΠΊΠΎΡ‚ΠΎΡ€Ρ‹ΠΉ ΠΎΡ‚Π΄Π°Ρ‘Ρ‚ источник Π² процСссС эксплуатации Π² зависимости ΠΎΡ‚ Ρ‚ΠΎΠΊΠ° Π½Π°Π³Ρ€ΡƒΠ·ΠΊΠΈ. Π­Ρ‚Ρƒ Π²Π΅Π»ΠΈΡ‡ΠΈΠ½Ρƒ ΠΌΠΎΠΆΠ½ΠΎ ΡΡ‡ΠΈΡ‚Π°Ρ‚ΡŒ постоянной условно. Π’Π°ΠΊ, стартСрный аккумулятор, ΠΈΠΌΠ΅ΡŽΡ‰ΠΈΠΉ Ρ€Π°Π·Ρ€ΡΠ΄Π½ΡƒΡŽ Ρ‘ΠΌΠΊΠΎΡΡ‚ΡŒ Π‘ = 55 А*Ρ‡, ΠΏΡ€ΠΈ Ρ‚ΠΎΠΊΠ΅ разряда 5,5 А ΠΏΡ€ΠΎΡ€Π°Π±ΠΎΡ‚Π°Π΅Ρ‚ 10 часов. ΠŸΡ€ΠΈ запусках Ρ…ΠΎΠ»ΠΎΠ΄Π½ΠΎΠ³ΠΎ ΠΈΠ»ΠΈ ΠΈΠΌΠ΅ΡŽΡ‰Π΅Π³ΠΎ Π½Π΅ΠΈΡΠΏΡ€Π°Π²Π½ΠΎΡΡ‚ΡŒ автомобиля аккумулятор ΠΌΠΎΠΆΠ½ΠΎ Ρ€Π°Π·Ρ€ΡΠ΄ΠΈΡ‚ΡŒ Π·Π° нСсколько ΠΌΠΈΠ½ΡƒΡ‚.

Для Ρ‚ΠΎΠ³ΠΎ Ρ‡Ρ‚ΠΎΠ±Ρ‹ Π½Π°ΠΉΡ‚ΠΈ ΠΎΡΡ‚Π°Ρ‚ΠΎΡ‡Π½ΡƒΡŽ Ρ€Π°Π·Ρ€ΡΠ΄Π½ΡƒΡŽ Ρ‘ΠΌΠΊΠΎΡΡ‚ΡŒ, производят Ρ†ΠΈΠΊΠ»Ρ‹ «заряд – разряд». Они Π²Ρ‹ΠΏΠΎΠ»Π½ΡΡŽΡ‚ΡΡ ΠΏΡ€ΠΈ ΠΏΠΎΠΌΠΎΡ‰ΠΈ Π½Π°Π³Ρ€ΡƒΠ·ΠΎΡ‡Π½Ρ‹Ρ… сопротивлСний. Разряд Π½Π° Π½Π°Π³Ρ€ΡƒΠ·ΠΎΡ‡Π½ΠΎΠ΅ сопротивлСниС производят Π΄ΠΎ минимально допустимых Π·Π½Π°Ρ‡Π΅Π½ΠΈΠΉ плотности элСктролита. ΠŸΡ€ΠΈ этом замСряСтся врСмя Ρ€Π°Π±ΠΎΡ‚Ρ‹ ΠΏΠΎΠ΄ Π½Π°Π³Ρ€ΡƒΠ·ΠΊΠΎΠΉ. Π­Ρ‚ΠΎ Π°ΠΊΡ‚ΡƒΠ°Π»ΡŒΠ½ΠΎ ΠΏΡ€ΠΈ сСзонном обслуТивании аккумуляторов для выявлСния процСссов саморазряда.

Π’Π½ΡƒΡ‚Ρ€Π΅Π½Π½Π΅Π΅ сопротивлСниС источников Ρ‚ΠΎΠΊΠ° – ваТная Π²Π΅Π»ΠΈΡ‡ΠΈΠ½Π°. ΠœΠ΅Ρ‚ΠΎΠ΄Ρ‹, примСняСмыС для Π΅Ρ‘ сниТСния, ΡΠ²Π»ΡΡŽΡ‚ΡΡ прямыми путями увСличСния ΠΎΡ‚Π΄Π°Π²Π°Π΅ΠΌΠΎΠΉ мощности источника, Π·Π½Π°Ρ‡ΠΈΡ‚, ΠΏΠΎΠ²Ρ‹ΡˆΠ΅Π½ΠΈΡ ΠΏΡ€ΠΎΠΈΠ·Π²ΠΎΠ΄ΠΈΡ‚Π΅Π»ΡŒΠ½ΠΎΡΡ‚ΠΈ Π΄Π²ΡƒΡ…ΠΏΠΎΠ»ΡŽΡΠ½ΠΈΠΊΠΎΠ². ΠŸΡ€Π°Π²ΠΈΠ»ΡŒΠ½ΠΎΠ΅ ΠΈΠ·ΠΌΠ΅Ρ€Π΅Π½ΠΈΠ΅ ΠΈ вычислСниС импСданса эквивалСнтных схСм ΠΏΠΎΠ·Π²ΠΎΠ»ΡΡŽΡ‚ ΠΏΡ€ΠΈΠ±Π»ΠΈΠ·ΠΈΡ‚ΡŒ Π΄Π²ΡƒΡ…ΠΏΠΎΠ»ΡŽΡΠ½ΠΈΠΊ ΠΊ ΠΈΠ΄Π΅Π°Π»ΡŒΠ½ΠΎΠΌΡƒ источнику.

Π’ΠΈΠ΄Π΅ΠΎ

Допустим, Π΅ΡΡ‚ΡŒ ΠΏΡ€ΠΎΡΡ‚Π΅ΠΉΡˆΠ°Ρ элСктричСская замкнутая Ρ†Π΅ΠΏΡŒ, Π²ΠΊΠ»ΡŽΡ‡Π°ΡŽΡ‰Π°Ρ Π² сСбя источник Ρ‚ΠΎΠΊΠ°, Π½Π°ΠΏΡ€ΠΈΠΌΠ΅Ρ€ Π³Π΅Π½Π΅Ρ€Π°Ρ‚ΠΎΡ€, Π³Π°Π»ΡŒΠ²Π°Π½ΠΈΡ‡Π΅ΡΠΊΠΈΠΉ элСмСнт ΠΈΠ»ΠΈ аккумулятор, ΠΈ рСзистор, ΠΎΠ±Π»Π°Π΄Π°ΡŽΡ‰ΠΈΠΉ сопротивлСниСм R. ΠŸΠΎΡΠΊΠΎΠ»ΡŒΠΊΡƒ Ρ‚ΠΎΠΊ Π² Ρ†Π΅ΠΏΠΈ Π½ΠΈΠ³Π΄Π΅ Π½Π΅ прСрываСтся, Ρ‚ΠΎ ΠΈ Π²Π½ΡƒΡ‚Ρ€ΠΈ источника ΠΎΠ½ Ρ‚Π΅Ρ‡Π΅Ρ‚.

Π’ Ρ‚Π°ΠΊΠΎΠΉ ситуации ΠΌΠΎΠΆΠ½ΠΎ ΡΠΊΠ°Π·Π°Ρ‚ΡŒ, Ρ‡Ρ‚ΠΎ любой источник ΠΎΠ±Π»Π°Π΄Π°Π΅Ρ‚ Π½Π΅ΠΊΠΎΡ‚ΠΎΡ€Ρ‹ΠΌ Π²Π½ΡƒΡ‚Ρ€Π΅Π½Π½ΠΈΠΌ сопротивлСниСм, ΠΏΡ€Π΅ΠΏΡΡ‚ΡΡ‚Π²ΡƒΡŽΡ‰ΠΈΠΌ Ρ‚ΠΎΠΊΡƒ. Π­Ρ‚ΠΎ Π²Π½ΡƒΡ‚Ρ€Π΅Π½Π½Π΅Π΅ сопротивлСниС Ρ…Π°Ρ€Π°ΠΊΡ‚Π΅Ρ€ΠΈΠ·ΡƒΠ΅Ρ‚ источник Ρ‚ΠΎΠΊΠ° ΠΈ обозначаСтся Π±ΡƒΠΊΠ²ΠΎΠΉ r. Для Π³Π°Π»ΡŒΠ²Π°Π½ΠΈΡ‡Π΅ΡΠΊΠΎΠ³ΠΎ элСмСнта ΠΈΠ»ΠΈ аккумулятора Π²Π½ΡƒΡ‚Ρ€Π΅Π½Π½Π΅Π΅ сопротивлСниС β€” это сопротивлСниС раствора элСктролита ΠΈ элСктродов, для Π³Π΅Π½Π΅Ρ€Π°Ρ‚ΠΎΡ€Π° β€” сопротивлСниС ΠΎΠ±ΠΌΠΎΡ‚ΠΎΠΊ статора ΠΈ Ρ‚. Π΄.

Π’Π°ΠΊΠΈΠΌ ΠΎΠ±Ρ€Π°Π·ΠΎΠΌ, источник Ρ‚ΠΎΠΊΠ° характСризуСтся ΠΊΠ°ΠΊ Π²Π΅Π»ΠΈΡ‡ΠΈΠ½ΠΎΠΉ Π­Π”Π‘, Ρ‚Π°ΠΊ ΠΈ Π²Π΅Π»ΠΈΡ‡ΠΈΠ½ΠΎΠΉ собствСнного Π²Π½ΡƒΡ‚Ρ€Π΅Π½Π½Π΅Π³ΠΎ сопротивлСния r – ΠΎΠ±Π΅ эти характСристики ΡΠ²ΠΈΠ΄Π΅Ρ‚Π΅Π»ΡŒΡΡ‚Π²ΡƒΡŽΡ‚ ΠΎ качСствС источника.

ЭлСктростатичСскиС Π²Ρ‹ΡΠΎΠΊΠΎΠ²ΠΎΠ»ΡŒΡ‚Π½Ρ‹Π΅ Π³Π΅Π½Π΅Ρ€Π°Ρ‚ΠΎΡ€Ρ‹ (ΠΊΠ°ΠΊ Π³Π΅Π½Π΅Ρ€Π°Ρ‚ΠΎΡ€ Π’Π°Π½ Π΄Π΅ Π“Ρ€Π°Π°Ρ„Π° ΠΈΠ»ΠΈ Π³Π΅Π½Π΅Ρ€Π°Ρ‚ΠΎΡ€ Π£ΠΈΠΌΡˆΡƒΡ€ΡΡ‚Π°), ΠΊ ΠΏΡ€ΠΈΠΌΠ΅Ρ€Ρƒ, ΠΎΡ‚Π»ΠΈΡ‡Π°ΡŽΡ‚ΡΡ ΠΎΠ³Ρ€ΠΎΠΌΠ½ΠΎΠΉ Π­Π”Π‘ измСряСмой ΠΌΠΈΠ»Π»ΠΈΠΎΠ½Π°ΠΌΠΈ Π²ΠΎΠ»ΡŒΡ‚, ΠΏΡ€ΠΈ этом ΠΈΡ… Π²Π½ΡƒΡ‚Ρ€Π΅Π½Π½Π΅Π΅ сопротивлСниС измСряСтся сотнями ΠΌΠ΅Π³Π°ΠΎΠΌ, ΠΏΠΎΡ‚ΠΎΠΌΡƒ ΠΎΠ½ΠΈ ΠΈ Π½Π΅ΠΏΡ€ΠΈΠ³ΠΎΠ΄Π½Ρ‹ для получСния Π±ΠΎΠ»ΡŒΡˆΠΈΡ… Ρ‚ΠΎΠΊΠΎΠ².

Π“Π°Π»ΡŒΠ²Π°Π½ΠΈΡ‡Π΅ΡΠΊΠΈΠ΅ элСмСнты (Ρ‚Π°ΠΊΠΈΠ΅ ΠΊΠ°ΠΊ Π±Π°Ρ‚Π°Ρ€Π΅ΠΉΠΊΠ°) β€” Π½Π°ΠΏΡ€ΠΎΡ‚ΠΈΠ² β€” ΠΈΠΌΠ΅ΡŽΡ‚ Π­Π”Π‘ порядка 1 Π²ΠΎΠ»ΡŒΡ‚Π°, хотя Π²Π½ΡƒΡ‚Ρ€Π΅Π½Π½Π΅Π΅ сопротивлСниС Ρƒ Π½ΠΈΡ… порядка Π΄ΠΎΠ»Π΅ΠΉ ΠΈΠ»ΠΈ максимум – дСсятка Ом, ΠΈ ΠΎΡ‚ Π³Π°Π»ΡŒΠ²Π°Π½ΠΈΡ‡Π΅ΡΠΊΠΈΡ… элСмСнтов поэтому ΠΌΠΎΠΆΠ½ΠΎ ΠΏΠΎΠ»ΡƒΡ‡Π°Ρ‚ΡŒ Ρ‚ΠΎΠΊΠΈ Π² Π΅Π΄ΠΈΠ½ΠΈΡ†Ρ‹ ΠΈ дСсятки Π°ΠΌΠΏΠ΅Ρ€.

На Π΄Π°Π½Π½ΠΎΠΉ схСмС ΠΏΠΎΠΊΠ°Π·Π°Π½ Ρ€Π΅Π°Π»ΡŒΠ½Ρ‹ΠΉ источник с присоСдинСнной Π½Π°Π³Ρ€ΡƒΠ·ΠΊΠΎΠΉ. Π—Π΄Π΅ΡΡŒ ΠΎΠ±ΠΎΠ·Π½Π°Ρ‡Π΅Π½Ρ‹ Π­Π”Π‘ источника, Π΅Π³ΠΎ Π²Π½ΡƒΡ‚Ρ€Π΅Π½Π½Π΅Π΅ сопротивлСниС, Π° Ρ‚Π°ΠΊΠΆΠ΅ сопротивлСниС Π½Π°Π³Ρ€ΡƒΠ·ΠΊΠΈ. Богласно Π·Π°ΠΊΠΎΠ½Ρƒ Ома для Π·Π°ΠΌΠΊΠ½ΡƒΡ‚ΠΎΠΉ Ρ†Π΅ΠΏΠΈ, Ρ‚ΠΎΠΊ Π² Π΄Π°Π½Π½ΠΎΠΉ Ρ†Π΅ΠΏΠΈ Π±ΡƒΠ΄Π΅Ρ‚ Ρ€Π°Π²Π΅Π½:

ΠŸΠΎΡΠΊΠΎΠ»ΡŒΠΊΡƒ участок внСшнСй Ρ†Π΅ΠΏΠΈ ΠΎΠ΄Π½ΠΎΡ€ΠΎΠ΄Π΅Π½, Ρ‚ΠΎ ΠΈΠ· Π·Π°ΠΊΠΎΠ½Π° Ома ΠΌΠΎΠΆΠ½ΠΎ Π½Π°ΠΉΡ‚ΠΈ напряТСниС Π½Π° Π½Π°Π³Ρ€ΡƒΠ·ΠΊΠ΅:

Π’Ρ‹Ρ€Π°Π·ΠΈΠ² ΠΈΠ· ΠΏΠ΅Ρ€Π²ΠΎΠ³ΠΎ уравнСния сопротивлСниС Π½Π°Π³Ρ€ΡƒΠ·ΠΊΠΈ, ΠΈ подставив Π΅Π³ΠΎ Π·Π½Π°Ρ‡Π΅Π½ΠΈΠ΅ Π²ΠΎ Π²Ρ‚ΠΎΡ€ΠΎΠ΅ ΡƒΡ€Π°Π²Π½Π΅Π½ΠΈΠ΅, ΠΏΠΎΠ»ΡƒΡ‡ΠΈΠΌ Π·Π°Π²ΠΈΡΠΈΠΌΠΎΡΡ‚ΡŒ напряТСния Π½Π° Π½Π°Π³Ρ€ΡƒΠ·ΠΊΠ΅ ΠΎΡ‚ Ρ‚ΠΎΠΊΠ° Π² Π·Π°ΠΌΠΊΠ½ΡƒΡ‚ΠΎΠΉ Ρ†Π΅ΠΏΠΈ:

Π’ Π·Π°ΠΌΠΊΠ½ΡƒΡ‚ΠΎΠΌ ΠΊΠΎΠ½Ρ‚ΡƒΡ€Π΅ Π­Π”Π‘ Ρ€Π°Π²Π½Π° суммС ΠΏΠ°Π΄Π΅Π½ΠΈΠΉ напряТСний Π½Π° элСмСнтах внСшнСй Ρ†Π΅ΠΏΠΈ ΠΈ Π½Π° Π²Π½ΡƒΡ‚Ρ€Π΅Π½Π½Π΅ΠΌ сопротивлСнии самого источника. Π—Π°Π²ΠΈΡΠΈΠΌΠΎΡΡ‚ΡŒ напряТСния Π½Π° Π½Π°Π³Ρ€ΡƒΠ·ΠΊΠ΅ ΠΎΡ‚ Ρ‚ΠΎΠΊΠ° Π½Π°Π³Ρ€ΡƒΠ·ΠΊΠΈ Π² идСальном случаС Π»ΠΈΠ½Π΅ΠΉΠ½Π°.

Π“Ρ€Π°Ρ„ΠΈΠΊ это ΠΏΠΎΠΊΠ°Π·Ρ‹Π²Π°Π΅Ρ‚, Π½ΠΎ ΡΠΊΡΠΏΠ΅Ρ€ΠΈΠΌΠ΅Π½Ρ‚Π°Π»ΡŒΠ½Ρ‹Π΅ Π΄Π°Π½Π½Ρ‹Π΅ Π½Π° Ρ€Π΅Π°Π»ΡŒΠ½ΠΎΠΌ рСзисторС (крСстики Π²ΠΎΠ·Π»Π΅ Π³Ρ€Π°Ρ„ΠΈΠΊΠ°) всСгда ΠΎΡ‚Π»ΠΈΡ‡Π°ΡŽΡ‚ΡΡ ΠΎΡ‚ ΠΈΠ΄Π΅Π°Π»Π°:

ЭкспСримСнты ΠΈ Π»ΠΎΠ³ΠΈΠΊΠ° ΠΏΠΎΠΊΠ°Π·Ρ‹Π²Π°ΡŽΡ‚, Ρ‡Ρ‚ΠΎ ΠΏΡ€ΠΈ Π½ΡƒΠ»Π΅Π²ΠΎΠΌ Ρ‚ΠΎΠΊΠ΅ Π½Π°Π³Ρ€ΡƒΠ·ΠΊΠΈ напряТСниС Π½Π° внСшнСй Ρ†Π΅ΠΏΠΈ Ρ€Π°Π²Π½ΠΎ Π­Π”Π‘ источника, Π° ΠΏΡ€ΠΈ Π½ΡƒΠ»Π΅Π²ΠΎΠΌ напряТСнии Π½Π° Π½Π°Π³Ρ€ΡƒΠ·ΠΊΠ΅ Ρ‚ΠΎΠΊ Π² Ρ†Π΅ΠΏΠΈ Ρ€Π°Π²Π΅Π½ Ρ‚ΠΎΠΊΡƒ ΠΊΠΎΡ€ΠΎΡ‚ΠΊΠΎΠ³ΠΎ замыкания. Π­Ρ‚ΠΎ свойство Ρ€Π΅Π°Π»ΡŒΠ½Ρ‹Ρ… Ρ†Π΅ΠΏΠ΅ΠΉ ΠΏΠΎΠΌΠΎΠ³Π°Π΅Ρ‚ ΡΠΊΡΠΏΠ΅Ρ€ΠΈΠΌΠ΅Π½Ρ‚Π°Π»ΡŒΠ½ΠΎ Π½Π°Ρ…ΠΎΠ΄ΠΈΡ‚ΡŒ Π­Π”Π‘ ΠΈ Π²Π½ΡƒΡ‚Ρ€Π΅Π½Π½Π΅Π΅ сопротивлСниС Ρ€Π΅Π°Π»ΡŒΠ½Ρ‹Ρ… источников.

Π­ΠΊΡΠΏΠ΅Ρ€ΠΈΠΌΠ΅Π½Ρ‚Π°Π»ΡŒΠ½ΠΎΠ΅ Π½Π°Ρ…ΠΎΠΆΠ΄Π΅Π½ΠΈΠ΅ Π²Π½ΡƒΡ‚Ρ€Π΅Π½Π½Π΅Π³ΠΎ сопротивлСния

Π§Ρ‚ΠΎΠ±Ρ‹ ΡΠΊΡΠΏΠ΅Ρ€ΠΈΠΌΠ΅Π½Ρ‚Π°Π»ΡŒΠ½ΠΎ ΠΎΠΏΡ€Π΅Π΄Π΅Π»ΠΈΡ‚ΡŒ Π΄Π°Π½Π½Ρ‹Π΅ характСристики, строят Π³Ρ€Π°Ρ„ΠΈΠΊ зависимости напряТСния Π½Π° Π½Π°Π³Ρ€ΡƒΠ·ΠΊΠ΅ ΠΎΡ‚ Π²Π΅Π»ΠΈΡ‡ΠΈΠ½Ρ‹ Ρ‚ΠΎΠΊΠ°, Π·Π°Ρ‚Π΅ΠΌ ΡΠΊΡΡ‚Ρ€Π°ΠΏΠΎΠ»ΠΈΡ€ΡƒΡŽΡ‚ Π΅Π³ΠΎ Π΄ΠΎ пСрСсСчСния с осями.

Π’ Ρ‚ΠΎΡ‡ΠΊΠ΅ пСрСсСчСния Π³Ρ€Π°Ρ„ΠΈΠΊΠ° с ΠΎΡΡ‚ΡŒΡŽ напряТСния находится Π·Π½Π°Ρ‡Π΅Π½ΠΈΠ΅ Π­Π”Π‘ источника, Π° Π² Ρ‚ΠΎΡ‡ΠΊΠ΅ пСрСсСчСния с осью Ρ‚ΠΎΠΊΠ° находится Π²Π΅Π»ΠΈΡ‡ΠΈΠ½Π° Ρ‚ΠΎΠΊΠ° ΠΊΠΎΡ€ΠΎΡ‚ΠΊΠΎΠ³ΠΎ замыкания. Π’ ΠΈΡ‚ΠΎΠ³Π΅ Π²Π½ΡƒΡ‚Ρ€Π΅Π½Π½Π΅Π΅ сопротивлСниС находится ΠΏΠΎ Ρ„ΠΎΡ€ΠΌΡƒΠ»Π΅:

РазвиваСмая источником полСзная ΠΌΠΎΡ‰Π½ΠΎΡΡ‚ΡŒ выдСляСтся Π½Π° Π½Π°Π³Ρ€ΡƒΠ·ΠΊΠ΅. Π“Ρ€Π°Ρ„ΠΈΠΊ зависимости этой мощности ΠΎΡ‚ сопротивлСния Π½Π°Π³Ρ€ΡƒΠ·ΠΊΠΈ ΠΏΡ€ΠΈΠ²Π΅Π΄Π΅Π½ Π½Π° рисункС. Π­Ρ‚Π° кривая начинаСтся ΠΎΡ‚ пСрСсСчСния осСй ΠΊΠΎΠΎΡ€Π΄ΠΈΠ½Π°Ρ‚ Π² Π½ΡƒΠ»Π΅Π²ΠΎΠΉ Ρ‚ΠΎΡ‡ΠΊΠ΅, Π·Π°Ρ‚Π΅ΠΌ возрастаСт Π΄ΠΎ максимального значСния мощности, послС Ρ‡Π΅Π³ΠΎ спадаСт Π΄ΠΎ нуля ΠΏΡ€ΠΈ сопротивлСнии Π½Π°Π³Ρ€ΡƒΠ·ΠΊΠΈ Ρ€Π°Π²Π½ΠΎΠΌ бСсконСчности.

Π§Ρ‚ΠΎΠ±Ρ‹ Π½Π°ΠΉΡ‚ΠΈ максимальноС сопротивлСниС Π½Π°Π³Ρ€ΡƒΠ·ΠΊΠΈ, ΠΏΡ€ΠΈ ΠΊΠΎΡ‚ΠΎΡ€ΠΎΠΌ тСорСтичСски Ρ€Π°Π·ΠΎΠ²ΡŒΠ΅Ρ‚ΡΡ максимальная ΠΌΠΎΡ‰Π½ΠΎΡΡ‚ΡŒ ΠΏΡ€ΠΈ Π΄Π°Π½Π½ΠΎΠΌ источникС, бСрСтся производная ΠΎΡ‚ Ρ„ΠΎΡ€ΠΌΡƒΠ»Ρ‹ мощности ΠΏΠΎ R ΠΈ приравниваСтся ΠΊ Π½ΡƒΠ»ΡŽ. Максимальная ΠΌΠΎΡ‰Π½ΠΎΡΡ‚ΡŒ Ρ€Π°Π·ΠΎΠ²ΡŒΠ΅Ρ‚ΡΡ ΠΏΡ€ΠΈ сопротивлСнии внСшнСй Ρ†Π΅ΠΏΠΈ, Ρ€Π°Π²Π½ΠΎΠΌ Π²Π½ΡƒΡ‚Ρ€Π΅Π½Π½Π΅ΠΌΡƒ ΡΠΎΠΏΡ€ΠΎΡ‚ΠΈΠ²Π»Π΅Π½ΠΈΡŽ источника:

Π­Ρ‚ΠΎ ΠΏΠΎΠ»ΠΎΠΆΠ΅Π½ΠΈΠ΅ ΠΎ максимальной мощности ΠΏΡ€ΠΈ R = r, позволяСт ΡΠΊΡΠΏΠ΅Ρ€ΠΈΠΌΠ΅Π½Ρ‚Π°Π»ΡŒΠ½ΠΎ Π½Π°ΠΉΡ‚ΠΈ Π²Π½ΡƒΡ‚Ρ€Π΅Π½Π½Π΅Π΅ сопротивлСниС источника, построив Π·Π°Π²ΠΈΡΠΈΠΌΠΎΡΡ‚ΡŒ мощности, выдСляСмой Π½Π° Π½Π°Π³Ρ€ΡƒΠ·ΠΊΠ΅, ΠΎΡ‚ Π²Π΅Π»ΠΈΡ‡ΠΈΠ½Ρ‹ сопротивлСния Π½Π°Π³Ρ€ΡƒΠ·ΠΊΠΈ. Найдя Ρ€Π΅Π°Π»ΡŒΠ½ΠΎΠ΅, Π° Π½Π΅ тСорСтичСскоС, сопротивлСниС Π½Π°Π³Ρ€ΡƒΠ·ΠΊΠΈ, ΠΎΠ±Π΅ΡΠΏΠ΅Ρ‡ΠΈΠ²Π°ΡŽΡ‰Π΅Π΅ ΠΌΠ°ΠΊΡΠΈΠΌΠ°Π»ΡŒΠ½ΡƒΡŽ ΠΌΠΎΡ‰Π½ΠΎΡΡ‚ΡŒ, ΠΎΠΏΡ€Π΅Π΄Π΅Π»ΡΡŽΡ‚ Ρ€Π΅Π°Π»ΡŒΠ½ΠΎΠ΅ Π²Π½ΡƒΡ‚Ρ€Π΅Π½Π½Π΅Π΅ сопротивлСниС источника питания.

ΠšΠŸΠ” источника Ρ‚ΠΎΠΊΠ° ΠΏΠΎΠΊΠ°Π·Ρ‹Π²Π°Π΅Ρ‚ ΠΎΡ‚Π½ΠΎΡˆΠ΅Π½ΠΈΠ΅ максимальной выдСляСмой Π½Π° Π½Π°Π³Ρ€ΡƒΠ·ΠΊΠ΅ мощности ΠΊ ΠΏΠΎΠ»Π½ΠΎΠΉ мощности, ΠΊΠΎΡ‚ΠΎΡ€ΡƒΡŽ Π² Π΄Π°Π½Π½Ρ‹ΠΉ ΠΌΠΎΠΌΠ΅Π½Ρ‚ Ρ€Π°Π·Π²ΠΈΠ²Π°Π΅Ρ‚ источник:

Ясно, Ρ‡Ρ‚ΠΎ Ссли источник Ρ€Π°Π·Π²ΠΈΠ²Π°Π΅Ρ‚ Ρ‚Π°ΠΊΡƒΡŽ ΠΌΠΎΡ‰Π½ΠΎΡΡ‚ΡŒ, Ρ‡Ρ‚ΠΎ Π½Π° Π½Π°Π³Ρ€ΡƒΠ·ΠΊΠ΅ получаСтся максимум Π²ΠΎΠ·ΠΌΠΎΠΆΠ½ΠΎΠΉ мощности для Π΄Π°Π½Π½ΠΎΠ³ΠΎ источника, Ρ‚ΠΎ ΠšΠŸΠ” источника окаТСтся Ρ€Π°Π²Π½Ρ‹ΠΌ 50%.

Как Π½Π°ΠΉΡ‚ΠΈ Π²Π½ΡƒΡ‚Ρ€Π΅Π½Π½Π΅Π΅ сопротивлСниС Ρ„ΠΎΡ€ΠΌΡƒΠ»Π° Π² Ρ„ΠΈΠ·ΠΈΠΊΠ΅. ЭлСктродвиТущая сила. Π’Π½ΡƒΡ‚Ρ€Π΅Π½Π½Π΅Π΅ сопротивлСниС источника Ρ‚ΠΎΠΊΠ°

ЦСль Ρ€Π°Π±ΠΎΡ‚Ρ‹: ΠΈΠ·ΡƒΡ‡ΠΈΡ‚ΡŒ ΠΌΠ΅Ρ‚ΠΎΠ΄ измСрСния Π­Π”Π‘ ΠΈ Π²Π½ΡƒΡ‚Ρ€Π΅Π½Π½Π΅Π³ΠΎ сопротивлСния источника Ρ‚ΠΎΠΊΠ° с ΠΏΠΎΠΌΠΎΡ‰ΡŒΡŽ Π°ΠΌΠΏΠ΅Ρ€ΠΌΠ΅Ρ‚Ρ€Π° ΠΈ Π²ΠΎΠ»ΡŒΡ‚ΠΌΠ΅Ρ‚Ρ€Π°.

ΠžΠ±ΠΎΡ€ΡƒΠ΄ΠΎΠ²Π°Π½ΠΈΠ΅: мСталличСский ΠΏΠ»Π°Π½ΡˆΠ΅Ρ‚, источник Ρ‚ΠΎΠΊΠ°, Π°ΠΌΠΏΠ΅Ρ€ΠΌΠ΅Ρ‚Ρ€, Π²ΠΎΠ»ΡŒΡ‚ΠΌΠ΅Ρ‚Ρ€, рСзистор, ΠΊΠ»ΡŽΡ‡, Π·Π°ΠΆΠΈΠΌΡ‹, ΡΠΎΠ΅Π΄ΠΈΠ½ΠΈΡ‚Π΅Π»ΡŒΠ½Ρ‹Π΅ ΠΏΡ€ΠΎΠ²ΠΎΠ΄Π°.

Для измСрСния Π­Π”Π‘ ΠΈ Π²Π½ΡƒΡ‚Ρ€Π΅Π½Π½Π΅Π³ΠΎ сопротивлСния источника Ρ‚ΠΎΠΊΠ° ΡΠΎΠ±ΠΈΡ€Π°ΡŽΡ‚ ΡΠ»Π΅ΠΊΡ‚Ρ€ΠΈΡ‡Π΅ΡΠΊΡƒΡŽ Ρ†Π΅ΠΏΡŒ, схСма ΠΊΠΎΡ‚ΠΎΡ€ΠΎΠΉ ΠΏΠΎΠΊΠ°Π·Π°Π½Π° Π½Π° рисункС 1.

К источнику Ρ‚ΠΎΠΊΠ° ΠΏΠΎΠ΄ΠΊΠ»ΡŽΡ‡Π°ΡŽΡ‚ Π°ΠΌΠΏΠ΅Ρ€ΠΌΠ΅Ρ‚Ρ€, сопротивлСниС ΠΈ ΠΊΠ»ΡŽΡ‡, соСдинСнныС ΠΏΠΎΡΠ»Π΅Π΄ΠΎΠ²Π°Ρ‚Π΅Π»ΡŒΠ½ΠΎ. ΠšΡ€ΠΎΠΌΠ΅ Ρ‚ΠΎΠ³ΠΎ, нСпосрСдствСн­но ΠΊ Π²Ρ‹Ρ…ΠΎΠ΄Π½Ρ‹ΠΌ Π³Π½Π΅Π·Π΄Π°ΠΌ источника ΠΏΠΎΠ΄ΠΊΠ»ΡŽΡ‡Π°ΡŽΡ‚ Π΅Ρ‰Π΅ ΠΈ Π²ΠΎΠ»ΡŒΡ‚ΠΌΠ΅Ρ‚Ρ€.

Π­Π”Π‘ ΠΈΠ·ΠΌΠ΅Ρ€ΡΡŽΡ‚ ΠΏΠΎ показанию Π²ΠΎΠ»ΡŒΡ‚ΠΌΠ΅Ρ‚Ρ€Π° ΠΏΡ€ΠΈ Ρ€Π°Π·ΠΎΠΌΠΊΠ½ΡƒΡ‚ΠΎΠΌ ΠΊΠ»ΡŽΡ‡Π΅. Π­Ρ‚ΠΎΡ‚ ΠΏΡ€ΠΈΠ΅ΠΌ опрСдСлСния Π­Π”Π‘ основан Π½Π° слСдствии ΠΈΠ· Π·Π°Β­ΠΊΠΎΠ½Π° Ома для ΠΏΠΎΠ»Π½ΠΎΠΉ Ρ†Π΅ΠΏΠΈ, согласно ΠΊΠΎΡ‚ΠΎΡ€ΠΎΠΌΡƒ ΠΏΡ€ΠΈ бСсконСчно большом сопротивлСнии внСшнСй Ρ†Π΅ΠΏΠΈ напряТСниС Π½Π° Π·Π°ΠΆΠΈΠΌΠ°Ρ… источника Ρ€Π°Π²Π½ΠΎ Π΅Π³ΠΎ Π­Π”Π‘. (Π‘ΠΌ. ΠΏΠ°Ρ€Π°Π³Ρ€Π°Ρ„ «Π—Π°ΠΊΠΎΠ½ Ома для ΠΏΠΎΠ»Π½ΠΎΠΉ Ρ†Π΅ΠΏΠΈ» ΡƒΡ‡Π΅Π±Π½ΠΈΠΊΠ° «Π€ΠΈΠ·ΠΈΠΊΠ° 10»).

Для опрСдСлСния Π²Π½ΡƒΡ‚Ρ€Π΅Π½Π½Π΅Π³ΠΎ сопротивлСния источника Π·Π°Β­ΠΌΡ‹ΠΊΠ°ΡŽΡ‚ ΠΊΠ»ΡŽΡ‡ К. ΠŸΡ€ΠΈ этом Π² Ρ†Π΅ΠΏΠΈ ΠΌΠΎΠΆΠ½ΠΎ условно Π²Ρ‹Π΄Π΅Π»ΠΈΡ‚ΡŒ Π΄Π²Π° участка: внСшний (Ρ‚ΠΎΡ‚, ΠΊΠΎΡ‚ΠΎΡ€Ρ‹ΠΉ ΠΏΠΎΠ΄ΠΊΠ»ΡŽΡ‡Π΅Π½ ΠΊ источнику) ΠΈ Π²Π½ΡƒΡ‚Ρ€Π΅Π½Π½ΠΈΠΉ (Ρ‚ΠΎΡ‚, ΠΊΠΎΡ‚ΠΎΡ€Ρ‹ΠΉ находится Π²Π½ΡƒΡ‚Ρ€ΠΈ источника Ρ‚ΠΎΠΊΠ°). ΠŸΠΎΡΠΊΠΎΠ»ΡŒΠΊΡƒ Π­Π”Π‘ источника Ρ€Π°Π²Π½Π° суммС падСния напряТСний Π½Π° Π²Π½ΡƒΡ‚Ρ€Π΅Π½Β­Π½Π΅ΠΌ ΠΈ внСшнСм участках Ρ†Π΅ΠΏΠΈ:

Ξ΅ = U r +U R , Ρ‚ΠΎ U r = Ξ΅ -U R (1)

По Π·Π°ΠΊΠΎΠ½Ρƒ Ома для участка Ρ†Π΅ΠΏΠΈ U r = IΒ· r (2). ΠŸΠΎΠ΄ΡΡ‚Π°Π²ΠΈΠ² равСнство (2) Π² (1) ΠΏΠΎΠ»ΡƒΡ‡Π°ΡŽΡ‚:

I Β· r = Ξ΅ U r , ΠΎΡ‚ΠΊΡƒΠ΄Π° r = (Ξ΅ U R )/ J

Π‘Π»Π΅Π΄ΠΎΠ²Π°Ρ‚Π΅Π»ΡŒΠ½ΠΎ, Ρ‡Ρ‚ΠΎΠ±Ρ‹ ΡƒΠ·Π½Π°Ρ‚ΡŒ Π²Π½ΡƒΡ‚Ρ€Π΅Π½Π½Π΅Π΅ сопротивлСниС источника Ρ‚ΠΎΠΊΠ°, Π½Π΅ΠΎΠ±Ρ…ΠΎΠ΄ΠΈΠΌΠΎ ΠΏΡ€Π΅Π΄Β­Π²Π°Ρ€ΠΈΡ‚Π΅Π»ΡŒΠ½ΠΎ ΠΎΠΏΡ€Π΅Π΄Π΅Π»ΠΈΡ‚ΡŒ Π΅Π³ΠΎ Π­Π”Π‘, Π·Π°Ρ‚Π΅ΠΌ Π·Π°ΠΌΠΊΠ½ΡƒΡ‚ΡŒ ΠΊΠ»ΡŽΡ‡ ΠΈ ΠΈΠ·ΠΌΠ΅Ρ€ΠΈΡ‚ΡŒ ΠΏΠ°Π΄Π΅Π½ΠΈΠ΅ напряТСния Π½Π° внСш­нСм сопротивлСнии, Π° Ρ‚Π°ΠΊΠΆΠ΅ силу Ρ‚ΠΎΠΊΠ° Π² Π½Π΅ΠΌ.

Π₯ΠΎΠ΄ Ρ€Π°Π±ΠΎΡ‚Ρ‹

1. ΠŸΠΎΠ΄Π³ΠΎΡ‚ΠΎΠ²ΡŒΡ‚Π΅ Ρ‚Π°Π±Π»ΠΈΡ†Ρƒ для записи Ρ€Π΅Π·ΡƒΠ»ΡŒΡ‚Π°Ρ‚ΠΎΠ² ΠΈΠ·ΠΌΠ΅Ρ€Π΅Π½ΠΈΠΉ ΠΈ вычислСний:

Ξ΅ ,Π²

U r , B

i,a

r , Ом

    НачСртитС Π² Ρ‚Π΅Ρ‚Ρ€Π°Π΄ΠΈ схСму для измСрСния Π­Π”Π‘ ΠΈ Π²Π½ΡƒΡ‚Ρ€Π΅Π½Π½Π΅Π³ΠΎ сопротивлСния источника.

    ПослС ΠΏΡ€ΠΎΠ²Π΅Ρ€ΠΊΠΈ схСмы собСритС ΡΠ»Π΅ΠΊΡ‚Ρ€ΠΈΡ‡Π΅ΡΠΊΡƒΡŽ Ρ†Π΅ΠΏΡŒ. ΠšΠ»ΡŽΡ‡ Ρ€Π°Π·ΠΎΠΌΠΊΠ½ΠΈΡ‚Π΅.

    Π˜Π·ΠΌΠ΅Ρ€ΡŒΡ‚Π΅ Π²Π΅Π»ΠΈΡ‡ΠΈΠ½Ρƒ Π­Π”Π‘ источника.

    Π—Π°ΠΌΠΊΠ½ΠΈΡ‚Π΅ ΠΊΠ»ΡŽΡ‡ ΠΈ ΠΎΠΏΡ€Π΅Π΄Π΅Π»ΠΈΡ‚Π΅ показания Π°ΠΌΠΏΠ΅Ρ€ΠΌΠ΅Ρ‚Ρ€Π° ΠΈ Π²ΠΎΠ»ΡŒΡ‚ΠΌΠ΅Ρ‚Ρ€Π°.

    ВычислитС Π²Π½ΡƒΡ‚Ρ€Π΅Π½Π½Π΅Π΅ сопротивлСниС источника.

  1. ΠžΠΏΡ€Π΅Π΄Π΅Π»Π΅Π½ΠΈΠ΅ эдс ΠΈ Π²Π½ΡƒΡ‚Ρ€Π΅Π½Π½Π΅Π³ΠΎ сопротивлСния источника Ρ‚ΠΎΠΊΠ° графичСским ΠΌΠ΅Ρ‚ΠΎΠ΄ΠΎΠΌ

ЦСль Ρ€Π°Π±ΠΎΡ‚Ρ‹: ΠΈΠ·ΡƒΡ‡ΠΈΡ‚ΡŒ измСрСния Π­Π”Π‘, Π²Π½ΡƒΡ‚Ρ€Π΅Π½Π½Π΅Π³ΠΎ сопротивлСния ΠΈ Ρ‚ΠΎΠΊΠ° ΠΊΠΎΡ€ΠΎΡ‚ΠΊΠΎΠ³ΠΎ замы­кания источника Ρ‚ΠΎΠΊΠ°, основанный Π½Π° Π°Π½Π°Π»ΠΈΠ·Π΅ Π³Ρ€Π°Ρ„ΠΈΠΊΠ° зависимости напряТС­ния Π½Π° Π²Ρ‹Ρ…ΠΎΠ΄Π΅ источника ΠΎΡ‚ силы Ρ‚ΠΎΠΊΠ° Π² Ρ†Π΅ΠΏΠΈ.

ΠžΠ±ΠΎΡ€ΡƒΠ΄ΠΎΠ²Π°Π½ΠΈΠ΅: Π³Π°Π»ΡŒΠ²Π°Π½ΠΈΡ‡Π΅ΡΠΊΠΈΠΉ элСмСнт, Π°ΠΌΠΏΠ΅Ρ€ΠΌΠ΅Ρ‚Ρ€, Π²ΠΎΠ»ΡŒΡ‚ΠΌΠ΅Ρ‚Ρ€, рСзистор R 1 , ΠΏΠ΅Ρ€Π΅ΠΌΠ΅Π½Π½Ρ‹ΠΉ рСзистор, ΠΊΠ»ΡŽΡ‡, Π·Π°ΠΆΠΈΠΌΡ‹, мСталличСский ΠΏΠ»Π°Π½ΡˆΠ΅Ρ‚, ΡΠΎΠ΅Π΄ΠΈΠ½ΠΈΡ‚Π΅Π»ΡŒΠ½Ρ‹Π΅ ΠΏΡ€ΠΎΠ²ΠΎΠ΄Π°.

Из Π·Π°ΠΊΠΎΠ½Π° Ома для ΠΏΠΎΠ»Π½ΠΎΠΉ Ρ†Π΅ΠΏΠΈ слСдуСт, Ρ‡Ρ‚ΠΎ напряТСниС Π½Π° Π²Ρ‹Ρ…ΠΎΠ΄Π΅ источника Ρ‚ΠΎΠΊΠ° зависит прямо ΠΏΡ€ΠΎΠΏΠΎΡ€Ρ†ΠΈΠΎΠ½Π°Π»ΡŒΠ½ΠΎ ΠΎΡ‚ силы Ρ‚ΠΎΠΊΠ° Π² Ρ†Π΅ΠΏΠΈ:

Ρ‚Π°ΠΊ ΠΊΠ°ΠΊ I =E/(R+r), Ρ‚ΠΎ IR + Ir = Π•, Π½ΠΎ IR = U, ΠΎΡ‚ΠΊΡƒΠ΄Π° U + Ir = Π• ΠΈΠ»ΠΈ U = Π• – Ir (1).

Если ΠΏΠΎΡΡ‚Ρ€ΠΎΠΈΡ‚ΡŒ Π³Ρ€Π°Ρ„ΠΈΠΊ зависимости U ΠΎΡ‚ I, Ρ‚ΠΎ ΠΏΠΎ Π΅Π³ΠΎ Ρ‚ΠΎΡ‡ΠΊΠ°ΠΌ пСрСсСчСния с осями ΠΊΠΎΠΎΡ€Π΄ΠΈΠ½Π°Ρ‚ ΠΌΠΎΠΆΠ½ΠΎ ΠΎΠΏΡ€Π΅Π΄Π΅Π»ΠΈΡ‚ΡŒ Π•, I К.Π—. — силу Ρ‚ΠΎΠΊΠ° ΠΊΠΎΡ€ΠΎΡ‚ΠΊΠΎΠ³ΠΎ замыкания (Ρ‚ΠΎΠΊ, ΠΊΠΎΡ‚ΠΎΡ€Ρ‹ΠΉ ΠΏΠΎΡ‚Π΅Ρ‡Π΅Ρ‚ Π² Ρ†Π΅ΠΏΠΈ источни­ка, ΠΊΠΎΠ³Π΄Π° внСшнСС сопротивлСниС R станСт Ρ€Π°Π²Π½Ρ‹ΠΌ Π½ΡƒΠ»ΡŽ).

Π­Π”Π‘ ΠΎΠΏΡ€Π΅Π΄Π΅Π»ΡΡŽΡ‚ ΠΏΠΎ Ρ‚ΠΎΡ‡ΠΊΠ΅ пСрСсСчСния Π³Ρ€Π°Ρ„ΠΈΠΊΠ° с осью напряТСний. Π­Ρ‚Π° Ρ‚ΠΎΡ‡ΠΊΠ° Π³Ρ€Π°Ρ„ΠΈΠΊΠ° со­отвСтствуСт ΡΠΎΡΡ‚ΠΎΡΠ½ΠΈΡŽ Ρ†Π΅ΠΏΠΈ, ΠΏΡ€ΠΈ ΠΊΠΎΡ‚ΠΎΡ€ΠΎΠΌ Ρ‚ΠΎΠΊ Π² Π½Π΅ΠΉ отсутствуСт ΠΈ, ΡΠ»Π΅Π΄ΠΎΠ²Π°Ρ‚Π΅Π»ΡŒΠ½ΠΎ, U = Π•.

Π‘ΠΈΠ»Ρƒ Ρ‚ΠΎΠΊΠ° ΠΊΠΎΡ€ΠΎΡ‚ΠΊΠΎΠ³ΠΎ замыкания ΠΎΠΏΡ€Π΅Π΄Π΅Π»ΡΡŽΡ‚ ΠΏΠΎ Ρ‚ΠΎΡ‡ΠΊΠ΅ пСрСсСчСния Π³Ρ€Π°Ρ„ΠΈΠΊΠ° с осью Ρ‚ΠΎΠΊΠΎΠ². Π’ этом случаС внСшнСС сопротивлСниС R = 0 ΠΈ, ΡΠ»Π΅Π΄ΠΎΠ²Π°Ρ‚Π΅Π»ΡŒΠ½ΠΎ, напряТСниС Π½Π° Π²Ρ‹Ρ…ΠΎΠ΄Π΅ источника U = 0.

Π’Π½ΡƒΡ‚Ρ€Π΅Π½Π½Π΅Π΅ сопротивлСниС источника находят ΠΏΠΎ тангСнсу ΡƒΠ³Π»Π° Π½Π°ΠΊΠ»ΠΎΠ½Π° Π³Ρ€Π°Ρ„ΠΈΠΊΠ° ΠΎΡ‚Π½ΠΎΡΠΈΒ­Ρ‚Π΅Π»ΡŒΠ½ΠΎ оси Ρ‚ΠΎΠΊΠΎΠ². (Π‘Ρ€Π°Π²Π½ΠΈΡ‚Π΅ Ρ„ΠΎΡ€ΠΌΡƒΠ»Ρƒ (1) с матСматичСской Ρ„ΡƒΠ½ΠΊΡ†ΠΈΠ΅ΠΉ Π²ΠΈΠ΄Π° Π£ = АΠ₯ +Π’ ΠΈ вспомни­тС смысл коэффициСнта ΠΏΡ€ΠΈ X).

Π₯ΠΎΠ΄ Ρ€Π°Π±ΠΎΡ‚Ρ‹

    Для записи Ρ€Π΅Π·ΡƒΠ»ΡŒΡ‚Π°Ρ‚ΠΎΠ² ΠΈΠ·ΠΌΠ΅Ρ€Π΅Π½ΠΈΠΉ ΠΏΠΎΠ΄Π³ΠΎΡ‚ΠΎΠ²ΡŒΡ‚Π΅ Ρ‚Π°Π±Π»ΠΈΡ†Ρƒ:

  1. ПослС ΠΏΡ€ΠΎΠ²Π΅Ρ€ΠΊΠΈ схСмы ΠΏΡ€Π΅ΠΏΠΎΠ΄Π°Π²Π°Ρ‚Π΅Π»Π΅ΠΌ собСритС ΡΠ»Π΅ΠΊΡ‚Ρ€ΠΈΡ‡Π΅ΡΠΊΡƒΡŽ Ρ†Π΅ΠΏΡŒ. ΠŸΠΎΠ»Π·ΡƒΠ½ΠΎΠΊ ΠΏΠ΅Ρ€Π΅ΠΌΠ΅Π½Π½ΠΎΠ³ΠΎ рСзистора установитС Π² ΠΏΠΎΠ»ΠΎΠΆΠ΅Π½ΠΈΠ΅, ΠΏΡ€ΠΈ ΠΊΠΎΡ‚ΠΎΡ€ΠΎΠΌ сопротивлСниС Ρ†Π΅ΠΏΠΈ, ΠΏΠΎΠ΄ΠΊΠ»ΡŽΡ‡Π΅Π½Π½ΠΎΠΉ ΠΊ источ­нику Ρ‚ΠΎΠΊΠ°, Π±ΡƒΠ΄Π΅Ρ‚ ΠΌΠ°ΠΊΡΠΈΠΌΠ°Π»ΡŒΠ½Ρ‹ΠΌ.
  2. ΠžΠΏΡ€Π΅Π΄Π΅Π»ΠΈΡ‚Π΅ Π·Π½Π°Ρ‡Π΅Π½ΠΈΠ΅ силы Ρ‚ΠΎΠΊΠ° Π² Ρ†Π΅ΠΏΠΈ ΠΈ напряТСниС Π½Π° Π·Π°ΠΆΠΈΠΌΠ°Ρ… источника ΠΏΡ€ΠΈ максимальной Π²Π΅Π»ΠΈΡ‡ΠΈΠ½Π΅ сопротивлСния ΠΏΠ΅Ρ€Π΅ΠΌΠ΅Π½Π½ΠΎΠ³ΠΎ рСзистора. Π”Π°Π½Π½Ρ‹Π΅ ΠΈΠ·ΠΌΠ΅Ρ€Π΅Π½ΠΈΠΉ занСситС Π² Ρ‚Π°Π±Π»ΠΈΡ†Ρƒ.

    ΠŸΠΎΠ²Ρ‚ΠΎΡ€ΠΈΡ‚Π΅ нСсколько Ρ€Π°Π· измСрСния силы Ρ‚ΠΎΠΊΠ° ΠΈ напряТСния, ΡƒΠΌΠ΅Π½ΡŒΡˆΠ°Ρ всякий Ρ€Π°Π· Π²Π΅Π»ΠΈΡ‡ΠΈΠ½Ρƒ ΠΏΠ΅Ρ€Π΅ΠΌΠ΅Π½Π½ΠΎΠ³ΠΎ сопротивлСния Ρ‚Π°ΠΊ, Ρ‡Ρ‚ΠΎΠ±Ρ‹ напряТСниС Π½Π° Π·Π°ΠΆΠΈΠΌΠ°Ρ… источника ΡƒΠΌΠ΅Π½ΡŒΡˆΠ°Π»ΠΎΡΡŒ Π½Π° 0,1Π’. Π˜Π·ΠΌΠ΅Ρ€Π΅Π½ΠΈΡ ΠΏΡ€Π΅ΠΊΡ€Π°Ρ‚ΠΈΡ‚Π΅, ΠΊΠΎΠ³Π΄Π° сила Ρ‚ΠΎΠΊΠ° Π² Ρ†Π΅ΠΏΠΈ достигнСт значСния Π² 1А.

    НанСситС ΠΏΠΎΠ»ΡƒΡ‡Π΅Π½Π½Ρ‹Π΅ Π² экспСримСнтС Ρ‚ΠΎΡ‡ΠΊΠΈ Π½Π° Π³Ρ€Π°Ρ„ΠΈΠΊ. НапряТСниС ΠΎΡ‚ΠΊΠ»Π°Π΄Ρ‹Π²Π°ΠΉΡ‚Π΅ ΠΏΠΎ Π²Π΅Ρ€Ρ‚ΠΈΒ­ΠΊΠ°Π»ΡŒΠ½ΠΎΠΉ оси, Π° силу Ρ‚ΠΎΠΊΠ° — ΠΏΠΎ Π³ΠΎΡ€ΠΈΠ·ΠΎΠ½Ρ‚Π°Π»ΡŒΠ½ΠΎΠΉ. ΠŸΡ€ΠΎΠ²Π΅Π΄ΠΈΡ‚Π΅ ΠΏΠΎ Ρ‚ΠΎΡ‡ΠΊΠ°ΠΌ ΠΏΡ€ΡΠΌΡƒΡŽ линию.

    ΠŸΡ€ΠΎΠ΄ΠΎΠ»ΠΆΠΈΡ‚Π΅ Π³Ρ€Π°Ρ„ΠΈΠΊ Π΄ΠΎ пСрСсСчСния с осями ΠΊΠΎΠΎΡ€Π΄ΠΈΠ½Π°Ρ‚ ΠΈ ΠΎΠΏΡ€Π΅Π΄Π΅Π»ΠΈΡ‚Π΅ Π²Π΅Π»ΠΈΡ‡ΠΈΠ½Ρ‹ Π• ΠΈ, I К.Π—.

    Π˜Π·ΠΌΠ΅Ρ€ΡŒΡ‚Π΅ Π­Π”Π‘ источника, ΠΏΠΎΠ΄ΠΊΠ»ΡŽΡ‡ΠΈΠ² Π²ΠΎΠ»ΡŒΡ‚ΠΌΠ΅Ρ‚Ρ€ ΠΊ Π΅Π³ΠΎ Π²Ρ‹Π²ΠΎΠ΄Π°ΠΌ ΠΏΡ€ΠΈ Ρ€Π°Π·ΠΎΠΌΠΊΠ½ΡƒΡ‚ΠΎΠΉ внСшнСй Ρ†Π΅Β­ΠΏΠΈ. Π‘ΠΎΠΏΠΎΡΡ‚Π°Π²ΡŒΡ‚Π΅ значСния Π­Π”Π‘, ΠΏΠΎΠ»ΡƒΡ‡Π΅Π½Π½Ρ‹Π΅ двумя способами, ΠΈ ΡƒΠΊΠ°ΠΆΠΈΡ‚Π΅ ΠΏΡ€ΠΈΡ‡ΠΈΠ½Ρƒ Π²ΠΎΠ·ΠΌΠΎΠΆΠ½ΠΎΠ³ΠΎ расхоТдСния Ρ€Π΅Π·ΡƒΠ»ΡŒΡ‚Π°Ρ‚ΠΎΠ².

    ΠžΠΏΡ€Π΅Π΄Π΅Π»ΠΈΡ‚Π΅ Π²Π½ΡƒΡ‚Ρ€Π΅Π½Π½Π΅Π΅ сопротивлСниС источника Ρ‚ΠΎΠΊΠ°. Для этого вычислитС тангСнс ΡƒΠ³Π»Π° Π½Π°Β­ΠΊΠ»ΠΎΠ½Π° построСнного Π³Ρ€Π°Ρ„ΠΈΠΊΠ° ΠΊ оси Ρ‚ΠΎΠΊΠΎΠ². Π’Π°ΠΊ ΠΊΠ°ΠΊ тангСнс ΡƒΠ³Π»Π° Π² ΠΏΡ€ΡΠΌΠΎΡƒΠ³ΠΎΠ»ΡŒΠ½ΠΎΠΌ Ρ‚Ρ€Π΅ΡƒΠ³ΠΎΠ»ΡŒΠ½ΠΈΠΊΠ΅ Ρ€Π°Π²Π΅Π½ ΠΎΡ‚Π½ΠΎΡˆΠ΅Π½ΠΈΡŽ ΠΏΡ€ΠΎΡ‚ΠΈΠ²ΠΎΠ»Π΅ΠΆΠ°Ρ‰Π΅Π³ΠΎ ΠΊΠ°Ρ‚Π΅Ρ‚Π° ΠΊ ΠΏΡ€ΠΈΠ»Π΅ΠΆΠ°Ρ‰Π΅ΠΌΡƒ, Ρ‚ΠΎ практичСски это ΠΌΠΎΠΆΠ½ΠΎ ΡΠ΄Π΅Π»Π°Ρ‚ΡŒ, найдя ΠΎΡ‚Π½ΠΎΡˆΠ΅Π½ΠΈΠ΅ Π• / I К.Π—

На ΠΊΠΎΠ½Ρ†Π°Ρ… ΠΏΡ€ΠΎΠ²ΠΎΠ΄Π½ΠΈΠΊΠ°, Π° Π·Π½Π°Ρ‡ΠΈΡ‚, ΠΈ Ρ‚ΠΎΠΊΠ° Π½Π΅ΠΎΠ±Ρ…ΠΎΠ΄ΠΈΠΌΠΎ Π½Π°Π»ΠΈΡ‡ΠΈΠ΅ сторонних сил нСэлСктричСской ΠΏΡ€ΠΈΡ€ΠΎΠ΄Ρ‹, с ΠΏΠΎΠΌΠΎΡ‰ΡŒΡŽ ΠΊΠΎΡ‚ΠΎΡ€Ρ‹Ρ… происходит Ρ€Π°Π·Π΄Π΅Π»Π΅Π½ΠΈΠ΅ элСктричСских зарядов .

Π‘Ρ‚ΠΎΡ€ΠΎΠ½Π½ΠΈΠΌΠΈ силами Π½Π°Π·Ρ‹Π²Π°ΡŽΡ‚ΡΡ Π»ΡŽΠ±Ρ‹Π΅ силы, Π΄Π΅ΠΉΡΡ‚Π²ΡƒΡŽΡ‰ΠΈΠ΅ Π½Π° элСктричСски заряТСнныС частицы Π² Ρ†Π΅ΠΏΠΈ, Π·Π° ΠΈΡΠΊΠ»ΡŽΡ‡Π΅Π½ΠΈΠ΅ΠΌ элСктростатичСских (Ρ‚. Π΅. кулоновских).

Π‘Ρ‚ΠΎΡ€ΠΎΠ½Π½ΠΈΠ΅ силы приводят Π² Π΄Π²ΠΈΠΆΠ΅Π½ΠΈΠ΅ заряТСнныС частицы Π²Π½ΡƒΡ‚-Ρ€ΠΈ всСх источников Ρ‚ΠΎΠΊΠ°: Π² Π³Π΅Π½Π΅Ρ€Π°Ρ‚ΠΎΡ€Π°Ρ…, Π½Π° элСктростанциях, Π² Π³Π°Π»ΡŒΠ²Π°Π½ΠΈΡ‡Π΅ΡΠΊΠΈΡ… элСмСнтах, аккумуляторах ΠΈ Ρ‚. Π΄.

ΠŸΡ€ΠΈ Π·Π°ΠΌΡ‹ΠΊΠ°Π½ΠΈΠΈ Ρ†Π΅ΠΏΠΈ создаСтся элСктричСскоС ΠΏΠΎΠ»Π΅ Π²ΠΎ всСх ΠΏΡ€ΠΎ-Π²ΠΎΠ΄Π½ΠΈΠΊΠ°Ρ… Ρ†Π΅ΠΏΠΈ. Π’Π½ΡƒΡ‚Ρ€ΠΈ источника Ρ‚ΠΎΠΊΠ° заряды двиТутся ΠΏΠΎΠ΄ дСйствиСм сторонних сил ΠΏΡ€ΠΎΡ‚ΠΈΠ² кулоновских сил (элСктроны Π΄Π²ΠΈΠΆΡƒΡ‚-ся ΠΎΡ‚ ΠΏΠΎΠ»ΠΎΠΆΠΈΡ‚Π΅Π»ΡŒΠ½ΠΎ заряТСнного элСктрода ΠΊ ΠΎΡ‚Ρ€ΠΈΡ†Π°Ρ‚Π΅Π»ΡŒΠ½ΠΎΠΌΡƒ), Π° Π²ΠΎ всСй ΠΎΡΡ‚Π°Π»ΡŒΠ½ΠΎΠΉ Ρ†Π΅ΠΏΠΈ ΠΈΡ… ΠΏΡ€ΠΈΠ²ΠΎΠ΄ΠΈΡ‚ Π° Π΄Π²ΠΈΠΆΠ΅Π½ΠΈΠ΅ элСктричСскоС ΠΏΠΎΠ»Π΅ (см. рис. Π²Ρ‹ΡˆΠ΅).

Π’ источниках Ρ‚ΠΎΠΊΠ° Π² процСссС Ρ€Π°Π±ΠΎΡ‚Ρ‹ ΠΏΠΎ Ρ€Π°Π·Π΄Π΅Π»Π΅Π½ΠΈΡŽ заряТСнных частиц происходит ΠΏΡ€Π΅Π²Ρ€Π°Ρ‰Π΅Π½ΠΈΠ΅ Ρ€Π°Π·Π½Ρ‹Ρ… Π²ΠΈΠ΄ΠΎΠ² энСргии Π² элСктричСс-ΠΊΡƒΡŽ. По Ρ‚ΠΈΠΏΡƒ ΠΏΡ€Π΅ΠΎΠ±Ρ€Π°Π·ΠΎΠ²Π°Π½Π½ΠΎΠΉ энСргии Ρ€Π°Π·Π»ΠΈΡ‡Π°ΡŽΡ‚ ΡΠ»Π΅Π΄ΡƒΡŽΡ‰ΠΈΠ΅ Π²ΠΈΠ΄Ρ‹ элСктродвиТущСй силы:

— элСктростатичСская β€” Π² элСктрофорной машинС, Π² ΠΊΠΎΡ‚ΠΎΡ€ΠΎΠΉ происходит ΠΏΡ€Π΅Π²Ρ€Π°Ρ‰Π΅Π½ΠΈΠ΅ мСханичСской энСргии ΠΏΡ€ΠΈ Ρ‚Ρ€Π΅Π½ΠΈΠΈ Π² ΡΠ»Π΅ΠΊΡ‚Ρ€ΠΈΡ‡Π΅ΡΠΊΡƒΡŽ;

— тСрмоэлСктричСская — Π² тСрмоэлСмСнтС β€” внутрСнняя энСргия Π½Π°Π³Ρ€Π΅Ρ‚ΠΎΠ³ΠΎ спая Π΄Π²ΡƒΡ… ΠΏΡ€ΠΎΠ²ΠΎΠ»ΠΎΠΊ, ΠΈΠ·Π³ΠΎΡ‚ΠΎΠ²Π»Π΅Π½Π½Ρ‹Ρ… ΠΈΠ· Ρ€Π°Π·Π½Ρ‹Ρ… ΠΌΠ΅Ρ‚Π°Π»Π»ΠΎΠ², прСвращаСтся Π² ΡΠ»Π΅ΠΊΡ‚Ρ€ΠΈΡ‡Π΅ΡΠΊΡƒΡŽ;

— фотоэлСктричСская β€” Π² фотоэлСмСнтС. Π—Π΄Π΅ΡΡŒ происходит ΠΏΡ€Π΅Π²Ρ€Π°Ρ‰Π΅Π½ΠΈΠ΅ энСргии свСта Π² элСк-Ρ‚Ρ€ΠΈΡ‡Π΅ΡΠΊΡƒΡŽ: ΠΏΡ€ΠΈ освСщСнии Π½Π΅ΠΊΠΎΡ‚ΠΎΡ€Ρ‹Ρ… вСщСств, Π½Π°ΠΏΡ€ΠΈΠΌΠ΅Ρ€, сСлСна, оксида ΠΌΠ΅Π΄ΠΈ (I) , крСмния Π½Π°Π±Π»ΡŽΠ΄Π°Π΅Ρ‚ΡΡ потСря ΠΎΡ‚Ρ€ΠΈΡ†Π°Ρ‚Π΅Π»ΡŒΠ½ΠΎΠ³ΠΎ элСктричСского заряда;

— химичСская β€” Π² Π³Π°Π»ΡŒΠ²Π°Π½ΠΈΡ‡Π΅ΡΠΊΠΈΡ… элСмСнтах, аккумуляторах ΠΈ Π΄Ρ€. источниках, Π² ΠΊΠΎΡ‚ΠΎΡ€Ρ‹Ρ… происходит ΠΏΡ€Π΅Π²Ρ€Π°Ρ‰Π΅Π½ΠΈΠ΅ химичСской энСргии Π² ΡΠ»Π΅ΠΊΡ‚Ρ€ΠΈΡ‡Π΅ΡΠΊΡƒΡŽ.

ЭлСктродвиТущая сила (Π­Π”Π‘) β€” характСристика источников Ρ‚ΠΎΠΊΠ°. ΠŸΠΎΠ½ΡΡ‚ΠΈΠ΅ Π­Π”Π‘ Π±Ρ‹Π»ΠΎ Π²Π²Π΅Π΄Π΅Π½ΠΎ Π“. Омом Π² 1827 Π³. для Ρ†Π΅ΠΏΠ΅ΠΉ постоянного Ρ‚ΠΎΠΊΠ°. Π’ 1857 Π³. ΠšΠΈΡ€Ρ…Π³ΠΎΡ„Ρ„ ΠΎΠΏΡ€Π΅Π΄Π΅Π»ΠΈΠ» Π­Π”Π‘ ΠΊΠ°ΠΊ Ρ€Π°Π±ΠΎΡ‚Ρƒ сторонних сил ΠΏΡ€ΠΈ пСрСносС Π΅Π΄ΠΈΠ½ΠΈΡ‡Π½ΠΎΠ³ΠΎ элСктричСского заряда вдоль Π·Π°ΠΌΠΊΠ½ΡƒΡ‚ΠΎΠ³ΠΎ ΠΊΠΎΠ½Ρ‚ΡƒΡ€Π°:

Ι› = A ст /q ,

Π³Π΄Π΅ Ι› β€” Π­Π”Π‘ источника Ρ‚ΠΎΠΊΠ°, А ст β€” Ρ€Π°Π±ΠΎΡ‚Π° сторонних сил , q β€” количСство ΠΏΠ΅Ρ€Π΅ΠΌΠ΅Ρ‰Π΅Π½Π½ΠΎΠ³ΠΎ заряда.

Π­Π»Π΅ΠΊΡ‚Ρ€ΠΎΠ΄Π²ΠΈΠΆΡƒΡ‰ΡƒΡŽ силу Π²Ρ‹Ρ€Π°ΠΆΠ°ΡŽΡ‚ Π² Π²ΠΎΠ»ΡŒΡ‚Π°Ρ….

МоТно Π³ΠΎΠ²ΠΎΡ€ΠΈΡ‚ΡŒ ΠΎΠ± элСктродвиТущСй силС Π½Π° любом участкС Ρ†Π΅ΠΏΠΈ. Π­Ρ‚ΠΎ ΡƒΠ΄Π΅Π»ΡŒΠ½Π°Ρ Ρ€Π°Π±ΠΎΡ‚Π° сторонних сил (Ρ€Π°Π±ΠΎΡ‚Π° ΠΏΠΎ ΠΏΠ΅Ρ€Π΅ΠΌΠ΅Ρ‰Π΅Π½ΠΈΡŽ Π΅Π΄ΠΈΠ½ΠΈΡ‡Π½ΠΎΠ³ΠΎ заряда) Π½Π΅ Π²ΠΎ всСм ΠΊΠΎΠ½Ρ‚ΡƒΡ€Π΅, Π° Ρ‚ΠΎΠ»ΡŒΠΊΠΎ Π½Π° Π΄Π°Π½Π½ΠΎΠΌ участкС.

Π’Π½ΡƒΡ‚Ρ€Π΅Π½Π½Π΅Π΅ сопротивлСниС источника Ρ‚ΠΎΠΊΠ°.

ΠŸΡƒΡΡ‚ΡŒ имССтся простая замкнутая Ρ†Π΅ΠΏΡŒ, состоящая ΠΈΠ· источника Ρ‚ΠΎΠΊΠ° (Π½Π°ΠΏΡ€ΠΈΠΌΠ΅Ρ€, Π³Π°Π»ΡŒΠ²Π°Π½ΠΈΡ‡Π΅ΡΠΊΠΎΠ³ΠΎ элСмСнта, аккумулятора ΠΈΠ»ΠΈ Π³Π΅Π½Π΅Ρ€Π°Ρ‚ΠΎΡ€Π°) ΠΈ рСзистора с сопротивлСниСм R . Π’ΠΎΠΊ Π² Π·Π°ΠΌΠΊΠ½Ρƒ-Ρ‚ΠΎΠΉ Ρ†Π΅ΠΏΠΈ Π½Π΅ прСрываСтся Π½ΠΈΠ³Π΄Π΅, ΡΠ»Π΅Π΄ΠΎΠ²Π°Ρ‚Π΅Π»ΡŒΠ½ΠΎ, oΠ½ сущСствуСт ΠΈ Π²Π½ΡƒΡ‚Ρ€ΠΈ источника Ρ‚ΠΎΠΊΠ°. Π›ΡŽΠ±ΠΎΠΉ источник прСдставляСт собой Π½Π΅ΠΊΠΎΡ‚ΠΎΡ€ΠΎΠ΅ сопротивлСниС Π΄Π»ΠΈ Ρ‚ΠΎΠΊΠ°. Оно называСтся Π²Π½ΡƒΡ‚Ρ€Π΅Π½Π½ΠΈΠΌ сопротивлСниСм источника Ρ‚ΠΎΠΊΠ° ΠΈ обозначаСтся Π±ΡƒΠΊΠ²ΠΎΠΉ r .

Π’ Π³Π΅Π½Π΅Ρ€Π°Ρ‚ΠΎΡ€Π΅ r β€” это сопротивлСниС ΠΎΠ±ΠΌΠΎΡ‚ΠΊΠΈ, Π² Π³Π°Π»ΡŒΠ²Π°Π½ΠΈΡ‡Π΅ΡΠΊΠΎΠΌ элСмСнтС β€” сопротивлСниС раствора элСктролита ΠΈ элСктродов.

Π’Π°ΠΊΠΈΠΌ ΠΎΠ±Ρ€Π°Π·ΠΎΠΌ, источник Ρ‚ΠΎΠΊΠ° характСризуСтся Π²Π΅Π»ΠΈΡ‡ΠΈΠ½Π°ΠΌΠΈ Π­Π”Π‘ ΠΈ Π²Π½ΡƒΡ‚Ρ€Π΅Π½Π½Π΅Π³ΠΎ сопротивлСнии, ΠΊΠΎΡ‚ΠΎΡ€Ρ‹Π΅ ΠΎΠΏΡ€Π΅Π΄Π΅Π»ΡΡŽΡ‚ Π΅Π³ΠΎ качСство. НапримСр, элСктростатичСскиС ΠΌΠ°ΡˆΠΈΠ½Ρ‹ ΠΈΠΌΠ΅ΡŽΡ‚ ΠΎΡ‡Π΅Π½ΡŒ Π±ΠΎΠ»ΡŒΡˆΡƒΡŽ Π­Π”Π‘ (Π΄ΠΎ дСсятков тысяч Π²ΠΎΠ»ΡŒΡ‚), Π½ΠΎ ΠΏΡ€ΠΈ этом ΠΈΡ… Π²Π½ΡƒΡ‚Ρ€Π΅Π½Π½Π΅Π΅ сопротивлСниС ΠΎΠ³Ρ€ΠΎΠΌΠ½ΠΎ (Π΄ΠΎ со-Ρ‚Π½ΠΈ Мом). ΠŸΠΎΡΡ‚ΠΎΠΌΡƒ ΠΎΠ½ΠΈ Π½Π΅ΠΏΡ€ΠΈΠ³ΠΎΠ΄Π½Ρ‹ для получСния ΡΠΈΠ»ΡŒΠ½Ρ‹Ρ… Ρ‚ΠΎΠΊΠΎΠ². Π£ Π³Π°Π»ΡŒΠ²Π°Π½ΠΈΡ‡Π΅ΡΠΊΠΈΡ… элСмСнтов Π­Π”Π‘ всСго лишь ΠΏΡ€ΠΈΠ±Π»ΠΈΠ·ΠΈΡ‚Π΅Π»ΡŒΠ½ΠΎ 1 Π’, Π½ΠΎ Π·Π°Ρ‚ΠΎ ΠΈ Π²Π½ΡƒΡ‚Ρ€Π΅Π½Π½Π΅Π΅ сопротивлСниС ΠΌΠ°Π»ΠΎ (ΠΏΡ€ΠΈΠ±Π»ΠΈΠ·ΠΈΡ‚Π΅Π»ΡŒΠ½ΠΎ 1 Ом ΠΈ мСньшС). Π­Ρ‚ΠΎ позволяСт с ΠΈΡ… ΠΏΠΎΠΌΠΎΡ‰ΡŒΡŽ ΠΏΠΎΠ»ΡƒΡ‡Π°Ρ‚ΡŒ Ρ‚ΠΎΠΊΠΈ, измСряСмыС Π°ΠΌΠΏΠ΅Ρ€Π°ΠΌΠΈ.

Π­Π”Π‘ ΠΈ напряТСниС. Π’Π½ΡƒΡ‚Ρ€Π΅Π½Π½Π΅Π΅ сопротивлСниС источников питания.
Π›ΠΈΠΊΠ±Π΅Π· Ρ‚Π°ΠΊ Π»ΠΈΠΊΠ±Π΅Π·!
Π—Π°ΠΊΠΎΠ½ Ома. Π’ΠΎΡ‚ я ΠΎ Ρ‡Π΅ΠΌ.
О Π·Π°ΠΊΠΎΠ½Π΅ Ома ΠΌΡ‹ ΡƒΠΆΠ΅ Π³ΠΎΠ²ΠΎΡ€ΠΈΠ»ΠΈ. ΠŸΠΎΠ³ΠΎΠ²ΠΎΡ€ΠΈΠΌ Π΅Ρ‰Π΅ Ρ€Π°Π· — с нСсколько ΠΈΠ½ΠΎΠΉ стороны. НС вдаваясь Π² физичСскиС подробности ΠΈ Π²Ρ‹Ρ€Π°ΠΆΠ°ΡΡΡŒ простым ΠΊΠΎΡˆΠ°Ρ‡ΡŒΠΈΠΌ языком, Π·Π°ΠΊΠΎΠ½ Ома гласит: Ρ‡Π΅ΠΌ большС э.Π΄.с. (элСктродвиТущая сила), Ρ‚Π΅ΠΌ большС Ρ‚ΠΎΠΊ, Ρ‡Π΅ΠΌ большС сопротивлСниС, Ρ‚Π΅ΠΌ мСньшС Ρ‚ΠΎΠΊ.
ΠŸΠ΅Ρ€Π΅Π²Π΅Π΄Ρ сиС Π·Π°ΠΊΠ»ΠΈΠ½Π°Π½ΠΈΠ΅ Π½Π° язык сухих Ρ„ΠΎΡ€ΠΌΡƒΠ» ΠΏΠΎΠ»ΡƒΡ‡Π°Π΅ΠΌ:

I=E/R

Π³Π΄Π΅:I — сила Ρ‚ΠΎΠΊΠ°,E — Π­.Π”.Π‘. — элСктродвиТущая силаR — сопротивлСниС
Π’ΠΎΠΊ измСряСтся Π² Π°ΠΌΠΏΠ΅Ρ€Π°Ρ…, э.Π΄.с. — Π² Π²ΠΎΠ»ΡŒΡ‚Π°Ρ…, Π° сопротивлСниС носит Π³ΠΎΡ€Π΄ΠΎΠ΅ имя Ρ‚ΠΎΠ²Π°Ρ€ΠΈΡ‰Π° Ома.Π­.Π΄.с. — это Π΅ΡΡ‚ΡŒ характСристика идСального Π³Π΅Π½Π΅Ρ€Π°Ρ‚ΠΎΡ€Π°, Π²Π½ΡƒΡ‚Ρ€Π΅Π½Π½Π΅ сопротивлСниС ΠΊΠΎΡ‚ΠΎΡ€ΠΎΠ³ΠΎ принято ΡΡ‡ΠΈΡ‚Π°Ρ‚ΡŒ бСсконСчно ΠΌΠ°Π»Ρ‹ΠΌ. Π’ Ρ€Π΅Π°Π»ΡŒΠ½ΠΎΠΉ ΠΆΠΈΠ·Π½ΠΈ Ρ‚Π°ΠΊΠΎΠ΅ Π±Ρ‹Π²Π°Π΅Ρ‚ Ρ€Π΅Π΄ΠΊΠΎ, поэтому Π² силу вступаСт Π·Π°ΠΊΠΎΠ½ Ома для ΠΏΠΎΡΠ»Π΅Π΄ΠΎΠ²Π°Ρ‚Π΅Π»ΡŒΠ½ΠΎΠΉ Ρ†Π΅ΠΏΠΈ (Π±ΠΎΠ»Π΅Π΅ Π·Π½Π°ΠΊΠΎΠΌΡ‹ΠΉ Π½Π°ΠΌ):

I=U/R

Π³Π΄Π΅:U — напряТСниС источника нСпосрСдствСнно Π½Π° Π΅Π³ΠΎ ΠΊΠ»Π΅ΠΌΠΌΠ°Ρ….
Рассмотрим простой ΠΏΡ€ΠΈΠΌΠ΅Ρ€.
ΠŸΡ€Π΅Π΄ΡΡ‚Π°Π²ΠΈΠΌ сСбС ΠΎΠ±Ρ‹Ρ‡Π½ΡƒΡŽ Π±Π°Ρ‚Π°Ρ€Π΅ΠΉΠΊΡƒ Π² Π²ΠΈΠ΄Π΅ источника э.Π΄.с. ΠΈ Π²ΠΊΠ»ΡŽΡ‡Π΅Π½Π½ΠΎΠ³ΠΎ ΠΏΠΎΡΠ»Π΅Π΄ΠΎΠ²Π°Ρ‚Π΅Π»ΡŒΠ½ΠΎ с Π½ΠΈΠΌ Π½Π΅ΠΊΠΎΠ΅Π³ΠΎ рСзистора, ΠΊΠΎΡ‚ΠΎΡ€Ρ‹ΠΉ Π±ΡƒΠ΄Π΅Ρ‚ ΠΎΠ»ΠΈΡ†Π΅Ρ‚Π²ΠΎΡ€ΡΡ‚ΡŒ собой Π²Π½ΡƒΡ‚Ρ€Π΅Π½Π½Π΅Π΅ сопротивлСниС Π±Π°Ρ‚Π°Ρ€Π΅ΠΉΠΊΠΈ. ΠŸΠΎΠ΄ΠΊΠ»ΡŽΡ‡ΠΈΠΌ ΠΏΠ°Ρ€Π°Π»Π»Π΅Π»ΡŒΠ½ΠΎ Π±Π°Ρ‚Π°Ρ€Π΅ΠΉΠΊΠ΅ Π²ΠΎΠ»ΡŒΡ‚ΠΌΠ΅Ρ‚Ρ€. Π•Π³ΠΎ Π²Ρ…ΠΎΠ΄Π½ΠΎΠ΅ сопротивлСниС Π·Π½Π°Ρ‡ΠΈΡ‚Π΅Π»ΡŒΠ½ΠΎ большС Π²Π½ΡƒΡ‚Ρ€Π΅Π½Π½Π΅Π³ΠΎ сопротивлСния Π±Π°Ρ‚Π°Ρ€Π΅ΠΉΠΊΠΈ, Π½ΠΎ Π½Π΅ бСсконСчно большоС — Ρ‚ΠΎ Π΅ΡΡ‚ΡŒ, Ρ‡Π΅Ρ€Π΅Π· Π½Π΅Π³ΠΎ ΠΏΠΎΡ‚Π΅Ρ‡Π΅Ρ‚ Ρ‚ΠΎΠΊ. Π’Π΅Π»ΠΈΡ‡ΠΈΠ½Π° напряТСния, ΠΊΠΎΡ‚ΠΎΡ€ΡƒΡŽ ΠΏΠΎΠΊΠ°ΠΆΠ΅Ρ‚ Π²ΠΎΠ»ΡŒΡ‚ΠΌΠ΅Ρ‚Ρ€ Π±ΡƒΠ΄Π΅Ρ‚ мСньшС Π²Π΅Π»ΠΈΡ‡ΠΈΠ½Ρ‹ э. Π΄.с. ΠΊΠ°ΠΊ Ρ€Π°Π· Π½Π° Π²Π΅Π»ΠΈΡ‡ΠΈΠ½Ρƒ падСния напряТСния Π½Π° Π²Π½ΡƒΡ‚Ρ€Π΅Π½Π½Π΅ΠΌ Π²ΠΎΠΎΠ±Ρ€Π°ΠΆΠ°Π΅ΠΌΠΎΠΌ рСзисторС ΠΏΡ€ΠΈ Π΄Π°Π½Π½ΠΎΠΌ Ρ‚ΠΎΠΊΠ΅.Но, Ρ‚Π΅ΠΌ Π½Π΅ ΠΌΠ΅Π½Π΅Π΅ ΠΈΠΌΠ΅Π½Π½ΠΎ эта Π²Π΅Π»ΠΈΡ‡ΠΈΠ½Π° ΠΈ принимаСтся Π·Π° напряТСниС Π±Π°Ρ‚Π°Ρ€Π΅ΠΉΠΊΠΈ.
Π€ΠΎΡ€ΠΌΡƒΠ»Π° ΠΊΠΎΠ½Π΅Ρ‡Π½ΠΎΠ³ΠΎ напряТСния ΠΏΡ€ΠΈ этом Π±ΡƒΠ΄Π΅Ρ‚ ΠΈΠΌΠ΅Ρ‚ΡŒ ΡΠ»Π΅Π΄ΡƒΡŽΡ‰ΠΈΠΉ Π²ΠΈΠ΄:

U(Π±Π°Ρ‚)=E-U(Π²Π½ΡƒΡ‚Ρ€)

Π’Π°ΠΊ ΠΊΠ°ΠΊ со Π²Ρ€Π΅ΠΌΠ΅Π½Π΅ΠΌ Ρƒ всСх элСмСнтов питания Π²Π½ΡƒΡ‚Ρ€Π΅Π½Π½Π΅Π΅ сопротивлСниС увСличиваСтся, Ρ‚ΠΎ ΠΈ ΠΏΠ°Π΄Π΅Π½ΠΈΠ΅ напряТСния Π½Π° Π²Π½ΡƒΡ‚Ρ€Π΅Π½Π½Π΅ΠΌ сопротивлСнии Ρ‚ΠΎΠΆΠ΅ увСличиваСтся. ΠŸΡ€ΠΈ этом напряТСниС Π½Π° ΠΊΠ»Π΅ΠΌΠΌΠ°Ρ… Π±Π°Ρ‚Π°Ρ€Π΅ΠΉΠΊΠΈ ΡƒΠΌΠ΅Π½ΡŒΡˆΠ°Π΅Ρ‚ΡΡ. ΠœΡΡƒ!
Π Π°Π·ΠΎΠ±Ρ€Π°Π»ΠΈΡΡŒ!
Π§Ρ‚ΠΎ ΠΆΠ΅ происходит, Ссли вмСсто Π²ΠΎΠ»ΡŒΡ‚ΠΌΠ΅Ρ‚Ρ€Π° ΠΊ Π±Π°Ρ‚Π°Ρ€Π΅ΠΉΠΊΠ΅ ΠΏΠΎΠ΄ΠΊΠ»ΡŽΡ‡ΠΈΡ‚ΡŒ Π°ΠΌΠΏΠ΅Ρ€ΠΌΠ΅Ρ‚Ρ€? Π’Π°ΠΊ ΠΊΠ°ΠΊ собствСнноС сопротивлСниС Π°ΠΌΠΏΠ΅Ρ€ΠΌΠ΅Ρ‚Ρ€Π° стрСмится ΠΊ Π½ΡƒΠ»ΡŽ, ΠΌΡ‹ фактичСски Π±ΡƒΠ΄Π΅ΠΌ ΠΈΠ·ΠΌΠ΅Ρ€ΡΡ‚ΡŒ Ρ‚ΠΎΠΊ, ΠΏΡ€ΠΎΡ‚Π΅ΠΊΠ°ΡŽΡ‰ΠΈΠΉ Ρ‡Π΅Ρ€Π΅Π· Π²Π½ΡƒΡ‚Ρ€Π΅Π½Π½Π΅Π΅ сопротивлСниС Π±Π°Ρ‚Π°Ρ€Π΅ΠΉΠΊΠΈ. Π’Π°ΠΊ ΠΊΠ°ΠΊ Π²Π½ΡƒΡ‚Ρ€Π΅Π½Π½Π΅ сопротивлСниС источника ΠΎΡ‡Π΅Π½ΡŒ нСбольшоС, ΠΈΠ·ΠΌΠ΅Ρ€Π΅Π½Π½Ρ‹ΠΉ ΠΏΡ€ΠΈ этом Ρ‚ΠΎΠΊ ΠΌΠΎΠΆΠ΅Ρ‚ Π΄ΠΎΡΡ‚ΠΈΠ³Π°Ρ‚ΡŒ Π½ Π΅ΡΠΊΠΎΠ»ΡŒΠΊΠΈΡ… Π°ΠΌΠΏΠ΅Ρ€.
Однако слСдуСт Π·Π°ΠΌΠ΅Ρ‚ΠΈΡ‚ΡŒ, Ρ‡Ρ‚ΠΎ Π²Π½ΡƒΡ‚Ρ€Π΅Π½Π½Π΅Π΅ сопротивлСниС источника являСтся Ρ‚Π°ΠΊΠΈΠΌ ΠΆΠ΅ элСмСнтом Ρ†Π΅ΠΏΠΈ, ΠΊΠ°ΠΊ ΠΈ всС ΠΎΡΡ‚Π°Π»ΡŒΠ½Ρ‹Π΅. ΠŸΠΎΡΡ‚ΠΎΠΌΡƒ ΠΏΡ€ΠΈ ΡƒΠ²Π΅Π»ΠΈΡ‡Π΅Π½ΠΈΠΈ Ρ‚ΠΎΠΊΠ° Π½Π°Π³Ρ€ΡƒΠ·ΠΊΠΈ ΠΏΠ°Π΄Π΅Π½ΠΈΠ΅ напряТСния Π½Π° Π²Π½ΡƒΡ‚Ρ€Π΅Π½Π½Π΅ΠΌ сопротивлСнии Ρ‚Π°ΠΊΠΆΠ΅ увСличится, Ρ‡Ρ‚ΠΎ ΠΏΡ€ΠΈΠ²ΠΎΠ΄ΠΈΡ‚ ΠΊ ΡƒΠΌΠ΅Π½ΡŒΡˆΠ΅Π½ΠΈΡŽ напряТСния Π½Π° Π½Π°Π³Ρ€ΡƒΠ·ΠΊΠ΅. Или ΠΊΠ°ΠΊ ΠΌΡ‹, Ρ€Π°Π΄ΠΈΠΎΠΊΠΎΡ‚Ρ‹, любим Π²Ρ‹Ρ€Π°ΠΆΠ°Ρ‚ΡŒΡΡ — ΠΊ просадкС Π½Π°ΠΏΡ€ΡƒΠ³ΠΈ.
Π§Ρ‚ΠΎΠ±Ρ‹ ΠΈΠ·ΠΌΠ΅Π½Π΅Π½ΠΈΠ΅ Π½Π°Π³Ρ€ΡƒΠ·ΠΊΠΈ ΠΊΠ°ΠΊ ΠΌΠΎΠΆΠ½ΠΎ мСньшС влияло Π½Π° Π²Ρ‹Ρ…ΠΎΠ΄Π½ΠΎΠ΅ напряТСниС источника Π΅Π³ΠΎ Π²Π½ΡƒΡ‚Ρ€Π΅Π½Π½Π΅Π΅ сопротивлСниС ΡΡ‚Π°Ρ€Π°ΡŽΡ‚ΡΡ свСсти ΠΊ ΠΌΠΈΠ½ΠΈΠΌΡƒΠΌΡƒ.
МоТно Ρ‚Π°ΠΊ ΠΏΠΎΠ΄ΠΎΠ±Ρ€Π°Ρ‚ΡŒ элСмСнты ΠΏΠΎΡΠ»Π΅Π΄ΠΎΠ²Π°Ρ‚Π΅Π»ΡŒΠ½ΠΎΠΉ Ρ†Π΅ΠΏΠΈ, Ρ‡Ρ‚ΠΎΠ±Ρ‹ Π½Π° ΠΊΠ°ΠΊΠΎΠΌ-Π½ΠΈΠ±ΡƒΠ΄ΡŒ ΠΈΠ· Π½ΠΈΡ… ΠΏΠΎΠ»ΡƒΡ‡ΠΈΡ‚ΡŒ напряТСниС, ΡƒΠΌΠ΅Π½ΡŒΡˆΠ΅Π½Π½ΠΎΠ΅, ΠΏΠΎ ΡΡ€Π°Π²Π½Π΅Π½ΠΈΡŽ с исходным, Π²ΠΎ сколько ΡƒΠ³ΠΎΠ΄Π½ΠΎ Ρ€Π°Π·.

Допустим, Π΅ΡΡ‚ΡŒ ΠΏΡ€ΠΎΡΡ‚Π΅ΠΉΡˆΠ°Ρ элСктричСская замкнутая Ρ†Π΅ΠΏΡŒ, Π²ΠΊΠ»ΡŽΡ‡Π°ΡŽΡ‰Π°Ρ Π² сСбя источник Ρ‚ΠΎΠΊΠ°, Π½Π°ΠΏΡ€ΠΈΠΌΠ΅Ρ€ Π³Π΅Π½Π΅Ρ€Π°Ρ‚ΠΎΡ€, Π³Π°Π»ΡŒΠ²Π°Π½ΠΈΡ‡Π΅ΡΠΊΠΈΠΉ элСмСнт ΠΈΠ»ΠΈ аккумулятор, ΠΈ рСзистор, ΠΎΠ±Π»Π°Π΄Π°ΡŽΡ‰ΠΈΠΉ сопротивлСниСм R. ΠŸΠΎΡΠΊΠΎΠ»ΡŒΠΊΡƒ Ρ‚ΠΎΠΊ Π² Ρ†Π΅ΠΏΠΈ Π½ΠΈΠ³Π΄Π΅ Π½Π΅ прСрываСтся, Ρ‚ΠΎ ΠΈ Π²Π½ΡƒΡ‚Ρ€ΠΈ источника ΠΎΠ½ Ρ‚Π΅Ρ‡Π΅Ρ‚.

Π’ Ρ‚Π°ΠΊΠΎΠΉ ситуации ΠΌΠΎΠΆΠ½ΠΎ ΡΠΊΠ°Π·Π°Ρ‚ΡŒ, Ρ‡Ρ‚ΠΎ любой источник ΠΎΠ±Π»Π°Π΄Π°Π΅Ρ‚ Π½Π΅ΠΊΠΎΡ‚ΠΎΡ€Ρ‹ΠΌ Π²Π½ΡƒΡ‚Ρ€Π΅Π½Π½ΠΈΠΌ сопротивлСниСм, ΠΏΡ€Π΅ΠΏΡΡ‚ΡΡ‚Π²ΡƒΡŽΡ‰ΠΈΠΌ Ρ‚ΠΎΠΊΡƒ. Π­Ρ‚ΠΎ Π²Π½ΡƒΡ‚Ρ€Π΅Π½Π½Π΅Π΅ сопротивлСниС Ρ…Π°Ρ€Π°ΠΊΡ‚Π΅Ρ€ΠΈΠ·ΡƒΠ΅Ρ‚ источник Ρ‚ΠΎΠΊΠ° ΠΈ обозначаСтся Π±ΡƒΠΊΠ²ΠΎΠΉ r. Для ΠΈΠ»ΠΈ аккумулятора Π²Π½ΡƒΡ‚Ρ€Π΅Π½Π½Π΅Π΅ сопротивлСниС — это сопротивлСниС раствора элСктролита ΠΈ элСктродов, для Π³Π΅Π½Π΅Ρ€Π°Ρ‚ΠΎΡ€Π° — сопротивлСниС ΠΎΠ±ΠΌΠΎΡ‚ΠΎΠΊ статора ΠΈ Ρ‚. Π΄.

Π’Π°ΠΊΠΈΠΌ ΠΎΠ±Ρ€Π°Π·ΠΎΠΌ, источник Ρ‚ΠΎΠΊΠ° характСризуСтся ΠΊΠ°ΠΊ Π²Π΅Π»ΠΈΡ‡ΠΈΠ½ΠΎΠΉ Π­Π”Π‘, Ρ‚Π°ΠΊ ΠΈ Π²Π΅Π»ΠΈΡ‡ΠΈΠ½ΠΎΠΉ собствСнного Π²Π½ΡƒΡ‚Ρ€Π΅Π½Π½Π΅Π³ΠΎ сопротивлСния r – ΠΎΠ±Π΅ эти характСристики ΡΠ²ΠΈΠ΄Π΅Ρ‚Π΅Π»ΡŒΡΡ‚Π²ΡƒΡŽΡ‚ ΠΎ качСствС источника.

ЭлСктростатичСскиС Π²Ρ‹ΡΠΎΠΊΠΎΠ²ΠΎΠ»ΡŒΡ‚Π½Ρ‹Π΅ Π³Π΅Π½Π΅Ρ€Π°Ρ‚ΠΎΡ€Ρ‹ (ΠΊΠ°ΠΊ Π³Π΅Π½Π΅Ρ€Π°Ρ‚ΠΎΡ€ Π’Π°Π½ Π΄Π΅ Π“Ρ€Π°Π°Ρ„Π° ΠΈΠ»ΠΈ Π³Π΅Π½Π΅Ρ€Π°Ρ‚ΠΎΡ€ Π£ΠΈΠΌΡˆΡƒΡ€ΡΡ‚Π°), ΠΊ ΠΏΡ€ΠΈΠΌΠ΅Ρ€Ρƒ, ΠΎΡ‚Π»ΠΈΡ‡Π°ΡŽΡ‚ΡΡ ΠΎΠ³Ρ€ΠΎΠΌΠ½ΠΎΠΉ Π­Π”Π‘ измСряСмой ΠΌΠΈΠ»Π»ΠΈΠΎΠ½Π°ΠΌΠΈ Π²ΠΎΠ»ΡŒΡ‚, ΠΏΡ€ΠΈ этом ΠΈΡ… Π²Π½ΡƒΡ‚Ρ€Π΅Π½Π½Π΅Π΅ сопротивлСниС измСряСтся сотнями ΠΌΠ΅Π³Π°ΠΎΠΌ, ΠΏΠΎΡ‚ΠΎΠΌΡƒ ΠΎΠ½ΠΈ ΠΈ Π½Π΅ΠΏΡ€ΠΈΠ³ΠΎΠ΄Π½Ρ‹ для получСния Π±ΠΎΠ»ΡŒΡˆΠΈΡ… Ρ‚ΠΎΠΊΠΎΠ².

Π“Π°Π»ΡŒΠ²Π°Π½ΠΈΡ‡Π΅ΡΠΊΠΈΠ΅ элСмСнты (Ρ‚Π°ΠΊΠΈΠ΅ ΠΊΠ°ΠΊ Π±Π°Ρ‚Π°Ρ€Π΅ΠΉΠΊΠ°) — Π½Π°ΠΏΡ€ΠΎΡ‚ΠΈΠ² — ΠΈΠΌΠ΅ΡŽΡ‚ Π­Π”Π‘ порядка 1 Π²ΠΎΠ»ΡŒΡ‚Π°, хотя Π²Π½ΡƒΡ‚Ρ€Π΅Π½Π½Π΅Π΅ сопротивлСниС Ρƒ Π½ΠΈΡ… порядка Π΄ΠΎΠ»Π΅ΠΉ ΠΈΠ»ΠΈ максимум — дСсятка Ом, ΠΈ ΠΎΡ‚ Π³Π°Π»ΡŒΠ²Π°Π½ΠΈΡ‡Π΅ΡΠΊΠΈΡ… элСмСнтов поэтому ΠΌΠΎΠΆΠ½ΠΎ ΠΏΠΎΠ»ΡƒΡ‡Π°Ρ‚ΡŒ Ρ‚ΠΎΠΊΠΈ Π² Π΅Π΄ΠΈΠ½ΠΈΡ†Ρ‹ ΠΈ дСсятки Π°ΠΌΠΏΠ΅Ρ€.

На Π΄Π°Π½Π½ΠΎΠΉ схСмС ΠΏΠΎΠΊΠ°Π·Π°Π½ Ρ€Π΅Π°Π»ΡŒΠ½Ρ‹ΠΉ источник с присоСдинСнной Π½Π°Π³Ρ€ΡƒΠ·ΠΊΠΎΠΉ. Π—Π΄Π΅ΡΡŒ ΠΎΠ±ΠΎΠ·Π½Π°Ρ‡Π΅Π½Ρ‹ , Π΅Π³ΠΎ Π²Π½ΡƒΡ‚Ρ€Π΅Π½Π½Π΅Π΅ сопротивлСниС, Π° Ρ‚Π°ΠΊΠΆΠ΅ сопротивлСниС Π½Π°Π³Ρ€ΡƒΠ·ΠΊΠΈ. Богласно , Ρ‚ΠΎΠΊ Π² Π΄Π°Π½Π½ΠΎΠΉ Ρ†Π΅ΠΏΠΈ Π±ΡƒΠ΄Π΅Ρ‚ Ρ€Π°Π²Π΅Π½:

ΠŸΠΎΡΠΊΠΎΠ»ΡŒΠΊΡƒ участок внСшнСй Ρ†Π΅ΠΏΠΈ ΠΎΠ΄Π½ΠΎΡ€ΠΎΠ΄Π΅Π½, Ρ‚ΠΎ ΠΈΠ· Π·Π°ΠΊΠΎΠ½Π° Ома ΠΌΠΎΠΆΠ½ΠΎ Π½Π°ΠΉΡ‚ΠΈ напряТСниС Π½Π° Π½Π°Π³Ρ€ΡƒΠ·ΠΊΠ΅:

Π’Ρ‹Ρ€Π°Π·ΠΈΠ² ΠΈΠ· ΠΏΠ΅Ρ€Π²ΠΎΠ³ΠΎ уравнСния сопротивлСниС Π½Π°Π³Ρ€ΡƒΠ·ΠΊΠΈ, ΠΈ подставив Π΅Π³ΠΎ Π·Π½Π°Ρ‡Π΅Π½ΠΈΠ΅ Π²ΠΎ Π²Ρ‚ΠΎΡ€ΠΎΠ΅ ΡƒΡ€Π°Π²Π½Π΅Π½ΠΈΠ΅, ΠΏΠΎΠ»ΡƒΡ‡ΠΈΠΌ Π·Π°Π²ΠΈΡΠΈΠΌΠΎΡΡ‚ΡŒ напряТСния Π½Π° Π½Π°Π³Ρ€ΡƒΠ·ΠΊΠ΅ ΠΎΡ‚ Ρ‚ΠΎΠΊΠ° Π² Π·Π°ΠΌΠΊΠ½ΡƒΡ‚ΠΎΠΉ Ρ†Π΅ΠΏΠΈ:

Π’ Π·Π°ΠΌΠΊΠ½ΡƒΡ‚ΠΎΠΌ ΠΊΠΎΠ½Ρ‚ΡƒΡ€Π΅ Π­Π”Π‘ Ρ€Π°Π²Π½Π° суммС ΠΏΠ°Π΄Π΅Π½ΠΈΠΉ напряТСний Π½Π° элСмСнтах внСшнСй Ρ†Π΅ΠΏΠΈ ΠΈ Π½Π° Π²Π½ΡƒΡ‚Ρ€Π΅Π½Π½Π΅ΠΌ сопротивлСнии самого источника. Π—Π°Π²ΠΈΡΠΈΠΌΠΎΡΡ‚ΡŒ напряТСния Π½Π° Π½Π°Π³Ρ€ΡƒΠ·ΠΊΠ΅ ΠΎΡ‚ Ρ‚ΠΎΠΊΠ° Π½Π°Π³Ρ€ΡƒΠ·ΠΊΠΈ Π² идСальном случаС Π»ΠΈΠ½Π΅ΠΉΠ½Π°.

Π“Ρ€Π°Ρ„ΠΈΠΊ это ΠΏΠΎΠΊΠ°Π·Ρ‹Π²Π°Π΅Ρ‚, Π½ΠΎ ΡΠΊΡΠΏΠ΅Ρ€ΠΈΠΌΠ΅Π½Ρ‚Π°Π»ΡŒΠ½Ρ‹Π΅ Π΄Π°Π½Π½Ρ‹Π΅ Π½Π° Ρ€Π΅Π°Π»ΡŒΠ½ΠΎΠΌ рСзисторС (крСстики Π²ΠΎΠ·Π»Π΅ Π³Ρ€Π°Ρ„ΠΈΠΊΠ°) всСгда ΠΎΡ‚Π»ΠΈΡ‡Π°ΡŽΡ‚ΡΡ ΠΎΡ‚ ΠΈΠ΄Π΅Π°Π»Π°:

ЭкспСримСнты ΠΈ Π»ΠΎΠ³ΠΈΠΊΠ° ΠΏΠΎΠΊΠ°Π·Ρ‹Π²Π°ΡŽΡ‚, Ρ‡Ρ‚ΠΎ ΠΏΡ€ΠΈ Π½ΡƒΠ»Π΅Π²ΠΎΠΌ Ρ‚ΠΎΠΊΠ΅ Π½Π°Π³Ρ€ΡƒΠ·ΠΊΠΈ напряТСниС Π½Π° внСшнСй Ρ†Π΅ΠΏΠΈ Ρ€Π°Π²Π½ΠΎ Π­Π”Π‘ источника, Π° ΠΏΡ€ΠΈ Π½ΡƒΠ»Π΅Π²ΠΎΠΌ напряТСнии Π½Π° Π½Π°Π³Ρ€ΡƒΠ·ΠΊΠ΅ Ρ‚ΠΎΠΊ Π² Ρ†Π΅ΠΏΠΈ Ρ€Π°Π²Π΅Π½ . Π­Ρ‚ΠΎ свойство Ρ€Π΅Π°Π»ΡŒΠ½Ρ‹Ρ… Ρ†Π΅ΠΏΠ΅ΠΉ ΠΏΠΎΠΌΠΎΠ³Π°Π΅Ρ‚ ΡΠΊΡΠΏΠ΅Ρ€ΠΈΠΌΠ΅Π½Ρ‚Π°Π»ΡŒΠ½ΠΎ Π½Π°Ρ…ΠΎΠ΄ΠΈΡ‚ΡŒ Π­Π”Π‘ ΠΈ Π²Π½ΡƒΡ‚Ρ€Π΅Π½Π½Π΅Π΅ сопротивлСниС Ρ€Π΅Π°Π»ΡŒΠ½Ρ‹Ρ… источников.

Π­ΠΊΡΠΏΠ΅Ρ€ΠΈΠΌΠ΅Π½Ρ‚Π°Π»ΡŒΠ½ΠΎΠ΅ Π½Π°Ρ…ΠΎΠΆΠ΄Π΅Π½ΠΈΠ΅ Π²Π½ΡƒΡ‚Ρ€Π΅Π½Π½Π΅Π³ΠΎ сопротивлСния

Π§Ρ‚ΠΎΠ±Ρ‹ ΡΠΊΡΠΏΠ΅Ρ€ΠΈΠΌΠ΅Π½Ρ‚Π°Π»ΡŒΠ½ΠΎ ΠΎΠΏΡ€Π΅Π΄Π΅Π»ΠΈΡ‚ΡŒ Π΄Π°Π½Π½Ρ‹Π΅ характСристики, строят Π³Ρ€Π°Ρ„ΠΈΠΊ зависимости напряТСния Π½Π° Π½Π°Π³Ρ€ΡƒΠ·ΠΊΠ΅ ΠΎΡ‚ Π²Π΅Π»ΠΈΡ‡ΠΈΠ½Ρ‹ Ρ‚ΠΎΠΊΠ°, Π·Π°Ρ‚Π΅ΠΌ ΡΠΊΡΡ‚Ρ€Π°ΠΏΠΎΠ»ΠΈΡ€ΡƒΡŽΡ‚ Π΅Π³ΠΎ Π΄ΠΎ пСрСсСчСния с осями.

Π’ Ρ‚ΠΎΡ‡ΠΊΠ΅ пСрСсСчСния Π³Ρ€Π°Ρ„ΠΈΠΊΠ° с ΠΎΡΡ‚ΡŒΡŽ напряТСния находится Π·Π½Π°Ρ‡Π΅Π½ΠΈΠ΅ Π­Π”Π‘ источника, Π° Π² Ρ‚ΠΎΡ‡ΠΊΠ΅ пСрСсСчСния с осью Ρ‚ΠΎΠΊΠ° находится Π²Π΅Π»ΠΈΡ‡ΠΈΠ½Π° Ρ‚ΠΎΠΊΠ° ΠΊΠΎΡ€ΠΎΡ‚ΠΊΠΎΠ³ΠΎ замыкания. Π’ ΠΈΡ‚ΠΎΠ³Π΅ Π²Π½ΡƒΡ‚Ρ€Π΅Π½Π½Π΅Π΅ сопротивлСниС находится ΠΏΠΎ Ρ„ΠΎΡ€ΠΌΡƒΠ»Π΅:

РазвиваСмая источником полСзная ΠΌΠΎΡ‰Π½ΠΎΡΡ‚ΡŒ выдСляСтся Π½Π° Π½Π°Π³Ρ€ΡƒΠ·ΠΊΠ΅. Π“Ρ€Π°Ρ„ΠΈΠΊ зависимости этой мощности ΠΎΡ‚ сопротивлСния Π½Π°Π³Ρ€ΡƒΠ·ΠΊΠΈ ΠΏΡ€ΠΈΠ²Π΅Π΄Π΅Π½ Π½Π° рисункС. Π­Ρ‚Π° кривая начинаСтся ΠΎΡ‚ пСрСсСчСния осСй ΠΊΠΎΠΎΡ€Π΄ΠΈΠ½Π°Ρ‚ Π² Π½ΡƒΠ»Π΅Π²ΠΎΠΉ Ρ‚ΠΎΡ‡ΠΊΠ΅, Π·Π°Ρ‚Π΅ΠΌ возрастаСт Π΄ΠΎ максимального значСния мощности, послС Ρ‡Π΅Π³ΠΎ спадаСт Π΄ΠΎ нуля ΠΏΡ€ΠΈ сопротивлСнии Π½Π°Π³Ρ€ΡƒΠ·ΠΊΠΈ Ρ€Π°Π²Π½ΠΎΠΌ бСсконСчности.

Π§Ρ‚ΠΎΠ±Ρ‹ Π½Π°ΠΉΡ‚ΠΈ максимальноС сопротивлСниС Π½Π°Π³Ρ€ΡƒΠ·ΠΊΠΈ, ΠΏΡ€ΠΈ ΠΊΠΎΡ‚ΠΎΡ€ΠΎΠΌ тСорСтичСски Ρ€Π°Π·ΠΎΠ²ΡŒΠ΅Ρ‚ΡΡ максимальная ΠΌΠΎΡ‰Π½ΠΎΡΡ‚ΡŒ ΠΏΡ€ΠΈ Π΄Π°Π½Π½ΠΎΠΌ источникС, бСрСтся производная ΠΎΡ‚ Ρ„ΠΎΡ€ΠΌΡƒΠ»Ρ‹ мощности ΠΏΠΎ R ΠΈ приравниваСтся ΠΊ Π½ΡƒΠ»ΡŽ. Максимальная ΠΌΠΎΡ‰Π½ΠΎΡΡ‚ΡŒ Ρ€Π°Π·ΠΎΠ²ΡŒΠ΅Ρ‚ΡΡ ΠΏΡ€ΠΈ сопротивлСнии внСшнСй Ρ†Π΅ΠΏΠΈ, Ρ€Π°Π²Π½ΠΎΠΌ Π²Π½ΡƒΡ‚Ρ€Π΅Π½Π½Π΅ΠΌΡƒ ΡΠΎΠΏΡ€ΠΎΡ‚ΠΈΠ²Π»Π΅Π½ΠΈΡŽ источника:

Π­Ρ‚ΠΎ ΠΏΠΎΠ»ΠΎΠΆΠ΅Π½ΠΈΠ΅ ΠΎ максимальной мощности ΠΏΡ€ΠΈ R = r, позволяСт ΡΠΊΡΠΏΠ΅Ρ€ΠΈΠΌΠ΅Π½Ρ‚Π°Π»ΡŒΠ½ΠΎ Π½Π°ΠΉΡ‚ΠΈ Π²Π½ΡƒΡ‚Ρ€Π΅Π½Π½Π΅Π΅ сопротивлСниС источника, построив Π·Π°Π²ΠΈΡΠΈΠΌΠΎΡΡ‚ΡŒ мощности, выдСляСмой Π½Π° Π½Π°Π³Ρ€ΡƒΠ·ΠΊΠ΅, ΠΎΡ‚ Π²Π΅Π»ΠΈΡ‡ΠΈΠ½Ρ‹ сопротивлСния Π½Π°Π³Ρ€ΡƒΠ·ΠΊΠΈ. Найдя Ρ€Π΅Π°Π»ΡŒΠ½ΠΎΠ΅, Π° Π½Π΅ тСорСтичСскоС, сопротивлСниС Π½Π°Π³Ρ€ΡƒΠ·ΠΊΠΈ, ΠΎΠ±Π΅ΡΠΏΠ΅Ρ‡ΠΈΠ²Π°ΡŽΡ‰Π΅Π΅ ΠΌΠ°ΠΊΡΠΈΠΌΠ°Π»ΡŒΠ½ΡƒΡŽ ΠΌΠΎΡ‰Π½ΠΎΡΡ‚ΡŒ, ΠΎΠΏΡ€Π΅Π΄Π΅Π»ΡΡŽΡ‚ Ρ€Π΅Π°Π»ΡŒΠ½ΠΎΠ΅ Π²Π½ΡƒΡ‚Ρ€Π΅Π½Π½Π΅Π΅ сопротивлСниС источника питания.

ΠšΠŸΠ” источника Ρ‚ΠΎΠΊΠ° ΠΏΠΎΠΊΠ°Π·Ρ‹Π²Π°Π΅Ρ‚ ΠΎΡ‚Π½ΠΎΡˆΠ΅Π½ΠΈΠ΅ максимальной выдСляСмой Π½Π° Π½Π°Π³Ρ€ΡƒΠ·ΠΊΠ΅ мощности ΠΊ ΠΏΠΎΠ»Π½ΠΎΠΉ мощности, ΠΊΠΎΡ‚ΠΎΡ€ΡƒΡŽ Π² Π΄Π°Π½Π½Ρ‹ΠΉ ΠΌΠΎΠΌΠ΅Π½Ρ‚ Ρ€Π°Π·Π²ΠΈΠ²Π°Π΅Ρ‚

ΠŸΠΎΠΏΡ€ΠΎΠ±ΡƒΠ΅ΠΌ Ρ€Π΅ΡˆΠΈΡ‚ΡŒ эту Π·Π°Π΄Π°Ρ‡Ρƒ Π½Π° ΠΊΠΎΠ½ΠΊΡ€Π΅Ρ‚Π½ΠΎΠΌ ΠΏΡ€ΠΈΠΌΠ΅Ρ€Π΅. ЭлСктродвиТущая сила источника питания составляСт 4,5 Π’. К Π½Π΅ΠΌΡƒ ΠΏΠΎΠ΄ΠΊΠ»ΡŽΡ‡ΠΈΠ»ΠΈ Π½Π°Π³Ρ€ΡƒΠ·ΠΊΡƒ, ΠΈ Ρ‡Π΅Ρ€Π΅Π· Π½Π΅Ρ‘ ΠΏΠΎΡ‚Ρ‘ΠΊ Ρ‚ΠΎΠΊ, Ρ€Π°Π²Π½Ρ‹ΠΉ 0,26 А. НапряТСниС ΠΏΡ€ΠΈ этом стало Ρ€Π°Π²Π½Ρ‹ΠΌ 3,7 Π’. ΠŸΠ΅Ρ€Π²Ρ‹ΠΌ Π΄Π΅Π»ΠΎΠΌ, прСдставим сСбС, Ρ‡Ρ‚ΠΎ Π²Π½ΡƒΡ‚Ρ€ΠΈ источника спрятана ΠΏΠΎΡΠ»Π΅Π΄ΠΎΠ²Π°Ρ‚Π΅Π»ΡŒΠ½Π°Ρ Ρ†Π΅ΠΏΡŒ ΠΈΠ· идСального источника напряТСния Π² 4,5 Π’, Π²Π½ΡƒΡ‚Ρ€Π΅Π½Π½Π΅Π΅ сопротивлСниС ΠΊΠΎΡ‚ΠΎΡ€ΠΎΠ³ΠΎ Ρ€Π°Π²Π½ΠΎ Π½ΡƒΠ»ΡŽ, Π° Ρ‚Π°ΠΊΠΆΠ΅ рСзистора, Π½ΠΎΠΌΠΈΠ½Π°Π» ΠΊΠΎΡ‚ΠΎΡ€ΠΎΠ³ΠΎ ΠΈ трСбуСтся Π½Π°ΠΉΡ‚ΠΈ. ΠŸΠΎΠ½ΡΡ‚Π½ΠΎ, Ρ‡Ρ‚ΠΎ Π½Π° самом Π΄Π΅Π»Π΅ это Π½Π΅ Ρ‚Π°ΠΊ, Π½ΠΎ для расчётов аналогия Π²ΠΏΠΎΠ»Π½Π΅ сойдёт.

2 шаг

Π—Π°ΠΏΠΎΠΌΠ½ΠΈΡ‚Π΅, Ρ‡Ρ‚ΠΎ Π±ΡƒΠΊΠ²ΠΎΠΉ U ΠΎΠ±ΠΎΠ·Π½Π°Ρ‡Π°ΡŽΡ‚ Ρ‚ΠΎΠ»ΡŒΠΊΠΎ напряТСниС ΠΏΠΎΠ΄ Π½Π°Π³Ρ€ΡƒΠ·ΠΊΠΎΠΉ. Для обозначСния ΠΆΠ΅ элСктродвиТущСй силы Π·Π°Ρ€Π΅Π·Π΅Ρ€Π²ΠΈΡ€ΠΎΠ²Π°Π½Π° другая Π±ΡƒΠΊΠ²Π° – E. ΠΠ±ΡΠΎΠ»ΡŽΡ‚Π½ΠΎ Ρ‚ΠΎΡ‡Π½ΠΎ Π΅Ρ‘ ΠΈΠ·ΠΌΠ΅Ρ€ΠΈΡ‚ΡŒ Π½Π΅Π²ΠΎΠ·ΠΌΠΎΠΆΠ½ΠΎ, ΠΏΠΎΡ‚ΠΎΠΌΡƒ Ρ‡Ρ‚ΠΎ потрСбуСтся Π²ΠΎΠ»ΡŒΡ‚ΠΌΠ΅Ρ‚Ρ€ с бСсконСчным Π²Ρ…ΠΎΠ΄Π½Ρ‹ΠΌ сопротивлСниСм. Π”Π°ΠΆΠ΅ Ρƒ элСктростатичСского Π²ΠΎΠ»ΡŒΡ‚ΠΌΠ΅Ρ‚Ρ€Π° (элСктромСтра) ΠΎΠ½ΠΎ ΠΎΠ³Ρ€ΠΎΠΌΠ½ΠΎ, Π½ΠΎ Π½Π΅ бСсконСчно. Но ΠΎΠ΄Π½ΠΎ Π΄Π΅Π»ΠΎ – Π°Π±ΡΠΎΠ»ΡŽΡ‚Π½ΠΎ Ρ‚ΠΎΡ‡Π½ΠΎ, Π° Π΄Ρ€ΡƒΠ³ΠΎΠ΅ – с Ρ‚ΠΎΡ‡Π½ΠΎΡΡ‚ΡŒΡŽ, ΠΏΡ€ΠΈΠ΅ΠΌΠ»Π΅ΠΌΠΎΠΉ Π½Π° ΠΏΡ€Π°ΠΊΡ‚ΠΈΠΊΠ΅. Π’Ρ‚ΠΎΡ€ΠΎΠ΅ Π²ΠΏΠΎΠ»Π½Π΅ осущСствимо: Π½ΡƒΠΆΠ½ΠΎ лишь, Ρ‡Ρ‚ΠΎΠ±Ρ‹ Π²Π½ΡƒΡ‚Ρ€Π΅Π½Π½Π΅Π΅ сопротивлСниС источника Π±Ρ‹Π»ΠΎ ΠΏΡ€Π΅Π½Π΅Π±Ρ€Π΅ΠΆΠΈΠΌΠΎ ΠΌΠ°Π»ΠΎ ΠΏΠΎ ΡΡ€Π°Π²Π½Π΅Π½ΠΈΡŽ с Π²Π½ΡƒΡ‚Ρ€Π΅Π½Π½ΠΈΠΌ сопротивлСниСм Π²ΠΎΠ»ΡŒΡ‚ΠΌΠ΅Ρ‚Ρ€Π°. А ΠΏΠΎΠΊΠ° ΡΡƒΡ‚ΡŒ Π΄Π° Π΄Π΅Π»ΠΎ, посчитаСм Ρ€Π°Π·Π½ΠΈΡ†Ρƒ ΠΌΠ΅ΠΆΠ΄Ρƒ Π­Π”Π‘ источника ΠΈ Π΅Π³ΠΎ напряТСниСм ΠΏΠΎΠ΄ Π½Π°Π³Ρ€ΡƒΠ·ΠΊΠΎΠΉ, ΠΏΠΎΡ‚Ρ€Π΅Π±Π»ΡΡŽΡ‰Π΅ΠΉ Ρ‚ΠΎΠΊ Π² 260 мА. E-U = 4,5-3,7 = 0,8. Π­Ρ‚ΠΎ ΠΈ Π±ΡƒΠ΄Π΅Ρ‚ ΠΏΠ°Π΄Π΅Π½ΠΈΠ΅ напряТСния Π½Π° Ρ‚ΠΎΠΌ β€œΠ²ΠΈΡ€Ρ‚ΡƒΠ°Π»ΡŒΠ½ΠΎΠΌ рСзисторС”.

3 шаг

Ну Π° дальшС всё просто, ΠΈΠ±ΠΎ Π² Π΄Π΅Π»ΠΎ вступаСт классичСский Π·Π°ΠΊΠΎΠ½ Ома. Помним, Ρ‡Ρ‚ΠΎ Ρ‚ΠΎΠΊ Ρ‡Π΅Ρ€Π΅Π· Π½Π°Π³Ρ€ΡƒΠ·ΠΊΡƒ ΠΈ β€œΠ²ΠΈΡ€Ρ‚ΡƒΠ°Π»ΡŒΠ½Ρ‹ΠΉ рСзистор” ΠΎΠ΄ΠΈΠ½Π°ΠΊΠΎΠ², вСдь ΠΎΠ½ΠΈ соСдинСны ΠΏΠΎΡΠ»Π΅Π΄ΠΎΠ²Π°Ρ‚Π΅Π»ΡŒΠ½ΠΎ. ПадСниС напряТСния Π½Π° послСднСм (0,8 Π’) Π΄Π΅Π»ΠΈΠΌ Π½Π° силу Ρ‚ΠΎΠΊΠ° (0,26 А) ΠΈ ΠΏΠΎΠ»ΡƒΡ‡Π°Π΅ΠΌ 3,08 Ома. Π’ΠΎΡ‚ ΠΈ Π³ΠΎΡ‚ΠΎΠ² ΠΎΡ‚Π²Π΅Ρ‚! МоТно Π΅Ρ‰Ρ‘ ΠΏΠΎΡΡ‡ΠΈΡ‚Π°Ρ‚ΡŒ, какая ΠΌΠΎΡ‰Π½ΠΎΡΡ‚ΡŒ рассСиваСтся Π½Π° Π½Π°Π³Ρ€ΡƒΠ·ΠΊΠ΅, Π° какая – бСсполСзно Π½Π° источникС. На Π½Π°Π³Ρ€ΡƒΠ·ΠΊΠ΅ рассСиваСтся: 3,7*0,26=0,962 Π’Ρ‚. На источникС: 0,8*0,26=0,208 Π’Ρ‚. ΠŸΡ€ΠΎΡ†Π΅Π½Ρ‚Π½ΠΎΠ΅ ΡΠΎΠΎΡ‚Π½ΠΎΡˆΠ΅Π½ΠΈΠ΅ ΠΌΠ΅ΠΆΠ΄Ρƒ Π½ΠΈΠΌΠΈ вычислитС ΡΠ°ΠΌΠΎΡΡ‚ΠΎΡΡ‚Π΅Π»ΡŒΠ½ΠΎ. Но эта Π½Π΅ СдинствСнный Π²ΠΈΠ΄ Π·Π°Π΄Π°Ρ‡ Π½Π° Π½Π°Ρ…ΠΎΠΆΠ΄Π΅Π½ΠΈΠ΅ Π²Π½ΡƒΡ‚Ρ€Π΅Π½Π½Π΅Π³ΠΎ сопротивлСния источника. Π•ΡΡ‚ΡŒ ΠΈ Ρ‚Π°ΠΊΠΈΠ΅, Π² ΠΊΠΎΡ‚ΠΎΡ€Ρ‹Ρ… вмСсто силы Ρ‚ΠΎΠΊΠ° ΡƒΠΊΠ°Π·Π°Π½ΠΎ сопротивлСниС Π½Π°Π³Ρ€ΡƒΠ·ΠΊΠΈ, Π° ΠΎΡΡ‚Π°Π»ΡŒΠ½Ρ‹Π΅ исходныС Π΄Π°Π½Π½Ρ‹Π΅ Ρ‚Π°ΠΊΠΈΠ΅ ΠΆΠ΅. Π’ΠΎΠ³Π΄Π° Π½Π°Π΄ΠΎ Π²Π½Π°Ρ‡Π°Π»Π΅ ΠΏΡ€ΠΎΠ΄Π΅Π»Π°Ρ‚ΡŒ Π΅Ρ‰Ρ‘ ΠΎΠ΄Π½ΠΎ вычислСниС. ΠŸΡ€ΠΈΠ²Π΅Π΄Ρ‘Π½Π½ΠΎΠ΅ Π² условии напряТСниС ΠΏΠΎΠ΄ Π½Π°Π³Ρ€ΡƒΠ·ΠΊΠΎΠΉ (Π½Π΅ Π­Π”Π‘!) ΠΏΠΎΠ΄Π΅Π»ΠΈΡ‚ΡŒ Π½Π° сопротивлСниС Π½Π°Π³Ρ€ΡƒΠ·ΠΊΠΈ. И получится сила Ρ‚ΠΎΠΊΠ° Π² Ρ†Π΅ΠΏΠΈ. ПослС Ρ‡Π΅Π³ΠΎ, ΠΊΠ°ΠΊ говорят Ρ„ΠΈΠ·ΠΈΠΊΠΈ, β€œΠ·Π°Π΄Π°Ρ‡Π° свСдСна ΠΊ прСдыдущСй”! ΠŸΠΎΠΏΡ€ΠΎΠ±ΡƒΠΉΡ‚Π΅ ΡΠΎΡΡ‚Π°Π²ΠΈΡ‚ΡŒ Ρ‚Π°ΠΊΡƒΡŽ Π·Π°Π΄Π°Ρ‡Ρƒ ΠΈ Ρ€Π΅ΡˆΠΈΡ‚ΡŒ Π΅Ρ‘.

Как Π½Π°ΠΉΡ‚ΠΈ Π²Π½ΡƒΡ‚Ρ€Π΅Π½Π½Π΅Π΅ сопротивлСниС Π°ΠΌΠΏΠ΅Ρ€ΠΌΠ΅Ρ‚Ρ€Π°

Лабораторная Ρ€Π°Π±ΠΎΡ‚Π° β„–2

Π’Π΅ΠΌΠ°. Π˜Π·ΠΌΠ΅Ρ€Π΅Π½ΠΈΠ΅ напряТСния Π² цСпях постоянного Ρ‚ΠΎΠΊΠ°

ЦСль. ΠŸΡ€ΠΎΠ²Π΅ΡΡ‚ΠΈ ΠΈΠ·ΠΌΠ΅Ρ€Π΅Π½ΠΈΠ΅ напряТСния Π² Ρ†Π΅ΠΏΠΈ постоянного Ρ‚ΠΎΠΊΠ° прямым ΠΌΠ΅Ρ‚ΠΎΠ΄ΠΎΠΌ. ΠžΠ·Π½Π°ΠΊΠΎΠΌΠΈΡ‚ΡΡ со способам Ρ€Π°ΡΡˆΠΈΡ€Π΅Π½ΠΈΡ Π²Π΅Ρ€Ρ…Π½ΠΈΡ… ΠΏΡ€Π΅Π΄Π΅Π»ΠΎΠ² ΠΈΠ·ΠΌΠ΅Ρ€Π΅Π½ΠΈΠΉ Π²ΠΎΠ»ΡŒΡ‚ΠΌΠ΅Ρ‚Ρ€ΠΎΠ² постоянного Ρ‚ΠΎΠΊΠ°, с ΠΌΠ΅Ρ‚ΠΎΠ΄Π°ΠΌ расчСта Π΄ΠΎΠ±Π°Π²ΠΎΡ‡Π½Ρ‹Ρ… сопротивлСний.

ΠžΠΏΡ€Π΅Π΄Π΅Π»ΠΈΡ‚ΡŒ Π²Π½ΡƒΡ‚Ρ€Π΅Π½Π½Π΅ сопротивлСниС Π²ΠΎΠ»ΡŒΡ‚ΠΌΠ΅Ρ‚Ρ€Π°.

ΠžΠΏΡ€Π΅Π΄Π΅Π»ΠΈΡ‚ΡŒ сопротивлСниС Π΄ΠΎΠ±Π°Π²ΠΎΡ‡Π½ΠΎΠ³ΠΎ рСзистора.

Π‘Π΄Π΅Π»Π°Ρ‚ΡŒ Π²Ρ‹Π²ΠΎΠ΄Ρ‹ ΠΏΠΎ Ρ€Π΅Π·ΡƒΠ»ΡŒΡ‚Π°Ρ‚Π°ΠΌ Ρ€Π°Π±ΠΎΡ‚Ρ‹.

Для измСрСния напряТСния U, Π΄Π΅ΠΉΡΡ‚Π²ΡƒΡŽΡ‰Π΅Π³ΠΎ ΠΌΠ΅ΠΆΠ΄Ρƒ ΠΊΠ°ΠΊΠΈΠΌΠΈ-Π»ΠΈΠ±ΠΎ двумя Ρ‚ΠΎΡ‡ΠΊΠ°ΠΌΠΈ элСктричСской Ρ†Π΅ΠΏΠΈ, Π²ΠΎΠ»ΡŒΡ‚ΠΌΠ΅Ρ‚Ρ€ 2 (рис. 1, Π°) ΠΏΡ€ΠΈΡΠΎΠ΅Π΄ΠΈΠ½ΡΡŽΡ‚ ΠΊ этим Ρ‚ΠΎΡ‡ΠΊΠ°ΠΌ, Ρ‚. Π΅. ΠΏΠ°Ρ€Π°Π»Π»Π΅Π»ΡŒΠ½ΠΎ источнику 1 элСктричСской энСргии ΠΈΠ»ΠΈ ΠΏΡ€ΠΈΠ΅ΠΌΠ½ΠΈΠΊΡƒ 3.

Для Ρ‚ΠΎΠ³ΠΎ Ρ‡Ρ‚ΠΎΠ±Ρ‹ Π²ΠΊΠ»ΡŽΡ‡Π΅Π½ΠΈΠ΅ Π²ΠΎΠ»ΡŒΡ‚ΠΌΠ΅Ρ‚Ρ€Π° Π½Π΅ ΠΎΠΊΠ°Π·Ρ‹Π²Π°Π»ΠΎ влияния Π½Π° Ρ€Π°Π±ΠΎΡ‚Ρƒ элСктричСских установок ΠΈ ΠΎΠ½ Π½Π΅ создавал Π±ΠΎΠ»ΡŒΡˆΠΈΡ… ΠΏΠΎΡ‚Π΅Ρ€ΡŒ энСргии, Π²ΠΎΠ»ΡŒΡ‚ΠΌΠ΅Ρ‚Ρ€Ρ‹ Π²Ρ‹ΠΏΠΎΠ»Π½ΡΡŽΡ‚ с большим сопротивлСниСм. ΠŸΠΎΡΡ‚ΠΎΠΌΡƒ практичСски ΠΌΠΎΠΆΠ½ΠΎ ΠΏΡ€Π΅Π½Π΅Π±Ρ€Π΅Π³Π°Ρ‚ΡŒ проходящим ΠΏΠΎ Π²ΠΎΠ»ΡŒΡ‚ΠΌΠ΅Ρ‚Ρ€Ρƒ Ρ‚ΠΎΠΊΠΎΠΌ.

Для Ρ€Π°ΡΡˆΠΈΡ€Π΅Π½ΠΈΡ ΠΏΡ€Π΅Π΄Π΅Π»ΠΎΠ² измСрСния Π²ΠΎΠ»ΡŒΡ‚ΠΌΠ΅Ρ‚Ρ€ΠΎΠ² ΠΏΠΎΡΠ»Π΅Π΄ΠΎΠ²Π°Ρ‚Π΅Π»ΡŒΠ½ΠΎ с ΠΎΠ±ΠΌΠΎΡ‚ΠΊΠΎΠΉ ΠΏΡ€ΠΈΠ±ΠΎΡ€Π° Π²ΠΊΠ»ΡŽΡ‡Π°ΡŽΡ‚ Π΄ΠΎΠ±Π°Π²ΠΎΡ‡Π½Ρ‹ΠΉ рСзистор 4 (RΠ”) (рис. 1, Π±). ΠŸΡ€ΠΈ этом Π½Π° ΠΏΡ€ΠΈΠ±ΠΎΡ€ приходится лишь Ρ‡Π°ΡΡ‚ΡŒ UV измСряСмого напряТСния U, ΠΏΡ€ΠΎΠΏΠΎΡ€Ρ†ΠΈΠΎΠ½Π°Π»ΡŒΠ½Π°Ρ ΡΠΎΠΏΡ€ΠΎΡ‚ΠΈΠ²Π»Π΅Π½ΠΈΡŽ ΠΏΡ€ΠΈΠ±ΠΎΡ€Π° RV.

Зная сопротивлСниС Π΄ΠΎΠ±Π°Π²ΠΎΡ‡Π½ΠΎΠ³ΠΎ рСзистора ΠΈ Π²ΠΎΠ»ΡŒΡ‚ΠΌΠ΅Ρ‚Ρ€Π°, ΠΌΠΎΠΆΠ½ΠΎ ΠΏΠΎ Π·Π½Π°Ρ‡Π΅Π½ΠΈΡŽ напряТСния UV, фиксируСмого Π²ΠΎΠ»ΡŒΡ‚ΠΌΠ΅Ρ‚Ρ€ΠΎΠΌ, ΠΎΠΏΡ€Π΅Π΄Π΅Π»ΠΈΡ‚ΡŒ напряТСниС, Π΄Π΅ΠΉΡΡ‚Π²ΡƒΡŽΡ‰Π΅Π΅ Π² Ρ†Π΅ΠΏΠΈ:

(1)

Π’Π΅Π»ΠΈΡ‡ΠΈΠ½Π° ΠΏΠΎΠΊΠ°Π·Ρ‹Π²Π°Π΅Ρ‚, Π²ΠΎ сколько Ρ€Π°Π· измСряСмоС напряТСниС U большС напряТСния UV, приходящСгося Π½Π° ΠΏΡ€ΠΈΠ±ΠΎΡ€, Ρ‚. Π΅. Π²ΠΎ сколько Ρ€Π°Π· увСличиваСтся ΠΏΡ€Π΅Π΄Π΅Π» измСрСния напряТСния Π²ΠΎΠ»ΡŒΡ‚ΠΌΠ΅Ρ‚Ρ€ΠΎΠΌ ΠΏΡ€ΠΈ ΠΏΡ€ΠΈΠΌΠ΅Π½Π΅Π½ΠΈΠΈ Π΄ΠΎΠ±Π°Π²ΠΎΡ‡Π½ΠΎΠ³ΠΎ рСзистора.

Π‘ΠΎΠΏΡ€ΠΎΡ‚ΠΈΠ²Π»Π΅Π½ΠΈΠ΅ Π΄ΠΎΠ±Π°Π²ΠΎΡ‡Π½ΠΎΠ³ΠΎ рСзистора, Π½Π΅ΠΎΠ±Ρ…ΠΎΠ΄ΠΈΠΌΠΎΠ΅ для измСрСния напряТСния U, Π² n Ρ€Π°Π· большСго напряТСния ΠΏΡ€ΠΈΠ±ΠΎΡ€Π° UV, опрСдСляСтся ΠΏΠΎ Ρ„ΠΎΡ€ΠΌΡƒΠ»Π΅

(2)

Π”ΠΎΠ±Π°Π²ΠΎΡ‡Π½Ρ‹ΠΉ рСзистор ΠΌΠΎΠΆΠ΅Ρ‚ Π²ΡΡ‚Ρ€Π°ΠΈΠ²Π°Ρ‚ΡŒΡΡ Π² ΠΏΡ€ΠΈΠ±ΠΎΡ€ ΠΈ ΠΎΠ΄Π½ΠΎΠ²Ρ€Π΅ΠΌΠ΅Π½Π½ΠΎ ΠΈΡΠΏΠΎΠ»ΡŒΠ·ΠΎΠ²Π°Ρ‚ΡŒΡΡ для ΡƒΠΌΠ΅Π½ΡŒΡˆΠ΅Π½ΠΈΡ влияния Ρ‚Π΅ΠΌΠΏΠ΅Ρ€Π°Ρ‚ΡƒΡ€Ρ‹ ΠΎΠΊΡ€ΡƒΠΆΠ°ΡŽΡ‰Π΅ΠΉ срСды Π½Π° показания ΠΏΡ€ΠΈΠ±ΠΎΡ€Π°. Для этой Ρ†Π΅Π»ΠΈ рСзистор выполняСтся ΠΈΠ· ΠΌΠ°Ρ‚Π΅Ρ€ΠΈΠ°Π»Π°, ΠΈΠΌΠ΅ΡŽΡ‰Π΅Π³ΠΎ ΠΌΠ°Π»Ρ‹ΠΉ Ρ‚Π΅ΠΌΠΏΠ΅Ρ€Π°Ρ‚ΡƒΡ€Π½Ρ‹ΠΉ коэффициСнт, ΠΈ Π΅Π³ΠΎ сопротивлСниС Π·Π½Π°Ρ‡ΠΈΡ‚Π΅Π»ΡŒΠ½ΠΎ ΠΏΡ€Π΅Π²Ρ‹ΡˆΠ°Π΅Ρ‚ сопротивлСниС ΠΊΠ°Ρ‚ΡƒΡˆΠΊΠΈ, вслСдствиС Ρ‡Π΅Π³ΠΎ ΠΎΠ±Ρ‰Π΅Π΅ сопротивлСниС ΠΏΡ€ΠΈΠ±ΠΎΡ€Π° становится ΠΏΠΎΡ‡Ρ‚ΠΈ нСзависимым ΠΎΡ‚ измСнСния Ρ‚Π΅ΠΌΠΏΠ΅Ρ€Π°Ρ‚ΡƒΡ€Ρ‹. По точности Π΄ΠΎΠ±Π°Π²ΠΎΡ‡Π½Ρ‹Π΅ рСзисторы ΠΏΠΎΠ΄Ρ€Π°Π·Π΄Π΅Π»ΡΡŽΡ‚ΡΡ Π½Π° Ρ‚Π΅ ΠΆΠ΅ классы точности, Ρ‡Ρ‚ΠΎ ΠΈ ΡˆΡƒΠ½Ρ‚Ρ‹.

Рис. 1. Π‘Ρ…Π΅ΠΌΡ‹ для измСрСния напряТСния

Π’Π½ΡƒΡ‚Ρ€Π΅Π½Π½Π΅Π΅ сопротивлСниС Π²ΠΎΠ»ΡŒΡ‚ΠΌΠ΅Ρ‚Ρ€Π°. Для опрСдСлСния Π²Π½ΡƒΡ‚Ρ€Π΅Π½Π½Π΅Π³ΠΎ сопротивлСния Π²ΠΎΠ»ΡŒΡ‚ΠΌΠ΅Ρ‚Ρ€Π° Π΅Π³ΠΎ Π½Π΅ΠΎΠ±Ρ…ΠΎΠ΄ΠΈΠΌΠΎ ΠΏΠΎΠ΄ΠΊΠ»ΡŽΡ‡ΠΈΡ‚ΡŒ ΠΏΠΎΡΠ»Π΅Π΄ΠΎΠ²Π°Ρ‚Π΅Π»ΡŒΠ½ΠΎ с рСзистором (с ) ΠΊ источнику напряТСния (рис. 2) . НапряТСниС источника подСлится ΠΌΠ΅ΠΆΠ΄Ρƒ рСзистором ΠΈ Π²ΠΎΠ»ΡŒΡ‚ΠΌΠ΅Ρ‚Ρ€ΠΎΠΌ ΠΏΡ€ΠΎΠΏΠΎΡ€Ρ†ΠΈΠΎΠ½Π°Π»ΡŒΠ½ΠΎ ΠΈΡ… сопротивлСниям

(3)

(4)

Рис. 2. Π‘Ρ…Π΅ΠΌΠ° для опрСдСлСния Π²Π½ΡƒΡ‚Ρ€Π΅Π½Π½Π΅Π³ΠΎ сопротивлСния Π²ΠΎΠ»ΡŒΡ‚ΠΌΠ΅Ρ‚Ρ€Π°

Π‘ΠΎΠ±Ρ€Π°Ρ‚ΡŒ схСму, ΠΊΠ°ΠΊ Π½Π° рис. 2.

БопротивлСния R Π²Ρ‹Π±Ρ€Π°Ρ‚ΡŒ с ΠΏΠΎΠΌΠΎΡ‰ΡŒΡŽ ΠΌΠ°Π³Π°Π·ΠΈΠ½ΠΎΠ² сопротивлСний Ρ‚Π°ΠΊ, Ρ‡Ρ‚ΠΎΠ±Ρ‹ напряТСния UR ΠΈ UV Π±Ρ‹Π»ΠΈ ΠΏΡ€ΠΈΠ±Π»ΠΈΠ·ΠΈΡ‚Π΅Π»ΡŒΠ½ΠΎ равнями ().

Π‘ΠΎΠ±Ρ€Π°Ρ‚ΡŒ схСму, ΠΊΠ°ΠΊ Π½Π° рис. 3 ( ).

Π Π°ΡΡΡ‡ΠΈΡ‚Π°Ρ‚ΡŒ Π·Π½Π°Ρ‡Π΅Π½ΠΈΠ΅ Π΄ΠΎΠ±Π°Π²ΠΎΡ‡Π½ΠΎΠ³ΠΎ сопротивлСния RΠ” (2) для ΠΊΠ°ΠΆΠ΄ΠΎΠ³ΠΎ ΠΈΠ· Π·Π½Π°Ρ‡Π΅Π½ΠΈΠΉ UИП ΠΈ коэффициСнт n.

Π˜ΡΠΏΠΎΠ»ΡŒΠ·ΡƒΡ ΠΏΠΎΠΊΠ°Π·Π°Π½ΠΈΠ΅ Π²ΠΎΠ»ΡŒΡ‚ΠΌΠ΅Ρ‚Ρ€Π° 15 Π’ ΠΈ Π·Π½Π°Ρ‡Π΅Π½ΠΈΠ΅ коэффициСнта n, ΠΎΠΏΡ€Π΅Π΄Π΅Π»ΠΈΡ‚ΡŒ напряТСниС UИП (1). Π£Π±Π΅Π΄ΠΈΡ‚ΡŒΡΡ, Ρ‡Ρ‚ΠΎ Π΅Π³ΠΎ Π·Π½Π°Ρ‡Π΅Π½ΠΈΠ΅ совпадаСт с Π΄Π΅ΠΉΡΡ‚Π²ΠΈΡ‚Π΅Π»ΡŒΠ½Ρ‹ΠΌ.

Π—Π°ΠΏΠΎΠ»Π½ΠΈΡ‚ΡŒ Ρ‚Π°Π±Π»ΠΈΡ†Ρƒ 1.

Рис. 3. ИспользованиС Π΄ΠΎΠ±Π°Π²ΠΎΡ‡Π½ΠΎΠ³ΠΎ сопротивлСния

Для контроля Π²Π΅Π»ΠΈΡ‡ΠΈΠ½Ρ‹ Ρ‚ΠΎΠΊΠ° примСняСтся ΠΏΡ€ΠΈΠ±ΠΎΡ€ Π½Π°Π·Ρ‹Π²Π°Π΅ΠΌΡ‹ΠΉ Π°ΠΌΠΏΠ΅Ρ€ΠΌΠ΅Ρ‚Ρ€ΠΎΠΌ. Из ΠΏΡ€Π°ΠΊΡ‚ΠΈΠΊΠΈ ΠΌΠΎΠ³Ρƒ ΡΠΊΠ°Π·Π°Ρ‚ΡŒ, Ρ‡Ρ‚ΠΎ Π½Π΅ всСгда ΠΏΠΎΠ΄ Ρ€ΡƒΠΊΠΎΠΉ оказываСтся ΠΏΡ€ΠΈΠ±ΠΎΡ€ с Π½ΡƒΠΆΠ½Ρ‹ΠΌ Π΄ΠΈΠ°ΠΏΠ°Π·ΠΎΠ½ΠΎΠΌ измСрСния. Как ΠΏΡ€Π°Π²ΠΈΠ»ΠΎ, Π΄ΠΈΠ°ΠΏΠ°Π·ΠΎΠ½ Π»ΠΈΠ±ΠΎ ΠΌΠ°Π», Π»ΠΈΠ±ΠΎ Π²Π΅Π»ΠΈΠΊ. Π—Π΄Π΅ΡΡŒ ΠΌΡ‹ Ρ€Π°Π·Π±Π΅Ρ€Π΅ΠΌ, ΠΊΠ°ΠΊ ΠΈΠ·ΠΌΠ΅Π½ΠΈΡ‚ΡŒ Ρ€Π°Π±ΠΎΡ‡ΠΈΠΉ Π΄ΠΈΠ°ΠΏΠ°Π·ΠΎΠ½ Π°ΠΌΠΏΠ΅Ρ€ΠΌΠ΅Ρ‚Ρ€Π°. АмпСрмСтры Π½Π° большиС Ρ‚ΠΎΠΊΠΈ ΠΎΡ‚ 20 Π°ΠΌΠΏΠ΅Ρ€ ΠΈ Π²Ρ‹ΡˆΠ΅ ΠΈΠΌΠ΅ΡŽΡ‚ Π² своём составС внСшний ΡˆΡƒΠ½Ρ‚ΠΈΡ€ΡƒΡŽΡ‰ΠΈΠΉ рСзистор. Он ΠΏΠΎΠ΄ΠΊΠ»ΡŽΡ‡Π°Π΅Ρ‚ΡΡ ΠΏΠ°Ρ€Π°Π»Π»Π΅Π»ΡŒΠ½ΠΎ Π°ΠΌΠΏΠ΅Ρ€ΠΌΠ΅Ρ‚Ρ€Ρƒ. На рисункС 1 ΠΏΡ€ΠΈΠ²Π΅Π΄Π΅Π½Π° схСма Π²ΠΊΠ»ΡŽΡ‡Π΅Π½ΠΈΡ Π°ΠΌΠΏΠ΅Ρ€ΠΌΠ΅Ρ‚Ρ€Π° с ΡˆΡƒΠ½Ρ‚ΠΈΡ€ΡƒΡŽΡ‰Π΅ΠΌ рСзистором.

Π’ качСствС ΠΏΡ€ΠΈΠΌΠ΅Ρ€Π° Π² экспСримСнтах Π±ΡƒΠ΄Π΅Ρ‚ использован Π°ΠΌΠΏΠ΅Ρ€ΠΌΠ΅Ρ‚Ρ€ M367 со шкалой Π΄ΠΎ 150 Π°ΠΌΠΏΠ΅Ρ€, соотвСтствСнно ΠΏΡ€ΠΈ Ρ‚Π°ΠΊΠΎΠΌ Ρ‚ΠΎΠΊΠ΅ Π°ΠΌΠΏΠ΅Ρ€ΠΌΠ΅Ρ‚Ρ€ ΠΈΡΠΏΠΎΠ»ΡŒΠ·ΡƒΠ΅Ρ‚ΡΡ с внСшним ΡˆΡƒΠ½Ρ‚ΠΈΡ€ΡƒΡŽΡ‰ΠΈΠΌ сопротивлСниСм.

Если ΡƒΠ±Ρ€Π°Ρ‚ΡŒ ΡˆΡƒΠ½Ρ‚ΠΈΡ€ΡƒΡŽΡ‰ΠΈΠΉ рСзистор, Ρ‚ΠΎ Π°ΠΌΠΏΠ΅Ρ€ΠΌΠ΅Ρ‚Ρ€ станСт ΠΌΠΈΠ»Π»ΠΈΠ°ΠΌΠΏΠ΅Ρ€ΠΌΠ΅Ρ‚Ρ€ΠΎΠΌ с ΠΌΠ°ΠΊΡΠΈΠΌΠ°Π»ΡŒΠ½Ρ‹ΠΌ Ρ‚ΠΎΠΊΠΎΠΌ отклонСния стрСлки 30 мА (Π΄Π°Π»Π΅Π΅ Π±ΡƒΠ΄Π΅Ρ‚ пояснСниС, ΠΎΡ‚ΠΊΡƒΠ΄Π° это Π·Π½Π°Ρ‡Π΅Π½ΠΈΠ΅ взялось). Π’Π°ΠΊΠΈΠΌ ΠΎΠ±Ρ€Π°Π·ΠΎΠΌ, ΠΈΡΠΏΠΎΠ»ΡŒΠ·ΡƒΡ Ρ€Π°Π·Π½Ρ‹Π΅ ΡˆΡƒΠ½Ρ‚ΠΈΡ€ΡƒΡŽΡ‰ΠΈΠ΅ сопротивлСния ΠΌΠΎΠΆΠ½ΠΎ ΡΠ΄Π΅Π»Π°Ρ‚ΡŒ Π°ΠΌΠΏΠ΅Ρ€ΠΌΠ΅Ρ‚Ρ€ практичСски с Π»ΡŽΠ±Ρ‹ΠΌ Π΄ΠΈΠ°ΠΏΠ°Π·ΠΎΠ½ΠΎΠΌ измСрСния.

Рассмотрим ΠΏΠΎΠ΄Ρ€ΠΎΠ±Π½Π΅Π΅ ΠΈΠΌΠ΅ΡŽΡ‰ΠΈΠΉΡΡ ΠΈΠ·ΠΌΠ΅Ρ€ΠΈΡ‚Π΅Π»ΡŒΠ½Ρ‹ΠΉ ΠΏΡ€ΠΈΠ±ΠΎΡ€. Из Π΅Π³ΠΎ ΠΌΠ°Ρ€ΠΊΠΈΡ€ΠΎΠ²ΠΎΠΊ ΠΌΠΎΠΆΠ½ΠΎ ΡƒΠ·Π½Π°Ρ‚ΡŒ ΡΠ»Π΅Π΄ΡƒΡŽΡ‰Π΅Π΅. ΠœΠ°Ρ€ΠΊΠΈΡ€ΠΎΠ²ΠΊΠ° Π² Π²Π΅Ρ€Ρ…Π½Π΅ΠΌ ΠΏΡ€Π°Π²ΠΎΠΌ ΡƒΠ³Π»Ρƒ (Ρ†ΠΈΡ„Ρ€Π° 1 Π½Π° ΠΈΠ·ΠΎΠ±Ρ€Π°ΠΆΠ΅Π½ΠΈΠΈ). МодСль ΠΈΠ·ΠΌΠ΅Ρ€ΠΈΡ‚Π΅Π»ΡŒΠ½ΠΎΠΉ Π³ΠΎΠ»ΠΎΠ²ΠΊΠΈ М367. Π‘Π΄Π΅Π»Π°Π½ Π½Π° краснодарском Π·Π°Π²ΠΎΠ΄Π΅ ΠΈΠ·ΠΌΠ΅Ρ€ΠΈΡ‚Π΅Π»ΡŒΠ½Ρ‹Ρ… ΠΏΡ€ΠΈΠ±ΠΎΡ€ΠΎΠ² (это ΠΌΠΎΠΆΠ½ΠΎ ΠΎΠΏΡ€Π΅Π΄Π΅Π»ΠΈΡ‚ΡŒ ΠΏΠΎ Ρ€ΠΎΠΌΠ±ΠΈΠΊΡƒ с Π±ΡƒΠΊΠΎΠ²ΠΊΠ°ΠΌΠΈ Π—Π˜ΠŸ). Π“ΠΎΠ΄ выпуска 1973. Π‘Π΅Ρ€ΠΈΠΉΠ½Ρ‹ΠΉ Π½ΠΎΠΌΠ΅Ρ€ 165266.

ΠœΠ°Ρ€ΠΊΠΈΡ€ΠΎΠ²ΠΊΠ° Π² Π½ΠΈΠΆΠ½Π΅ΠΌ Π»Π΅Π²ΠΎΠΌ ΡƒΠ³Π»Ρƒ (Ρ†ΠΈΡ„Ρ€Π° 2 Π½Π° ΠΈΠ·ΠΎΠ±Ρ€Π°ΠΆΠ΅Π½ΠΈΠΈ). Π‘Π»Π΅Π²Π° Π½Π° ΠΏΡ€Π°Π²ΠΎ. ΠŸΡ€ΠΈΠ±ΠΎΡ€ ΠΏΡ€Π΅Π΄Π½Π°Π·Π½Π°Ρ‡Π΅Π½ для измСрСния постоянного Ρ‚ΠΎΠΊΠ°. ΠœΠ°Π³Π½ΠΈΡ‚ΠΎΡΠ»Π΅ΠΊΡ‚Ρ€ΠΈΡ‡Π΅ΡΠΊΠΈΠΉ ΠΏΡ€ΠΈΠ±ΠΎΡ€ с ΠΏΠΎΠ΄Π²ΠΈΠΆΠ½ΠΎΠΉ Ρ€Π°ΠΌΠΊΠΎΠΉ. НапряТСниС ΠΌΠ΅ΠΆΠ΄Ρƒ корпусом ΠΈ мангнитоэлСктричСской систСмой Π½Π΅ Π΄ΠΎΠ»ΠΆΠ½ΠΎ ΠΏΡ€Π΅Π²Ρ‹ΡˆΠ°Ρ‚ΡŒ 2 ΠšΠ’. Π Π°Π±ΠΎΡ‡Π΅Π΅ ΠΏΠΎΠ»ΠΎΠΆΠ΅Π½ΠΈΠ΅ ΡˆΠΊΠ°Π»Ρ‹ ΠΏΡ€ΠΈΠ±ΠΎΡ€Π° Π²Π΅Ρ€Ρ‚ΠΈΠΊΠ°Π»ΡŒΠ½ΠΎΠ΅. Класс точности ΠΏΡ€ΠΈΠ±ΠΎΡ€Π° Π² ΠΏΡ€ΠΎΡ†Π΅Π½Ρ‚Π°Ρ… 1,5. Π“ΠžΠ‘Π’8711-60. Π˜Π·ΠΌΠ΅Ρ€ΠΈΡ‚Π΅Π»ΡŒΠ½Π°Ρ Π³ΠΎΠ»ΠΎΠ²ΠΊΠ° рассчитана Π½Π° измСрСния силы Ρ‚ΠΎΠΊΠ° Π΄ΠΎ 150 Π°ΠΌΠΏΠ΅Ρ€ с использованиСм внСшнСго ΡˆΡƒΠ½Ρ‚ΠΈΡ€ΡƒΡŽΡ‰Π΅Π³ΠΎ сопротивлСния рассчитанного Π½Π° ΠΏΠ°Π΄Π΅Π½ΠΈΠ΅ Π½Π° Π½Ρ‘ΠΌ напряТСния Π½ΠΎΠΌΠΈΠ½Π°Π»ΠΎΠΌ Π² 75 ΠΌΠΈΠ»Π»ΠΈΠ²ΠΎΠ»ΡŒΡ‚.

Π˜Ρ‚Π°ΠΊ, это максимум Ρ‡Ρ‚ΠΎ ΡƒΠ΄Π°Π»ΠΎΡΡŒ ΡƒΠ·Π½Π°Ρ‚ΡŒ ΠΈΠ· ΠΌΠ°Ρ€ΠΊΠΈΡ€ΠΎΠ²ΠΊΠΈ Π°ΠΌΠΏΠ΅Ρ€ΠΌΠ΅Ρ‚Ρ€Π°. Π’Π΅ΠΏΠ΅Ρ€ΡŒ ΠΏΠ΅Ρ€Π΅ΠΉΠ΄Ρ‘ΠΌ ΠΊ расчСтам. Π‘ΠΎΠΏΡ€ΠΎΡ‚ΠΈΠ²Π»Π΅Π½ΠΈΠ΅ ΡˆΡƒΠ½Ρ‚Π° опрСдСляСтся ΠΏΠΎ Ρ„ΠΎΡ€ΠΌΡƒΠ»Π΅:

Π³Π΄Π΅ :
Rш – сопротивлСниС ΡˆΡƒΠ½Ρ‚ΠΈΡ€ΡƒΡŽΡ‰Π΅Π³ΠΎ рСзистора;
RΠΏΡ€ΠΈΠ± – Π²Π½ΡƒΡ‚Ρ€Π΅Π½Π½Π΅Π΅ сопротивлСниС Π°ΠΌΠΏΠ΅Ρ€ΠΌΠ΅Ρ‚Ρ€Π°;
IΠΏΡ€ΠΈΠ± – максимально ΠΈΠ·ΠΌΠ΅Ρ€ΠΈΠΌΡ‹ΠΉ Ρ‚ΠΎΠΊ Π°ΠΌΠΏΠ΅Ρ€ΠΌΠ΅Ρ‚Ρ€ΠΎΠΌ Π±Π΅Π· ΡˆΡƒΠ½Ρ‚Π°;
IΡ€Π°Π± – максимально ΠΈΠ·ΠΌΠ΅Ρ€ΠΈΠΌΡ‹ΠΉ Ρ‚ΠΎΠΊ с ΡˆΡƒΠ½Ρ‚ΠΎΠΌ (Ρ‚Ρ€Π΅Π±ΡƒΠ΅ΠΌΠΎΠ΅ Π·Π½Π°Ρ‡Π΅Π½ΠΈΠ΅)

Если всС Π΄Π°Π½Π½Ρ‹Π΅ для расчёта ΠΈΠΌΠ΅ΡŽΡ‚ΡΡ, Ρ‚ΠΎ ΠΌΠΎΠΆΠ½ΠΎ ΠΏΡ€ΠΈΡΡ‚ΡƒΠΏΠ°Ρ‚ΡŒ ΠΊ самому расчёту. Для упрощСния ΠΌΠΎΠΆΠ½ΠΎ Π²ΠΎΡΠΏΠΎΠ»ΡŒΠ·ΠΎΠ²Π°Ρ‚ΡŒΡΡ ΠΎΠ½Π»Π°ΠΉΠ½ ΠΊΠ°Π»ΡŒΠΊΡƒΠ»ΡΡ‚ΠΎΡ€ΠΎΠΌ Π½ΠΈΠΆΠ΅:

Π’ нашСм случаС ΠΈΠ· Ρ„ΠΎΡ€ΠΌΡƒΠ»Ρ‹ Π²ΠΈΠ΄Π½ΠΎ, Ρ‡Ρ‚ΠΎ Π΄Π°Π½Π½Ρ‹Ρ… Π½Π΅ достаточно. Нам извСстСн Ρ‚ΠΎΠ»ΡŒΠΊΠΎ ΠΌΠ°ΠΊΡΠΈΠΌΠ°Π»ΡŒΠ½Ρ‹ΠΉ измСряСмый Ρ‚ΠΎΠΊ с ΡˆΡƒΠ½Ρ‚ΠΎΠΌ. Π’ΠΎ Π΅ΡΡ‚ΡŒ, Ρ‚ΠΎ, Ρ‡Ρ‚ΠΎ ΠΌΡ‹ Ρ…ΠΎΡ‚ΠΈΠΌ Π²ΠΈΠ΄Π΅Ρ‚ΡŒ Π² случаС максимального отклонСния стрСлки Π°ΠΌΠΏΠ΅Ρ€ΠΌΠ΅Ρ‚Ρ€Π°.

Из ΠΌΠ°Ρ€ΠΊΠΈΡ€ΠΎΠ²ΠΊΠΈ ΠΏΡ€ΠΈΠ±ΠΎΡ€Π° ΡƒΠ΄Π°Π»ΠΎΡΡŒ ΡƒΠ·Π½Π°Ρ‚ΡŒ ΠΏΠ°Π΄Π΅Π½ΠΈΠ΅ напряТСния Π½Π° ΡˆΡƒΠ½Ρ‚ΠΈΡ€ΡƒΡŽΡ‰Π΅ΠΌ сопротивлСнии. И это ΡƒΠΆΠ΅ Ρ‡Ρ‚ΠΎ-Ρ‚ΠΎ. Из этого ΠΏΠ°Ρ€Π°ΠΌΠ΅Ρ‚Ρ€Π° ясно, Ρ‡Ρ‚ΠΎ ΠΏΡ€ΠΈ ΠΏΠΎΠ΄Π°Ρ‡Π΅ Π½Π° ΠΏΡ€ΠΈΠ±ΠΎΡ€ напряТСния Π½ΠΎΠΌΠΈΠ½Π°Π»ΠΎΠΌ 0,075 Π²ΠΎΠ»ΡŒΡ‚ (75ΠΌΠ’) стрСлка ΠΎΡ‚ΠΊΠ»ΠΎΠ½ΠΈΡ‚ΡŒΡΡ Π΄ΠΎ ΠΊΡ€Π°ΠΉΠ½Π΅Π³ΠΎ значСния Π½Π° шкалС 150 Π°ΠΌΠΏΠ΅Ρ€. Π’Π°ΠΊΠΈΠΌ ΠΎΠ±Ρ€Π°Π·ΠΎΠΌ, получаСтся, Ρ‡Ρ‚ΠΎ максимальноС ΠΎΡ‚ΠΊΠ»ΠΎΠ½Π΅Π½ΠΈΠ΅ стрСлки ΠΏΡ€ΠΈΠ±ΠΎΡ€Π° достигаСтся ΠΏΠΎΠ΄Π°Ρ‡Π΅ΠΉ напряТСния 75 ΠΌΠ’. Π’Ρ€ΠΎΠ΄Π΅ ΠΊΠ°ΠΊ Π΄Π°Π½Π½Ρ‹Ρ… для расчСта ΠΏΠΎ-ΠΏΡ€Π΅ΠΆΠ½Π΅ΠΌΡƒ Π½Π΅ Ρ…Π²Π°Ρ‚Π°Π΅Ρ‚. НСобходимо ΡƒΠ·Π½Π°Ρ‚ΡŒ сопротивлСниС ΠΏΡ€ΠΈΠ±ΠΎΡ€Π° ΠΈ Ρ‚ΠΎΠΊ, ΠΏΡ€ΠΈ ΠΊΠΎΡ‚ΠΎΡ€ΠΎΠΌ стрСлка откланяСтся Π΄ΠΎ максимального значСния Π±Π΅Π· ΡˆΡƒΠ½Ρ‚ΠΈΡ€ΡƒΡŽΡ‰Π΅Π³ΠΎ рСзистора. Π”Π°Π»Π΅Π΅ ΠΏΡ€Π΅Π΄Π»Π°Π³Π°ΡŽ нСсколько способов для опрСдСлСния Π½ΡƒΠΆΠ½Ρ‹Ρ… ΠΏΠ°Ρ€Π°ΠΌΠ΅Ρ‚Ρ€ΠΎΠ² ΠΈ Ρ€Π΅ΡˆΠ΅Π½ΠΈΡ Π·Π°Π΄Π°Ρ‡ΠΈ.

Бпособ ΠΏΠ΅Ρ€Π²Ρ‹ΠΉ. ΠŸΡ€ΠΈ ΠΏΠΎΠΌΠΎΡ‰ΠΈ Π±Π»ΠΎΠΊΠ° питания выясняСм максимальноС ΠΎΡ‚ΠΊΠ»ΠΎΠ½Π΅Π½ΠΈΠ΅ стрСлки ΠΏΠΎ Ρ‚ΠΎΠΊΡƒ ΠΈ Π½Π°ΠΏΡ€ΡΠΆΠ΅Π½ΠΈΡŽ Π±Π΅Π· ΡˆΡƒΠ½Ρ‚Π°. Π’ нашСм случаС напряТСниС ΡƒΠΆΠ΅ извСстно. Π•Π³ΠΎ Π·Π°ΠΌΠ΅Ρ€ΡΡ‚ΡŒ Π½Π΅ Π±ΡƒΠ΄Π΅ΠΌ. Π˜Π·ΠΌΠ΅Ρ€ΡΠ΅ΠΌ Ρ‚ΠΎΠΊ ΠΈ ΠΎΡ‚ΠΊΠ»ΠΎΠ½Π΅Π½ΠΈΠ΅ стрСлки. Π’Π°ΠΊ ΠΊΠ°ΠΊ Π±Π»ΠΎΠΊΠ° питания ΠΏΠΎΠ΄ Ρ€ΡƒΠΊΠΎΠΉ Π½Π΅ оказалось, Ρ‚ΠΎ ΠΏΡ€ΠΈΡˆΠ»ΠΎΡΡŒ Π²ΠΎΡΠΏΠΎΠ»ΡŒΠ·ΠΎΠ²Π°Ρ‚ΡŒΡΡ ΠΎΡ‡Π΅Π½ΡŒ разряТСной Π±Π°Ρ‚Π°Ρ€Π΅ΠΉΠΊΠΎΠΉ Ρ‚ΠΈΠΏΠ° АА. Π’ΠΎΠΊ, ΠΊΠΎΡ‚ΠΎΡ€Ρ‹ΠΉ Π±Π°Ρ‚Π°Ρ€Π΅ΠΉΠΊΠ° ΠΌΠΎΠ³Π»Π° Π΅Ρ‰Ρ‘ ΠΎΡ‚Π΄Π°Ρ‚ΡŒ, составил 12 мА (ΠΏΠΎ показаниям ΠΌΡƒΠ»ΡŒΡ‚ΠΈΠΌΠ΅Ρ‚Ρ€Π°). ΠŸΡ€ΠΈ этом Ρ‚ΠΎΠΊΠ΅ стрСлка ΠΏΡ€ΠΈΠ±ΠΎΡ€Π° ΠΎΡ‚ΠΊΠ»ΠΎΠ½ΠΈΠ»Π°ΡΡŒ Π΄ΠΎ значСния Π½Π° Ρ†ΠΈΡ„Π΅Ρ€Π±Π»Π°Ρ‚Π΅ 60А. Π”Π°Π»Π΅Π΅ опрСдСляСм Ρ†Π΅Π½Ρƒ дСлСния ΠΈ рассчитываСм ΠΏΠΎΠ»Π½ΠΎΠ΅ (максимальноС) ΠΎΡ‚ΠΊΠ»ΠΎΠ½Π΅Π½ΠΈΠ΅ стрСлки. ΠŸΠΎΡΠΊΠΎΠ»ΡŒΠΊΡƒ шкала Ρ†ΠΈΡ„Π΅Ρ€Π±Π»Π°Ρ‚Π° Π°ΠΌΠΏΠ΅Ρ€ΠΌΠ΅Ρ‚Ρ€Π° Ρ€Π°Π·ΠΌΠ΅Ρ‡Π΅Π½Π° Ρ€Π°Π²Π½ΠΎΠΌΠ΅Ρ€Π½ΠΎ, Ρ‚ΠΎ Π½Π΅ составит Ρ‚Ρ€ΡƒΠ΄Π° ΡƒΠ·Π½Π°Ρ‚ΡŒ (Ρ€Π°ΡΡΡ‡ΠΈΡ‚Π°Ρ‚ΡŒ) Ρ‚ΠΎΠΊ максимального отклонСния стрСлки.

Π¦Π΅Π½Π° дСлСния ΠΏΡ€ΠΈΠ±ΠΎΡ€Π° рассчитываСтся ΠΏΠΎ Ρ„ΠΎΡ€ΠΌΡƒΠ»Π΅:

Π³Π΄Π΅:
Ρ…1 – мСньшСС Π·Π½Π°Ρ‡Π΅Π½ΠΈΠ΅,
Ρ…2 – большСС Π·Π½Π°Ρ‡Π΅Π½ΠΈΠ΅,
n – количСство ΠΏΡ€ΠΎΠΌΠ΅ΠΆΡƒΡ‚ΠΊΠΎΠ² (ΠΎΡ‚Ρ€Π΅Π·ΠΊΠΎΠ²) ΠΌΠ΅ΠΆΠ΄Ρƒ значСниями

Для упрощСния ΠΌΠΎΠΆΠ½ΠΎ Π²ΠΎΡΠΏΠΎΠ»ΡŒΠ·ΠΎΠ²Π°Ρ‚ΡŒΡΡ ΠΎΠ½Π»Π°ΠΉΠ½ ΠΊΠ°Π»ΡŒΠΊΡƒΠ»ΡΡ‚ΠΎΡ€ΠΎΠΌ Π½ΠΈΠΆΠ΅:

Расчёт ΠΏΠΎΠΊΠ°Π·Π°Π», Ρ‡Ρ‚ΠΎ Ρ†Π΅Π½Π° дСлСния ΠΏΡ€ΠΈΠ±ΠΎΡ€Π° ΡˆΡ‚Π°Ρ‚Π½ΠΎΠΉ ΡˆΠΊΠ°Π»Ρ‹ составляСт 5 Π°ΠΌΠΏΠ΅Ρ€. ΠŸΡ€ΠΈ Ρ‚ΠΎΠΊΠ΅ 12 мА стрСлка ΠΎΡ‚ΠΊΠ»ΠΎΠ½ΡΠ»Π°ΡΡŒ Π΄ΠΎ показания 60А. Π’Π°ΠΊΠΈΠΌ ΠΎΠ±Ρ€Π°Π·ΠΎΠΌ, Ρ†Π΅Π½Π° ΠΎΠ΄Π½ΠΎΠ³ΠΎ дСлСния Π±Π΅Π· ΡˆΡƒΠ½Ρ‚Π° составляСт 1 мА. ВсСго Π΄Π΅Π»Π΅Π½ΠΈΠΉ 30, соотвСтствСнно максимальноС ΠΎΡ‚ΠΊΠ»ΠΎΠ½Π΅Π½ΠΈΠ΅ стрСлки Π΄ΠΎ значСния 150А Π±Π΅Π· ΡˆΡƒΠ½Ρ‚Π° составляСт 30 мА.

Π”Π°Π»Π΅Π΅ ΠΏΡ€ΠΈ ΠΏΠΎΠΌΠΎΡ‰ΠΈ Π·Π°ΠΊΠΎΠ½Π° Ома Π½Π°Ρ…ΠΎΠ΄ΠΈΠΌ сопротивлСниС ΠΏΡ€ΠΈΠ±ΠΎΡ€Π°. 0,075/0,03=2,5 Ом

Расчёт:
Rш=RΠΏΡ€ΠΈΠ±*IΠΏΡ€ΠΈΠ±/(IΡ€Π°Π±-IΠΏΡ€ΠΈΠ±)=2,5*0,03/(10-0,03)=0,00752 Ом для ΡˆΠΊΠ°Π»Ρ‹ 10А ΠΌΠ°Ρ…
Rш=RΠΏΡ€ΠΈΠ±*IΠΏΡ€ΠΈΠ±/(IΡ€Π°Π±-IΠΏΡ€ΠΈΠ±)=2,5*0,03/(5-0,03)=0,01509 Ом для ΡˆΠΊΠ°Π»Ρ‹ 5А ΠΌΠ°Ρ…
Rш=RΠΏΡ€ΠΈΠ±*IΠΏΡ€ΠΈΠ±/(IΡ€Π°Π±-IΠΏΡ€ΠΈΠ±)=2,5*0,03/(3-0,03)=0,02525 Ом для ΡˆΠΊΠ°Π»Ρ‹ 3А ΠΌΠ°Ρ…

Для упрощСния ΠΌΠΎΠΆΠ½ΠΎ Π²ΠΎΡΠΏΠΎΠ»ΡŒΠ·ΠΎΠ²Π°Ρ‚ΡŒΡΡ ΠΎΠ½Π»Π°ΠΉΠ½ ΠΊΠ°Π»ΡŒΠΊΡƒΠ»ΡΡ‚ΠΎΡ€ΠΎΠΌ расчёта сопротивлСния ΡˆΡƒΠ½Ρ‚ΠΈΡ€ΡƒΡŽΡ‰Π΅Π³ΠΎ сопротивлСния Π²Ρ‹ΡˆΠ΅.

Π’Ρ‚ΠΎΡ€ΠΎΠΉ Π²Π°Ρ€ΠΈΠ°Π½Ρ‚. ΠŸΡ€ΠΈ ΠΏΠΎΠΌΠΎΡ‰ΠΈ ΠΏΡ€Π΅Ρ†ΠΈΠ·ΠΈΠΎΠ½Π½ΠΎΠ³ΠΎ ΠΌΡƒΠ»ΡŒΡ‚ΠΈΠΌΠ΅Ρ‚Ρ€Π° замСряСм сопротивлСниС Π°ΠΌΠΏΠ΅Ρ€ΠΌΠ΅Ρ‚Ρ€Π° ΠΈ Π΄Π°Π»Π΅Π΅ ΠΏΡ€ΠΈ ΠΏΠΎΠΌΠΎΡ‰ΠΈ Π·Π°ΠΊΠΎΠ½Π° Ома (зная напряТСниС максимального отклонСния стрСлки) Π½Π°Ρ…ΠΎΠ΄ΠΈΠΌ Ρ‚ΠΎΠΊ максимального отклонСния стрСлки. Π˜Π·ΠΌΠ΅Ρ€Π΅Π½ΠΈΡ Π²Ρ‹ΠΏΠΎΠ»Π½ΡΠ»ΠΈΡΡŒ ΠΏΡ€Π΅Ρ†ΠΈΠ·ΠΈΠΎΠ½Π½Ρ‹ΠΌΠΈ ΠΌΡƒΠ»ΡŒΡ‚ΠΈΠΌΠ΅Ρ‚Ρ€Π°ΠΌΠΈ Mastech MS8218 ΠΈ Uni-t UT71E. ΠŸΡ€ΠΈ ΠΈΠ·ΠΌΠ΅Ρ€Π΅Π½ΠΈΠΈ сопротивлСния Π°ΠΌΠΏΠ΅Ρ€ΠΌΠ΅Ρ‚Ρ€Π° Π·Π½Π°Ρ‡Π΅Π½ΠΈΠ΅ составило 2,50-2,52 Ом ΠΏΡ€ΠΈΠ±ΠΎΡ€ΠΎΠΌ UT71E ΠΈ 2,52-2,53 ΠΏΡ€ΠΈΠ±ΠΎΡ€ΠΎΠΌ MS8218.

Π€ΠΎΡ€ΠΌΡƒΠ»Π° для расчёта Ρ‚ΠΎΠΊΠ° отклонСния стрСлки Π΄ΠΎ максимального значСния:

Для упрощСния вычислСний максимального Ρ‚ΠΎΠΊΠ° отклонСния стрСлки Π°ΠΌΠΏΠ΅Ρ€ΠΌΠ΅Ρ‚Ρ€Π° ΠΌΠΎΠΆΠ½ΠΎ Π²ΠΎΡΠΏΠΎΠ»ΡŒΠ·ΠΎΠ²Π°Ρ‚ΡŒΡΡ ΠΊΠ°Π»ΡŒΠΊΡƒΠ»ΡΡ‚ΠΎΡ€ΠΎΠΌ Π½ΠΈΠΆΠ΅:

Π”Π°Π»Π΅Π΅, ΠΊΠ°ΠΊ ΠΈ Π² ΠΏΠ΅Ρ€Π²ΠΎΠΌ Π²Π°Ρ€ΠΈΠ°Π½Ρ‚Π΅ выполняСм расчёт сопротивлСния ΡˆΡƒΠ½Ρ‚ΠΈΡ€ΡƒΡŽΡ‰Π΅Π³ΠΎ рСзистора (ΠΊΠ°Π»ΡŒΠΊΡƒΠ»ΡΡ‚ΠΎΡ€ Π²Ρ‹ΡˆΠ΅). Для расчёта Π±Ρ‹Π»ΠΎ принято срСднСС ΠΏΠΎΠΊΠ°Π·Π°Π½ΠΈΠ΅ ΠΈΠ·ΠΌΠ΅Ρ€Π΅Π½Π½ΠΎΠ³ΠΎ сопротивлСния Π°ΠΌΠΏΠ΅Ρ€ΠΌΠ΅Ρ‚Ρ€Π° двумя ΠΌΡƒΠ»ΡŒΡ‚ΠΈΠΌΠ΅Ρ‚Ρ€Π°ΠΌΠΈ RΠΏΡ€ΠΈΠ± = 2,52Ом

Расчёт:
Rш=RΠΏΡ€ΠΈΠ±*IΠΏΡ€ΠΈΠ±/(IΡ€Π°Π±-IΠΏΡ€ΠΈΠ±)=2,52*0,02976/(10-0,02976)=0,00752 Ом для ΡˆΠΊΠ°Π»Ρ‹ 10А ΠΌΠ°Ρ…
Rш=RΠΏΡ€ΠΈΠ±*IΠΏΡ€ΠΈΠ±/(IΡ€Π°Π±-IΠΏΡ€ΠΈΠ±)=2,52*0,02976/(5-0,02976)=0,01508 Ом для ΡˆΠΊΠ°Π»Ρ‹ 5А ΠΌΠ°Ρ…
Rш=RΠΏΡ€ΠΈΠ±*IΠΏΡ€ΠΈΠ±/(IΡ€Π°Π±-IΠΏΡ€ΠΈΠ±)=2,52*0,02976/(3-0,02976)=0,02524 Ом для ΡˆΠΊΠ°Π»Ρ‹ 3А ΠΌΠ°Ρ…

Если ΡΡ€Π°Π²Π½ΠΈΡ‚ΡŒ расчёты Π΄Π²ΡƒΡ… ΠΌΠ΅Ρ‚ΠΎΠ΄ΠΈΠΊ ΠΌΠ΅ΠΆΠ΄Ρƒ собой, Ρ‚ΠΎ ΠΏΠΎΠ»ΡƒΡ‡ΠΈΠ»ΠΈΡΡŒ совпадСниС Π΄Π°Π½Π½Ρ‹Ρ… Π΄ΠΎ Ρ‡Π΅Ρ‚Π²Ρ‘Ρ€Ρ‚ΠΎΠ³ΠΎ Π·Π½Π°ΠΊΠ° послС запятой, Π° Π² Π½Π΅ΠΊΠΎΡ‚ΠΎΡ€Ρ‹Ρ… случаях Π΄Π°ΠΆΠ΅ Π΄ΠΎ пяти Π·Π½Π°ΠΊΠΎΠ².

О тонкостях изготовлСния ΡˆΡƒΠ½Ρ‚ΠΈΡ€ΡƒΡŽΡ‰Π΅Π³ΠΎ сопротивлСния расскаТу Π² ΡΠ»Π΅Π΄ΡƒΡŽΡ‰Π΅ΠΉ ΡΡ‚Π°Ρ‚ΡŒΠ΅.

1. НСобходимо ΠΈΠ·ΠΌΠ΅Ρ€ΠΈΡ‚ΡŒ Ρ‚ΠΎΠΊ потрСбитСля Π² ΠΏΡ€Π΅Π΄Π΅Π»Π°Ρ… 20 – 25 А. Π˜ΠΌΠ΅Π΅Ρ‚ΡΡ ΠΌΠΈΠΊΡ€ΠΎΠ°ΠΌΠΏΠ΅Ρ€ΠΌΠ΅Ρ‚Ρ€ с ΠΏΡ€Π΅Π΄Π΅Π»ΠΎΠΌ измСрСния 200мкА, Π²Π½ΡƒΡ‚Ρ€Π΅Π½Π½ΠΈΠΌ сопротивлСниСм 300 Ом ΠΈ ΠΌΠ°ΠΊΡΠΈΠΌΠ°Π»ΡŒΠ½Ρ‹ΠΌ числом Π΄Π΅Π»Π΅Π½ΠΈΠΉ 100. ΠžΠΏΡ€Π΅Π΄Π΅Π»ΠΈΡ‚ΡŒ сопротивлСниС ΡˆΡƒΠ½Ρ‚Π° для Ρ€Π°ΡΡˆΠΈΡ€Π΅Π½ΠΈΡ ΠΏΡ€Π΅Π΄Π΅Π»Π° измСрСния Π΄ΠΎ 30 А ΠΈ ΠΎΠΏΡ€Π΅Π΄Π΅Π»ΠΈΡ‚ΡŒ ΠΎΡ‚Π½ΠΎΡΠΈΡ‚Π΅Π»ΡŒΠ½ΡƒΡŽ ΠΏΠΎΠ³Ρ€Π΅ΡˆΠ½ΠΎΡΡ‚ΡŒ измСрСния Π½Π° ΠΎΡ‚ΠΌΠ΅Ρ‚ΠΊΠ΅ 85 Π΄Π΅Π»Π΅Π½ΠΈΠΉ, Ссли класс точности ΠΏΡ€ΠΈΠ±ΠΎΡ€Π° 1,0.

РСшСниС:

2. ΠŸΡ€Π΅Π΄Π΅Π» измСрСния IΠΏΡ€ Π°ΠΌΠΏΠ΅Ρ€ΠΌΠ΅Ρ‚Ρ€Π° с Π²Π½ΡƒΡ‚Ρ€Π΅Π½Π½ΠΈΠΌ сопротивлСниСм RА Π΄ΠΎΠ»ΠΆΠ΅Π½ Π±Ρ‹Ρ‚ΡŒ Ρ€Π°ΡΡˆΠΈΡ€Π΅Π½ Π΄ΠΎ значСния 8 IΠΏΡ€. Найти Π·Π½Π°Ρ‡Π΅Π½ΠΈΠ΅ RΠ¨.

РСшСниС:

3. ΠŸΡ€Π΅Π΄Π΅Π» измСрСния ΠΌΠΈΠΊΡ€ΠΎΠ°ΠΌΠΏΠ΅Ρ€ΠΌΠ΅Ρ‚Ρ€Π° Π½Π° 150 мкА Π΄ΠΎΠ»ΠΆΠ΅Π½ Π±Ρ‹Ρ‚ΡŒ Ρ€Π°ΡΡˆΠΈΡ€Π΅Π½ Π΄ΠΎ 15 А. ΠžΠΏΡ€Π΅Π΄Π΅Π»ΠΈΡ‚ΡŒ сопротивлСниС ΡˆΡƒΠ½Ρ‚Π°, Ссли Π²Π½ΡƒΡ‚Ρ€Π΅Π½Π½Π΅Π΅ Π΅Π³ΠΎ сопротивлСниС RА = 400 А.

РСшСниС:

4. АмпСрмСтр с Π²Π½ΡƒΡ‚Ρ€Π΅Π½Π½ΠΈΠΌ сопротивлСниСм RА = 0,015 Ом ΠΈ ΠΏΡ€Π΅Π΄Π΅Π»ΠΎΠΌ измСрСния 20 А ΠΈΠΌΠ΅Π΅Ρ‚ ΡˆΡƒΠ½Ρ‚ сопротивлСниСм 0,005 Ом. ΠžΠΏΡ€Π΅Π΄Π΅Π»ΠΈΡ‚ΡŒ ΠΏΡ€Π΅Π΄Π΅Π» измСрСния Π°ΠΌΠΏΠ΅Ρ€ΠΌΠ΅Ρ‚Ρ€Π° с ΡˆΡƒΠ½Ρ‚ΠΎΠΌ, Π° Ρ‚Π°ΠΊΠΆΠ΅ Ρ‚ΠΎΠΊ Π² Ρ†Π΅ΠΏΠΈ, Ссли Π΅Π³ΠΎ ΠΏΠΎΠΊΠ°Π·Π°Π½ΠΈΠ΅ Ρ€Π°Π²Π½ΠΎ 12 А.

РСшСниС:

5. ΠœΠΈΠ»Π»ΠΈΠ²ΠΎΠ»ΡŒΡ‚ΠΌΠ΅Ρ‚Ρ€ с ΠΏΡ€Π΅Π΄Π΅Π»ΠΎΠΌ измСрСния 75 ΠΌΠ’ ΠΈ Π²Π½ΡƒΡ‚Ρ€Π΅Π½Π½ΠΈΠΌ сопротивлСниСм RΠ’ = 25 Ом ΠΈΠΌΠ΅Π΅Ρ‚ 150 Π΄Π΅Π»Π΅Π½ΠΈΠΉ ΡˆΠΊΠ°Π»Ρ‹. ΠžΠΏΡ€Π΅Π΄Π΅Π»ΠΈΡ‚ΡŒ сопротивлСниС ΡˆΡƒΠ½Ρ‚Π°, Ρ‡Ρ‚ΠΎΠ±Ρ‹ ΠΏΡ€ΠΈΠ±ΠΎΡ€ΠΎΠΌ ΠΌΠΎΠΆΠ½ΠΎ Π±Ρ‹Π»ΠΎ ΠΈΠ·ΠΌΠ΅Ρ€ΡΡ‚ΡŒ ΠΏΡ€Π΅Π΄Π΅Π»ΡŒΠ½ΠΎΠ΅ Π·Π½Π°Ρ‡Π΅Π½ΠΈΠ΅ Ρ‚ΠΎΠΊΠ° 30 А. ΠžΠΏΡ€Π΅Π΄Π΅Π»ΠΈΡ‚ΡŒ Ρ†Π΅Π½Ρƒ дСлСния ΠΏΡ€ΠΈΠ±ΠΎΡ€Π° Π² ΠΎΠ±ΠΎΠΈΡ… случаях.

РСшСниС:

6. ΠœΠΈΠΊΡ€ΠΎΠ°ΠΌΠΏΠ΅Ρ€ΠΌΠ΅Ρ‚Ρ€ с ΠΏΡ€Π΅Π΄Π΅Π»ΠΎΠΌ измСрСния 1000 мкА ΠΈ Π²Π½ΡƒΡ‚Ρ€Π΅Π½Π½ΠΈΠΌ сопротивлСниСм RА =300 Ом Π½Π΅ΠΎΠ±Ρ…ΠΎΠ΄ΠΈΠΌΠΎ ΠΈΡΠΏΠΎΠ»ΡŒΠ·ΠΎΠ²Π°Ρ‚ΡŒ Π² качСствС Π²ΠΎΠ»ΡŒΡ‚ΠΌΠ΅Ρ‚Ρ€Π° Π½Π° ΠΏΡ€Π΅Π΄Π΅Π» 30 Π’. ΠžΠΏΡ€Π΅Π΄Π΅Π»ΠΈΡ‚ΡŒ Π΄ΠΎΠ±Π°Π²ΠΎΡ‡Π½ΠΎΠ΅ сопротивлСниС.

РСшСниС:

7. ΠœΠΈΠ»Π»ΠΈΠ²ΠΎΠ»ΡŒΡ‚ΠΌΠ΅Ρ‚Ρ€ с ΠΏΡ€Π΅Π΄Π΅Π»ΠΎΠΌ измСрСния 750 ΠΌΠ’ Π½Π΅ΠΎΠ±Ρ…ΠΎΠ΄ΠΈΠΌΠΎ ΠΏΠ΅Ρ€Π΅Π΄Π΅Π»Π°Ρ‚ΡŒ Π² ΠΌΠ½ΠΎΠ³ΠΎΠΏΡ€Π΅Π΄Π΅Π»ΡŒΠ½Ρ‹ΠΉ Π²ΠΎΠ»ΡŒΡ‚ΠΌΠ΅Ρ‚Ρ€ с ΠΏΡ€Π΅Π΄Π΅Π»Π°ΠΌΠΈ 7,5; 15; 75; 150Π’. Π”ΠΎΠ±Π°Π²ΠΎΡ‡Π½ΠΎΠ΅ сопротивлСниС Π½Π° ΠΏΡ€Π΅Π΄Π΅Π»Π΅ 7,5 Π’ составляСт 1350 Ом. ΠžΠΏΡ€Π΅Π΄Π΅Π»ΠΈΡ‚ΡŒ Π΄ΠΎΠ±Π°Π²ΠΎΡ‡Π½ΠΎΠ΅ сопротивлСниС Π½Π° ΠΊΠ°ΠΆΠ΄ΠΎΠΌ ΠΈΠ· ΠΏΡ€Π΅Π΄Π΅Π»ΠΎΠ², сопротивлСниС ΠΈ Ρ‚ΠΎΠΊ ΠΏΠΎΠ»Π½ΠΎΠ³ΠΎ отклонСния ΠΏΡ€ΠΈΠ±ΠΎΡ€Π°.

РСшСниС:

8. Π£ Π²ΠΎΠ»ΡŒΡ‚ΠΌΠ΅Ρ‚Ρ€Π° элСктродинамичСской систСмы с ΠΏΡ€Π΅Π΄Π΅Π»ΠΎΠΌ измСрСния U = 300 Π’ ΠΈ Π²Π½ΡƒΡ‚Ρ€Π΅Π½Π½ΠΈΠΌ сопротивлСниСм RΠ² = 30 кОм Π½Π΅ΠΎΠ±Ρ…ΠΎΠ΄ΠΈΠΌΠΎ Ρ€Π°ΡΡˆΠΈΡ€ΠΈΡ‚ΡŒ ΠΏΡ€Π΅Π΄Π΅Π» Π΄ΠΎ 1500 Π’. ΠžΠΏΡ€Π΅Π΄Π΅Π»ΠΈΡ‚ΡŒ Π΄ΠΎΠ±Π°Π²ΠΎΡ‡Π½ΠΎΠ΅ сопротивлСниС Π²ΠΎΠ»ΡŒΡ‚ΠΌΠ΅Ρ‚Ρ€Π° ΠΈ ΠΌΠ°ΠΊΡΠΈΠΌΠ°Π»ΡŒΠ½ΡƒΡŽ ΠΏΠΎΡ‚Ρ€Π΅Π±Π»ΡΠ΅ΠΌΡƒΡŽ ΠΌΠΎΡ‰Π½ΠΎΡΡ‚ΡŒ Π½Π° основном ΠΈ Ρ€Π°ΡΡˆΠΈΡ€Π΅Π½Π½ΠΎΠΌ ΠΏΡ€Π΅Π΄Π΅Π»Π°Ρ….

РСшСниС:

9. ΠŸΡ€Π΅Π΄Π΅Π» измСрСния Π²ΠΎΠ»ΡŒΡ‚ΠΌΠ΅Ρ‚Ρ€Π° элСктромагнитной систСмы составляСт 7,5 Π’ ΠΏΡ€ΠΈ Π²Π½ΡƒΡ‚Ρ€Π΅Π½Π½Π΅ΠΌ сопротивлСнии RΠ² = 200 Ом. ΠžΠΏΡ€Π΅Π΄Π΅Π»ΠΈΡ‚ΡŒ Π΄ΠΎΠ±Π°Π²ΠΎΡ‡Π½ΠΎΠ΅ сопротивлСниС, ΠΊΠΎΡ‚ΠΎΡ€ΠΎΠ΅ Π½Π΅ΠΎΠ±Ρ…ΠΎΠ΄ΠΈΠΌΠΎ Π²ΠΊΠ»ΡŽΡ‡ΠΈΡ‚ΡŒ для Ρ€Π°ΡΡˆΠΈΡ€Π΅Π½ΠΈΡ ΠΏΡ€Π΅Π΄Π΅Π»Π° измСрСния Π΄ΠΎ 600 Π’.

РСшСниС:

10. АмпСрмСтром с Π²Π½ΡƒΡ‚Ρ€Π΅Π½Π½ΠΈΠΌ сопротивлСниСм RA = 1 Ом слСдуСт ΠΈΠ·ΠΌΠ΅Ρ€ΠΈΡ‚ΡŒ Ρ‚ΠΎΠΊ Π² 10, 100 ΠΈ 1000 Ρ€Π°Π· большС Π΅Π³ΠΎ номинального значСния. Найти ΡΠΎΠΎΡ‚Π½ΠΎΡˆΠ΅Π½ΠΈΠ΅ ΠΌΠ΅ΠΆΠ΄Ρƒ сопротивлСниями Π°ΠΌΠΏΠ΅Ρ€ΠΌΠ΅Ρ‚Ρ€Π° ΠΈ ΡˆΡƒΠ½Ρ‚ΠΎΠ², ΠΏΠΎΠ΄ΠΎΠ±Ρ€Π°Π½Π½Ρ‹Ρ… для выполнСния ΡƒΠΊΠ°Π·Π°Π½Π½Ρ‹Ρ… ΠΈΠ·ΠΌΠ΅Ρ€Π΅Π½ΠΈΠΉ.

РСшСниС:

11. АмпСрмСтр, ΠΈΠΌΠ΅ΡŽΡ‰ΠΈΠΉ Π²Π½ΡƒΡ‚Ρ€Π΅Π½Π½Π΅Π΅ сопротивлСниС 0,2 Ом ΠΈ ΠΏΡ€Π΅Π΄Π΅Π» измСрСния 10 А, Π½Π΅ΠΎΠ±Ρ…ΠΎΠ΄ΠΈΠΌΠΎ ΠΈΡΠΏΠΎΠ»ΡŒΠ·ΠΎΠ²Π°Ρ‚ΡŒ для измСрСния Ρ‚ΠΎΠΊΠ° Π΄ΠΎ 500 А. ΠžΠΏΡ€Π΅Π΄Π΅Π»ΠΈΡ‚ΡŒ сопротивлСниС ΡˆΡƒΠ½Ρ‚Π° ΠΏΡ€ΠΈΠ±ΠΎΡ€Π° ΠΈ ΠΏΠ°Π΄Π΅Π½ΠΈΠ΅ напряТСния Π½Π° Π°ΠΌΠΏΠ΅Ρ€ΠΌΠ΅Ρ‚Ρ€Π΅ ΠΈ ΡˆΡƒΠ½Ρ‚Π΅.

РСшСниС:

12. ΠΠΎΠΌΠΈΠ½Π°Π»ΡŒΠ½Ρ‹ΠΉ Ρ‚ΠΎΠΊ Π°ΠΌΠΏΠ΅Ρ€ΠΌΠ΅Ρ‚Ρ€Π° IΠ½ = 1 А, сопротивлСниС ΡˆΡƒΠ½Ρ‚Π° Rш = 0,5 Ом ΠžΠΏΡ€Π΅Π΄Π΅Π»ΠΈΡ‚ΡŒ сопротивлСниС Π°ΠΌΠΏΠ΅Ρ€ΠΌΠ΅Ρ‚Ρ€Π°, Ссли номинальноС Π·Π½Π°Ρ‡Π΅Π½ΠΈΠ΅ Ρ‚ΠΎΠΊΠ° Π² Π½Π΅ΠΌ Π±Ρ‹Π»ΠΎ ΠΏΡ€ΠΈ ΠΎΠ±Ρ‰Π΅ΠΌ Ρ‚ΠΎΠΊΠ΅ Ρ†Π΅ΠΏΠΈ 5 А.

РСшСниС:

13. ΠΠΎΠΌΠΈΠ½Π°Π»ΡŒΠ½Ρ‹ΠΉ Ρ‚ΠΎΠΊ Π°ΠΌΠΏΠ΅Ρ€ΠΌΠ΅Ρ‚Ρ€Π° 1 А, Π΅Π³ΠΎ Π²Π½ΡƒΡ‚Ρ€Π΅Π½Π½Π΅Π΅ сопротивлСниС 0,08 Ом. Какой Ρ‚ΠΎΠΊ ΠΏΡ€ΠΎΡ…ΠΎΠ΄ΠΈΡ‚ Π² элСктричСской Ρ†Π΅ΠΏΠΈ, Ссли Π°ΠΌΠΏΠ΅Ρ€ΠΌΠ΅Ρ‚Ρ€ с ΡˆΡƒΠ½Ρ‚ΠΎΠΌ сопротивлСниСм 0,03 Ом ΠΏΠΎΠΊΠ°Π·Ρ‹Π²Π°Π΅Ρ‚ Ρ‚ΠΎΠΊ 0,9 А?

РСшСниС:

14. НСобходимо ΠΈΠ·ΠΌΠ΅Ρ€ΠΈΡ‚ΡŒ напряТСниС Π² ΠΏΡ€Π΅Π΄Π΅Π»Π°Ρ… 30-40 Π’. Какой ΠΈΠ· Π²ΠΎΠ»ΡŒΡ‚ΠΌΠ΅Ρ‚Ρ€ΠΎΠ² позволяСт произвСсти ΠΈΠ·ΠΌΠ΅Ρ€Π΅Π½ΠΈΠ΅ с большСй Ρ‚ΠΎΡ‡Π½ΠΎΡΡ‚ΡŒΡŽ:

1 ) с Π²Π΅Ρ€Ρ…Π½ΠΈΠΌ ΠΏΡ€Π΅Π΄Π΅Π»ΠΎΠΌ 50 Π’ ΠΈ классом точности 2,5;

2) с Π²Π΅Ρ€Ρ…Π½ΠΈΠΌ ΠΏΡ€Π΅Π΄Π΅Π»ΠΎΠΌ 100 Π’ ΠΈ классом точности 1,5;

3) с Π²Π΅Ρ€Ρ…Π½ΠΈΠΌ ΠΏΡ€Π΅Π΄Π΅Π»ΠΎΠΌ 300 Π’ ΠΈ классом точности 0,5; 4) с Π²Π΅Ρ€Ρ…Π½ΠΈΠΌ ΠΏΡ€Π΅Π΄Π΅Π»ΠΎΠΌ 150 Π’ ΠΈ классом точности 1.

РСшСниС:

15. Π’ΠΎΠ»ΡŒΡ‚ΠΌΠ΅Ρ‚Ρ€ΠΎΠΌ с Π²Π½ΡƒΡ‚Ρ€Π΅Π½Π½ΠΈΠΌ сопротивлСниСм Rv трСбуСтся ΠΈΠ·ΠΌΠ΅Ρ€ΠΈΡ‚ΡŒ напряТСниС Π² 10, 100 ΠΈ 1000 Ρ€Π°Π· большС Π΅Π³ΠΎ номинального значСния. Найти ΡΠΎΠΎΡ‚Π½ΠΎΡˆΠ΅Π½ΠΈΠ΅ ΠΌΠ΅ΠΆΠ΄Ρƒ Π²Π½ΡƒΡ‚Ρ€Π΅Π½Π½ΠΈΠΌ сопротивлСниСм Π²ΠΎΠ»ΡŒΡ‚ΠΌΠ΅Ρ‚Ρ€Π° ΠΈ сопротивлСниями Π΄ΠΎΠ±Π°Π²ΠΎΡ‡Π½Ρ‹Ρ… рСзисторов, ΠΏΠΎΠ΄ΠΎΠ±Ρ€Π°Π½Π½Ρ‹Ρ… для выполнСния ΡƒΠΊΠ°Π·Π°Π½Π½Ρ‹Ρ… условий.

РСшСниС:

16. НоминальноС напряТСниС Π²ΠΎΠ»ΡŒΡ‚ΠΌΠ΅Ρ‚Ρ€Π° 10 Π’, Π²Π½ΡƒΡ‚Ρ€Π΅Π½Π½Π΅Π΅ Π΅Π³ΠΎ сопротивлСниС 5 кОм. КакоС допустимоС напряТСниС ΠΌΠΎΠΆΠ΅Ρ‚ Π±Ρ‹Ρ‚ΡŒ Π² измСряСмой Ρ†Π΅ΠΏΠΈ, Ссли ΠΊ Π²ΠΎΠ»ΡŒΡ‚ΠΌΠ΅Ρ‚Ρ€Ρƒ ΠΏΠΎΠ΄ΠΊΠ»ΡŽΡ‡Π΅Π½ Π΄ΠΎΠ±Π°Π²ΠΎΡ‡Π½Ρ‹ΠΉ рСзистор, сопротивлСниС ΠΊΠΎΡ‚ΠΎΡ€ΠΎΠ³ΠΎ 150 кОм?

РСшСниС:

17. Π’ΠΎΠ»ΡŒΡ‚ΠΌΠ΅Ρ‚Ρ€ рассчитан для измСрСния напряТСния Π΄ΠΎ 15 Π’. ΠžΠΏΡ€Π΅Π΄Π΅Π»ΠΈΡ‚ΡŒ сопротивлСниС Π΄ΠΎΠ±Π°Π²ΠΎΡ‡Π½ΠΎΠ³ΠΎ рСзистора, ΠΊΠΎΡ‚ΠΎΡ€Ρ‹ΠΉ Π½Π΅ΠΎΠ±Ρ…ΠΎΠ΄ΠΈΠΌΠΎ ΠΏΠΎΠ΄ΠΊΠ»ΡŽΡ‡ΠΈΡ‚ΡŒ ΠΊ Π²ΠΎΠ»ΡŒΡ‚ΠΌΠ΅Ρ‚Ρ€Ρƒ с Rv = 50 кОм, Ρ‡Ρ‚ΠΎΠ±Ρ‹ с Π΅Π³ΠΎ ΠΏΠΎΠΌΠΎΡ‰ΡŒΡŽ ΠΈΠ·ΠΌΠ΅Ρ€ΡΡ‚ΡŒ напряТСниС 220 Π’. ΠšΠ°ΠΊΠΎΠ²Ρ‹ ΠΏΡ€ΠΈ этом ΠΏΠΎΡ‚Π΅Ρ€ΠΈ мощности Π² ΠΎΠ±ΠΌΠΎΡ‚ΠΊΠ΅ Π²ΠΎΠ»ΡŒΡ‚ΠΌΠ΅Ρ‚Ρ€Π° ΠΈ Π΄ΠΎΠ±Π°Π²ΠΎΡ‡Π½ΠΎΠΌ рСзисторС?

РСшСниС:

Рис. 3.1

18. НоминальноС напряТСниС Π²ΠΎΠ»ΡŒΡ‚ΠΌΠ΅Ρ‚Ρ€Π° 30 Π’, Π΅Π³ΠΎ Π²Π½ΡƒΡ‚Ρ€Π΅Π½Π½Π΅Π΅ сопротивлСниС 10 кОм. Каково напряТСниС Π² измСряСмой Ρ†Π΅ΠΏΠΈ, Ссли ΠΏΠΎΠΊΠ°Π·Π°Π½ΠΈΠ΅ Π²ΠΎΠ»ΡŒΡ‚ΠΌΠ΅Ρ‚Ρ€Π° 10 Π’ соотвСтствуСт ΡΠΎΠΏΡ€ΠΎΡ‚ΠΈΠ²Π»Π΅Π½ΠΈΡŽ Π΄ΠΎΠ±Π°Π²ΠΎΡ‡Π½ΠΎΠ³ΠΎ рСзистора 50 кОм?

РСшСниС:

19. Для измСрСния Π­Π”Π‘ Π³Π΅Π½Π΅Ρ€Π°Ρ‚ΠΎΡ€Π° ΠΊ Π΅Π³ΠΎ Π·Π°ΠΆΠΈΠΌΠ°ΠΌ присоСдинСн Π²ΠΎΠ»ΡŒΡ‚ΠΌΠ΅Ρ‚Ρ€ (рис, 3.1), сопротивлСниС ΠΊΠΎΡ‚ΠΎΡ€ΠΎΠ³ΠΎ Rv = 10000 Ом. Π‘ΠΎΠΏΡ€ΠΎΡ‚ΠΈΠ²Π»Π΅Π½ΠΈΠ΅ якоря Π³Π΅Π½Π΅Ρ€Π°Ρ‚ΠΎΡ€Π° RΠ° = 0,2 Ом. ΠžΠΏΡ€Π΅Π΄Π΅Π»ΠΈΡ‚ΡŒ, Π½Π° сколько ΠΏΡ€ΠΎΡ†Π΅Π½Ρ‚ΠΎΠ² Π΄Π΅Π»Π°Π΅ΠΌ ΠΎΡˆΠΈΠ±ΠΊΡƒ, считая ΠΏΠΎΠΊΠ°Π·Π°Π½ΠΈΠ΅ Π²ΠΎΠ»ΡŒΡ‚ΠΌΠ΅Ρ‚Ρ€Π°, Ρ€Π°Π²Π½Ρ‹ΠΌ Π­Π”Π‘ Π³Π΅Π½Π΅Ρ€Π°Ρ‚ΠΎΡ€Π°.

РСшСниС:

20. АмпСрмСтр, сопротивлСниС ΠΊΠΎΡ‚ΠΎΡ€ΠΎΠ³ΠΎ RА = 0,3 Ом, ΠΈΠΌΠ΅Π΅Ρ‚ ΡˆΠΊΠ°Π»Ρƒ Π² 150 Π΄Π΅Π»Π΅Π½ΠΈΠΉ ΠΈ постоянная ΠΏΡ€ΠΈΠ±ΠΎΡ€Π° БА = 0,001 А/Π΄Π΅Π». ΠžΠΏΡ€Π΅Π΄Π΅Π»ΠΈΡ‚ΡŒ сопротивлСниС ΡˆΡƒΠ½Ρ‚Π° RΠ¨, ΠΏΡ€ΠΈ ΠΏΠΎΠΌΠΎΡ‰ΠΈ ΠΊΠΎΡ‚ΠΎΡ€ΠΎΠ³ΠΎ ΠΌΠΎΠΆΠ½ΠΎ Π±Ρ‹Π»ΠΎ ΠΈΠ·ΠΌΠ΅Ρ€ΡΡ‚ΡŒ Ρ‚ΠΎΠΊ Π΄ΠΎ 300 А.

РСшСниС:

21. АмпСрмСтр, сопротивлСниС ΠΊΠΎΡ‚ΠΎΡ€ΠΎΠ³ΠΎ RА = 0,3 Ом, ΠΈΠΌΠ΅Π΅Ρ‚ ΡˆΠΊΠ°Π»Ρƒ Π² 150 Π΄Π΅Π»Π΅Π½ΠΈΠΉ ΠΈ постоянная ΠΏΡ€ΠΈΠ±ΠΎΡ€Π° БА = 0,001 А/Π΄Π΅Π». ΠžΠΏΡ€Π΅Π΄Π΅Π»ΠΈΡ‚ΡŒ; ΠΊΠ°ΠΊΠΎΠ΅ сопротивлСниС RΠ” Π½Π΅ΠΎΠ±Ρ…ΠΎΠ΄ΠΈΠΌΠΎ ΠΏΠΎΡΠ»Π΅Π΄ΠΎΠ²Π°Ρ‚Π΅Π»ΡŒΠ½ΠΎ Π²ΠΊΠ»ΡŽΡ‡ΠΈΡ‚ΡŒ с Π°ΠΌΠΏΠ΅Ρ€ΠΌΠ΅Ρ‚Ρ€ΠΎΠΌ, Ρ‡Ρ‚ΠΎΠ±Ρ‹ этим ΠΏΡ€ΠΈΠ±ΠΎΡ€ΠΎΠΌ ΠΌΠΎΠΆΠ½ΠΎ Π±Ρ‹Π»ΠΎ ΠΈΠ·ΠΌΠ΅Ρ€ΡΡ‚ΡŒ напряТСниС Π΄ΠΎ 150Π’.

РСшСниС:

22. Какой Ρ‚ΠΎΠΊ ΠΌΠΎΠΆΠ½ΠΎ ΠΈΠ·ΠΌΠ΅Ρ€ΡΡ‚ΡŒ Π°ΠΌΠΏΠ΅Ρ€ΠΌΠ΅Ρ‚Ρ€ΠΎΠΌ (сопротивлСниС RА = 0,3 Ом, шкала ΠΈΠΌΠ΅Π΅Ρ‚ 150 Π΄Π΅Π»Π΅Π½ΠΈΠΉ ΠΈ постоянная ΠΏΡ€ΠΈΠ±ΠΎΡ€Π° БА =0,001 А/Π΄Π΅Π»), Ссли имССтся ΡˆΡƒΠ½Ρ‚ с сопротивлСниСм Ом?

РСшСниС:

23. НСобходимо ΠΏΠΎΠ΄ΠΎΠ±Ρ€Π°Ρ‚ΡŒ ΠΊ Π°ΠΌΠΏΠ΅Ρ€ΠΌΠ΅Ρ‚Ρ€Ρƒ с сопротивлСниСм RА ΡˆΡƒΠ½Ρ‚Ρ‹, Ρ€Π°ΡΡˆΠΈΡ€ΡΡŽΡ‰ΠΈΠ΅ ΠΏΡ€Π΅Π΄Π΅Π»Ρ‹ измСрСния Π² 10 ΠΈ 100 Ρ€Π°Π·. ΠšΠ°ΠΊΠΎΠ²Ρ‹ Π±ΡƒΠ΄ΡƒΡ‚ ΡΠΎΠΎΡ‚Π½ΠΎΡˆΠ΅Π½ΠΈΡ ΠΌΠ΅ΠΆΠ΄Ρƒ сопротивлСниСм Π°ΠΌΠΏΠ΅Ρ€ΠΌΠ΅Ρ‚Ρ€Π° ΠΈ ΡˆΡƒΠ½Ρ‚Π°?

РСшСниС:

24. Π’ΠΎΠ»ΡŒΡ‚ΠΌΠ΅Ρ‚Ρ€ с Ρ†Π΅Π½ΠΎΠΉ дСлСния 1Π’/Π΄Π΅Π», шкала ΠΊΠΎΡ‚ΠΎΡ€ΠΎΠ³ΠΎ содСрТит 150 Π΄Π΅Π»Π΅Π½ΠΈΠΉ, ΠΈΠΌΠ΅Π΅Ρ‚ сопротивлСниС RΠ’ = 10 000 Ом. КакоС Π΄ΠΎΠ±Π°Π²ΠΎΡ‡Π½ΠΎΠ΅ сопротивлСниС RΠ” Π½Π΅ΠΎΠ±Ρ…ΠΎΠ΄ΠΈΠΌΠΎ Π²ΠΊΠ»ΡŽΡ‡ΠΈΡ‚ΡŒ ΠΏΠΎΡΠ»Π΅Π΄ΠΎΠ²Π°Ρ‚Π΅Π»ΡŒΠ½ΠΎ с Π²ΠΎΠ»ΡŒΡ‚ΠΌΠ΅Ρ‚Ρ€ΠΎΠΌ, Ρ‡Ρ‚ΠΎΠ±Ρ‹ ΠΈΠΌ ΠΌΠΎΠΆΠ½ΠΎ Π±Ρ‹Π»ΠΎ ΠΈΠ·ΠΌΠ΅Ρ€ΡΡ‚ΡŒ напряТСниС Π΄ΠΎ 600 Π’?

РСшСниС:

25. Π§Π΅Ρ€Π΅Π· Π°ΠΌΠΏΠ΅Ρ€ΠΌΠ΅Ρ‚Ρ€, Π²ΠΊΠ»ΡŽΡ‡Π΅Π½Π½Ρ‹ΠΉ Π² Ρ†Π΅ΠΏΡŒ Ρ‡Π΅Ρ€Π΅Π· трансформатор Ρ‚ΠΎΠΊΠ° Π’Π›-35К, рассчитанный Π½Π° Ρ‚ΠΎΠΊ 600/5 А, ΠΏΡ€ΠΎΡ…ΠΎΠ΄ΠΈΡ‚ Ρ‚ΠΎΠΊ I2 = 4,25 А, ΠžΠΏΡ€Π΅Π΄Π΅Π»ΠΈΡ‚ΡŒ Ρ‚ΠΎΠΊ Π² ΠΏΠ΅Ρ€Π²ΠΈΡ‡Π½ΠΎΠΉ Ρ†Π΅ΠΏΠΈ.

РСшСниС:

26. АмпСрмСтр, рассчитанный Π½Π° 5А, со шкалой 0 β€” 500 Π΄Π΅Π»Π΅Π½ΠΈΠΉ Π²ΠΊΠ»ΡŽΡ‡Π΅Π½ Π² Ρ†Π΅ΠΏΡŒ Ρ‡Π΅Ρ€Π΅Π· трансформатор Ρ‚ΠΎΠΊΠ° 400/5 А. Какой Ρ‚ΠΎΠΊ ΠΏΡ€ΠΎΡ…ΠΎΠ΄ΠΈΡ‚ Π² ΠΏΠ΅Ρ€Π²ΠΈΡ‡Π½ΠΎΠΉ ΠΈ Π²Ρ‚ΠΎΡ€ΠΈΡ‡Π½ΠΎΠΉ ΠΎΠ±ΠΌΠΎΡ‚ΠΊΠ°Ρ… трансформатора, Ссли Π°ΠΌΠΏΠ΅Ρ€ΠΌΠ΅Ρ‚Ρ€ ΠΏΠΎΠΊΠ°Π·Ρ‹Π²Π°Π΅Ρ‚ 350 Π΄Π΅Π»Π΅Π½ΠΈΠΉ?

РСшСниС:

27. Π’Π°Ρ‚Ρ‚ΠΌΠ΅Ρ‚Ρ€, ΠΈΠΌΠ΅ΡŽΡ‰ΠΈΠΉ ΠΏΡ€Π΅Π΄Π΅Π»Ρ‹ измСрСния U = 150 Π’, I = 5 А ΠΈ число Π΄Π΅Π»Π΅Π½ΠΈΠΉ ΡˆΠΊΠ°Π»Ρ‹ 150, Π²ΠΊΠ»ΡŽΡ‡Π΅Π½Ρ‹ Ρ‡Π΅Ρ€Π΅Π· ΠΈΠ·ΠΌΠ΅Ρ€ΠΈΡ‚Π΅Π»ΡŒΠ½Ρ‹ΠΉ трансформатор, напряТСния 6000/100 Π’ ΠΈ трансформатор Ρ‚ΠΎΠΊΠ° 500/5 А. ΠžΠΏΡ€Π΅Π΄Π΅Π»ΠΈΡ‚ΡŒ ΠΌΠΎΡ‰Π½ΠΎΡΡ‚ΡŒ ΠΏΠ΅Ρ€Π²ΠΈΡ‡Π½ΠΎΠΉ Ρ†Π΅ΠΏΠΈ, Ссли показания Π²Π°Ρ‚Ρ‚ΠΌΠ΅Ρ‚Ρ€Π° β€” 124 дСлСния.

РСшСниС:

28. Π§Π΅Ρ€Π΅Π· трансформатор Ρ‚ΠΎΠΊΠ° 500/5А ΠΈ трансформатор напряТСния 6000/100Π’ Π² ΡΠ΅Ρ‚ΡŒ ΠΏΠ΅Ρ€Π΅ΠΌΠ΅Π½Π½ΠΎΠ³ΠΎ Ρ‚ΠΎΠΊΠ° Π²ΠΊΠ»ΡŽΡ‡Π΅Π½Ρ‹ Π°ΠΌΠΏΠ΅Ρ€ΠΌΠ΅Ρ‚Ρ€, Π²ΠΎΠ»ΡŒΡ‚ΠΌΠ΅Ρ‚Ρ€ ΠΈ Π²Π°Ρ‚Ρ‚ΠΌΠ΅Ρ‚Ρ€. ΠžΠΏΡ€Π΅Π΄Π΅Π»ΠΈΡ‚ΡŒ Ρ‚ΠΎΠΊ, напряТСниС, Π°ΠΊΡ‚ΠΈΠ²Π½ΡƒΡŽ ΠΌΠΎΡ‰Π½ΠΎΡΡ‚ΡŒ ΠΈ коэффициСнт мощности Ρ†Π΅ΠΏΠΈ, Ссли Π°ΠΌΠΏΠ΅Ρ€ΠΌΠ΅Ρ‚Ρ€ ΠΏΠΎΠΊΠ°Π·Π°Π» I = 4 А, Π²ΠΎΠ»ΡŒΡ‚ΠΌΠ΅Ρ‚Ρ€ β€” U = 100 Π’, Π° Π²Π°Ρ‚Ρ‚ΠΌΠ΅Ρ‚Ρ€ β€” 350 Π’Ρ‚.

РСшСниС:

29. ΠžΠ±ΠΌΠΎΡ‚ΠΊΠΈ Π²Π°Ρ‚Ρ‚ΠΌΠ΅Ρ‚Ρ€Π°, рассчитанныС Π½Π° Π½ΠΎΠΌΠΈΠ½Π°Π»ΡŒΠ½ΡƒΡŽ ΠΌΠΎΡ‰Π½ΠΎΡΡ‚ΡŒ Π Π½ΠΎΠΌ = 500 Π’Ρ‚, присоСдинСны ΠΊ сСти ΠΏΠ΅Ρ€Π΅ΠΌΠ΅Π½Π½ΠΎΠ³ΠΎ Ρ‚ΠΎΠΊΠ° Ρ‡Π΅Ρ€Π΅Π· ΠΈΠ·ΠΌΠ΅Ρ€ΠΈΡ‚Π΅Π»ΡŒΠ½Ρ‹ΠΉ трансформатор напряТСния НОМ-6 (3000/100 Π’) ΠΈ трансформатор Ρ‚ΠΎΠΊΠ° Π’ΠŸΠ›-10 (400/5 А). ΠžΠΏΡ€Π΅Π΄Π΅Π»ΡΡ‚ΡŒ ΠΌΠΎΡ‰Π½ΠΎΡΡ‚ΡŒ ΠΏΠ΅Ρ€Π²ΠΈΡ‡Π½ΠΎΠΉ Ρ†Π΅ΠΏΠΈ, Ссли ΠΌΠΎΡ‰Π½ΠΎΡΡ‚ΡŒ Π² Ρ†Π΅ΠΏΠΈ Π²Π°Ρ‚Ρ‚ΠΌΠ΅Ρ‚Ρ€Π° Π  = 380 Π’Ρ‚.

РСшСниС:

30. К трансформатору напряТСния НОМ-10 (номинальноС напряТСниС ΠΏΠ΅Ρ€Π²ΠΈΡ‡Π½ΠΎΠΉ ΠΎΠ±ΠΌΠΎΡ‚ΠΊΠΈ 10 000 Π’) ΠΏΠΎΠ΄ΠΊΠ»ΡŽΡ‡ΠΈΠ»ΠΈ Π²ΠΎΠ»ΡŒΡ‚ΠΌΠ΅Ρ‚Ρ€, рассчитанный Π½Π° 150 Π’. ΠžΠΏΡ€Π΅Π΄Π΅Π»ΠΈΡ‚ΡŒ напряТСниС Π½Π° Π²ΠΎΠ»ΡŒΡ‚ΠΌΠ΅Ρ‚Ρ€Π΅, Ссли напряТСниС Π² ΠΏΠ΅Ρ€Π²ΠΈΡ‡Π½ΠΎΠΉ Ρ†Π΅ΠΏΠΈ понизилось Π΄ΠΎ 9950 Π’.

РСшСниС:

НС нашли Ρ‚ΠΎ, Ρ‡Ρ‚ΠΎ искали? Π’ΠΎΡΠΏΠΎΠ»ΡŒΠ·ΡƒΠΉΡ‚Π΅ΡΡŒ поиском:

Π›ΡƒΡ‡ΡˆΠΈΠ΅ изрСчСния: Для студСнта самоС Π³Π»Π°Π²Π½ΠΎΠ΅ Π½Π΅ ΡΠ΄Π°Ρ‚ΡŒ экзамСн, Π° воврСмя Π²ΡΠΏΠΎΠΌΠ½ΠΈΡ‚ΡŒ ΠΏΡ€ΠΎ Π½Π΅Π³ΠΎ. 9825 – | 7406 – ΠΈΠ»ΠΈ Ρ‡ΠΈΡ‚Π°Ρ‚ΡŒ всС.

91.146.8.87 Β© studopedia.ru НС являСтся Π°Π²Ρ‚ΠΎΡ€ΠΎΠΌ ΠΌΠ°Ρ‚Π΅Ρ€ΠΈΠ°Π»ΠΎΠ², ΠΊΠΎΡ‚ΠΎΡ€Ρ‹Π΅ Ρ€Π°Π·ΠΌΠ΅Ρ‰Π΅Π½Ρ‹. Но прСдоставляСт Π²ΠΎΠ·ΠΌΠΎΠΆΠ½ΠΎΡΡ‚ΡŒ бСсплатного использования. Π•ΡΡ‚ΡŒ Π½Π°Ρ€ΡƒΡˆΠ΅Π½ΠΈΠ΅ авторского ΠΏΡ€Π°Π²Π°? ΠΠ°ΠΏΠΈΡˆΠΈΡ‚Π΅ Π½Π°ΠΌ | ΠžΠ±Ρ€Π°Ρ‚Π½Π°Ρ связь.

ΠžΡ‚ΠΊΠ»ΡŽΡ‡ΠΈΡ‚Π΅ adBlock!
ΠΈ ΠΎΠ±Π½ΠΎΠ²ΠΈΡ‚Π΅ страницу (F5)

ΠΎΡ‡Π΅Π½ΡŒ Π½ΡƒΠΆΠ½ΠΎ

Найти Π²Π½ΡƒΡ‚Ρ€Π΅Π½Π½Π΅Π΅ сопротивлСниС ΠΈ Π­Π”Π‘ Π±Π°Ρ‚Π°Ρ€Π΅ΠΈ аккумуляторов

Π’ΠΠ Π˜ΠΠΠ’ 10-15-1.

  1. Найти Π²Π½ΡƒΡ‚Ρ€Π΅Π½Π½Π΅Π΅ сопротивлСниС ΠΈ Π­Π”Π‘ Π±Π°Ρ‚Π°Ρ€Π΅ΠΈ аккумуляторов, Ссли ΠΏΡ€ΠΈ сопротивлСнии внСшнСй части Ρ†Π΅ΠΏΠΈ 2,0 Ом Ρ‚ΠΎΠΊ Ρ€Π°Π²Π΅Π½ 0,80 А, Π° ΠΏΡ€ΠΈ сопротивлСнии

3 Ом — 0,60 А.

  1. Π’Ρ€ΠΈ потрСбитСля эл.энСргии сопротивлСниСм 12, 9 ΠΈ 3 Ом соСдинСны ΠΏΠΎΡΠ»Π΅Π΄ΠΎΠ²Π°Ρ‚Π΅Π»ΡŒΠ½ΠΎ. Напря­ТСниС Π½Π° ΠΊΠΎΠ½Ρ†Π°Ρ… Ρ†Π΅ΠΏΠΈ 120 Π’. Найти Ρ‚ΠΎΠΊ Π² Ρ†Π΅ΠΏΠΈ ΠΈ ΠΏΠ°Π΄Π΅Π½ΠΈΠ΅ напряТСния Π½Π° ΠΊΠ°ΠΆΠ΄ΠΎΠΌ ΠΏΠΎΡ‚Ρ€Π΅Π±ΠΈΡ‚Π΅Π»Π΅.

  1. Каково ΡƒΠ΄Π΅Π»ΡŒΠ½ΠΎΠ΅ сопротивлСниС ΠΏΡ€ΠΎΠ²ΠΎΠ΄Π°, Ссли Π΅Π³ΠΎ Π΄Π»ΠΈΠ½Π° 20 ΠΌ, ΠΏΠ»ΠΎΡ‰Π°Π΄ΡŒ ΠΏΠΎΠΏΠ΅Ρ€Π΅Ρ‡Π½ΠΎΠ³ΠΎ сСчСния 1,0 ΠΌΠΌ2 ΠΈ сопротивлСниС 10 Ом?

  2. Π§Π΅Ρ‚Ρ‹Ρ€Π΅ ΠΏΡ€ΠΎΠ²ΠΎΠ΄Π½ΠΈΠΊΠ° сопротивлСниСм 1 Ом ; 2 Ом ; 3 Ом ; 4 Ом соСдинСны ΠΏΠ°Ρ€Π°Π»Π»Π΅Π»ΡŒΠ½ΠΎ. ΠžΠΏΡ€Π΅Π΄Π΅Π»ΠΈΡ‚ΡŒ ΠΎΠ±Ρ‰Π΅Π΅ сопротивлСниС.

Π’ΠΠ Π˜ΠΠΠ’ 10-15-2

  1. По Ρ†Π΅ΠΏΠΈ, состоящСй ΠΈΠ· источника Ρ‚ΠΎΠΊΠ° с Π­Π”Π‘ 2 Π’ ΠΈ Π²Π½ΡƒΡ‚Ρ€Π΅Π½Π½ΠΈΠΌ сопротивлСниСм

2 Ом ΠΈ рСостата, ΠΈΠ΄Π΅Ρ‚ Ρ‚ΠΎΠΊ силой 0,5 А. Какой силы ΠΏΠΎΠΉΠ΄Π΅Ρ‚ Ρ‚ΠΎΠΊ ΠΏΡ€ΠΈ ΡƒΠΌΠ΅Π½ΡŒΡˆΠ΅Π½ΠΈΠΈ сопротивлСния рСостата Π² Π΄Π²Π° Ρ€Π°Π·Π°?

  1. ЦСпь состоит ΠΈΠ· Ρ‚Ρ€Π΅Ρ… сопротивлСний 10, 20 ΠΈ 30 Ом, соСдинСнных ΠΏΠΎΡΠ»Π΅Π΄ΠΎΠ²Π°Ρ‚Π΅Π»ΡŒΠ½ΠΎ. ПадСниС напряТСния Π½Π° ΠΏΠ΅Ρ€Π²ΠΎΠΌ сопротивлСнии 20 Π’. Най­ти ΠΏΠ°Π΄Π΅Π½ΠΈΠ΅ напряТСния Π½Π° ΠΎΡΡ‚Π°Π»ΡŒΠ½Ρ‹Ρ… сопротив­лСниях ΠΈ напряТСниС Π½Π° ΠΊΠΎΠ½Ρ†Π°Ρ… Ρ†Π΅ΠΏΠΈ.

  2. Π’ΠΎΡΠ΅ΠΌΡŒ ΠΏΡ€ΠΎΠ²ΠΎΠ΄Π½ΠΈΠΊΠΎΠ² сопротивлСниСм ΠΏΠΎ 20 0ΠΌ ΠΊΠ°ΠΆΠ΄Ρ‹ΠΉ соСдинСны ΠΏΠΎ Π΄Π²Π° ΠΏΠΎΡΠ»Π΅Π΄ΠΎΠ²Π°Ρ‚Π΅Π»ΡŒΠ½ΠΎ Π² Ρ‡Π΅Ρ‚Ρ‹Ρ€Π΅ ΠΏΠ°Ρ€Π°Π»Π»Π΅Π»ΡŒΠ½Ρ‹Π΅ Ρ†Π΅ΠΏΠΈ. ΠžΠΏΡ€Π΅Π΄Π΅Π»ΠΈΡ‚ΡŒ ΠΎΠ±Ρ‰Π΅Π΅ сопротивлСниС.

  3. Π”Π²Π° элСктронагрСватСля сопротивлСниСм 25 Ом ΠΈ 20 Ом находятся ΠΏΠΎΠ΄ напряТСниСм 100 Π’. КакоС количСство Ρ‚Π΅ΠΏΠ»ΠΎΡ‚Ρ‹ Π² Ρ‚Π΅Ρ‡Π΅Π½ΠΈΠ΅ 3 ΠΌΠΈΠ½ΡƒΡ‚ выдСлится нагрСватСлями ΠΏΡ€ΠΈ ΠΈΡ… ΠΏΠΎΡΠ»Π΅Π΄ΠΎΠ²Π°Ρ‚Π΅Π»ΡŒΠ½ΠΎΠΌ соСдинСнии?

Π’ΠΠ Π˜ΠΠΠ’ 10-15-3

  1. Π“Π°Π»ΡŒΠ²Π°Π½ΠΈΡ‡Π΅ΡΠΊΠΈΠΉ элСмСнт с Π­Π”Π‘ 14,0 Π’ ΠΈ Π²Π½ΡƒΡ‚Ρ€Π΅Π½Π½ΠΈΠΌ сопротивлСниСм 1,0 Ом Π·Π°ΠΌΠΊΠ½ΡƒΡ‚ Π½Π° внСшнСС сопротивлСниС 20,0 Ом. Π§Π΅ΠΌΡƒ Ρ€Π°Π²Π½ΠΎ напряТСниС Π½Π° внСшнСм сопротивлСнии?

  2. ΠŸΠΎΡΠ»Π΅Π΄ΠΎΠ²Π°Ρ‚Π΅Π»ΡŒΠ½ΠΎ Π΄ΡƒΠ³ΠΎΠ²ΠΎΠΉ Π»Π°ΠΌΠΏΠ΅ сопротивлСниСм 4 Ом Π²ΠΊΠ»ΡŽΡ‡Π΅Π½ рСостат сопротивлСниСм 8 Ом. ΠžΠΏΡ€Π΅Π΄Π΅Π»ΠΈΡ‚ΡŒ силу Ρ‚ΠΎΠΊΠ° Π² Π»Π°ΠΌΠΏΠ΅, Ссли напряТСниС Π² сСти 120 Π’.

  3. Какого сопротивлСния ΠΏΡ€ΠΎΠ²ΠΎΠ΄Π½ΠΈΠΊ Π½ΡƒΠΆΠ½ΠΎ ΡΠΎΠ΅Π΄ΠΈΒ­Π½ΠΈΡ‚ΡŒ ΠΏΠ°Ρ€Π°Π»Π»Π΅Π»ΡŒΠ½ΠΎ с ΠΏΡ€ΠΎΠ²ΠΎΠ΄Π½ΠΈΠΊΠΎΠΌ

300 Ом, Ρ‡Ρ‚ΠΎΠ±Ρ‹ ΠΎΠ±Ρ‰Π΅Π΅ сопротивлСниС ΠΈΡ… стало Ρ€Π°Π²Π½Ρ‹ΠΌ 120 Ом?

  1. Π’ ΠΊΠ°ΠΊΠΎΠΌ ΠΈΠ· Π΄Π²ΡƒΡ… сопротивлСний 2 Ом ΠΈ 10 Ом, соСдинСнных ΠΏΠΎΡΠ»Π΅Π΄ΠΎΠ²Π°Ρ‚Π΅Π»ΡŒΠ½ΠΎ, ΠΏΡ€ΠΈ ΠΏΡ€ΠΎΡ‚Π΅ΠΊΠ°Π½ΠΈΠΈ Ρ‚ΠΎΠΊΠ° выдСляСтся большСС количСство Ρ‚Π΅ΠΏΠ»ΠΎΡ‚Ρ‹?

Π’ΠΠ Π˜ΠΠΠ’ 10-15-4

  1. Π­Π”Π‘ источника напряТСния 6,0 Π’. ΠŸΡ€ΠΈ внСш­нСм сопротивлСнии Ρ†Π΅ΠΏΠΈ 1,0 Ом Ρ‚ΠΎΠΊ Ρ€Π°Π²Π΅Π½ 3,0 А. Каким Π±ΡƒΠ΄Π΅Ρ‚ Ρ‚ΠΎΠΊ ΠΊΠΎΡ€ΠΎΡ‚ΠΊΠΎΠ³ΠΎ замыкания?

  2. ΠžΠΏΡ€Π΅Π΄Π΅Π»ΠΈΡ‚ΡŒ сопротивлСниС Ρ†Π΅ΠΏΠΈ, состоящСй ΠΈΠ· эл.Π»Π°ΠΌΠΏΠΎΡ‡ΠΊΠΈ сопротивлСниСм

9,5 Ом, рСостата сопротивлСниСм 12 Ом ΠΈ ΠΌΠ΅Π΄Π½ΠΎΠ³ΠΎ ΠΏΡ€ΠΎΠ²ΠΎΠ΄Π½ΠΈΠΊΠ° Π΄Π»ΠΈΠ½ΠΎΠΉ 400 см ΠΈ сСчСниСм 0,4 ΠΌΠΌ2, соСдинСнных ΠΏΠΎΡΠ»Π΅Π΄ΠΎΠ²Π°Ρ‚Π΅Π»ΡŒΠ½ΠΎ.

  1. Π˜ΠΌΠ΅ΡŽΡ‚ΡΡ Ρ‚Ρ€ΠΈ ΠΏΡ€ΠΎΠ²ΠΎΠ΄Π½ΠΈΠΊΠ° сопротивлСниСм ΠΏΠΎ 2 Ом ΠΊΠ°ΠΆΠ΄Ρ‹ΠΉ. Как Π½ΡƒΠΆΠ½ΠΎ ΡΠΎΠ΅Π΄ΠΈΠ½ΠΈΡ‚ΡŒ ΠΈΡ… ΠΌΠ΅ΠΆΠ΄Ρƒ собой, Ρ‡Ρ‚ΠΎΠ±Ρ‹ ΠΎΠ±Ρ‰Π΅Π΅ сопротивлСниС Π±Ρ‹Π»ΠΎ Ρ€Π°Π²Π½ΠΎ 3 Ом.

  2. Π”Π²Π° элСктронагрСватСля 3 Ом ΠΈ 4 Ом соСдинСны ΠΏΠ°Ρ€Π°Π»Π»Π΅Π»ΡŒΠ½ΠΎ. НапряТСниС Π² Ρ†Π΅ΠΏΠΈ 220 Π’. КакоС количСство Ρ‚Π΅ΠΏΠ»ΠΎΡ‚Ρ‹ выдСлится нагрСватСлями Π·Π° 1 ΠΌΠΈΠ½ΡƒΡ‚Ρƒ?

Π’ΠΠ Π˜ΠΠΠ’ 10-15-5

  1. ΠžΠΏΡ€Π΅Π΄Π΅Π»ΠΈΡ‚ΡŒ ΠΏΠΎΡ‚Π΅Ρ€ΡŽ напряТСния Π²Π½ΡƒΡ‚Ρ€ΠΈ источника Ρ‚ΠΎΠΊΠ° ΠΈ Π΅Π³ΠΎ Π­Π”Π‘, Ссли потСря напряТСния Π½Π° внСшнСй части Ρ†Π΅ΠΏΠΈ 1,2 Π’, сопротивлСниС внСш­нСй части Ρ†Π΅ΠΏΠΈ

1,5 Ом, Π²Π½ΡƒΡ‚Ρ€Π΅Π½Π½Π΅Π΅ сопротивлС­ниС источника Ρ‚ΠΎΠΊΠ° 0,3 Ом.

  1. Π”Π²Π° сопротивлСния ΠΏΠΎ 4 Ом ΠΊΠ°ΠΆΠ΄Ρ‹ΠΉ соСдинСны ΠΏΠ°Ρ€Π°Π»Π»Π΅Π»ΡŒΠ½ΠΎ. ΠŸΠΎΡΠ»Π΅Π΄ΠΎΠ²Π°Ρ‚Π΅Π»ΡŒΠ½ΠΎ ΠΊ Π½ΠΈΠΌ ΠΏΠΎΠ΄ΠΊΠ»ΡŽΡ‡Π΅Π½ΠΎ сопротивлСниС 6 Ом. ΠžΠΏΡ€Π΅Π΄Π΅Π»ΠΈΡ‚ΡŒ ΠΎΠ±Ρ‰Π΅Π΅ сопротив­лСниС.

  2. Каково ΡƒΠ΄Π΅Π»ΡŒΠ½ΠΎΠ΅ сопротивлСниС ΠΏΡ€ΠΎΠ²ΠΎΠ΄Π°, Ссли Π΅Π³ΠΎ Π΄Π»ΠΈΠ½Π° 10,0 ΠΊΠΌ, ΠΏΠ»ΠΎΡ‰Π°Π΄ΡŒ ΠΏΠΎΠΏΠ΅Ρ€Π΅Ρ‡Π½ΠΎΠ³ΠΎ сСчС­ния 70,0 ΠΌΠΌ2 ΠΈ сопротивлСниС 4,0 0ΠΌ.

  3. Π”Π²Π° элСктронагрСватСля сопротивлСниСм 6 0ΠΌ ΠΈ 4 0ΠΌ соСдинСны ΠΏΠΎΡΠ»Π΅Π΄ΠΎΠ²Π°Ρ‚Π΅Π»ΡŒΠ½ΠΎ ΠΈ находятся ΠΏΠΎΠ΄ напряТСниСм 110 Π’, КакоС количСство Ρ‚Π΅ΠΏΠ»ΠΎΡ‚Ρ‹ выдСлится ΠΈΠΌΠΈ Π·Π°

2 ΠΌΠΈΠ½ΡƒΡ‚Ρ‹?

Π’ΠΠ Π˜ΠΠΠ’ 10-15-6

  1. Π’ ΠΏΡ€ΠΎΠ²ΠΎΠ΄Π½ΠΈΠΊΠ΅ сопротивлСниСм 2 0ΠΌ, ΠΏΠΎΠ΄ΠΊΠ»ΡŽΡ‡Π΅Π½Β­Π½ΠΎΠΌ ΠΊ элСмСнту с Π­Π”Π‘=1,1 Π’, ΠΈΠ΄Π΅Ρ‚ Ρ‚ΠΎΠΊ 0,5 А, Какова сила Ρ‚ΠΎΠΊΠ° ΠΏΡ€ΠΈ ΠΊΠΎΡ€ΠΎΡ‚ΠΊΠΎΠΌ Π·Π°ΠΌΡ‹ΠΊΠ°Π½ΠΈΠΈ элСмСнта?

  2. Π”Π²Π° сопротивлСния ΠΏΠΎ 5 0ΠΌ ΠΊΠ°ΠΆΠ΄Ρ‹ΠΉ соСдинСны ΠΏΠ°Ρ€Π°Π»Π»Π΅Π»ΡŒΠ½ΠΎ. ΠžΠΏΡ€Π΅Π΄Π΅Π»ΠΈΡ‚ΡŒ силу Ρ‚ΠΎΠΊΠ° Π² Ρ†Π΅ΠΏΠΈ, Ссли напряТСниС Ρ€Π°Π²Π½ΠΎ 25 Π’.

  3. ΠœΠ΅Π΄Π½Ρ‹ΠΉ ΠΈ Π°Π»ΡŽΠΌΠΈΠ½ΠΈΠ΅Π²Ρ‹ΠΉ ΠΏΡ€ΠΎΠ²ΠΎΠ΄Π½ΠΈΠΊΠΈ ΠΈΠΌΠ΅ΡŽΡ‚ ΠΎΠ΄ΠΈΠ½Π°ΠΊΠΎΠ²Ρ‹Π΅ сСчСния ΠΈ сопротивлСния. Какой ΠΏΡ€ΠΎΠ²ΠΎΠ΄Π½ΠΈΠΊ Π΄Π»ΠΈΠ½Π½Π΅Π΅ ΠΈ Π²ΠΎ сколько Ρ€Π°Π·?

  4. КакоС количСство Ρ‚Π΅ΠΏΠ»ΠΎΡ‚Ρ‹ выдСлится эл.ΠΏΠ»ΠΈΡ‚ΠΊΠΎΠΉ сопротивлСниСм 10 0ΠΌ, Ссли ΠΏΠΎ Ρ†Π΅ΠΏΠΈ Ρ‚Π΅Ρ‡Π΅Ρ‚ Ρ‚ΠΎΠΊ 3 А Π·Π° 5 ΠΌΠΈΠ½ΡƒΡ‚? 10 ΠΌΠΈΠ½ΡƒΡ‚?

НахоТдСниС Π²Π½ΡƒΡ‚Ρ€Π΅Π½Π½Π΅Π³ΠΎ сопротивлСния ΠΈ Π­Π”Π‘ источника.

Π’ ΡΡ‚Π°Ρ‚ΡŒΠ΅Β Ρ€Π°ΡΡ‡Ρ‘Ρ‚ Π² ΠΌΠ°Ρ‚ΠΊΠ°Π΄Π΅ ΠΏΠ΅Ρ€Π΅Ρ…ΠΎΠ΄Π½Ρ‹Ρ… процСссов Π² ёмкостном Ρ„ΠΈΠ»ΡŒΡ‚Ρ€Π΅Β ΠΈΡΡΠ»Π΅Π΄ΠΎΠ²Π°Π»ΡΡ ΠΏΠ΅Ρ€Π΅Ρ…ΠΎΠ΄Π½Ρ‹ΠΉ процСсс Π² Ρ„ΠΈΠ»ΡŒΡ‚Ρ€Π΅ поставлСнном Π½Π° Π²Ρ‹Ρ…ΠΎΠ΄Π΅ ΠΎΠ΄Π½ΠΎΡ„Π°Π·Π½ΠΎΠ³ΠΎ однополупСриодного выпрямитСля, ΠΏΡ€ΠΈ этом Π² схСмС замСщСния Π²Ρ‹ΠΏΡ€ΡΠΌΠΈΡ‚Π΅Π»ΡŒ с источником ΠΏΠ΅Ρ€Π΅ΠΌΠ΅Π½Π½ΠΎΠ³ΠΎ напряТСния Π·Π°ΠΌΠ΅Π½Π΅Π½Ρ‹ ΠΏΠΎΡΠ»Π΅Π΄ΠΎΠ²Π°Ρ‚Π΅Π»ΡŒΠ½Ρ‹ΠΌ соСдинСниСм источника Π­Π”Π‘ ΠΈ рСзистора, такая Π·Π°ΠΌΠ΅Π½Π° Π΄Π΅Π»Π°Π΅Ρ‚ Π²ΠΎΠ·ΠΌΠΎΠΆΠ½Ρ‹ΠΌ расчёт схСм Π½ΠΎ ΠΏΡ€ΠΈ этом для расчётов трСбуСтся Π½Π°ΠΉΡ‚ΠΈ Π­Π”Π‘ источника ΠΈ Π΅Π³ΠΎ Π²Π½ΡƒΡ‚Ρ€Π΅Π½Π½Π΅Π΅ сопротивлСниС. Найти Π­Π”Π‘ источника ΠΈ Π΅Π³ΠΎ Π²Π½ΡƒΡ‚Ρ€Π΅Π½Π½Π΅Π΅ сопротивлСниС ΡΠΊΡΠΏΠ΅Ρ€Π΅ΠΌΠ΅Π½Ρ‚Π°Π»ΡŒΠ½ΠΎ ΠΌΠΎΠΆΠ½ΠΎ ΠΏΡ€ΠΎΠ΄Π΅Π»Π°Π² ΠΎΠΏΡ‹Ρ‚ холостого Ρ…ΠΎΠ΄Π° ΠΈ ΠΎΠΏΡ‹Ρ‚ ΠΊΠΎΡ€ΠΎΡ‚ΠΊΠΎΠ³ΠΎ замыкания Π½ΠΎ это Π½Π΅ всСгда Π²ΠΎΠ·ΠΌΠΎΠΆΠ½ΠΎ, Π½Π°ΠΏΡ€ΠΈΠΌΠ΅Ρ€ ΠΊΠΎΠ³Π΄Π° Π½Π΅ΠΎΠ±Ρ…ΠΎΠ΄ΠΈΠΌΠΎ Π½Π°ΠΉΡ‚ΠΈ Π­Π”Π‘ ΠΈ Π²Π½ΡƒΡ‚Ρ€Π΅Π½Π½Π΅Π΅ сопротивлСниС источника ΠΏΡ€Π΅Π΄ΡΡ‚Π°Π²Π»ΡΡŽΡ‰Π΅Π³ΠΎ собой Π²Ρ‚ΠΎΡ€ΠΈΡ‡Π½ΡƒΡŽ ΠΎΠ±ΠΌΠΎΡ‚ΠΊΡƒ трансформатора, поэтому Π±Ρ‹Π²Π°Π΅Ρ‚ Π½Π΅ΠΎΠ±Ρ…ΠΎΠ΄ΠΈΠΌΠΎ ΠΎΠΏΡ€Π΅Π΄Π΅Π»ΠΈΡ‚ΡŒ ΠΏΠ°Ρ€Π°ΠΌΠ΅Ρ‚Ρ€Ρ‹ схСмы замСщСния источника Π½Π΅ внося Π±ΠΎΠ»ΡŒΡˆΠΈΡ… ΠΈΠ·ΠΌΠ΅Π½Π΅Π½ΠΈΠΉ сопротивлСния Π½Π°Π³Ρ€ΡƒΠ·ΠΊΠΈ Π² схСму. Рассмотрим схСму Π½Π° рисункС 1:

Рисунок 1 — Π‘Ρ…Π΅ΠΌΠ° для опрСдСлСния ΠΏΠ°Ρ€Π°ΠΌΠ΅Ρ‚Ρ€ΠΎΠ² схСмы замСщСния источника.

Π’ этой схСмС значСния Π­Π”Π‘ источника ΠΈ Π΅Π³ΠΎ Π²Π½ΡƒΡ‚Ρ€Π΅Π½Π½Π΅Π³ΠΎ сопротивлСния нСизвСстны, извСстны Ρ‚ΠΎΠ»ΡŒΠΊΠΎ показания Π°ΠΌΠΏΠ΅Ρ€ΠΌΠ΅Ρ‚Ρ€Π° ΠΈ Π²ΠΎΠ»ΡŒΡ‚ΠΌΠ΅Ρ‚Ρ€Π°. Π£Ρ‡Ρ‚Ρ‘ΠΌ Ρ‡Ρ‚ΠΎ Ρ‚ΠΎΠΊΠ° Π² Ρ†Π΅ΠΏΠΈ Π²ΠΎΠ»ΡŒΡ‚ΠΌΠ΅Ρ‚Ρ€Π° Π½Π΅Ρ‚, Ρ‚Π°ΠΊ ΠΊΠ°ΠΊ Ρƒ Π½Π΅Π³ΠΎ большоС сопротивлСниС ΠΈ Π΅Π³ΠΎ ΠΏΡ€ΠΎΠ²ΠΎΠ΄ΠΈΠΌΠΎΡΡ‚ΡŒΡŽ ΠΌΠΎΠΆΠ½ΠΎ ΠΏΡ€Π΅Π½Π΅Π±Ρ€Π΅Ρ‡ а сопротивлСниС Π°ΠΌΠΏΠ΅Ρ€ΠΌΠ΅Ρ‚Ρ€Π° Π½Π°ΡΡ‚ΠΎΠ»ΡŒΠΊΠΎ ΠΌΠ°Π»ΠΎ Ρ‡Ρ‚ΠΎ ΠΈΠΌ Ρ‚ΠΎΠΆΠ΅Β ΠΌΠΎΠΆΠ½ΠΎ ΠΏΡ€Π΅Π½Π΅Π±Ρ€Π΅Ρ‡ ΠΈ Π·Π°ΠΌΠ΅Π½ΠΈΡ‚ΡŒ Π°ΠΌΠΏΠ΅Ρ€ΠΌΠ΅Ρ‚Ρ€ ΠΏΠ΅Ρ€Π΅ΠΌΡ‹Ρ‡ΠΊΠΎΠΉ. Π’ΠΎΠΊ Π² этой Ρ†Π΅ΠΏΠΈ ΠΎΠ±ΠΎΠ·Π½Π°Ρ‡ΠΈΠΌ ΠΊΠ°ΠΊ I1 (Π΅Π³ΠΎ ΠΏΠΎΠΊΠ°Π·Ρ‹Π²Π°Π΅Ρ‚ Π°ΠΌΠΏΠ΅Ρ€ΠΌΠ΅Ρ‚Ρ€) Π° напряТСниС Π½Π° R1 ΠΈΒ GΒ ΠΎΠ±ΠΎΠ·Π½Π°Ρ‡ΠΈΠΌ ΠΊΠ°ΠΊΒ U1 (Π΅Π³ΠΎ ΠΏΠΎΠΊΠ°Π·Ρ‹Π²Π°Π΅Ρ‚ Π²ΠΎΠ»ΡŒΡ‚ΠΌΠ΅Ρ‚Ρ€) ΠΏΡ€ΠΈ этом Π±ΡƒΠ΄Π΅ΠΌ ΡΡ‡ΠΈΡ‚Π°Ρ‚ΡŒ Ρ‡Ρ‚ΠΎΒ Ρ‚ΠΎΠΊ Π½Π°ΠΏΡ€Π°Π²Π»Π΅Π½ ΠΊΠ°ΠΊ ΠΏΠΎΠΊΠ°Π·Π°Π½ΠΎ Π½Π° рисункС 1, Π° напряТСния Π½Π°Β R1 ΠΈΒ r Π½Π°ΠΏΡ€Π°Π²Π»Π΅Π½Ρ‹ Π² Ρ‚ΡƒΠΆΠ΅ сторону Ρ‡Ρ‚ΠΎ ΠΈ Ρ‚ΠΎΠΊ. Рассмотрим схСму Π½Π° рисункС 2 Π² ΠΊΠΎΡ‚ΠΎΡ€ΠΎΠΉ ΠΈΠ·ΠΌΠ΅Π½Π΅Π½ΠΎ (Π² нашСм случаС ΡƒΠ²Π΅Π»ΠΈΡ‡Π΅Π½ΠΎ) сопротивлСниС рСостата:

Рисунок 2 — Π‘Ρ…Π΅ΠΌΠ° для опрСдСлСния ΠΏΠ°Ρ€Π°ΠΌΠ΅Ρ‚Ρ€ΠΎΠ² схСмы замСщСния источника с ΠΈΠ·ΠΌΠ΅Π½Ρ‘Π½Π½Ρ‹ΠΌ сопротивлСниСм рСостата.

Π’ этой схСмС ΠΏΠΎΠΊΠ°Π·Π°Π½ΠΈΠ΅ Π°ΠΌΠΏΠ΅Ρ€ΠΌΠ΅Ρ‚Ρ€Π° ΠΎΠ±ΠΎΠ·Π½Π°Ρ‡ΠΈΠΌ ΠΊΠ°ΠΊ I2 Π° ΠΏΠΎΠΊΠ°Π·Π°Π½ΠΈΠ΅ Π²ΠΎΠ»ΡŒΡ‚ΠΌΠ΅Ρ‚Ρ€Π° ΠΊΠ°ΠΊ U2.

Из схСмы Π½Π° рисункС 1, составим ΡƒΡ€Π°Π²Π½Π΅Π½ΠΈΠ΅ ΠΏΠΎ Π²Ρ‚ΠΎΡ€ΠΎΠΌΡƒ Π·Π°ΠΊΠΎΠ½Ρƒ ΠšΠΈΡ€Ρ…Π³ΠΎΡ„Π° для контура ΠΊΠΎΡ‚ΠΎΡ€Ρ‹ΠΉ остаётся Ссли Π·Π°ΠΌΠ΅Π½ΠΈΡ‚ΡŒ Π²ΠΎΠ»ΡŒΡ‚ΠΌΠ΅Ρ‚Ρ€ Ρ€Π°Π·Ρ€Ρ‹Π²ΠΎΠΌο»Ώ:

Π—Π΄Π΅ΡΡŒ E — Π­Π”Π‘ источника, U1 — напряТСниС Π½Π° рСостатС (ΠΏΠΎΠΊΠ°Π·Ρ‹Π²Π°Π΅Ρ‚ Π²ΠΎΠ»ΡŒΡ‚ΠΌΠ΅Ρ‚Ρ€), I1 — Ρ‚ΠΎΠΊ Π² Ρ†Π΅ΠΏΠΈ (ΠΏΠΎΠΊΠ°Π·Ρ‹Π²Π°Π΅Ρ‚ Π°ΠΌΠΏΠ΅Ρ€ΠΌΠ΅Ρ‚Ρ€), r — Π²Π½ΡƒΡ‚Ρ€Π΅Π½Π½Π΅Π΅ сопротивлСниС источника. Π’Ρ‹Ρ€Π°Π·ΠΈΠΌ ΠΈΠ· уравнСния (1) напряТСниС U1:

Аналогично Π½Π°ΠΉΠ΄Ρ‘ΠΌ U2, ΠΈΡΠΏΠΎΠ»ΡŒΠ·ΡƒΡ схСму Π½Π° рисункС 2:

ΠŸΠΎΠ΄ΡΡ‚Π°Π²ΠΈΠΌ (1) Π² (3):

Π’Ρ‹Ρ€Π°Π·ΠΈΠΌ ΠΈΠ· уравнСния (4) Π²Π½ΡƒΡ‚Ρ€Π΅Π½Π½Π΅Π΅ сопротивлСниС источника r:

Β ΠŸΠΎΠ΄ΡΡ‚Π°Π²ΠΈΠΌ (6) Π² (1) ΠΈ Π½Π°ΠΉΠ΄Ρ‘ΠΌ Π­Π”Π‘ источника:

По Ρ„ΠΎΡ€ΠΌΡƒΠ»Π°ΠΌ (6) ΠΈ (7) находятся ΠΏΠ°Ρ€Π°ΠΌΠ΅Ρ‚Ρ€Ρ‹ схСмы замСщСния источника элСктричСской энСргии (ΠΏΠΎ Ρ„ΠΎΡ€ΠΌΡƒΠ»Π΅ (7) Π΅Π³ΠΎ Π­Π”Π‘, ΠΏΠΎ Ρ„ΠΎΡ€ΠΌΡƒΠ»Π΅ (6) Π΅Π³ΠΎ Π²Π½ΡƒΡ‚Ρ€Π΅Π½Π½Π΅Π΅ сопротивлСниС). ΠŸΠΎΡΠ»Π΅Π΄ΠΎΠ²Π°Ρ‚Π΅Π»ΡŒΠ½ΠΎ с рСостатом ΠΌΠΎΠΆΠ½ΠΎ ΠΏΠΎΡΡ‚Π°Π²ΠΈΡ‚ΡŒ ΠΈΠ·ΠΌΠ΅Ρ€ΠΈΡ‚Π΅Π»ΡŒΠ½Ρ‹ΠΉ рСзистор ΠΈ ΠΈΡΠΏΠΎΠ»ΡŒΠ·ΠΎΠ²Π°Ρ‚ΡŒ Π΅Π³ΠΎ для измСрСния Ρ‚ΠΎΠΊΠ° Π²ΠΎΠ»ΡŒΡ‚ΠΌΠ΅Ρ‚Ρ€ΠΎΠΌ Ρ‚ΠΎΠ³Π΄Π° измСрСния ΠΌΠΎΠΆΠ½ΠΎ ΠΏΡ€ΠΎΠ²ΠΎΠ΄ΠΈΡ‚ΡŒ ΠΎΠ΄Π½ΠΈΠΌ Π²ΠΎΠ»ΡŒΡ‚ΠΌΠ΅Ρ‚Ρ€ΠΎΠΌ сначала ΠΏΠΎΠ΄ΠΊΠ»ΡŽΡ‡Π°Ρ Π΅Π³ΠΎ ΠΏΠ°Ρ€Π°Π»Π΅Π»ΡŒΠ½ΠΎ источнику G, Π° ΠΏΠΎΡ‚ΠΎΠΌ ΠΏΠ°Ρ€Π°Π»Π΅Π»ΡŒΠ½ΠΎ ΠΈΠ·ΠΌΠ΅Ρ€ΠΈΡ‚Π΅Π»ΡŒΠ½ΠΎΠΌΡƒ рСзистору.
Для расчёта Π²Π½ΡƒΡ‚Ρ€Π΅Π½Π½Π΅Π³ΠΎ сопротивлСния ΠΈ Π­Π”Π‘ источника ΠΌΠΎΠΆΠ½ΠΎ Π²ΠΎΡΠΏΠΎΠ»ΡŒΠ·ΠΎΠ²Π°Ρ‚ΡŒΡΡ ΠΏΡ€ΠΎΠ³Ρ€Π°ΠΌΠΌΠΎΠΉ:
ΠŸΠ΅Ρ€Π²ΠΎΠ΅ ΠΈΠ·ΠΌΠ΅Ρ€Π΅Π½ΠΈΠ΅ Π΄ΠΎΠ»ΠΆΠ½ΠΎ Π±Ρ‹Ρ‚ΡŒ с мСньшим сопротивлСниСм рСостата, Π° Π²Ρ‚ΠΎΡ€ΠΎΠ΅ с большим.

эдс источника Π²Π½ΡƒΡ‚Ρ€Π΅Π½Π½Π΅Π΅ сопротивлСниС

эдс источника Π²Π½ΡƒΡ‚Ρ€Π΅Π½Π½Π΅Π΅ сопротивлСниС


Π—Π°Π΄Π°Ρ‡Π° 13496

На рисункС Ξ΅1 = 10 Π’, Ξ΅2 = 20 Π’, Ξ΅3 = 40 Π’, Π° сопротивлСния R1 = R2 = R3 = 10 Ом. ΠžΠΏΡ€Π΅Π΄Π΅Π»ΠΈΡ‚Π΅ силу Ρ‚ΠΎΠΊΠΎΠ², ΠΏΡ€ΠΎΡ‚Π΅ΠΊΠ°ΡŽΡ‰ΠΈΡ… Ρ‡Π΅Ρ€Π΅Π· сопротивлСния (I) ΠΈ Ρ‡Π΅Ρ€Π΅Π· источники Π­Π”Π‘ (I’). Π’Π½ΡƒΡ‚Ρ€Π΅Π½Π½Π΅Π΅ сопротивлСниС источников Π­Π”Π‘ Π½Π΅ ΡƒΡ‡ΠΈΡ‚Ρ‹Π²Π°Ρ‚ΡŒ.
.


Π—Π°Π΄Π°Ρ‡Π° 10083

ΠžΠΏΡ€Π΅Π΄Π΅Π»ΠΈΡ‚ΡŒ Π­Π”Π‘ ΠΈ Π²Π½ΡƒΡ‚Ρ€Π΅Π½Π½Π΅Π΅ сопротивлСниС источника, Ссли ΠΏΡ€ΠΈ ΠΏΠΎΠ΄ΠΊΠ»ΡŽΡ‡Π΅Π½ΠΈΠΈ ΠΊ этому источнику Π½Π°Π³Ρ€ΡƒΠ·ΠΊΠΈ сопротивлСниСм 5 Ом напряТСниС Π½Π° Π½Π΅ΠΉ составляСт 10 Π’, Π° ΠΏΡ€ΠΈ ΠΏΠΎΠ΄ΠΊΠ»ΡŽΡ‡Π΅Π½ΠΈΠΈ Π½Π°Π³Ρ€ΡƒΠ·ΠΊΠΈ сопротивлСниСм 20 Ом напряТСниС Π½Π° Π½Π°Π³Ρ€ΡƒΠ·ΠΊΠ΅ составляСт 20 Π’.


Π—Π°Π΄Π°Ρ‡Π° 60002

ПолСзная ΠΌΠΎΡ‰Π½ΠΎΡΡ‚ΡŒ, выдСляСмая Π²ΠΎ внСшнСй части Ρ†Π΅ΠΏΠΈ, достигаСт наибольшСго значСния 5 Π’Ρ‚ ΠΏΡ€ΠΈ силС Ρ‚ΠΎΠΊΠ° 5 А. Найти Π²Π½ΡƒΡ‚Ρ€Π΅Π½Π½Π΅Π΅ сопротивлСниС ΠΈ Π­Π”Π‘ источника.


Π—Π°Π΄Π°Ρ‡Π° 60089

ΠŸΡ€ΠΈ ΠΊΠ°ΠΊΠΎΠΌ внСшнСм сопротивлСнии потрСбляСмая ΠΌΠΎΡ‰Π½ΠΎΡΡ‚ΡŒ Π±ΡƒΠ΄Π΅Ρ‚ максимальной, Ссли Π΄Π²Π° ΠΎΠ΄ΠΈΠ½Π°ΠΊΠΎΠ²Ρ‹Ρ… источника с Π­Π”Π‘ 6 Π’ ΠΈ Π²Π½ΡƒΡ‚Ρ€Π΅Π½Π½ΠΈΠΌ сопротивлСниСм 1 Ом ΠΊΠ°ΠΆΠ΄Ρ‹ΠΉ соСдинСны ΠΏΠΎΡΠ»Π΅Π΄ΠΎΠ²Π°Ρ‚Π΅Π»ΡŒΠ½ΠΎ? Π§Π΅ΠΌΡƒ Ρ€Π°Π²Π½Π° эта ΠΌΠΎΡ‰Π½ΠΎΡΡ‚ΡŒ?


Π—Π°Π΄Π°Ρ‡Π° 60124

ΠžΠΏΡ€Π΅Π΄Π΅Π»ΠΈΡ‚ΡŒ Π­Π”Π‘ ΠΈ Π²Π½ΡƒΡ‚Ρ€Π΅Π½Π½Π΅Π΅ сопротивлСниС источника Ρ‚ΠΎΠΊΠ°, Ссли ΠΏΡ€ΠΈ внСшнСм сопротивлСнии 3,9 Ом сила Ρ‚ΠΎΠΊΠ° Π² Ρ†Π΅ΠΏΠΈ Ρ€Π°Π²Π½Π° 0,5 А, Π° ΠΏΡ€ΠΈ внСшнСм сопротивлСнии 1,9 Ом β€” 1 А.


Π—Π°Π΄Π°Ρ‡Π° 60377

Π‘ΠΎΠΏΡ€ΠΎΡ‚ΠΈΠ²Π»Π΅Π½ΠΈΠ΅ R1 = 5 Ом, Π²ΠΎΠ»ΡŒΡ‚ΠΌΠ΅Ρ‚Ρ€ ΠΈ источник Ρ‚ΠΎΠΊΠ° соСдинСны ΠΏΠ°Ρ€Π°Π»Π»Π΅Π»ΡŒΠ½ΠΎ. Π’ΠΎΠ»ΡŒΡ‚ΠΌΠ΅Ρ‚Ρ€ ΠΏΠΎΠΊΠ°Π·Ρ‹Π²Π°Π΅Ρ‚ напряТСниС U1 = 10 Π’. Если Π·Π°ΠΌΠ΅Π½ΠΈΡ‚ΡŒ сопротивлСниС R1 Π½Π° R2 = 12 Ом, Ρ‚ΠΎ Π²ΠΎΠ»ΡŒΡ‚ΠΌΠ΅Ρ‚Ρ€ ΠΏΠΎΠΊΠ°ΠΆΠ΅Ρ‚ напряТСниС U2 = 12 Π’. ΠžΠΏΡ€Π΅Π΄Π΅Π»ΠΈΡ‚ΡŒ Π­Π”Π‘ ΠΈ Π²Π½ΡƒΡ‚Ρ€Π΅Π½Π½Π΅Π΅ сопротивлСниС источника Ρ‚ΠΎΠΊΠ°. Π’ΠΎΠΊΠΎΠΌ Ρ‡Π΅Ρ€Π΅Π· Π²ΠΎΠ»ΡŒΡ‚ΠΌΠ΅Ρ‚Ρ€ ΠΏΡ€Π΅Π½Π΅Π±Ρ€Π΅Ρ‡ΡŒ.


Π—Π°Π΄Π°Ρ‡Π° 60435

ΠŸΡ€ΠΈ ΠΊΠ°ΠΊΠΎΠΌ внСшнСм сопротивлСнии потрСбляСмая ΠΌΠΎΡ‰Π½ΠΎΡΡ‚ΡŒ Π±ΡƒΠ΄Π΅Ρ‚ максимальна, Ссли Π΄Π²Π° ΠΎΠ΄ΠΈΠ½Π°ΠΊΠΎΠ²Ρ‹Ρ… источника с Π­Π”Π‘ 6 Π’ ΠΈ Π²Π½ΡƒΡ‚Ρ€Π΅Π½Π½ΠΈΠΌ сопротивлСниСм 1 Ом ΠΊΠ°ΠΆΠ΄Ρ‹ΠΉ соСдинСны ΠΏΠ°Ρ€Π°Π»Π»Π΅Π»ΡŒΠ½ΠΎ? Π§Π΅ΠΌΡƒ Ρ€Π°Π²Π½Π° эта ΠΌΠΎΡ‰Π½ΠΎΡΡ‚ΡŒ?


Π—Π°Π΄Π°Ρ‡Π° 12316

ΠžΠΏΡ€Π΅Π΄Π΅Π»ΠΈΡ‚ΡŒ эдс ΠΈ Π²Π½ΡƒΡ‚Ρ€Π΅Π½Π½Π΅Π΅ сопротивлСниС источника питания, Ссли Π² Ρ€Π΅ΠΆΠΈΠΌΠ΅ холостого Ρ…ΠΎΠ΄Π° напряТСниС Π½Π° Π²Ρ‹Π²ΠΎΠ΄Π°Ρ… 15 Π’, Π° Π² Ρ€Π΅ΠΆΠΈΠΌΠ΅ ΠΊΠΎΡ€ΠΎΡ‚ΠΊΠΎΠ³ΠΎ замыкания Ρ‚ΠΎΠΊ 0,5 А. Найти Ρ‚ΠΎΠΊ Π² этой Ρ†Π΅ΠΏΠΈ ΠΏΡ€ΠΈ ΠΏΠΎΠ΄ΠΊΠ»ΡŽΡ‡Π΅Π½ΠΈΠΈ рСзистора сопротивлСниСм 120 Ом.


Π—Π°Π΄Π°Ρ‡Π° 16205

Π’ Π·Π°ΠΌΠΊΠ½ΡƒΡ‚ΠΎΠΉ Ρ†Π΅ΠΏΠΈ ΠΏΡ€ΠΈ сопротивлСнии Π½Π°Π³Ρ€ΡƒΠ·ΠΊΠΈ R1 = 3 Ом Ρ‚ΠΎΠΊ I1 = 3 А, Π° ΠΏΡ€ΠΈ R2 = 2 Ом, I2 = 4 А. ΠžΠΏΡ€Π΅Π΄Π΅Π»ΠΈΡ‚ΡŒ эдс источника ΠΈ Π΅Π³ΠΎ Π²Π½ΡƒΡ‚Ρ€Π΅Π½Π½Π΅Π΅ сопротивлСниС.


Π—Π°Π΄Π°Ρ‡Π° 16262

БатарСя ΠΈΠ· Π΄Π²ΡƒΡ… ΠΏΠ°Ρ€Π°Π»Π»Π΅Π»ΡŒΠ½Ρ‹Ρ… источников с Π­Π”Π‘ 2 ΠΈ 1,8 Π’ ΠΈ Π²Π½ΡƒΡ‚Ρ€Π΅Π½Π½ΠΈΠΌ сопротивлСниСм 50 мОм ΠΊΠ°ΠΆΠ΄Ρ‹ΠΉ Π·Π°ΠΌΠΊΠ½ΡƒΡ‚Π° Π½Π° сопротивлСниС 2 Ом. Найти Π²Π΅Π»ΠΈΡ‡ΠΈΠ½Ρƒ Ρ‚ΠΎΠΊΠ°, проходящСго Ρ‡Π΅Ρ€Π΅Π· сопротивлСниС ΠΈ Ρ‡Π΅Ρ€Π΅Π· источники.


Π—Π°Π΄Π°Ρ‡Π° 17294

ΠœΠΎΡ‰Π½ΠΎΡΡ‚ΡŒ, Π²Ρ‹Π΄Π΅Π»ΡΡŽΡ‰Π°ΡΡΡ Π½Π° Π½Π°Π³Ρ€ΡƒΠ·ΠΊΠ΅, ΠΏΠΎΠ΄ΠΊΠ»ΡŽΡ‡Π΅Π½Π½ΠΎΠΉ ΠΊ источнику с Π­Π”Π‘ 5 Π’ ΠΈ Π²Π½ΡƒΡ‚Ρ€Π΅Π½Π½ΠΈΠΌ сопротивлСниСм 2 Ом, составляСт 2 Π’Ρ‚. ΠžΠΏΡ€Π΅Π΄Π΅Π»ΠΈΡ‚ΡŒ Ρ‚ΠΎΠΊ Π² Ρ†Π΅ΠΏΠΈ ΠΈ сопротивлСниС Π½Π°Π³Ρ€ΡƒΠ·ΠΊΠΈ.


Π—Π°Π΄Π°Ρ‡Π° 21481

Π’ΠΎΠΊ ΠΊΠΎΡ€ΠΎΡ‚ΠΊΠΎΠ³ΠΎ замыкания источника 48 А. ΠŸΡ€ΠΈ ΠΏΠΎΠ΄ΠΊΠ»ΡŽΡ‡Π΅Π½ΠΈΠΈ ΠΊ источнику рСзистора сопротивлСниСм 19,5 Ом Ρ‚ΠΎΠΊ Π² Ρ†Π΅ΠΏΠΈ ΡƒΠΌΠ΅Π½ΡŒΡˆΠΈΠ»ΡΡ Π΄ΠΎ 1,2 А. ΠžΠΏΡ€Π΅Π΄Π΅Π»ΠΈΡ‚ΡŒ Π­Π”Π‘ источника, Π΅Π³ΠΎ Π²Π½ΡƒΡ‚Ρ€Π΅Π½Π½Π΅Π΅ сопротивлСниС, Π° Ρ‚Π°ΠΊΠΆΠ΅ ΠΌΠΎΡ‰Π½ΠΎΡΡ‚ΡŒ источника Π² ΠΎΠ±ΠΎΠΈΡ… случаях.


Π—Π°Π΄Π°Ρ‡Π° 21499

Π’ΠΎΠΊ ΠΊΠΎΡ€ΠΎΡ‚ΠΊΠΎΠ³ΠΎ замыкания источника 48 А. ΠŸΡ€ΠΈ ΠΏΠΎΠ΄ΠΊΠ»ΡŽΡ‡Π΅Π½ΠΈΠΈ ΠΊ источнику рСзистора сопротивлСниСм R = 19,5 Ом Ρ‚ΠΎΠΊ Π² Ρ†Π΅ΠΏΠΈ ΡƒΠΌΠ΅Π½ΡŒΡˆΠΈΠ»ΡΡ Π΄ΠΎ 1,2 А. ΠžΠΏΡ€Π΅Π΄Π΅Π»ΠΈΡ‚ΡŒ Π­Π”Π‘ источника ΠΈ Π΅Π³ΠΎ Π²Π½ΡƒΡ‚Ρ€Π΅Π½Π½Π΅Π΅ сопротивлСниС.


Π˜Π·ΠΌΠ΅Ρ€Π΅Π½ΠΈΠ΅ Π²Π½ΡƒΡ‚Ρ€Π΅Π½Π½Π΅Π³ΠΎ сопротивлСния Π±Π°Ρ‚Π°Ρ€Π΅ΠΉ

Π”ΠΎΠ±Π°Π²Π»Π΅Π½ΠΎ Π² ΠΈΠ·Π±Ρ€Π°Π½Π½ΠΎΠ΅ Π›ΡŽΠ±ΠΈΠΌΡ‹ΠΉ 7

Π’Π½ΡƒΡ‚Ρ€Π΅Π½Π½Π΅Π΅ сопротивлСниС

ΠŸΡ€ΠΈ ΠΏΡ€ΠΎΠ΅ΠΊΡ‚ΠΈΡ€ΠΎΠ²Π°Π½ΠΈΠΈ схСмы с аккумулятором ΠΌΡ‹ часто ΠΏΡ€Π΅Π΄ΠΏΠΎΠ»Π°Π³Π°Π΅ΠΌ, Ρ‡Ρ‚ΠΎ аккумулятор являСтся ΠΈΠ΄Π΅Π°Π»ΡŒΠ½Ρ‹ΠΌ источником напряТСния. Π­Ρ‚ΠΎ ΠΎΠ·Π½Π°Ρ‡Π°Π΅Ρ‚, Ρ‡Ρ‚ΠΎ нСзависимо ΠΎΡ‚ Ρ‚ΠΎΠ³ΠΎ, ΠΊΠ°ΠΊΡƒΡŽ Π½Π°Π³Ρ€ΡƒΠ·ΠΊΡƒ ΠΌΡ‹ ΠΏΡ€ΠΈΠ»Π°Π³Π°Π΅ΠΌ ΠΊ аккумулятору, напряТСниС Π½Π° Π²Ρ‹Π²ΠΎΠ΄Π°Ρ… источника всСгда остаСтся Π½Π΅ΠΈΠ·ΠΌΠ΅Π½Π½Ρ‹ΠΌ.

Если ΠΌΡ‹ ΠΌΠΎΠ΄Π΅Π»ΠΈΡ€ΡƒΠ΅ΠΌ эту Π±Π°Ρ‚Π°Ρ€Π΅ΡŽ ΠΊΠ°ΠΊ ΠΈΠ΄Π΅Π°Π»ΡŒΠ½Ρ‹ΠΉ источник напряТСния, ΠΈΠ·ΠΌΠ΅Π½Π΅Π½ΠΈΠ΅ значСния R L Π½Π΅ влияСт Π½Π° напряТСниС ΠΌΠ΅ΠΆΠ΄Ρƒ Π²Ρ‹Π²ΠΎΠ΄Π°ΠΌΠΈ Π±Π°Ρ‚Π°Ρ€Π΅ΠΈ

На самом Π΄Π΅Π»Π΅ нСсколько Ρ„Π°ΠΊΡ‚ΠΎΡ€ΠΎΠ² ΠΌΠΎΠ³ΡƒΡ‚ ΠΎΠ³Ρ€Π°Π½ΠΈΡ‡ΠΈΡ‚ΡŒ ΡΠΏΠΎΡΠΎΠ±Π½ΠΎΡΡ‚ΡŒ Π±Π°Ρ‚Π°Ρ€Π΅ΠΈ Π΄Π΅ΠΉΡΡ‚Π²ΠΎΠ²Π°Ρ‚ΡŒ ΠΊΠ°ΠΊ ΠΈΠ΄Π΅Π°Π»ΡŒΠ½Ρ‹ΠΉ источник напряТСния.Π Π°Π·ΠΌΠ΅Ρ€ Π±Π°Ρ‚Π°Ρ€Π΅ΠΈ, химичСскиС свойства, возраст ΠΈ Ρ‚Π΅ΠΌΠΏΠ΅Ρ€Π°Ρ‚ΡƒΡ€Π° — всС это влияСт Π½Π° Π²Π΅Π»ΠΈΡ‡ΠΈΠ½Ρƒ Ρ‚ΠΎΠΊΠ°, ΠΊΠΎΡ‚ΠΎΡ€ΡƒΡŽ батарСя ΠΌΠΎΠΆΠ΅Ρ‚ Π²Ρ‹Π΄Π°Π²Π°Ρ‚ΡŒ. Π’ Ρ€Π΅Π·ΡƒΠ»ΡŒΡ‚Π°Ρ‚Π΅ ΠΌΡ‹ ΠΌΠΎΠΆΠ΅ΠΌ ΡΠΎΠ·Π΄Π°Ρ‚ΡŒ Π»ΡƒΡ‡ΡˆΡƒΡŽ модСль Π±Π°Ρ‚Π°Ρ€Π΅ΠΈ с ΠΈΠ΄Π΅Π°Π»ΡŒΠ½Ρ‹ΠΌ источником напряТСния ΠΈ рСзистором, Π²ΠΊΠ»ΡŽΡ‡Π΅Π½Π½Ρ‹ΠΌ Π² ΡΠ΅Ρ€ΠΈΡŽ .

Π‘Π°Ρ‚Π°Ρ€Π΅ΠΈ ΠΌΠΎΠΆΠ½ΠΎ ΡΠΌΠΎΠ΄Π΅Π»ΠΈΡ€ΠΎΠ²Π°Ρ‚ΡŒ ΠΊΠ°ΠΊ ΠΈΠ΄Π΅Π°Π»ΡŒΠ½Ρ‹ΠΉ источник напряТСния с ΠΏΠΎΡΠ»Π΅Π΄ΠΎΠ²Π°Ρ‚Π΅Π»ΡŒΠ½Ρ‹ΠΌ рСзистором (ΠΎΠ±ΠΎΠ·Π½Π°Ρ‡Π΅Π½Π½Ρ‹ΠΌ R I )

ΠœΡ‹ ΠΌΠΎΠΆΠ΅ΠΌ ΠΈΠ·ΠΌΠ΅Ρ€ΠΈΡ‚ΡŒ напряТСниС аккумулятора Π½Π° Π΅Π³ΠΎ Π²Ρ‹Π²ΠΎΠ΄Π°Ρ… Π±Π΅Π· ΠΏΠΎΠ΄ΠΊΠ»ΡŽΡ‡Π΅Π½Π½ΠΎΠΉ Π½Π°Π³Ρ€ΡƒΠ·ΠΊΠΈ. Π­Ρ‚ΠΎ извСстно ΠΊΠ°ΠΊ напряТСниС холостого Ρ…ΠΎΠ΄Π° (Π’ OC ).

Π˜Π·ΠΌΠ΅Ρ€Π΅Π½ΠΈΠ΅ напряТСния Ρ‰Π΅Π»ΠΎΡ‡Π½ΠΎΠ³ΠΎ элСмСнта AA Π±Π΅Π· Π½Π°Π³Ρ€ΡƒΠ·ΠΊΠΈ

ΠžΠ±Ρ€Π°Ρ‚ΠΈΡ‚Π΅ Π²Π½ΠΈΠΌΠ°Π½ΠΈΠ΅, Ρ‡Ρ‚ΠΎ, ΠΏΠΎΡΠΊΠΎΠ»ΡŒΠΊΡƒ Ρ‡Π΅Ρ€Π΅Π· Π²Π½ΡƒΡ‚Ρ€Π΅Π½Π½ΠΈΠΉ рСзистор Π½Π΅ Ρ‚Π΅Ρ‡Π΅Ρ‚ Ρ‚ΠΎΠΊ, ΠΏΠ°Π΄Π΅Π½ΠΈΠ΅ напряТСния Π½Π° Π½Π΅ΠΌ Ρ€Π°Π²Π½ΠΎ 0 Π’. Π‘Π»Π΅Π΄ΠΎΠ²Π°Ρ‚Π΅Π»ΡŒΠ½ΠΎ, ΠΌΡ‹ ΠΌΠΎΠΆΠ΅ΠΌ ΠΏΡ€Π΅Π΄ΠΏΠΎΠ»ΠΎΠΆΠΈΡ‚ΡŒ, Ρ‡Ρ‚ΠΎ V OC Ρ€Π°Π²Π½ΠΎ Π½Π°ΠΏΡ€ΡΠΆΠ΅Π½ΠΈΡŽ идСального источника напряТСния Π² Π±Π°Ρ‚Π°Ρ€Π΅Π΅.

Если ΠΌΡ‹ ΠΏΠΎΠ΄ΠΊΠ»ΡŽΡ‡ΠΈΠΌ Π½Π°Π³Ρ€ΡƒΠ·ΠΊΡƒ ΠΊ аккумулятору, напряТСниС Π½Π° Π²Ρ‹Π²ΠΎΠ΄Π°Ρ… ΡƒΠΏΠ°Π΄Π΅Ρ‚.

Π—Π΄Π΅ΡΡŒ ΠΌΡ‹ измСряСм ΠΏΠ°Π΄Π΅Π½ΠΈΠ΅ напряТСния Π½Π° рСзисторС 4 Ом

Π­Ρ‚ΠΎ ΠΏΠ°Π΄Π΅Π½ΠΈΠ΅ напряТСния Π²Ρ‹Π·Π²Π°Π½ΠΎ Π²Π½ΡƒΡ‚Ρ€Π΅Π½Π½ΠΈΠΌ сопротивлСниСм Π±Π°Ρ‚Π°Ρ€Π΅ΠΈ.ΠœΡ‹ ΠΌΠΎΠΆΠ΅ΠΌ Ρ€Π°ΡΡΡ‡ΠΈΡ‚Π°Ρ‚ΡŒ Π²Π½ΡƒΡ‚Ρ€Π΅Π½Π½Π΅Π΅ сопротивлСниС, Ссли снимСм показания напряТСния холостого Ρ…ΠΎΠ΄Π° ΠΈ напряТСния Π½Π° ΠΊΠ»Π΅ΠΌΠΌΠ°Ρ… Π±Π°Ρ‚Π°Ρ€Π΅ΠΈ с ΠΏΠΎΠ΄ΠΊΠ»ΡŽΡ‡Π΅Π½Π½ΠΎΠΉ Π½Π°Π³Ρ€ΡƒΠ·ΠΊΠΎΠΉ.

Для Π½Π°Ρ‡Π°Π»Π° создадим схСму, ΠΏΠΎΠΊΠ°Π·Ρ‹Π²Π°ΡŽΡ‰ΡƒΡŽ Π½Π°ΡˆΡƒ схСму.

Π’ΠΎΡ‚ наша схСма. ΠœΡ‹ Ρ…ΠΎΡ‚ΠΈΠΌ Ρ€Π°ΡΡΡ‡ΠΈΡ‚Π°Ρ‚ΡŒ R I .

ΠœΡ‹ ΠΌΠΎΠΆΠ΅ΠΌ ΠΏΠΎΠ΄ΠΊΠ»ΡŽΡ‡ΠΈΡ‚ΡŒ ΠΈΠ·ΠΌΠ΅Ρ€Π΅Π½Π½ΠΎΠ΅ Π½Π°ΠΌΠΈ напряТСниС ΠΏΠΎΠ΄ Π½Π°Π³Ρ€ΡƒΠ·ΠΊΠΎΠΉ (V L ) ΠΈ Π·Π½Π°Ρ‡Π΅Π½ΠΈΠ΅ рСзистора (R L ) Π² Π·Π°ΠΊΠΎΠ½ Ома, Ρ‡Ρ‚ΠΎΠ±Ρ‹ ΠΏΠΎΠ»ΡƒΡ‡ΠΈΡ‚ΡŒ Ρ‚ΠΎΠΊ, ΠΏΡ€ΠΎΡ‚Π΅ΠΊΠ°ΡŽΡ‰ΠΈΠΉ ΠΏΠΎ Ρ†Π΅ΠΏΠΈ (I).

Нам Ρ‚Π°ΠΊΠΆΠ΅ Π½ΡƒΠΆΠ½ΠΎ ΠΏΠΎΠ»ΡƒΡ‡ΠΈΡ‚ΡŒ напряТСниС Π½Π° Π²Π½ΡƒΡ‚Ρ€Π΅Π½Π½Π΅ΠΌ рСзисторС. ΠœΡ‹ ΠΌΠΎΠΆΠ΅ΠΌ ΡΠ΄Π΅Π»Π°Ρ‚ΡŒ это, ΠΈΡΠΏΠΎΠ»ΡŒΠ·ΡƒΡ Π·Π°ΠΊΠΎΠ½ напряТСния ΠšΠΈΡ€Ρ…Π³ΠΎΡ„Π°. Π£ΠΏΡ€ΠΎΡ‰Π΅Π½Π½ΠΎ для этой схСмы, ΠΌΡ‹ ΠΌΠΎΠΆΠ΅ΠΌ ΡΠΊΠ°Π·Π°Ρ‚ΡŒ, Ρ‡Ρ‚ΠΎ ΠΏΠ°Π΄Π΅Π½ΠΈΠ΅ напряТСния Π½Π° ΠΎΠ±ΠΎΠΈΡ… рСзисторах Π΄ΠΎΠ»ΠΆΠ½ΠΎ ΡΠΎΡΡ‚Π°Π²Π»ΡΡ‚ΡŒ Π² суммС напряТСниС идСального источника напряТСния.

Π’Π΅ΠΏΠ΅Ρ€ΡŒ, ΠΊΠΎΠ³Π΄Π° ΠΌΡ‹ Π·Π½Π°Π΅ΠΌ ΠΏΠ°Π΄Π΅Π½ΠΈΠ΅ напряТСния Π½Π° Π²Π½ΡƒΡ‚Ρ€Π΅Π½Π½Π΅ΠΌ рСзисторС ΠΈ Ρ‚ΠΎΠΊ Ρ‡Π΅Ρ€Π΅Π· Π½Π΅Π³ΠΎ, ΠΌΡ‹ ΠΌΠΎΠΆΠ΅ΠΌ снова ΠΈΡΠΏΠΎΠ»ΡŒΠ·ΠΎΠ²Π°Ρ‚ΡŒ Π·Π°ΠΊΠΎΠ½ Ома, Ρ‡Ρ‚ΠΎΠ±Ρ‹ Π½Π°ΠΉΡ‚ΠΈ Π΅Π³ΠΎ сопротивлСниС.

ΠžΡ‚ΡΡŽΠ΄Π° Π²ΠΈΠ΄Π½ΠΎ, Ρ‡Ρ‚ΠΎ Π²Π½ΡƒΡ‚Ρ€Π΅Π½Π½Π΅Π΅ сопротивлСниС (Π½Π° Π΄Π°Π½Π½Ρ‹ΠΉ ΠΌΠΎΠΌΠ΅Π½Ρ‚) элСмСнта AA составляСт 0.273 Ом .

ΠŸΠ Π˜ΠœΠ•Π§ΠΠΠ˜Π• : Π‘ ΠΏΠΎΠΌΠΎΡ‰ΡŒΡŽ этого ΠΌΠ΅Ρ‚ΠΎΠ΄Π° ΠΌΡ‹ ΠΌΠΎΠΆΠ΅ΠΌ Ρ‚ΠΎΠ»ΡŒΠΊΠΎ ΡΠ΄Π΅Π»Π°Ρ‚ΡŒ снимок Π²Π½ΡƒΡ‚Ρ€Π΅Π½Π½Π΅Π³ΠΎ сопротивлСния. Π’Π½ΡƒΡ‚Ρ€Π΅Π½Π½Π΅Π΅ сопротивлСниС ΠΌΠΎΠΆΠ΅Ρ‚ Π²Π°Ρ€ΡŒΠΈΡ€ΠΎΠ²Π°Ρ‚ΡŒΡΡ Π² зависимости ΠΎΡ‚ возраста Π±Π°Ρ‚Π°Ρ€Π΅ΠΈ ΠΈ Ρ‚Π΅ΠΌΠΏΠ΅Ρ€Π°Ρ‚ΡƒΡ€Ρ‹. Π§Π΅Ρ€Π΅Π· 10 ΠΌΠΈΠ½ΡƒΡ‚ Π·Π½Π°Ρ‡Π΅Π½ΠΈΠ΅ сопротивлСния ΠΌΠΎΠΆΠ΅Ρ‚ ΠΈΠ·ΠΌΠ΅Π½ΠΈΡ‚ΡŒΡΡ! ΠžΠ±Ρ‹Ρ‡Π½Π°Ρ щСлочная батарСя AA ΠΌΠΎΠΆΠ΅Ρ‚ ΠΈΠΌΠ΅Ρ‚ΡŒ Π²Π½ΡƒΡ‚Ρ€Π΅Π½Π½Π΅Π΅ сопротивлСниС ΠΎΡ‚ 0,1 Ом Π΄ΠΎ 0,9 Ом.


← ΠŸΡ€Π΅Π΄Ρ‹Π΄ΡƒΡ‰Π°Ρ страница
Π’Π²Π΅Π΄Π΅Π½ΠΈΠ΅

ΠžΠΏΡ€Π΅Π΄Π΅Π»Π΅Π½ΠΈΠ΅ Π­Π”Π‘ ΠΈ Π²Π½ΡƒΡ‚Ρ€Π΅Π½Π½Π΅Π³ΠΎ сопротивлСния ячСйки — ЭлСктричСскиС источники ΠΈ Π²Π½ΡƒΡ‚Ρ€Π΅Π½Π½Π΅Π΅ сопротивлСниС — Π’Ρ‹ΡΡˆΠ°Ρ физичСская вСрсия

Π’ΠΎΠ»ΡŒΡ‚ΠΌΠ΅Ρ‚Ρ€ ΠΈ ΠΏΠ΅Ρ€Π΅ΠΌΠ΅Π½Π½Ρ‹ΠΉ рСзистор

ΠŸΠΎΡΠΌΠΎΡ‚Ρ€ΠΈΡ‚Π΅ это Π²ΠΈΠ΄Π΅ΠΎ, Ρ‡Ρ‚ΠΎΠ±Ρ‹ ΡƒΠ·Π½Π°Ρ‚ΡŒ, ΠΊΠ°ΠΊ ΠΌΠΎΠΆΠ½ΠΎ ΠΈΠ·ΠΌΠ΅Ρ€ΠΈΡ‚ΡŒ Π­Π”Π‘ ΠΈ Π²Π½ΡƒΡ‚Ρ€Π΅Π½Π½Π΅Π΅ сопротивлСниС ячСйки.

Как ΠΏΠΎΠΊΠ°Π·Π°Π½ΠΎ Π½Π° Π²ΠΈΠ΄Π΅ΠΎ, Ρ‡Ρ‚ΠΎΠ±Ρ‹ Π½Π°ΠΉΡ‚ΠΈ Π­Π”Π‘ ΠΈ Π²Π½ΡƒΡ‚Ρ€Π΅Π½Π½Π΅Π΅ сопротивлСниС ячСйки, создаСтся ΡΠ»Π΅Π΄ΡƒΡŽΡ‰Π°Ρ схСма.

Π˜Π·ΠΌΠ΅Π½ΡΠ΅Ρ‚ΡΡ ΠΏΠ΅Ρ€Π΅ΠΌΠ΅Π½Π½Ρ‹ΠΉ рСзистор ΠΈ ΡΠ½ΠΈΠΌΠ°ΡŽΡ‚ΡΡ показания разности ΠΏΠΎΡ‚Π΅Π½Ρ†ΠΈΠ°Π»ΠΎΠ² Π½Π° Π²Ρ‹Π²ΠΎΠ΄Π°Ρ… элСмСнта ΠΈ Ρ‚ΠΎΠΊΠ° Ρ‡Π΅Ρ€Π΅Π· элСмСнт. Π—Π°Ρ‚Π΅ΠΌ строится Π³Ρ€Π°Ρ„ΠΈΠΊ этих Ρ€Π΅Π·ΡƒΠ»ΡŒΡ‚Π°Ρ‚ΠΎΠ², ΠΊΠ°ΠΊ ΠΏΠΎΠΊΠ°Π·Π°Π½ΠΎ Π½ΠΈΠΆΠ΅.

Π’ Ρ‚ΠΎΡ‡ΠΊΠ΅, Π² ΠΊΠΎΡ‚ΠΎΡ€ΠΎΠΉ линия пСрСсСкаСт ось ΠΊΠΎΠ½Π΅Ρ‡Π½ΠΎΠΉ разности ΠΏΠΎΡ‚Π΅Π½Ρ†ΠΈΠ°Π»ΠΎΠ², Π½Π΅Ρ‚ Ρ‚ΠΎΠΊΠ° , ΠΏΡ€ΠΎΡ‚Π΅ΠΊΠ°ΡŽΡ‰Π΅Π³ΠΎ ΠΈΠ· ячСйки. Π­Ρ‚ΠΎ называСтся ΠΏΠΎΡ‚Π΅Π½Ρ†ΠΈΠ°Π»ΠΎΠΌ Ρ€Π°Π·ΠΎΠΌΠΊΠ½ΡƒΡ‚ΠΎΠΉ Ρ†Π΅ΠΏΠΈ Ρ†Π΅ΠΏΠΈ.

ΠŸΠΎΡΠΊΠΎΠ»ΡŒΠΊΡƒ Π½Π΅Ρ‚ Ρ‚ΠΎΠΊΠ°, Π½Π΅ Π±ΡƒΠ΄Π΅Ρ‚ ΠΏΠΎΡ‚Π΅Ρ€ΡŒ напряТСния, поэтому это Π·Π½Π°Ρ‡Π΅Π½ΠΈΠ΅ являСтся Π­Π”Π‘ ячСйки. Π’Π°ΠΊΠΈΠΌ ΠΎΠ±Ρ€Π°Π·ΠΎΠΌ, Ρ‚ΠΎΡ‡ΠΊΠ°, Π³Π΄Π΅ линия пСрСсСкаСт ось ΠΊΠΎΠ½Π΅Ρ‡Π½ΠΎΠΉ разности ΠΏΠΎΡ‚Π΅Π½Ρ†ΠΈΠ°Π»ΠΎΠ² (Ρ‚ΠΎΡ‡ΠΊΠ° пСрСсСчСния оси Y), являСтся Π­Π”Π‘ ячСйки.

Π’ Ρ‚ΠΎΡ‡ΠΊΠ΅, Π³Π΄Π΅ линия пСрСсСкаСт Ρ‚Π΅ΠΊΡƒΡ‰ΡƒΡŽ ось (Ρ‚ΠΎΡ‡ΠΊΠ° пСрСсСчСния оси x), ΠΈΠ· ячСйки бСрСтся ΠΌΠ°ΠΊΡΠΈΠΌΠ°Π»ΡŒΠ½Ρ‹ΠΉ Ρ‚ΠΎΠΊ. Π­Ρ‚ΠΎ происходит ΠΏΡ€ΠΈ сопротивлСнии Π½Π°Π³Ρ€ΡƒΠ·ΠΊΠΈ \ (R = 0 \ Omega. \)

. Π­Ρ‚ΠΎ ΠΌΠΎΠΆΠ΅Ρ‚ Π±Ρ‹Ρ‚ΡŒ достигнуто ΠΏΡƒΡ‚Π΅ΠΌ ΠΊΠΎΡ€ΠΎΡ‚ΠΊΠΎΠ³ΠΎ замыкания ячСйки (этого слСдуСт ΠΈΠ·Π±Π΅Π³Π°Ρ‚ΡŒ, ΠΏΠΎΡΠΊΠΎΠ»ΡŒΠΊΡƒ ячСйка ΠΌΠΎΠΆΠ΅Ρ‚ ΠΏΠ΅Ρ€Π΅Π³Ρ€Π΅Ρ‚ΡŒΡΡ ΠΈ это ΠΏΠΎΡ‚Π΅Π½Ρ†ΠΈΠ°Π»ΡŒΠ½ΠΎ опасно).ΠœΠ°ΠΊΡΠΈΠΌΠ°Π»ΡŒΠ½Ρ‹ΠΉ Ρ‚ΠΎΠΊ называСтся Ρ‚ΠΎΠΊΠΎΠΌ ΠΊΠΎΡ€ΠΎΡ‚ΠΊΠΎΠ³ΠΎ замыкания , \ (I_ {SC} \).

Для опрСдСлСния Π²Π½ΡƒΡ‚Ρ€Π΅Π½Π½Π΅Π³ΠΎ сопротивлСния ячСйки рассчитываСтся Π³Ρ€Π°Π΄ΠΈΠ΅Π½Ρ‚ Π»ΠΈΠ½ΠΈΠΈ. Π­Ρ‚ΠΎ ΠΎΡ‚Ρ€ΠΈΡ†Π°Ρ‚Π΅Π»ΡŒΠ½ΠΎΠ΅ Π·Π½Π°Ρ‡Π΅Π½ΠΈΠ΅. Π’Π½ΡƒΡ‚Ρ€Π΅Π½Π½Π΅Π΅ сопротивлСниС ячСйки Ρ‚Π°ΠΊΠΎΠ΅ ΠΆΠ΅, Π½ΠΎ Π±Π΅Π· Π·Π½Π°ΠΊΠ° минус. НапримСр, Ссли Π½Π°ΠΊΠ»ΠΎΠ½ Π»ΠΈΠ½ΠΈΠΈ Ρ€Π°Π²Π΅Π½ \ (- 4 \), Ρ‚ΠΎ Π²Π½ΡƒΡ‚Ρ€Π΅Π½Π½Π΅Π΅ сопротивлСниС Ρ€Π°Π²Π½ΠΎ \ (4 \ Omega \).

Π’Π½ΡƒΡ‚Ρ€Π΅Π½Π½Π΅Π΅ сопротивлСниС Ρ‚Π°ΠΊΠΆΠ΅ ΠΌΠΎΠΆΠ½ΠΎ Π½Π°ΠΉΡ‚ΠΈ, Ρ€Π°Π·Π΄Π΅Π»ΠΈΠ² Π­Π”Π‘ \ (E \) Π½Π° Ρ‚ΠΎΠΊ ΠΊΠΎΡ€ΠΎΡ‚ΠΊΠΎΠ³ΠΎ замыкания \ (I_ {SC} \).

Как Ρ€Π°ΡΡΡ‡ΠΈΡ‚Π°Ρ‚ΡŒ Π­Π”Π‘ | Sciencing

ОбновлСно 2 ноября 2020 Π³.

Π›ΠΈ ДТонсон

ЭлСктродвиТущая сила (Π­Π”Π‘) — понятиС Π½Π΅Π·Π½Π°ΠΊΠΎΠΌΠΎΠ΅ для Π±ΠΎΠ»ΡŒΡˆΠΈΠ½ΡΡ‚Π²Π° людСй, Π½ΠΎ ΠΎΠ½ΠΎ тСсно связано с Π±ΠΎΠ»Π΅Π΅ Π·Π½Π°ΠΊΠΎΠΌΡ‹ΠΌ понятиСм напряТСния. ПониманиС Ρ€Π°Π·Π½ΠΈΡ†Ρ‹ ΠΌΠ΅ΠΆΠ΄Ρƒ Π½ΠΈΠΌΠΈ ΠΈ Ρ‚ΠΎΠ³ΠΎ, Ρ‡Ρ‚ΠΎ ΠΎΠ·Π½Π°Ρ‡Π°Π΅Ρ‚ Π­Π”Π‘, Π΄Π°Π΅Ρ‚ Π²Π°ΠΌ инструмСнты, Π½Π΅ΠΎΠ±Ρ…ΠΎΠ΄ΠΈΠΌΡ‹Π΅ для Ρ€Π΅ΡˆΠ΅Π½ΠΈΡ ΠΌΠ½ΠΎΠ³ΠΈΡ… ΠΏΡ€ΠΎΠ±Π»Π΅ΠΌ Π² Ρ„ΠΈΠ·ΠΈΠΊΠ΅ ΠΈ элСктроникС, Π° Ρ‚Π°ΠΊΠΆΠ΅ Π·Π½Π°ΠΊΠΎΠΌΠΈΡ‚ с ΠΊΠΎΠ½Ρ†Π΅ΠΏΡ†ΠΈΠ΅ΠΉ Π²Π½ΡƒΡ‚Ρ€Π΅Π½Π½Π΅Π³ΠΎ сопротивлСния Π±Π°Ρ‚Π°Ρ€Π΅ΠΈ. Π­Π”Π‘ сообщаСт Π²Π°ΠΌ напряТСниС Π±Π°Ρ‚Π°Ρ€Π΅ΠΈ Π±Π΅Π· ΡƒΠΌΠ΅Π½ΡŒΡˆΠ΅Π½ΠΈΡ Π²Π½ΡƒΡ‚Ρ€Π΅Π½Π½Π΅Π³ΠΎ сопротивлСния, ΠΊΠ°ΠΊ это происходит ΠΏΡ€ΠΈ ΠΎΠ±Ρ‹Ρ‡Π½Ρ‹Ρ… измСрСниях разности ΠΏΠΎΡ‚Π΅Π½Ρ†ΠΈΠ°Π»ΠΎΠ².Π’Ρ‹ ΠΌΠΎΠΆΠ΅Ρ‚Π΅ Ρ€Π°ΡΡΡ‡ΠΈΡ‚Π°Ρ‚ΡŒ Π΅Π³ΠΎ нСсколькими способами, Π² зависимости ΠΎΡ‚ Ρ‚ΠΎΠ³ΠΎ, какая информация Ρƒ вас Π΅ΡΡ‚ΡŒ.

TL; DR (слишком Π΄ΠΎΠ»Π³ΠΎ; Π½Π΅ Ρ‡ΠΈΡ‚Π°Π»)

РассчитайтС Π­Π”Π‘ ΠΏΠΎ Ρ„ΠΎΡ€ΠΌΡƒΠ»Π΅:

Ξ΅ = V + Ir

Π—Π΄Π΅ΡΡŒ (Π’) ΠΎΠ·Π½Π°Ρ‡Π°Π΅Ρ‚ напряТСниС элСмСнта, (I) ΠΎΠ·Π½Π°Ρ‡Π°Π΅Ρ‚ Ρ‚ΠΎΠΊ Π² Ρ†Π΅ΠΏΠΈ, Π° (r) ΠΎΠ·Π½Π°Ρ‡Π°Π΅Ρ‚ Π²Π½ΡƒΡ‚Ρ€Π΅Π½Π½Π΅Π΅ сопротивлСниС ячСйки.

Π§Ρ‚ΠΎ Ρ‚Π°ΠΊΠΎΠ΅ Π­Π”Π‘?

ЭлСктродвиТущая сила — это Ρ€Π°Π·Π½ΠΎΡΡ‚ΡŒ ΠΏΠΎΡ‚Π΅Π½Ρ†ΠΈΠ°Π»ΠΎΠ² (Ρ‚. Π•. НапряТСниС) Π½Π° ΠΊΠ»Π΅ΠΌΠΌΠ°Ρ… Π±Π°Ρ‚Π°Ρ€Π΅ΠΈ ΠΏΡ€ΠΈ отсутствии Ρ‚ΠΎΠΊΠ°.ΠœΠΎΠΆΠ΅Ρ‚ ΠΏΠΎΠΊΠ°Π·Π°Ρ‚ΡŒΡΡ, Ρ‡Ρ‚ΠΎ это Π½Π΅ ΠΈΠΌΠ΅Π΅Ρ‚ значСния, Π½ΠΎ каТдая батарСя ΠΈΠΌΠ΅Π΅Ρ‚ Β«Π²Π½ΡƒΡ‚Ρ€Π΅Π½Π½Π΅Π΅ сопротивлСниС». Π­Ρ‚ΠΎ ΠΏΠΎΡ…ΠΎΠΆΠ΅ Π½Π° ΠΎΠ±Ρ‹Ρ‡Π½ΠΎΠ΅ сопротивлСниС, ΠΊΠΎΡ‚ΠΎΡ€ΠΎΠ΅ сниТаСт Ρ‚ΠΎΠΊ Π² Ρ†Π΅ΠΏΠΈ, Π½ΠΎ ΠΎΠ½ΠΎ сущСствуСт Π²Π½ΡƒΡ‚Ρ€ΠΈ самой Π±Π°Ρ‚Π°Ρ€Π΅ΠΈ. Π­Ρ‚ΠΎ связано с Ρ‚Π΅ΠΌ, Ρ‡Ρ‚ΠΎ ΠΌΠ°Ρ‚Π΅Ρ€ΠΈΠ°Π»Ρ‹, ΠΈΠ· ΠΊΠΎΡ‚ΠΎΡ€Ρ‹Ρ… состоят элСмСнты Π² Π±Π°Ρ‚Π°Ρ€Π΅Π΅, ΠΈΠΌΠ΅ΡŽΡ‚ собствСнноС сопротивлСниС (Ρ‚Π°ΠΊ ΠΊΠ°ΠΊ практичСски всС ΠΌΠ°Ρ‚Π΅Ρ€ΠΈΠ°Π»Ρ‹ ΠΈΠΌΠ΅ΡŽΡ‚).

Когда Ρ‚ΠΎΠΊ Π½Π΅ Ρ‚Π΅Ρ‡Π΅Ρ‚ Ρ‡Π΅Ρ€Π΅Π· элСмСнт, это Π²Π½ΡƒΡ‚Ρ€Π΅Π½Π½Π΅Π΅ сопротивлСниС Π½ΠΈΡ‡Π΅Π³ΠΎ Π½Π΅ мСняСт, ΠΏΠΎΡ‚ΠΎΠΌΡƒ Ρ‡Ρ‚ΠΎ Π½Π΅Ρ‚ Ρ‚ΠΎΠΊΠ° для Π΅Π³ΠΎ замСдлСния.Π’ Π½Π΅ΠΊΠΎΡ‚ΠΎΡ€ΠΎΠΌ смыслС, Π­Π”Π‘ ΠΌΠΎΠΆΠ½ΠΎ Ρ€Π°ΡΡΠΌΠ°Ρ‚Ρ€ΠΈΠ²Π°Ρ‚ΡŒ ΠΊΠ°ΠΊ ΠΌΠ°ΠΊΡΠΈΠΌΠ°Π»ΡŒΠ½ΡƒΡŽ Ρ€Π°Π·Π½ΠΎΡΡ‚ΡŒ ΠΏΠΎΡ‚Π΅Π½Ρ†ΠΈΠ°Π»ΠΎΠ² Π½Π° ΠΊΠ»Π΅ΠΌΠΌΠ°Ρ… Π² идСальной ситуации, ΠΈ Π½Π° ΠΏΡ€Π°ΠΊΡ‚ΠΈΠΊΠ΅ ΠΎΠ½Π° всСгда большС, Ρ‡Π΅ΠΌ напряТСниС Π±Π°Ρ‚Π°Ρ€Π΅ΠΈ.

УравнСния для расчСта Π­Π”Π‘

Π•ΡΡ‚ΡŒ Π΄Π²Π° основных уравнСния для расчСта Π­Π”Π‘. НаиболСС Ρ„ΡƒΠ½Π΄Π°ΠΌΠ΅Π½Ρ‚Π°Π»ΡŒΠ½ΠΎΠ΅ ΠΎΠΏΡ€Π΅Π΄Π΅Π»Π΅Π½ΠΈΠ΅ — это количСство Π΄ΠΆΠΎΡƒΠ»Π΅ΠΉ энСргии (E), ΠΊΠΎΡ‚ΠΎΡ€ΠΎΠ΅ ΠΊΠ°ΠΆΠ΄Ρ‹ΠΉ ΠΊΡƒΠ»ΠΎΠ½ заряда (Q) Π·Π°Π±ΠΈΡ€Π°Π΅Ρ‚ ΠΏΡ€ΠΈ ΠΏΡ€ΠΎΡ…ΠΎΠΆΠ΄Π΅Π½ΠΈΠΈ Ρ‡Π΅Ρ€Π΅Π· ячСйку:

Π“Π΄Π΅ (Ξ΅) — символ элСктродвиТущСй силы, (E) — энСргия Π² Ρ†Π΅ΠΏΠΈ, Π° (Q) — заряд Ρ†Π΅ΠΏΠΈ.Если Π²Ρ‹ Π·Π½Π°Π΅Ρ‚Π΅ Ρ€Π΅Π·ΡƒΠ»ΡŒΡ‚ΠΈΡ€ΡƒΡŽΡ‰ΡƒΡŽ ΡΠ½Π΅Ρ€Π³ΠΈΡŽ ΠΈ количСство заряда, проходящСго Ρ‡Π΅Ρ€Π΅Π· ячСйку, это самый простой способ Ρ€Π°ΡΡΡ‡ΠΈΡ‚Π°Ρ‚ΡŒ Π­Π”Π‘, Π½ΠΎ Π² Π±ΠΎΠ»ΡŒΡˆΠΈΠ½ΡΡ‚Π²Π΅ случаСв Ρƒ вас Π½Π΅Ρ‚ этой ΠΈΠ½Ρ„ΠΎΡ€ΠΌΠ°Ρ†ΠΈΠΈ.

ВмСсто этого Π²Ρ‹ ΠΌΠΎΠΆΠ΅Ρ‚Π΅ ΠΈΡΠΏΠΎΠ»ΡŒΠ·ΠΎΠ²Π°Ρ‚ΡŒ ΠΎΠΏΡ€Π΅Π΄Π΅Π»Π΅Π½ΠΈΠ΅, большС ΠΏΠΎΡ…ΠΎΠΆΠ΅Π΅ Π½Π° Π·Π°ΠΊΠΎΠ½ Ома (V = IR). Π­Ρ‚ΠΎ ΠΌΠΎΠΆΠ΅Ρ‚ Π±Ρ‹Ρ‚ΡŒ Π²Ρ‹Ρ€Π°ΠΆΠ΅Π½ΠΎ ΠΊΠ°ΠΊ:

\ epsilon = I (R + r)

, Π³Π΄Π΅ (I) ΠΎΠ·Π½Π°Ρ‡Π°Π΅Ρ‚ Ρ‚ΠΎΠΊ, (R) — сопротивлСниС рассматриваСмой Ρ†Π΅ΠΏΠΈ, Π° (r) — Π²Π½ΡƒΡ‚Ρ€Π΅Π½Π½Π΅Π΅ сопротивлСниС ячСйки. Π Π°ΡΡˆΠΈΡ€Π΅Π½ΠΈΠ΅ этого ΠΏΠΎΠΊΠ°Π·Ρ‹Π²Π°Π΅Ρ‚ Ρ‚Π΅ΡΠ½ΡƒΡŽ связь с Π·Π°ΠΊΠΎΠ½ΠΎΠΌ Ома:

\ epsilon = IR + Ir = V + Ir

Π­Ρ‚ΠΎ ΠΏΠΎΠΊΠ°Π·Ρ‹Π²Π°Π΅Ρ‚, Ρ‡Ρ‚ΠΎ Π²Ρ‹ ΠΌΠΎΠΆΠ΅Ρ‚Π΅ Ρ€Π°ΡΡΡ‡ΠΈΡ‚Π°Ρ‚ΡŒ Π­Π”Π‘, Ссли Π²Ρ‹ Π·Π½Π°Π΅Ρ‚Π΅ напряТСниС Π½Π° ΠΊΠ»Π΅ΠΌΠΌΠ°Ρ… (напряТСниС, ΠΈΡΠΏΠΎΠ»ΡŒΠ·ΡƒΠ΅ΠΌΠΎΠ΅ Π² Ρ€Π΅Π°Π»ΡŒΠ½Ρ‹Ρ… ситуациях) , ΠΏΡ€ΠΎΡ‚Π΅ΠΊΠ°ΡŽΡ‰ΠΈΠΉ Ρ‚ΠΎΠΊ ΠΈ Π²Π½ΡƒΡ‚Ρ€Π΅Π½Π½Π΅Π΅ сопротивлСниС ячСйки.

Как Ρ€Π°ΡΡΡ‡ΠΈΡ‚Π°Ρ‚ΡŒ Π­Π”Π‘: ΠΏΡ€ΠΈΠΌΠ΅Ρ€

Π’ качСствС ΠΏΡ€ΠΈΠΌΠ΅Ρ€Π° ΠΏΡ€Π΅Π΄ΡΡ‚Π°Π²ΡŒΡ‚Π΅, Ρ‡Ρ‚ΠΎ Ρƒ вас Π΅ΡΡ‚ΡŒ Ρ†Π΅ΠΏΡŒ с Ρ€Π°Π·Π½ΠΎΡΡ‚ΡŒΡŽ ΠΏΠΎΡ‚Π΅Π½Ρ†ΠΈΠ°Π»ΠΎΠ² 3,2 Π’, ΠΏΡ€ΠΎΡ‚Π΅ΠΊΠ°ΡŽΡ‰ΠΈΠΌ Ρ‚ΠΎΠΊΠΎΠΌ 0,6 А ΠΈ Π²Π½ΡƒΡ‚Ρ€Π΅Π½Π½ΠΈΠΌ сопротивлСниСм Π±Π°Ρ‚Π°Ρ€Π΅ΠΈ 0,5 Ом. Π˜ΡΠΏΠΎΠ»ΡŒΠ·ΡƒΡ Ρ„ΠΎΡ€ΠΌΡƒΠ»Ρƒ Π²Ρ‹ΡˆΠ΅:

\ epsilon = V + Ir = 3.2 \ text {V} + (0.6 \ text {A}) (0.5 \ text {} \ Omega) = 3.5 \ text {V}

Π˜Ρ‚Π°ΠΊ, EMF этой Ρ†Π΅ΠΏΠΈ составляСт 3,5 Π’.

Π’Π½ΡƒΡ‚Ρ€Π΅Π½Π½Π΅Π΅ сопротивлСниС, Π­Π”Π‘ ΠΈ Ρ€Π°Π·Π½ΠΎΡΡ‚ΡŒ ΠΏΠΎΡ‚Π΅Π½Ρ†ΠΈΠ°Π»ΠΎΠ²

Π’ любой Ρ†Π΅ΠΏΠΈ Π΅ΡΡ‚ΡŒ ΠΊΠΎΠΌΠΏΠΎΠ½Π΅Π½Ρ‚Ρ‹, ΠΊΠΎΡ‚ΠΎΡ€Ρ‹Π΅ ΠΏΠ΅Ρ€Π΅Π΄Π°ΡŽΡ‚ ΡΠ½Π΅Ρ€Π³ΠΈΡŽ ΠΈΠ· Π² схСму, ΠΈ ΠΊΠΎΠΌΠΏΠΎΠ½Π΅Π½Ρ‚Ρ‹, ΠΊΠΎΡ‚ΠΎΡ€Ρ‹Π΅ ΠΎΡ‚Π±ΠΈΡ€Π°ΡŽΡ‚ ΡΠ½Π΅Ρ€Π³ΠΈΡŽ ΠΈΠ· .Π‘ этого ΠΌΠΎΠΌΠ΅Π½Ρ‚Π° ΠΌΡ‹ Π±ΡƒΠ΄Π΅ΠΌ Π³ΠΎΠ²ΠΎΡ€ΠΈΡ‚ΡŒ, Ρ‡Ρ‚ΠΎ любоС устройство, ΠΏΠ΅Ρ€Π΅Π΄Π°ΡŽΡ‰Π΅Π΅ ΡΠ½Π΅Ρ€Π³ΠΈΡŽ Π² Ρ†Π΅ΠΏΡŒ, обСспСчиваСт ΡΠ»Π΅ΠΊΡ‚Ρ€ΠΎΠ΄Π²ΠΈΠΆΡƒΡ‰ΡƒΡŽ силу (Π­Π”Π‘) , ΠΈ любоС устройство, выводящСС Π΅Π΅, ΠΈΠΌΠ΅Π΅Ρ‚ Ρ€Π°Π·Π½ΠΎΡΡ‚ΡŒ ΠΏΠΎΡ‚Π΅Π½Ρ†ΠΈΠ°Π»ΠΎΠ² (pd) Π½Π° Π½Π΅ΠΌ.

И Π­Π”Π‘, ΠΈ pd ΠΈΠ·ΠΌΠ΅Ρ€ΡΡŽΡ‚ΡΡ Π² Π²ΠΎΠ»ΡŒΡ‚Π°Ρ…, Π’, , ΠΏΠΎΡΠΊΠΎΠ»ΡŒΠΊΡƒ ΠΎΠ½ΠΈ ΠΎΠΏΠΈΡΡ‹Π²Π°ΡŽΡ‚, сколько энСргии вкладываСтся ΠΈΠ»ΠΈ снимаСтся Π½Π° ΠΎΠ΄ΠΈΠ½ ΠΊΡƒΠ»ΠΎΠ½ заряда, проходящСго Ρ‡Π΅Ρ€Π΅Π· этот участок Ρ†Π΅ΠΏΠΈ.

Π›ΡƒΡ‡ΡˆΠ΅ всСго Π΄ΡƒΠΌΠ°Ρ‚ΡŒ ΠΎ Π½ΠΈΡ…:

Π­Π”Π‘ — это количСство энСргии любой Ρ„ΠΎΡ€ΠΌΡ‹, ΠΊΠΎΡ‚ΠΎΡ€ΠΎΠ΅ прСвращаСтся Π² ΡΠ»Π΅ΠΊΡ‚Ρ€ΠΈΡ‡Π΅ΡΠΊΡƒΡŽ ΡΠ½Π΅Ρ€Π³ΠΈΡŽ Π½Π° ΠΊΡƒΠ»ΠΎΠ½ заряда.

pd — количСство элСктричСской энСргии, которая прСобразуСтся Π² Π΄Ρ€ΡƒΠ³ΠΈΠ΅ Ρ„ΠΎΡ€ΠΌΡ‹ энСргии Π½Π° ΠΎΠ΄ΠΈΠ½ ΠΊΡƒΠ»ΠΎΠ½ заряда.

Π˜ΡΡ‚ΠΎΡ‡Π½ΠΈΠΊΠΈ Π­Π”Π‘:

Π­Π»Π΅ΠΌΠ΅Π½Ρ‚, аккумулятор (комбинация элСмСнтов), солнСчный элСмСнт, Π³Π΅Π½Π΅Ρ€Π°Ρ‚ΠΎΡ€, Π΄ΠΈΠ½Π°ΠΌΠΎ-машина, Ρ‚Π΅Ρ€ΠΌΠΎΠΏΠ°Ρ€Π°.

Π­Π»Π΅ΠΌΠ΅Π½Ρ‚Ρ‹ ΠΈ Π±Π°Ρ‚Π°Ρ€Π΅ΠΈ Π½Π΅ ΠΈΠ΄Π΅Π°Π»ΡŒΠ½Ρ‹ (Ρ‡Ρ‚ΠΎ, ΠΊΠΎΠ½Π΅Ρ‡Π½ΠΎ, Π½Π΅ считая ΠΌΠΎΠΌΠ΅Π½Ρ‚Π° окончания вашСго послСднСго экзамСна?). Π˜ΡΠΏΠΎΠ»ΡŒΠ·ΡƒΠΉΡ‚Π΅ ΠΈΡ… Π½Π΅ΠΊΠΎΡ‚ΠΎΡ€ΠΎΠ΅ врСмя, ΠΈ Π²Ρ‹ Π·Π°ΠΌΠ΅Ρ‚ΠΈΡ‚Π΅, Ρ‡Ρ‚ΠΎ ΠΎΠ½ΠΈ Π½Π°Π³Ρ€Π΅Π²Π°ΡŽΡ‚ΡΡ.

ΠžΡ‚ΠΊΡƒΠ΄Π° поступаСт тСпловая энСргия?

Π­Ρ‚ΠΎ ΠΎΡ‚ Ρ‚ΠΎΠΊΠ°, проходящСго Ρ‡Π΅Ρ€Π΅Π· Π²Π½ΡƒΡ‚Ρ€Π΅Π½Π½ΡŽΡŽ Ρ‡Π°ΡΡ‚ΡŒ ячСйки.Π‘ΠΎΠΏΡ€ΠΎΡ‚ΠΈΠ²Π»Π΅Π½ΠΈΠ΅ Π²Π½ΡƒΡ‚Ρ€ΠΈ ячСйки ΠΏΡ€Π΅Π²Ρ€Π°Ρ‰Π°Π΅Ρ‚ Ρ‡Π°ΡΡ‚ΡŒ ΠΏΡ€ΠΎΠΈΠ·Π²ΠΎΠ΄ΠΈΠΌΠΎΠΉ Сю элСктроэнСргии Π² Ρ‚Π΅ΠΏΠ»ΠΎΠ²ΡƒΡŽ ΠΏΡ€ΠΈ Π΄Π²ΠΈΠΆΠ΅Π½ΠΈΠΈ элСктронов Ρ‡Π΅Ρ€Π΅Π· Π½Π΅Π΅.

Π­Ρ‚ΠΎ Π»Π΅Π³ΠΊΠΎ ΠΎΠ±ΡŠΡΡΠ½ΠΈΡ‚ΡŒ, Ссли ΠΏΡ€Π΅Π΄ΡΡ‚Π°Π²ΠΈΡ‚ΡŒ, Ρ‡Ρ‚ΠΎ каТдая ячСйка идСальна, Π·Π° ΠΈΡΠΊΠ»ΡŽΡ‡Π΅Π½ΠΈΠ΅ΠΌ Ρ‚ΠΎΠ³ΠΎ, Ρ‡Ρ‚ΠΎ ΠΏΠΎ ΠΊΠ°ΠΊΠΎΠΉ-Ρ‚ΠΎ странной ΠΏΡ€ΠΈΡ‡ΠΈΠ½Π΅ (вСроятно, Ρ‡Π°ΡΡ‚ΡŒ Π·Π°Π³ΠΎΠ²ΠΎΡ€Π° ΠΏΠΎ Π·Π°Ρ…Π²Π°Ρ‚Ρƒ ΠΌΠΈΡ€Π°, Π·Π°Π΄ΡƒΠΌΠ°Π½Π½ΠΎΠ³ΠΎ Π΄ΠΎΠΊΡ‚ΠΎΡ€ΠΎΠΌ Π—Π»ΠΎΠΌ) ΠΏΡ€ΠΎΠΈΠ·Π²ΠΎΠ΄ΠΈΡ‚Π΅Π»ΠΈ Π²ΠΊΠ»ΡŽΡ‡ΠΈΠ»ΠΈ рСзистор ΠΏΠΎΡΠ»Π΅Π΄ΠΎΠ²Π°Ρ‚Π΅Π»ΡŒΠ½ΠΎ с ячСйкой Π²Π½ΡƒΡ‚Ρ€ΠΈ ячСйки. ΠΊΠΎΠΆΡƒΡ….

Π’Π°ΠΊΠΈΠΌ ΠΎΠ±Ρ€Π°Π·ΠΎΠΌ, Π²Π½ΡƒΡ‚Ρ€ΠΈ ячСйки энСргия пСрСдаСтся Π² Ρ†Π΅ΠΏΡŒ ячСйкой (Π­Π”Π‘), Π½ΠΎ нСкоторая Ρ‡Π°ΡΡ‚ΡŒ этой энСргии отбираСтся ΠΈΠ· Ρ†Π΅ΠΏΠΈ Π²Π½ΡƒΡ‚Ρ€Π΅Π½Π½ΠΈΠΌ рСзистором (pd).

Π’Π°ΠΊΠΈΠΌ ΠΎΠ±Ρ€Π°Π·ΠΎΠΌ, pd, доступный для ΠΎΡΡ‚Π°Π»ΡŒΠ½ΠΎΠΉ части схСмы (внСшнСй Ρ†Π΅ΠΏΠΈ, ΠΏΠΎΡΠΊΠΎΠ»ΡŒΠΊΡƒ Π½Π΅ΠΊΠΎΡ‚ΠΎΡ€Ρ‹Π΅ вопросы ΠΌΠΎΠ³ΡƒΡ‚ ΠΎΡ‚Π½ΠΎΡΠΈΡ‚ΡŒΡΡ ΠΊ Π½Π΅ΠΉ), это Π­Π”Π‘ минус pd, потСрянный Π²Π½ΡƒΡ‚Ρ€ΠΈ ячСйки:

V = E — Ir

Π“Π΄Π΅ :

Π’ = pd ΠΏΠΎ внСшнСй Ρ†Π΅ΠΏΠΈ (Π’)

E = Π­Π”Π‘ ячСйки (Π’)

I = Ρ‚ΠΎΠΊ Ρ‡Π΅Ρ€Π΅Π· элСмСнт (А)

r = Π·Π½Π°Ρ‡Π΅Π½ΠΈΠ΅ Π²Π½ΡƒΡ‚Ρ€Π΅Π½Π½Π΅Π³ΠΎ сопротивлСния (& Omega;)

( Ir = p.d. Π½Π° Π²Π½ΡƒΡ‚Ρ€Π΅Π½Π½Π΅ΠΌ рСзисторС)

ΠŸΡ€ΠΈΠΌΠ΅Ρ‡Π°Π½ΠΈΠ΅: Π’ ΠΈΠ½ΠΎΠ³Π΄Π° Π½Π°Π·Ρ‹Π²Π°ΡŽΡ‚ ΠΊΠ»Π΅ΠΌΠΌΠΎΠΉ pd , ΠΏΠΎΡΠΊΠΎΠ»ΡŒΠΊΡƒ это pd Π½Π° ΠΊΠ»Π΅ΠΌΠΌΠ°Ρ… ячСйки

ΠŸΡ€ΠΈΠΌΠ΅Ρ€ 1:

Π§Ρ‚ΠΎ Ρ‚Π°ΠΊΠΎΠ΅ ΠΊΠ»Π΅ΠΌΠΌΠ° p.d. для ячСйки Π­Π”Π‘ 2Π’ ΠΈ Π²Π½ΡƒΡ‚Ρ€Π΅Π½Π½Π΅Π³ΠΎ сопротивлСния 1 Ом ΠΏΡ€ΠΈ ΠΏΠΎΠ΄ΠΊΠ»ΡŽΡ‡Π΅Π½ΠΈΠΈ ΠΊ рСзистору 9 Ом?

ΠžΡ‚Π²Π΅Ρ‚:

ΠŸΡ€ΠΎΡΡ‚ΠΎ ΠΏΡ€Π΅Π΄ΡΡ‚Π°Π²ΡŒΡ‚Π΅, Ρ‡Ρ‚ΠΎ Π²Π½ΡƒΡ‚Ρ€Π΅Π½Π½Π΅Π΅ сопротивлСниС являСтся ΠΎΠ΄Π½ΠΈΠΌ ΠΈΠ· Π½ΠΎΡ€ΠΌΠ°Π»ΡŒΠ½Ρ‹Ρ… рСзисторов Π² Ρ†Π΅ΠΏΠΈ.НарисуйтС Π΅Π³ΠΎ Π½Π° ΠΏΡ€ΠΈΠ½Ρ†ΠΈΠΏΠΈΠ°Π»ΡŒΠ½ΠΎΠΉ схСмС рядом с ячСйкой, Ρ‡Ρ‚ΠΎΠ±Ρ‹ вСсь Ρ‚ΠΎΠΊ, проходящий Ρ‡Π΅Ρ€Π΅Π· ячСйку, Ρ‚Π°ΠΊΠΆΠ΅ ΠΏΡ€ΠΎΡ…ΠΎΠ΄ΠΈΠ» Ρ‡Π΅Ρ€Π΅Π· рСзистор.

Π§Ρ‚ΠΎΠ±Ρ‹ Π½Π°ΠΉΡ‚ΠΈ V, ΠΊΠ»Π΅ΠΌΠΌΡƒ pd (ΠΈΠ»ΠΈ напряТСниС, доступноС для внСшнСй Ρ†Π΅ΠΏΠΈ), вычислитС Ρ‚ΠΎΠΊ I для всСй Ρ†Π΅ΠΏΠΈ:

ΠŸΡ€ΠΈΠΌΠ΅Ρ‡Π°Π½ΠΈΠ΅: V T ΠΈ R T — это напряТСниС ΠΈ сопротивлСниС для всСй Ρ†Π΅ΠΏΠΈ, Π²ΠΊΠ»ΡŽΡ‡Π°Ρ внСшнСС ΠΈ Π²Π½ΡƒΡ‚Ρ€Π΅Π½Π½Π΅Π΅ сопротивлСниС.

Π’Π°ΠΊΠΈΠΌ ΠΎΠ±Ρ€Π°Π·ΠΎΠΌ, 9 & Omega; рСзистор ΠΏΠΎΠ»ΡƒΡ‡Π°Π΅Ρ‚ V = IR = 0.2 x 9 = 1,8 Π’

Π’Π°ΠΊΠΈΠΌ ΠΎΠ±Ρ€Π°Π·ΠΎΠΌ, эта ячСйка Π­Π”Π‘ Π½Π° 2 Π’ фактичСски ΠΏΠΎΠ΄Π°Π΅Ρ‚ 1,8 Π’ Π²ΠΎ внСшнюю Ρ†Π΅ΠΏΡŒ.

ΠŸΡ€ΠΈΠΌΠ΅Ρ€ 2:

Π’Π΅ΠΏΠ΅Ρ€ΡŒ помСняйтС мСстами 9 & Omega; рСзистор Π² послСднСм ΠΏΡ€ΠΈΠΌΠ΅Ρ€Π΅ для 1 & Omega; рСзистор.

ΠžΡ‚Π²Π΅Ρ‚:

НайдитС V, Ρ‚Π΅Ρ€ΠΌΠΈΠ½Π°Π» pd, ΠΈΡΠΏΠΎΠ»ΡŒΠ·ΡƒΡ Ρ‚ΠΎΡ‚ ΠΆΠ΅ ΠΌΠ΅Ρ‚ΠΎΠ΄ снова:

Π’Π΅ΠΏΠ΅Ρ€ΡŒ ячСйка Π­Π”Π‘ 2V ΠΏΠΎΠ΄Π°Π΅Ρ‚ Ρ‚ΠΎΠ»ΡŒΠΊΠΎ 1V Π²ΠΎ внСшнюю Ρ†Π΅ΠΏΡŒ !!! Π”Ρ€ΡƒΠ³ΠΎΠΉ 1 Π’ тСряСтся, ΠΈΠ·-Π·Π° Ρ‡Π΅Π³ΠΎ ячСйка нагрСваСтся. НС ΠΎΡ‡Π΅Π½ΡŒ работоспособСн.

ΠŸΡ€ΠΈΠΌΠ΅Ρ‡Π°Π½ΠΈΠ΅: Π’Π°ΠΌ Π½Π΅ΠΎΠ±Ρ…ΠΎΠ΄ΠΈΠΌΠΎ ΡƒΡ‡ΠΈΡ‚Ρ‹Π²Π°Ρ‚ΡŒ Π²Π½ΡƒΡ‚Ρ€Π΅Π½Π½Π΅Π΅ сопротивлСниС ΠΏΡ€ΠΈ принятии Ρ€Π΅ΡˆΠ΅Π½ΠΈΡ ΠΎ Ρ‚ΠΎΠΌ, ΠΏΠΎΠ΄Ρ…ΠΎΠ΄ΠΈΡ‚ Π»ΠΈ ячСйка для использования Π² ΠΊΠΎΠ½ΠΊΡ€Π΅Ρ‚Π½ΠΎΠΉ Ρ†Π΅ΠΏΠΈ. Для максимальной эффСктивности внСшнСС сопротивлСниС Π΄ΠΎΠ»ΠΆΠ½ΠΎ Π±Ρ‹Ρ‚ΡŒ Π½Π°ΠΌΠ½ΠΎΠ³ΠΎ большС Π²Π½ΡƒΡ‚Ρ€Π΅Π½Π½Π΅Π³ΠΎ сопротивлСния ячСйки. Однако для максимальной мощности, ΠΏΠ΅Ρ€Π΅Π΄Π°Π²Π°Π΅ΠΌΠΎΠΉ Π²ΠΎ внСшнюю Ρ†Π΅ΠΏΡŒ, Π²Π½ΡƒΡ‚Ρ€Π΅Π½Π½Π΅Π΅ сопротивлСниС Π΄ΠΎΠ»ΠΆΠ½ΠΎ Π±Ρ‹Ρ‚ΡŒ Ρ€Π°Π²Π½ΠΎ ΡΠΎΠΏΡ€ΠΎΡ‚ΠΈΠ²Π»Π΅Π½ΠΈΡŽ внСшнСй Ρ†Π΅ΠΏΠΈ, хотя ΡΡ„Ρ„Π΅ΠΊΡ‚ΠΈΠ²Π½ΠΎΡΡ‚ΡŒ ячСйки Π±ΡƒΠ΄Π΅Ρ‚ Ρ‚ΠΎΠ»ΡŒΠΊΠΎ 50%.

Π˜ΡΡ‚ΠΎΡ‡Π½ΠΈΠΊΠΈ питания с Π½ΠΈΠ·ΠΊΠΈΠΌ напряТСниСм ΠΈ Π±ΠΎΠ»Π΅Π΅ высокими Ρ‚ΠΎΠΊΠ°ΠΌΠΈ, Π½Π°ΠΏΡ€ΠΈΠΌΠ΅Ρ€ Π°Π²Ρ‚ΠΎΠΌΠΎΠ±ΠΈΠ»ΡŒΠ½Ρ‹ΠΉ аккумулятор, Π΄ΠΎΠ»ΠΆΠ½Ρ‹ ΠΈΠΌΠ΅Ρ‚ΡŒ Π½ΠΈΠ·ΠΊΠΎΠ΅ Π²Π½ΡƒΡ‚Ρ€Π΅Π½Π½Π΅Π΅ сопротивлСниС, ΠΊΠ°ΠΊ ΠΏΠΎΠΊΠ°Π·Π°Π½ΠΎ Π²Ρ‹ΡˆΠ΅.Π˜ΡΡ‚ΠΎΡ‡Π½ΠΈΠΊΠΈ питания высокого напряТСния, Π²Ρ‹Ρ€Π°Π±Π°Ρ‚Ρ‹Π²Π°ΡŽΡ‰ΠΈΠ΅ тысячи Π²ΠΎΠ»ΡŒΡ‚, Π΄ΠΎΠ»ΠΆΠ½Ρ‹ ΠΈΠΌΠ΅Ρ‚ΡŒ Ρ‡Ρ€Π΅Π·Π²Ρ‹Ρ‡Π°ΠΉΠ½ΠΎ высокоС Π²Π½ΡƒΡ‚Ρ€Π΅Π½Π½Π΅Π΅ сопротивлСниС, Ρ‡Ρ‚ΠΎΠ±Ρ‹ ΠΎΠ³Ρ€Π°Π½ΠΈΡ‡ΠΈΡ‚ΡŒ Ρ‚ΠΎΠΊ, ΠΊΠΎΡ‚ΠΎΡ€Ρ‹ΠΉ ΠΌΠΎΠΆΠ΅Ρ‚ ΠΏΡ€ΠΎΡ‚Π΅ΠΊΠ°Ρ‚ΡŒ Π² случаС случайного ΠΊΠΎΡ€ΠΎΡ‚ΠΊΠΎΠ³ΠΎ замыкания.

ΠŸΠΎΡΠΊΠΎΠ»ΡŒΠΊΡƒ V = E — Ir, Ссли Π²Ρ‹ построитС Π³Ρ€Π°Ρ„ΠΈΠΊ зависимости pd, V ΠΎΡ‚ Ρ‚ΠΎΠΊΠ°, I, Π³Ρ€Π°Π΄ΠΈΠ΅Π½Ρ‚ Π³Ρ€Π°Ρ„ΠΈΠΊΠ° Π±ΡƒΠ΄Π΅Ρ‚ Ρ€Π°Π²Π΅Π½ Π²Π½ΡƒΡ‚Ρ€Π΅Π½Π½Π΅ΠΌΡƒ ΡΠΎΠΏΡ€ΠΎΡ‚ΠΈΠ²Π»Π΅Π½ΠΈΡŽ ячСйки. (ΠΎΡ‚Ρ€ΠΈΡ†Π°Ρ‚Π΅Π»ΡŒΠ½Ρ‹ΠΉ, ΠΏΠΎΡ‚ΠΎΠΌΡƒ Ρ‡Ρ‚ΠΎ Π³Ρ€Π°Ρ„ΠΈΠΊ наклоняСтся Π²Π½ΠΈΠ·)

Записывая значСния Ρ‚ΠΎΠΊΠ° ΠΈ ΠΊΠ»Π΅ΠΌΠΌΡ‹ pd ΠΏΠΎ ΠΌΠ΅Ρ€Π΅ измСнСния внСшнСго сопротивлСния, Π²Ρ‹ ΠΌΠΎΠΆΠ΅Ρ‚Π΅ ΠΏΠΎΡΡ‚Ρ€ΠΎΠΈΡ‚ΡŒ Π³Ρ€Π°Ρ„ΠΈΠΊ ΠΈ Π½Π°ΠΉΡ‚ΠΈ Π²Π½ΡƒΡ‚Ρ€Π΅Π½Π½Π΅Π΅ сопротивлСниС ΠΈ Π­Π”Π‘ ячСйки.

Если ΠΏΠΎΡΠ»Π΅Π΄ΠΎΠ²Π°Ρ‚Π΅Π»ΡŒΠ½ΠΎ ΠΏΠΎΠ΄ΠΊΠ»ΡŽΡ‡Π΅Π½ΠΎ Π±ΠΎΠ»Π΅Π΅ ΠΎΠ΄Π½ΠΎΠΉ ячСйки, Π½Π΅ΠΎΠ±Ρ…ΠΎΠ΄ΠΈΠΌΠΎ ΡΠ»ΠΎΠΆΠΈΡ‚ΡŒ Π²Π½ΡƒΡ‚Ρ€Π΅Π½Π½ΠΈΠ΅ сопротивлСния ячССк.

Π’Π½ΡƒΡ‚Ρ€Π΅Π½Π½Π΅Π΅ сопротивлСниС Π±Π°Ρ‚Π°Ρ€Π΅ΠΉΠ½ΠΎΠ³ΠΎ ΠΊΠ°Π»ΡŒΠΊΡƒΠ»ΡΡ‚ΠΎΡ€Π° β€’ ЭлСктричСскиС, радиочастотныС ΠΈ элСктронныС ΠΊΠ°Π»ΡŒΠΊΡƒΠ»ΡΡ‚ΠΎΡ€Ρ‹ β€’ Онлайн-ΠΏΡ€Π΅ΠΎΠ±Ρ€Π°Π·ΠΎΠ²Π°Ρ‚Π΅Π»ΠΈ Π΅Π΄ΠΈΠ½ΠΈΡ†

Π­Ρ‚ΠΎΡ‚ ΠΊΠ°Π»ΡŒΠΊΡƒΠ»ΡΡ‚ΠΎΡ€ опрСдСляСт Π²Π½ΡƒΡ‚Ρ€Π΅Π½Π½Π΅Π΅ сопротивлСниС элСктричСской Π±Π°Ρ‚Π°Ρ€Π΅ΠΈ ΠΏΠΎ падСнию напряТСния Π½Π° Π½Π°Π³Ρ€ΡƒΠ·ΠΎΡ‡Π½ΠΎΠΌ рСзисторС с извСстным сопротивлСниСм, Π° Ρ‚Π°ΠΊΠΆΠ΅ ΠΏΠΎ Π½Π°ΠΏΡ€ΡΠΆΠ΅Π½ΠΈΡŽ ΠΈΠ»ΠΈ Ρ‚ΠΎΠΊΡƒ холостого Ρ…ΠΎΠ΄Π°. Π² Π½Π°Π³Ρ€ΡƒΠ·ΠΎΡ‡Π½ΠΎΠΌ рСзисторС.

ΠŸΡ€ΠΈΠΌΠ΅Ρ€ 1: РассчитайтС Π²Π½ΡƒΡ‚Ρ€Π΅Π½Π½Π΅Π΅ сопротивлСниС Li-PO Π±Π°Ρ‚Π°Ρ€Π΅ΠΈ, Ссли Π΅Π΅ напряТСниС Π±Π΅Π· Π½Π°Π³Ρ€ΡƒΠ·ΠΊΠΈ Ρ€Π°Π²Π½ΠΎ 3.90 Π’, Π° ΠΏΡ€ΠΈ Π½Π°Π³Ρ€ΡƒΠ·ΠΊΠ΅ 10 Ом — 3,89 Π’. НиТС Π²Ρ‹ Π½Π°ΠΉΠ΄Π΅Ρ‚Π΅ Π΅Ρ‰Π΅ ΠΏΡΡ‚ΡŒ ΠΏΡ€ΠΈΠΌΠ΅Ρ€ΠΎΠ².

Π Π°ΡΡΡ‡ΠΈΡ‚Π°Ρ‚ΡŒ

R I ΠΈ I ΠΎΡ‚ U NL , R L ΠΈ U L R I U NL , R L ΠΈ I R I ΠΈ R L ΠΎΡ‚ U NL , U 9 ΠΈ 9033 L L ΠΈ I ΠΎΡ‚ U NL , R I ΠΈ R L R L ΠΈ I ΠΎΡ‚ UL 903 934 9000 R UL 903 934 9000 R I ΠΈ U L R L ΠΈ U L ΠΎΡ‚ U NL , R I ΠΈ I 9333 U 903 33 I ΠΈΠ· R I , R L ΠΈ U L U NL ΠΈ U L ΠΈΠ· R I , ΠΈ I U NL ΠΈ R L ΠΎΡ‚ R I , U L ΠΈ I

НапряТСниС Π½Π° Π±Π°Ρ‚Π°Ρ€Π΅Π΅, Π±Π΅Π· Π½Π°Π³Ρ€ΡƒΠ·ΠΊΠΈ

NL ΠΌΠΈΠΊΡ€ΠΎΠ²ΠΎΠ»ΡŒΡ‚ (ΠΌΠΊΠ’) ΠΌΠΈΠ»Π»ΠΈΠ²ΠΎΠ»ΡŒΡ‚ (ΠΌΠ’) Π²ΠΎΠ»ΡŒΡ‚ (Π’) ΠΊΠΈΠ»ΠΎΠ²ΠΎΠ»ΡŒΡ‚ (ΠΊΠ’) ΠΌΠ΅Π³Π°Π²ΠΎΠ»ΡŒΡ‚ (ΠœΠ’)

Π’Π½ΡƒΡ‚Ρ€Π΅Π½Π½Π΅Π΅ сопротивлСниС Π±Π°Ρ‚Π°Ρ€Π΅ΠΈ

R I миллиом (мОм) (Ом) кОм) мСгаом (МОм)

Для расчСта Π²Π²Π΅Π΄ΠΈΡ‚Π΅ Π»ΡŽΠ±Ρ‹Π΅ Ρ‚Ρ€ΠΈ ΠΈΠ· пяти Π·Π½Π°Ρ‡Π΅Π½ΠΈΠΉ ΠΈ Π½Π°ΠΆΠΌΠΈΡ‚Π΅ ΠΈΠ»ΠΈ ΠΊΠΎΡΠ½ΠΈΡ‚Π΅ΡΡŒ ΠΊΠ½ΠΎΠΏΠΊΠΈ Π’Ρ‹Ρ‡ΠΈΡΠ»ΠΈΡ‚ΡŒ .Π˜ΡΠΊΠ»ΡŽΡ‡Π΅Π½ΠΈΠ΅: ΠΏΡ€ΠΈ Π²Π²ΠΎΠ΄Π΅ Ρ‚ΠΎΠ»ΡŒΠΊΠΎ Ρ‚Ρ€Π΅Ρ… ΠΏΠ°Ρ€Π°ΠΌΠ΅Ρ‚Ρ€ΠΎΠ² Π½Π°Π³Ρ€ΡƒΠ·ΠΊΠΈ R L , U L ΠΈ I Π½Π΅Π²ΠΎΠ·ΠΌΠΎΠΆΠ½ΠΎ Ρ€Π°ΡΡΡ‡ΠΈΡ‚Π°Ρ‚ΡŒ ΠΏΠ°Ρ€Π°ΠΌΠ΅Ρ‚Ρ€Ρ‹ Π±Π°Ρ‚Π°Ρ€Π΅ΠΈ U NL ΠΈ R I ΠΈ Π½ΠΈΠΊΠ°ΠΊΠΈΡ… расчСтов Π²Ρ‹ΠΏΠΎΠ»Π½ΡΡŽΡ‚ΡΡ.

ΠžΠΏΡ€Π΅Π΄Π΅Π»Π΅Π½ΠΈΡ ΠΈ Ρ„ΠΎΡ€ΠΌΡƒΠ»Ρ‹

Богласно Ρ‚Π΅ΠΎΡ€Π΅ΠΌΠ΅ Π“Π΅Π»ΡŒΠΌΠ³ΠΎΠ»ΡŒΡ†Π° – Π’Π΅Π²Π΅Π½ΠΈΠ½Π° Π»ΡŽΠ±ΡƒΡŽ Π»ΠΈΠ½Π΅ΠΉΠ½ΡƒΡŽ ΡΠ΅Ρ‚ΡŒ с Π»ΡŽΠ±Ρ‹ΠΌ количСством источников напряТСния (Π½Π°ΠΏΡ€ΠΈΠΌΠ΅Ρ€, ΡˆΠ΅ΡΡ‚ΡŒ Π³Π°Π»ΡŒΠ²Π°Π½ΠΈΡ‡Π΅ΡΠΊΠΈΡ… элСмСнтов, соСдинСнных ΠΏΠΎΡΠ»Π΅Π΄ΠΎΠ²Π°Ρ‚Π΅Π»ΡŒΠ½ΠΎ Π² Π°Π²Ρ‚ΠΎΠΌΠΎΠ±ΠΈΠ»ΡŒΠ½ΠΎΠΌ аккумуляторС), ΠΌΠΎΠΆΠ½ΠΎ Π·Π°ΠΌΠ΅Π½ΠΈΡ‚ΡŒ элСктродвиТущСй силой (Π­Π”Π‘ ). ) ΠΈΠ»ΠΈ эквивалСнтноС напряТСниС холостого Ρ…ΠΎΠ΄Π° U NL источник ΠΏΠΎΡΠ»Π΅Π΄ΠΎΠ²Π°Ρ‚Π΅Π»ΡŒΠ½ΠΎ с Π²Π½ΡƒΡ‚Ρ€Π΅Π½Π½ΠΈΠΌ сопротивлСниСм R I ΠΈΠ»ΠΈ импСдансом Z I .НапряТСниС U NL ΠΏΠΈΡ‚Π°Π΅Ρ‚ внСшнюю Π½Π°Π³Ρ€ΡƒΠ·ΠΊΡƒ R L Ρ‚ΠΎΠΊΠΎΠΌ I .

Π’ΠΎΠΊ, ΠΏΠΎΠ΄Π°Π²Π°Π΅ΠΌΡ‹ΠΉ Π±Π°Ρ‚Π°Ρ€Π΅Π΅ΠΉ Π½Π° Π½Π°Π³Ρ€ΡƒΠ·ΠΊΡƒ, Π±ΡƒΠ΄Π΅Ρ‚ ΠΎΠΏΡ€Π΅Π΄Π΅Π»ΡΡ‚ΡŒΡΡ сопротивлСниСм внСшнСй Π½Π°Π³Ρ€ΡƒΠ·ΠΊΠΈ, ΠΈ Π² Ρ‚ΠΎ ΠΆΠ΅ врСмя этот Ρ‚ΠΎΠΊ Π±ΡƒΠ΄Π΅Ρ‚ ΠΎΠ³Ρ€Π°Π½ΠΈΡ‡Π΅Π½ Π²Π½ΡƒΡ‚Ρ€Π΅Π½Π½ΠΈΠΌ сопротивлСниСм Π±Π°Ρ‚Π°Ρ€Π΅ΠΈ. Π’Π½ΡƒΡ‚Ρ€Π΅Π½Π½Π΅Π΅ сопротивлСниС складываСтся ΠΈΠ· сопротивлСния пластин Π±Π°Ρ‚Π°Ρ€Π΅ΠΈ, Π΅Π΅ Π°ΠΊΡ‚ΠΈΠ²Π½ΠΎΠ³ΠΎ ΠΌΠ°Ρ‚Π΅Ρ€ΠΈΠ°Π»Π° ΠΈ элСктролита.

Π‘Π²ΠΈΠ½Ρ†ΠΎΠ²ΠΎ-кислотныС Π±Π°Ρ‚Π°Ρ€Π΅ΠΈ ΠΈΠΌΠ΅ΡŽΡ‚ ΠΎΡ‡Π΅Π½ΡŒ малСнькоС Π²Π½ΡƒΡ‚Ρ€Π΅Π½Π½Π΅Π΅ сопротивлСниС (ΠΎΠ±Ρ‹Ρ‡Π½ΠΎ 0.01 Ом) — ΠΈΠΌΠ΅Π½Π½ΠΎ поэтому ΠΎΠ½ΠΈ способны ΠΎΠ±Π΅ΡΠΏΠ΅Ρ‡ΠΈΡ‚ΡŒ большой Ρ‚ΠΎΠΊ, Π½Π΅ΠΎΠ±Ρ…ΠΎΠ΄ΠΈΠΌΡ‹ΠΉ для запуска двигатСля. Π’Π½ΡƒΡ‚Ρ€Π΅Π½Π½Π΅Π΅ сопротивлСниС свинцово-кислотных ячССк Π½Π°ΡΡ‚ΠΎΠ»ΡŒΠΊΠΎ ΠΌΠ°Π»ΠΎ, ΠΏΠΎΡ‚ΠΎΠΌΡƒ Ρ‡Ρ‚ΠΎ Π² ΠΊΠ°ΠΆΠ΄ΠΎΠΉ ячСйкС Π΅ΡΡ‚ΡŒ нСсколько ΠΎΡ‚Ρ€ΠΈΡ†Π°Ρ‚Π΅Π»ΡŒΠ½Ρ‹Ρ… ΠΈ ΠΏΠΎΠ»ΠΎΠΆΠΈΡ‚Π΅Π»ΡŒΠ½Ρ‹Ρ… пластин, соСдинСнных ΠΏΠ°Ρ€Π°Π»Π»Π΅Π»ΡŒΠ½ΠΎ. ΠšΡ€ΠΎΠΌΠ΅ Ρ‚ΠΎΠ³ΠΎ, расстояниС ΠΌΠ΅ΠΆΠ΄Ρƒ ΠΎΡ‚Ρ€ΠΈΡ†Π°Ρ‚Π΅Π»ΡŒΠ½ΠΎΠΉ ΠΈ ΠΏΠΎΠ»ΠΎΠΆΠΈΡ‚Π΅Π»ΡŒΠ½ΠΎΠΉ пластинами ΠΎΡ‡Π΅Π½ΡŒ ΠΌΠ°Π»ΠΎ, ΠΈ, ΡΠ»Π΅Π΄ΠΎΠ²Π°Ρ‚Π΅Π»ΡŒΠ½ΠΎ, Ρ‚ΠΎΠ»Ρ‰ΠΈΠ½Π° слоя элСктролита ΠΌΠ΅ΠΆΠ΄Ρƒ Π½ΠΈΠΌΠΈ ΡƒΠΌΠ΅Π½ΡŒΡˆΠ°Π΅Ρ‚ΡΡ, Ρ‡Ρ‚ΠΎ, Π² свою ΠΎΡ‡Π΅Ρ€Π΅Π΄ΡŒ, Π΄Π΅Π»Π°Π΅Ρ‚ ΠΈΡ… Π²Π½ΡƒΡ‚Ρ€Π΅Π½Π½Π΅Π΅ сопротивлСниС Π΅Ρ‰Π΅ мСньшС. Когда батарСя Π²Ρ‹Π΄Π°Π΅Ρ‚ большой Ρ‚ΠΎΠΊ, это Π²Π½ΡƒΡ‚Ρ€Π΅Π½Π½Π΅Π΅ сопротивлСниС рассСиваСт Ρ‚Π΅ΠΏΠ»ΠΎ, ΠΈ батарСя нагрСваСтся.

Π’Π½ΡƒΡ‚Ρ€Π΅Π½Π½Π΅Π΅ сопротивлСниС Π±Π°Ρ‚Π°Ρ€Π΅ΠΈ ΠΌΠΎΠΆΠ½ΠΎ Ρ€Π°ΡΡΡ‡ΠΈΡ‚Π°Ρ‚ΡŒ ΠΏΠΎ Π΅Π΅ Π½Π°ΠΏΡ€ΡΠΆΠ΅Π½ΠΈΡŽ холостого Ρ…ΠΎΠ΄Π° U NL , Π½Π°ΠΏΡ€ΡΠΆΠ΅Π½ΠΈΡŽ, ΠΈΠ·ΠΌΠ΅Ρ€Π΅Π½Π½ΠΎΠΌΡƒ Π½Π° Π½Π°Π³Ρ€ΡƒΠ·ΠΊΠ΅ U L , ΠΈ ΡΠΎΠΏΡ€ΠΎΡ‚ΠΈΠ²Π»Π΅Π½ΠΈΡŽ Π½Π°Π³Ρ€ΡƒΠ·ΠΊΠΈ R L . Π­Ρ‚ΠΎ напряТСниС холостого Ρ…ΠΎΠ΄Π° эквивалСнтно элСктродвиТущСй силС Π±Π°Ρ‚Π°Ρ€Π΅ΠΈ.

Π’ΠΎΠΊ, ΠΏΡ€ΠΎΡ‚Π΅ΠΊΠ°ΡŽΡ‰ΠΈΠΉ Ρ‡Π΅Ρ€Π΅Π· Π½Π°Π³Ρ€ΡƒΠ·ΠΎΡ‡Π½Ρ‹ΠΉ рСзистор:

ПадСниС напряТСния Π½Π° Π²Π½ΡƒΡ‚Ρ€Π΅Π½Π½Π΅ΠΌ сопротивлСнии:

Π’Π½ΡƒΡ‚Ρ€Π΅Π½Π½Π΅Π΅ сопротивлСниС:

Полная Ρ„ΠΎΡ€ΠΌΡƒΠ»Π°:

Π’ качСствС Π°Π»ΡŒΡ‚Π΅Ρ€Π½Π°Ρ‚ΠΈΠ²Ρ‹ Π²Π½ΡƒΡ‚Ρ€Π΅Π½Π½Π΅Π΅ сопротивлСниС Π±Π°Ρ‚Π°Ρ€Π΅ΠΈ ΠΌΠΎΠΆΠ½ΠΎ Ρ€Π°ΡΡΡ‡ΠΈΡ‚Π°Ρ‚ΡŒ ΠΏΠΎ Ρ‚ΠΎΠΊΡƒ I L Ρ‡Π΅Ρ€Π΅Π· сопротивлСниС Π½Π°Π³Ρ€ΡƒΠ·ΠΊΠΈ, напряТСниС холостого Ρ…ΠΎΠ΄Π° аккумулятора ΠΈ сопротивлСниС Π½Π°Π³Ρ€ΡƒΠ·ΠΊΠΈ.

НапряТСниС Π½Π° Π½Π°Π³Ρ€ΡƒΠ·ΠΎΡ‡Π½ΠΎΠΌ рСзисторС

ПадСниС напряТСния Π½Π° Π²Π½ΡƒΡ‚Ρ€Π΅Π½Π½Π΅ΠΌ сопротивлСнии:

Π’Π½ΡƒΡ‚Ρ€Π΅Π½Π½Π΅Π΅ сопротивлСниС:

Полная Ρ„ΠΎΡ€ΠΌΡƒΠ»Π°:

Как ΠΈΠ·ΠΌΠ΅Ρ€ΠΈΡ‚ΡŒ Π²Π½ΡƒΡ‚Ρ€Π΅Π½Π½Π΅Π΅ сопротивлСниС Π±Π°Ρ‚Π°Ρ€Π΅ΠΈ

As Как ΠΌΡ‹ объяснили Π²Ρ‹ΡˆΠ΅, для опрСдСлСния Π²Π½ΡƒΡ‚Ρ€Π΅Π½Π½Π΅Π³ΠΎ сопротивлСния Π½Π°ΠΌ Π½ΡƒΠΆΠ½Ρ‹ Ρ‚Ρ€ΠΈ значСния:

  • напряТСниС холостого Ρ…ΠΎΠ΄Π° Π±Π°Ρ‚Π°Ρ€Π΅ΠΈ U NL , напряТСниС, ΠΈΠ·ΠΌΠ΅Ρ€Π΅Π½Π½ΠΎΠ΅ Π½Π° Π½Π°Π³Ρ€ΡƒΠ·ΠΊΠ΅ U L , ΠΈ сопротивлСниС Π½Π°Π³Ρ€ΡƒΠ·ΠΊΠΈ R L

ΠΈΠ»ΠΈ

  • Ρ‚ΠΎΠΊ I L Ρ‡Π΅Ρ€Π΅Π· сопротивлСниС Π½Π°Π³Ρ€ΡƒΠ·ΠΊΠΈ, напряТСниС холостого Ρ…ΠΎΠ΄Π° Π±Π°Ρ‚Π°Ρ€Π΅ΠΈ U NL ΠΈ сопротивлСниС Π½Π°Π³Ρ€ΡƒΠ·ΠΊΠΈ R L .

Π§Ρ‚ΠΎΠ±Ρ‹ ΠΏΡ€Π°Π²ΠΈΠ»ΡŒΠ½ΠΎ ΠΎΠΏΡ€Π΅Π΄Π΅Π»ΠΈΡ‚ΡŒ Π²Π½ΡƒΡ‚Ρ€Π΅Π½Π½Π΅Π΅ сопротивлСниС, Π½ΡƒΠΆΠ½ΠΎ произвСсти нСсколько ΠΈΠ·ΠΌΠ΅Ρ€Π΅Π½ΠΈΠΉ с Ρ€Π°Π·Π½Ρ‹ΠΌΠΈ рСзисторами. ΠšΡ€ΠΎΠΌΠ΅ Ρ‚ΠΎΠ³ΠΎ, Π²Π½ΡƒΡ‚Ρ€Π΅Π½Π½Π΅Π΅ сопротивлСниС ΠΌΠΎΠΆΠ΅Ρ‚ Π²Π°Ρ€ΡŒΠΈΡ€ΠΎΠ²Π°Ρ‚ΡŒΡΡ Π² зависимости ΠΎΡ‚ Ρ‚Π΅ΠΌΠΏΠ΅Ρ€Π°Ρ‚ΡƒΡ€Ρ‹, возраста Π±Π°Ρ‚Π°Ρ€Π΅ΠΈ ΠΈ ряда Π΄Ρ€ΡƒΠ³ΠΈΡ… Ρ„Π°ΠΊΡ‚ΠΎΡ€ΠΎΠ². Π’Π°ΠΊΠΈΠΌ ΠΎΠ±Ρ€Π°Π·ΠΎΠΌ, вашС ΠΈΠ·ΠΌΠ΅Ρ€Π΅Π½ΠΈΠ΅ являСтся лишь ΠΏΡ€ΠΈΠ±Π»ΠΈΠ·ΠΈΡ‚Π΅Π»ΡŒΠ½Ρ‹ΠΌ, ΠΈ Π½Π΅ сущСствуСт Ρ‚Π°ΠΊΠΎΠ³ΠΎ понятия, ΠΊΠ°ΠΊ «истинноС» Π²Π½ΡƒΡ‚Ρ€Π΅Π½Π½Π΅Π΅ сопротивлСниС, ΠΊΠΎΡ‚ΠΎΡ€ΠΎΠ΅ ΠΌΠΎΠΆΠ½ΠΎ Π±Ρ‹Π»ΠΎ Π±Ρ‹ Ρ‚ΠΎΡ‡Π½ΠΎ ΠΈΠ·ΠΌΠ΅Ρ€ΠΈΡ‚ΡŒ.

На Π²Π½ΡƒΡ‚Ρ€Π΅Π½Π½Π΅Π΅ сопротивлСниС Π±Π°Ρ‚Π°Ρ€Π΅ΠΉ влияСт нСсколько Ρ„Π°ΠΊΡ‚ΠΎΡ€ΠΎΠ², Π²ΠΊΠ»ΡŽΡ‡Π°Ρ ΠΈΡ… Π΅ΠΌΠΊΠΎΡΡ‚ΡŒ, химичСский состав, качСство элСмСнтов, возраст, Ρ‚Π΅ΠΌΠΏΠ΅Ρ€Π°Ρ‚ΡƒΡ€Ρƒ ΠΈ ΡΠΊΠΎΡ€ΠΎΡΡ‚ΡŒ разряда.Π”ΠΎΠΏΠΎΠ»Π½ΠΈΡ‚Π΅Π»ΡŒΠ½ΡƒΡŽ ΠΈΠ½Ρ„ΠΎΡ€ΠΌΠ°Ρ†ΠΈΡŽ ΠΎ батарСях Π²Ρ‹ Π½Π°ΠΉΠ΄Π΅Ρ‚Π΅ Π² Π½Π°ΡˆΠΈΡ… ΠΊΠ°Π»ΡŒΠΊΡƒΠ»ΡΡ‚ΠΎΡ€Π°Ρ… энСргии ΠΈ Π²Ρ€Π΅ΠΌΠ΅Π½ΠΈ Ρ€Π°Π±ΠΎΡ‚Ρ‹ ΠΎΡ‚ Π±Π°Ρ‚Π°Ρ€Π΅ΠΉ ΠΈ ΠΊΠ°Π»ΡŒΠΊΡƒΠ»ΡΡ‚ΠΎΡ€Π΅ LiPo Π±Π°Ρ‚Π°Ρ€Π΅ΠΉ для Π΄Ρ€ΠΎΠ½Π°.

Для измСрСния напряТСния Π½Π° Π½Π°Π³Ρ€ΡƒΠ·ΠΊΠ΅, ΠΏΠΎΠ΄ΠΊΠ»ΡŽΡ‡Π΅Π½Π½ΠΎΠΉ ΠΊ Π±Π°Ρ‚Π°Ρ€Π΅Π΅ , Π²ΠΎΠ»ΡŒΡ‚ΠΌΠ΅Ρ‚Ρ€ ΠΏΠΎΠ΄ΠΊΠ»ΡŽΡ‡Π°Π΅Ρ‚ΡΡ ΠΏΠ°Ρ€Π°Π»Π»Π΅Π»ΡŒΠ½ΠΎ Π½Π°Π³Ρ€ΡƒΠ·ΠΊΠ΅ ΠΈΠ»ΠΈ ΠΊΠ»Π΅ΠΌΠΌΠ°ΠΌ Π±Π°Ρ‚Π°Ρ€Π΅ΠΈ. Если сопротивлСниС Π½Π°Π³Ρ€ΡƒΠ·ΠΊΠΈ ΠΎΡ‚Π½ΠΎΡΠΈΡ‚Π΅Π»ΡŒΠ½ΠΎ Π½ΠΈΠ·ΠΊΠΎΠ΅ ΠΏΠΎ ΡΡ€Π°Π²Π½Π΅Π½ΠΈΡŽ с Π²Π½ΡƒΡ‚Ρ€Π΅Π½Π½ΠΈΠΌ сопротивлСниСм измСритСля, Π²Ρ‹ ΠΏΠΎΠ»ΡƒΡ‡ΠΈΡ‚Π΅ достаточно Ρ‚ΠΎΡ‡Π½ΠΎΠ΅ ΠΏΠΎΠΊΠ°Π·Π°Π½ΠΈΠ΅ напряТСния Π½Π°Π³Ρ€ΡƒΠ·ΠΊΠΈ.

Для измСрСния Ρ‚ΠΎΠΊΠ° , ΠΏΠΎΠ΄Π°Π²Π°Π΅ΠΌΠΎΠ³ΠΎ Π½Π° Π½Π°Π³Ρ€ΡƒΠ·ΠΊΡƒ, ΠΏΠΎΠ΄ΠΊΠ»ΡŽΡ‡Π΅Π½Π½ΡƒΡŽ ΠΊ Π±Π°Ρ‚Π°Ρ€Π΅Π΅ , ΠΌΠ΅ΠΆΠ΄Ρƒ Π½Π°Π³Ρ€ΡƒΠ·ΠΊΠΎΠΉ ΠΈ Π±Π°Ρ‚Π°Ρ€Π΅Π΅ΠΉ ΠΏΠΎΠ΄ΠΊΠ»ΡŽΡ‡Π°Π΅Ρ‚ΡΡ Π°ΠΌΠΏΠ΅Ρ€ΠΌΠ΅Ρ‚Ρ€, ΠΊΠ°ΠΊ ΠΏΠΎΠΊΠ°Π·Π°Π½ΠΎ Π½Π° рисункС.Если Π΅Π³ΠΎ Π²Π½ΡƒΡ‚Ρ€Π΅Π½Π½Π΅Π΅ сопротивлСниС ΠΎΡ‚Π½ΠΎΡΠΈΡ‚Π΅Π»ΡŒΠ½ΠΎ ΠΌΠ°Π»ΠΎ ΠΏΠΎ ΡΡ€Π°Π²Π½Π΅Π½ΠΈΡŽ с сопротивлСниСм Π½Π°Π³Ρ€ΡƒΠ·ΠΊΠΈ, Π²Ρ‹ ΠΌΠΎΠΆΠ΅Ρ‚Π΅ ΠΏΡ€Π΅Π΄ΠΏΠΎΠ»ΠΎΠΆΠΈΡ‚ΡŒ, Ρ‡Ρ‚ΠΎ ваши измСрСния Ρ‚ΠΎΡ‡Π½Ρ‹.

ΠšΠΎΠ½Π΅Ρ‡Π½ΠΎ, тСорСтичСски ΠΈ Π΄Π°ΠΆΠ΅ практичСски (Π½Π°ΠΏΡ€ΠΈΠΌΠ΅Ρ€, для ΡƒΠ³ΠΎΠ»ΡŒΠ½ΠΎ-Ρ†ΠΈΠ½ΠΊΠΎΠ²ΠΎΠΉ Π±Π°Ρ‚Π°Ρ€Π΅ΠΈ) Ρ‚ΠΎΠΊ ΠΊΠΎΡ€ΠΎΡ‚ΠΊΠΎΠ³ΠΎ замыкания Π±Π°Ρ‚Π°Ρ€Π΅ΠΈ ΠΈΠ·ΠΌΠ΅Ρ€ΠΈΡ‚ΡŒ ΠΏΡƒΡ‚Π΅ΠΌ ΠΊΠΎΡ€ΠΎΡ‚ΠΊΠΎΠ³ΠΎ замыкания Π±Π°Ρ‚Π°Ρ€Π΅ΠΈ Π°ΠΌΠΏΠ΅Ρ€ΠΌΠ΅Ρ‚Ρ€ΠΎΠΌ Π²ΠΏΠΎΠ»Π½Π΅ Π²ΠΎΠ·ΠΌΠΎΠΆΠ½ΠΎ. Однако, Ссли батарСя способна Π²Ρ‹Π΄Π°Π²Π°Ρ‚ΡŒ Π·Π½Π°Ρ‡ΠΈΡ‚Π΅Π»ΡŒΠ½Ρ‹ΠΉ Ρ‚ΠΎΠΊ, ΠΎΠ½Π° ΠΌΠΎΠΆΠ΅Ρ‚ ΠΏΠ΅Ρ€Π΅Π³Ρ€Π΅Ρ‚ΡŒΡΡ ΠΈΠ»ΠΈ Π΄Π°ΠΆΠ΅ Π·Π°Π³ΠΎΡ€Π΅Ρ‚ΡŒΡΡ ΠΏΡ€ΠΈ ΠΊΠΎΡ€ΠΎΡ‚ΠΊΠΎΠΌ Π·Π°ΠΌΡ‹ΠΊΠ°Π½ΠΈΠΈ. Π›ΠΈΡ‚ΠΈΠΉ-ΠΈΠΎΠ½Π½Ρ‹Π΅ Π±Π°Ρ‚Π°Ρ€Π΅ΠΈ ΠΌΠΎΠ³ΡƒΡ‚ Π΄Π°ΠΆΠ΅ Π²Π·ΠΎΡ€Π²Π°Ρ‚ΡŒΡΡ, Ссли ΠΈΡ… ΠΊΠ»Π΅ΠΌΠΌΡ‹ Π·Π°ΠΊΠΎΡ€ΠΎΡ‡Π΅Π½Ρ‹.Π‘Π»Π΅Π΄ΠΎΠ²Π°Ρ‚Π΅Π»ΡŒΠ½ΠΎ, Ρ‚ΠΎΠΊ ΠΏΠΎΡ‡Ρ‚ΠΈ всСгда измСряСтся, ΠΊΠΎΠ³Π΄Π° батарСя ΠΏΠΎΠ΄ΠΊΠ»ΡŽΡ‡Π΅Π½Π° ΠΊ Ρ€Π°Π·ΡƒΠΌΠ½ΠΎΠΉ Π½Π°Π³Ρ€ΡƒΠ·ΠΊΠ΅.

Для измСрСния напряТСния холостого Ρ…ΠΎΠ΄Π° аккумулятора ΠΊ Π΅Π³ΠΎ ΠΊΠ»Π΅ΠΌΠΌΠ°ΠΌ Π±Π΅Π· Π½Π°Π³Ρ€ΡƒΠ·ΠΊΠΈ ΠΏΠΎΠ΄ΠΊΠ»ΡŽΡ‡Π°Π΅Ρ‚ΡΡ Π²ΠΎΠ»ΡŒΡ‚ΠΌΠ΅Ρ‚Ρ€. Π­Ρ‚ΠΎ напряТСниС Ρ‚Π°ΠΊΠΆΠ΅ называСтся напряТСниСм холостого Ρ…ΠΎΠ΄Π°. Если Π²Π½ΡƒΡ‚Ρ€Π΅Π½Π½Π΅Π΅ сопротивлСниС Π²ΠΎΠ»ΡŒΡ‚ΠΌΠ΅Ρ‚Ρ€Π° Π½Π°ΠΌΠ½ΠΎΠ³ΠΎ Π²Ρ‹ΡˆΠ΅ Π²Π½ΡƒΡ‚Ρ€Π΅Π½Π½Π΅Π³ΠΎ сопротивлСния аккумулятора, ΠΌΠΎΠΆΠ½ΠΎ ΠΏΡ€Π΅Π΄ΠΏΠΎΠ»ΠΎΠΆΠΈΡ‚ΡŒ, Ρ‡Ρ‚ΠΎ напряТСниС холостого Ρ…ΠΎΠ΄Π° измСряСтся ΠΎΡ‚Π½ΠΎΡΠΈΡ‚Π΅Π»ΡŒΠ½ΠΎ Ρ‚ΠΎΡ‡Π½ΠΎ.

Π’Π°ΠΊΠΆΠ΅ Π½Π΅ΠΎΠ±Ρ…ΠΎΠ΄ΠΈΠΌΠΎ ΠΈΠ·ΠΌΠ΅Ρ€ΠΈΡ‚ΡŒ сопротивлСниС Π½Π°Π³Ρ€ΡƒΠ·ΠΊΠΈ , Ссли Π²Ρ‹ Π½Π΅ ΠΈΡΠΏΠΎΠ»ΡŒΠ·ΡƒΠ΅Ρ‚Π΅ ΠΏΡ€Π΅Ρ†ΠΈΠ·ΠΈΠΎΠ½Π½Ρ‹ΠΉ рСзистор.ΠŸΠΎΠΌΠ½ΠΈΡ‚Π΅, Ρ‡Ρ‚ΠΎ Ссли Π½Π°Π³Ρ€ΡƒΠ·ΠΎΡ‡Π½Ρ‹ΠΉ рСзистор нагрСваСтся, Π΅Π³ΠΎ сопротивлСниС увСличиваСтся, поэтому ΠΈΠ·ΠΌΠ΅Ρ€Π΅Π½ΠΈΠ΅ Ρ‚ΠΎΠΊΠ° Π½Π΅ΠΎΠ±Ρ…ΠΎΠ΄ΠΈΠΌΠΎ ΠΏΡ€ΠΎΠ²ΠΎΠ΄ΠΈΡ‚ΡŒ быстро.

Π’Π΅ΠΏΠ΅Ρ€ΡŒ Π²Ρ‹ ΠΌΠΎΠΆΠ΅Ρ‚Π΅ ΠΏΠΎΠΌΠ΅ΡΡ‚ΠΈΡ‚ΡŒ Ρ€Π΅Π·ΡƒΠ»ΡŒΡ‚Π°Ρ‚Ρ‹ Π²Π°ΡˆΠΈΡ… ΠΈΠ·ΠΌΠ΅Ρ€Π΅Π½ΠΈΠΉ Π² наш ΠΊΠ°Π»ΡŒΠΊΡƒΠ»ΡΡ‚ΠΎΡ€ ΠΈ ΠΏΠΎΠ»ΡƒΡ‡ΠΈΡ‚ΡŒ Π²Π½ΡƒΡ‚Ρ€Π΅Π½Π½Π΅Π΅ сопротивлСниС вашСй Π±Π°Ρ‚Π°Ρ€Π΅ΠΈ. ΠšΠΎΠ½Π΅Ρ‡Π½ΠΎ, Π² ΠΏΡ€ΠΎΠ΄Π°ΠΆΠ΅ имССтся мноТСство ΡΠΏΠ΅Ρ†ΠΈΠ°Π»ΡŒΠ½Ρ‹Ρ… ΠΈΠ·ΠΌΠ΅Ρ€ΠΈΡ‚Π΅Π»Π΅ΠΉ Π²Π½ΡƒΡ‚Ρ€Π΅Π½Π½Π΅Π³ΠΎ сопротивлСния. ΠšΡ€ΠΎΠΌΠ΅ Ρ‚ΠΎΠ³ΠΎ, Π±ΠΎΠ»Π΅Π΅ ΠΌΠΎΡ‰Π½Ρ‹Π΅ зарядныС устройства ΠΌΠΎΠ³ΡƒΡ‚ ΠΈΠ·ΠΌΠ΅Ρ€ΡΡ‚ΡŒ Π²Π½ΡƒΡ‚Ρ€Π΅Π½Π½Π΅Π΅ сопротивлСниС Π±Π°Ρ‚Π°Ρ€Π΅ΠΈ.

Π§Ρ‚ΠΎΠ±Ρ‹ ΠΏΠΎΠ»ΡƒΡ‡ΠΈΡ‚ΡŒ ΠΏΠΎΠ»Π½ΡƒΡŽ ΠΊΠ°Ρ€Ρ‚ΠΈΠ½Ρƒ, ΠΌΡ‹ ΠΌΠΎΠΆΠ΅ΠΌ ΠΎΡ‚ΠΌΠ΅Ρ‚ΠΈΡ‚ΡŒ, Ρ‡Ρ‚ΠΎ каТдая батарСя ΠΈΠΌΠ΅Π΅Ρ‚ спСктр Π²Π½ΡƒΡ‚Ρ€Π΅Π½Π½ΠΈΡ… сопротивлСний ΠΈΠ»ΠΈ, скорСС, импСдансов, ΠΈ для ΠΈΡ… измСрСния часто ΠΈΡΠΏΠΎΠ»ΡŒΠ·ΡƒΠ΅Ρ‚ΡΡ Π±ΠΎΠ»Π΅Π΅ слоТная схСма, которая питаСтся ΠΎΡ‚ источника ΠΏΠ΅Ρ€Π΅ΠΌΠ΅Π½Π½ΠΎΠ³ΠΎ Ρ‚ΠΎΠΊΠ° с частотой измСняСтся ΠΎΡ‚ ΠΎΡ‡Π΅Π½ΡŒ Π½ΠΈΠ·ΠΊΠΎΠ³ΠΎ Π΄ΠΎ Π½Π΅ΡΠΊΠΎΠ»ΡŒΠΊΠΈΡ… ΠΊΠΈΠ»ΠΎΠ³Π΅Ρ€Ρ†.Π’Π½ΡƒΡ‚Ρ€Π΅Π½Π½Π΅Π΅ сопротивлСниС ΠΎΠ±Ρ‹Ρ‡Π½ΠΎ характСризуСтся Π³Ρ€Π°Ρ„ΠΈΠΊΠ°ΠΌΠΈ, ΠΏΠΎΠΊΠ°Π·Ρ‹Π²Π°ΡŽΡ‰ΠΈΠΌΠΈ Π΅Π³ΠΎ Π·Π°Π²ΠΈΡΠΈΠΌΠΎΡΡ‚ΡŒ ΠΎΡ‚ Ρ€Π°Π·Π»ΠΈΡ‡Π½Ρ‹Ρ… Ρ„Π°ΠΊΡ‚ΠΎΡ€ΠΎΠ².

ΠŸΡ€ΠΈΠΌΠ΅Ρ€Ρ‹ расчСтов

ΠŸΡ€ΠΈΠΌΠ΅Ρ€ 2. БатарСя с Π­Π”Π‘ = 14,5 Π’ Π²Ρ‹Π΄Π°Π΅Ρ‚ 25 Π’Ρ‚ мощности Π½Π° внСшний Π½Π°Π³Ρ€ΡƒΠ·ΠΎΡ‡Π½Ρ‹ΠΉ рСзистор. НапряТСниС Π½Π° Π²Ρ‹Π²ΠΎΠ΄Π΅ аккумулятора составляСт 11,9 Π’. ΠžΠΏΡ€Π΅Π΄Π΅Π»ΠΈΡ‚Π΅ Π²Π½ΡƒΡ‚Ρ€Π΅Π½Π½Π΅Π΅ сопротивлСниС аккумулятора. НамСк. Π’ΠΎΡΠΏΠΎΠ»ΡŒΠ·ΡƒΠΉΡ‚Π΅ΡΡŒ нашим ΠΊΠ°Π»ΡŒΠΊΡƒΠ»ΡΡ‚ΠΎΡ€ΠΎΠΌ Π·Π°ΠΊΠΎΠ½Π° Ома, Ρ‡Ρ‚ΠΎΠ±Ρ‹ ΠΎΠΏΡ€Π΅Π΄Π΅Π»ΠΈΡ‚ΡŒ Ρ‚ΠΎΠΊ Ρ‡Π΅Ρ€Π΅Π· Π½Π°Π³Ρ€ΡƒΠ·ΠΎΡ‡Π½Ρ‹ΠΉ рСзистор. Π—Π°Ρ‚Π΅ΠΌ ΠΈΡΠΏΠΎΠ»ΡŒΠ·ΡƒΠΉΡ‚Π΅ этот ΠΊΠ°Π»ΡŒΠΊΡƒΠ»ΡΡ‚ΠΎΡ€ для опрСдСлСния Π²Π½ΡƒΡ‚Ρ€Π΅Π½Π½Π΅Π³ΠΎ сопротивлСния.

ΠŸΡ€ΠΈΠΌΠ΅Ρ€ 3. Π›Π°ΠΌΠΏΠ° накаливания Π½Π° 4 Ом ΠΏΠΎΠ΄ΠΊΠ»ΡŽΡ‡Π΅Π½Π° ΠΊ Π±Π°Ρ‚Π°Ρ€Π΅Π΅ с Π²Π½ΡƒΡ‚Ρ€Π΅Π½Π½ΠΈΠΌ сопротивлСниСм 0,15 Ом. Π’ΠΎΠ»ΡŒΡ‚ΠΌΠ΅Ρ‚Ρ€, ΠΏΠΎΠ΄ΠΊΠ»ΡŽΡ‡Π΅Π½Π½Ρ‹ΠΉ ΠΊ ΠΊΠ»Π΅ΠΌΠΌΠ°ΠΌ аккумулятора, ΠΏΠΎΠΊΠ°Π·Ρ‹Π²Π°Π΅Ρ‚ 11,5 Π’. Какая Π­Π”Π‘ аккумулятора?

ΠŸΡ€ΠΈΠΌΠ΅Ρ€ 4. Π”Π²Π΅ Π³Π°Π»ΠΎΠ³Π΅Π½Π½Ρ‹Π΅ Π»Π°ΠΌΠΏΡ‹ Π³ΠΎΠ»ΠΎΠ²Π½ΠΎΠ³ΠΎ свСта ΠΌΠΎΡ‰Π½ΠΎΡΡ‚ΡŒΡŽ 55 Π’Ρ‚ ΠΏΠΎΠ΄ΠΊΠ»ΡŽΡ‡Π΅Π½Ρ‹ ΠΏΠ°Ρ€Π°Π»Π»Π΅Π»ΡŒΠ½ΠΎ ΠΊ ΠΊΠ»Π΅ΠΌΠΌΠ°ΠΌ аккумуляторной Π±Π°Ρ‚Π°Ρ€Π΅ΠΈ Π³Ρ€ΡƒΠ·ΠΎΠ²ΠΎΠ³ΠΎ автомобиля, ΠΈΠΌΠ΅ΡŽΡ‰Π΅ΠΉ Π²Π½ΡƒΡ‚Ρ€Π΅Π½Π½Π΅Π΅ сопротивлСниС 0,02 Ом. НапряТСниС Π½Π° Π²Ρ‹Π²ΠΎΠ΄Π°Ρ… аккумулятора 23,6 Π’. Какая Π­Π”Π‘ β„° аккумулятора? Подсказка: ΠΈΡΠΏΠΎΠ»ΡŒΠ·ΡƒΠΉΡ‚Π΅ наш ΠΊΠ°Π»ΡŒΠΊΡƒΠ»ΡΡ‚ΠΎΡ€ мощности постоянного Ρ‚ΠΎΠΊΠ°, Ρ‡Ρ‚ΠΎΠ±Ρ‹ ΠΎΠΏΡ€Π΅Π΄Π΅Π»ΠΈΡ‚ΡŒ сопротивлСниС горячСй Π»Π°ΠΌΠΏΡ‹.Π—Π°Ρ‚Π΅ΠΌ ΠΈΡΠΏΠΎΠ»ΡŒΠ·ΡƒΠΉΡ‚Π΅ наш ΠΊΠ°Π»ΡŒΠΊΡƒΠ»ΡΡ‚ΠΎΡ€ ΠΏΠ°Ρ€Π°Π»Π»Π΅Π»ΡŒΠ½ΠΎΠ³ΠΎ сопротивлСния, Ρ‡Ρ‚ΠΎΠ±Ρ‹ ΠΎΠΏΡ€Π΅Π΄Π΅Π»ΠΈΡ‚ΡŒ сопротивлСниС Π΄Π²ΡƒΡ… ΠΏΠ°Ρ€Π°Π»Π»Π΅Π»ΡŒΠ½ΠΎ соСдинСнных Π»Π°ΠΌΠΏ. Π—Π°Ρ‚Π΅ΠΌ ΠΈΡΠΏΠΎΠ»ΡŒΠ·ΡƒΠΉΡ‚Π΅ этот ΠΊΠ°Π»ΡŒΠΊΡƒΠ»ΡΡ‚ΠΎΡ€ для опрСдСлСния Π­Π”Π‘ аккумулятора.

ΠŸΡ€ΠΈΠΌΠ΅Ρ€ 5. ΠžΠΏΡ€Π΅Π΄Π΅Π»ΠΈΡ‚Π΅ Ρ‚ΠΎΠΊ ΠΊΠΎΡ€ΠΎΡ‚ΠΊΠΎΠ³ΠΎ замыкания Π°Π²Ρ‚ΠΎΠΌΠΎΠ±ΠΈΠ»ΡŒΠ½ΠΎΠ³ΠΎ аккумулятора Π½Π° 12 Π’ с Π­Π”Π‘ = 13,5 Π’ ΠΈ Π²Π½ΡƒΡ‚Ρ€Π΅Π½Π½ΠΈΠΌ сопротивлСниСм 0,04 Ом. Подсказка: 12 Π’ — это номинальноС напряТСниС аккумулятора, ΠΈ это число Π½Π΅ ΠΈΡΠΏΠΎΠ»ΡŒΠ·ΡƒΠ΅Ρ‚ΡΡ ΠΏΡ€ΠΈ Ρ€Π΅ΡˆΠ΅Π½ΠΈΠΈ этой ΠΏΡ€ΠΎΠ±Π»Π΅ΠΌΡ‹.

ΠŸΡ€ΠΈΠΌΠ΅Ρ€ 6. БатарСя с Π­Π”Π‘ β„° = 1.5 Π’ Π·Π°ΠΌΠΊΠ½ΡƒΡ‚ Π½Π°ΠΊΠΎΡ€ΠΎΡ‚ΠΊΠΎ с ΠΏΠΎΠΌΠΎΡ‰ΡŒΡŽ нСидСального Π°ΠΌΠΏΠ΅Ρ€ΠΌΠ΅Ρ‚Ρ€Π° с Π²Π½ΡƒΡ‚Ρ€Π΅Π½Π½ΠΈΠΌ сопротивлСниСм 0,02 Ом, ΠΊΠΎΡ‚ΠΎΡ€ΠΎΠ΅ ΠΏΠΎΠΊΠ°Π·Ρ‹Π²Π°Π΅Ρ‚ 2,7 А. ΠžΠΏΡ€Π΅Π΄Π΅Π»ΠΈΡ‚Π΅ Π΅Π³ΠΎ Π²Π½ΡƒΡ‚Ρ€Π΅Π½Π½Π΅Π΅ сопротивлСниС ΠΈ ΠΌΠΎΡ‰Π½ΠΎΡΡ‚ΡŒ, Ρ€Π°ΡΡΠ΅ΠΈΠ²Π°Π΅ΠΌΡƒΡŽ Π²Π½ΡƒΡ‚Ρ€ΠΈ Π±Π°Ρ‚Π°Ρ€Π΅ΠΈ. Подсказка: сначала ΠΈΡΠΏΠΎΠ»ΡŒΠ·ΡƒΠΉΡ‚Π΅ этот ΠΊΠ°Π»ΡŒΠΊΡƒΠ»ΡΡ‚ΠΎΡ€ для опрСдСлСния Π²Π½ΡƒΡ‚Ρ€Π΅Π½Π½Π΅Π³ΠΎ сопротивлСния Π±Π°Ρ‚Π°Ρ€Π΅ΠΈ, Π° Π·Π°Ρ‚Π΅ΠΌ Π²ΠΎΡΠΏΠΎΠ»ΡŒΠ·ΡƒΠΉΡ‚Π΅ΡΡŒ нашим ΠΊΠ°Π»ΡŒΠΊΡƒΠ»ΡΡ‚ΠΎΡ€ΠΎΠΌ мощности постоянного Ρ‚ΠΎΠΊΠ°, Ρ‡Ρ‚ΠΎΠ±Ρ‹ ΠΎΠΏΡ€Π΅Π΄Π΅Π»ΠΈΡ‚ΡŒ ΠΌΠΎΡ‰Π½ΠΎΡΡ‚ΡŒ, Ρ€Π°ΡΡΠ΅ΠΈΠ²Π°Π΅ΠΌΡƒΡŽ Π² Π±Π°Ρ‚Π°Ρ€Π΅Π΅.

ΠŸΡ€ΠΈΠΌΠ΅Ρ€ 7. ΠšΠΎΠ½Ρ‚Ρ€ΠΎΠ»Π»Π΅Ρ€ запуска ΠΌΠΎΠ΄Π΅Π»ΠΈ, ΠΊΠΎΡ‚ΠΎΡ€Ρ‹ΠΉ ΠΈΡΠΏΠΎΠ»ΡŒΠ·ΡƒΠ΅Ρ‚ΡΡ для запуска Ρ€Π°ΠΊΠ΅Ρ‚Π½ΠΎΠ³ΠΎ двигатСля ΠΏΡƒΡ‚Π΅ΠΌ Π½Π°Π³Ρ€Π΅Π²Π° Π½ΠΈΡ…Ρ€ΠΎΠΌΠΎΠ²ΠΎΠΉ ΠΏΡ€ΠΎΠ²ΠΎΠ»ΠΎΠΊΠΈ воспламСнитСля, питаСтся ΠΎΡ‚ Ρ‡Π΅Ρ‚Ρ‹Ρ€Π΅Ρ… AA 1.Π‘Π°Ρ‚Π°Ρ€Π΅ΠΈ 5 Π’ соСдинСны ΠΏΠΎΡΠ»Π΅Π΄ΠΎΠ²Π°Ρ‚Π΅Π»ΡŒΠ½ΠΎ. КаТдая батарСя ΠΈΠΌΠ΅Π΅Ρ‚ Π²Π½ΡƒΡ‚Ρ€Π΅Π½Π½Π΅Π΅ сопротивлСниС 200 мОм. Π‘ΠΎΠΏΡ€ΠΎΡ‚ΠΈΠ²Π»Π΅Π½ΠΈΠ΅ Π΄Π²ΡƒΡ… Ρ€Π°Π·Π½Ρ‹Ρ… воспламСнитСлСй Ρ€Π°ΠΊΠ΅Ρ‚Π½Ρ‹Ρ… Π΄Π²ΠΈΠ³Π°Ρ‚Π΅Π»Π΅ΠΉ составляСт 0,7 ΠΈ 3 Ом. ΠžΠΏΡ€Π΅Π΄Π΅Π»ΠΈΡ‚Π΅ Ρ‚ΠΎΠΊ, ΠΏΠΎΠ΄Π°Π²Π°Π΅ΠΌΡ‹ΠΉ Π½Π° Π²ΠΎΡΠΏΠ»Π°ΠΌΠ΅Π½ΠΈΡ‚Π΅Π»ΡŒ 0,7 Ом ΠΈ Π²ΠΎΡΠΏΠ»Π°ΠΌΠ΅Π½ΠΈΡ‚Π΅Π»ΡŒ 3 Ом. Подсказка: напряТСниС Ρ‡Π΅Ρ‚Ρ‹Ρ€Π΅Ρ… ΠΏΠΎΡΠ»Π΅Π΄ΠΎΠ²Π°Ρ‚Π΅Π»ΡŒΠ½ΠΎ соСдинСнных Π±Π°Ρ‚Π°Ρ€Π΅ΠΉ составляСт 1,5 Γ— 4 = 6 Π’, Π° ΠΈΡ… ΠΎΠ±Ρ‰Π΅Π΅ Π²Π½ΡƒΡ‚Ρ€Π΅Π½Π½Π΅Π΅ сопротивлСниС составляСт 200 Γ— 4 = 0,8 Ом.

Π­Ρ‚Ρƒ ΡΡ‚Π°Ρ‚ΡŒΡŽ написал Анатолий Π—ΠΎΠ»ΠΎΡ‚ΠΊΠΎΠ²

Π’Π½ΡƒΡ‚Ρ€Π΅Π½Π½Π΅Π΅ сопротивлСниС — ΠΊΠΎΠ³Π΄Π° ΠΎΠ½ΠΎ Π½Π΅Π·Π½Π°Ρ‡ΠΈΡ‚Π΅Π»ΡŒΠ½ΠΎ, Π° ΠΊΠΎΠ³Π΄Π° Π΅Π³ΠΎ Π½Π΅Ρ‚

Π‘Π°Ρ‚Π°Ρ€Π΅ΠΉΠΊΠΈ ΠΏΠΎΡ‚Π΅ΡŽΡ‚, Ссли Π·Π°ΡΡ‚Π°Π²Π»ΡΡ‚ΡŒ ΠΈΡ… Ρ€Π°Π±ΠΎΡ‚Π°Ρ‚ΡŒ слишком ΠΌΠ½ΠΎΠ³ΠΎ

Аккумулятор ΠΏΡ€Π΅ΠΎΠ±Ρ€Π°Π·ΡƒΠ΅Ρ‚ Ρ…ΠΈΠΌΠΈΡ‡Π΅ΡΠΊΡƒΡŽ ΡΠ½Π΅Ρ€Π³ΠΈΡŽ Π² ΡΠ»Π΅ΠΊΡ‚Ρ€ΠΈΡ‡Π΅ΡΠΊΡƒΡŽ.Π­Ρ‚ΠΎ ΠΏΡ€Π΅ΠΎΠ±Ρ€Π°Π·ΠΎΠ²Π°Π½ΠΈΠ΅ Π²Ρ‹Π·Π²Π°Π½ΠΎ химичСскими рСакциями Π²Π½ΡƒΡ‚Ρ€ΠΈ Π±Π°Ρ‚Π°Ρ€Π΅ΠΈ. Π§Π΅ΠΌ быстрСС батарСя Π²Ρ‹Ρ€Π°Π±Π°Ρ‚Ρ‹Π²Π°Π΅Ρ‚ ΡΠ½Π΅Ρ€Π³ΠΈΡŽ, Ρ‚Π΅ΠΌ быстрСС Π΄ΠΎΠ»ΠΆΠ½Ρ‹ ΠΏΡ€ΠΎΠΈΡΡ…ΠΎΠ΄ΠΈΡ‚ΡŒ химичСскиС Ρ€Π΅Π°ΠΊΡ†ΠΈΠΈ.

Если Π²Ρ‹ заставляСтС аккумулятор Ρ€Π°Π±ΠΎΡ‚Π°Ρ‚ΡŒ интСнсивно, Ρ‚ΠΎ Ρ‡Π°ΡΡ‚ΡŒ химичСской энСргии прСобразуСтся Π² ΡΠ»Π΅ΠΊΡ‚Ρ€ΠΈΡ‡Π΅ΡΠΊΡƒΡŽ, Π° Ρ‡Π°ΡΡ‚ΡŒ — Π² Ρ‚Π΅ΠΏΠ»ΠΎΠ²ΡƒΡŽ. ВСпловая энСргия Π½Π°Π³Ρ€Π΅Π²Π°Π΅Ρ‚ аккумулятор.

Π§Π΅ΠΌ тяТСлСС Ρ€Π°Π±ΠΎΡ‚Π°Π΅Ρ‚ аккумулятор, Ρ‚Π΅ΠΌ большС химичСской энСргии прСобразуСтся Π² Ρ‚Π΅ΠΏΠ»ΠΎΠ²ΡƒΡŽ ΠΈ Ρ‚Π΅ΠΌ сильнСС нагрСваСтся аккумулятор.

Анимация, ΠΏΠΎΠΊΠ°Π·Ρ‹Π²Π°ΡŽΡ‰Π°Ρ Ρ€Π°Π·Π»ΠΈΡ‡Π½Ρ‹Π΅ способы ΠΊΠΎΡ€ΠΎΡ‚ΠΊΠΎΠ³ΠΎ замыкания всСй Ρ†Π΅ΠΏΠΈ, ΠΏΠΎΠ΄ΠΊΠ»ΡŽΡ‡ΠΈΠ² ΠΎΠ΄Π½Ρƒ ΠΊΠ»Π΅ΠΌΠΌΡƒ Π±Π°Ρ‚Π°Ρ€Π΅ΠΈ Π½Π°ΠΏΡ€ΡΠΌΡƒΡŽ ΠΊ Π΄Ρ€ΡƒΠ³ΠΎΠΉ.

Как Π·Π°ΡΡ‚Π°Π²ΠΈΡ‚ΡŒ аккумулятор Ρ€Π°Π±ΠΎΡ‚Π°Ρ‚ΡŒ

Если Π²Ρ‹ заставляСтС Π±Π°Ρ‚Π°Ρ€Π΅ΡŽ Ρ€Π°Π±ΠΎΡ‚Π°Ρ‚ΡŒ с ΠΎΠ΄Π½ΠΈΠΌ ΠΊΠΎΠΌΠΏΠΎΠ½Π΅Π½Ρ‚ΠΎΠΌ, ΠΊΠΎΡ‚ΠΎΡ€Ρ‹ΠΉ ΠΎΡ‡Π΅Π½ΡŒ быстро потрСбляСт ΡΠ½Π΅Ρ€Π³ΠΈΡŽ, Π½Π°ΠΏΡ€ΠΈΠΌΠ΅Ρ€ ΠΎΡ‡Π΅Π½ΡŒ яркой Π»Π°ΠΌΠΏΠΎΡ‡ΠΊΠΎΠΉ, ΠΈΠ»ΠΈ с мноТСством ΠΊΠΎΠΌΠΏΠΎΠ½Π΅Π½Ρ‚ΠΎΠ², Π²ΠΊΠ»ΡŽΡ‡Π΅Π½Π½Ρ‹Ρ… ΠΏΠ°Ρ€Π°Π»Π»Π΅Π»ΡŒΠ½ΠΎ, Ρ‚ΠΎΠ³Π΄Π° батарСя Π΄ΠΎΠ»ΠΆΠ½Π° Ρ€Π°Π±ΠΎΡ‚Π°Ρ‚ΡŒ ΠΎΡ‡Π΅Π½ΡŒ тяТСло, ΠΏΠΎΡ‚ΠΎΠΌΡƒ Ρ‡Ρ‚ΠΎ ΠΎΠ½Π° Π΄ΠΎΠ»ΠΆΠ½Π° ΠΎΡ‡Π΅Π½ΡŒ быстро ΠΏΠΎΠ΄Π°Π²Π°Ρ‚ΡŒ ΡΠ½Π΅Ρ€Π³ΠΈΡŽ.

ΠŸΠΎΠΌΠ½ΠΈΡ‚Π΅, Ρ‡Ρ‚ΠΎ Π±Π°Ρ‚Π°Ρ€Π΅ΠΈ ΡΠ²Π»ΡΡŽΡ‚ΡΡ (ΠΈΠ»ΠΈ ΡΡ‚Π°Ρ€Π°ΡŽΡ‚ΡΡ Π±Ρ‹Ρ‚ΡŒ) поставщиками постоянного напряТСния. Π‘ΠΈΠ»Π° Ρ‚ΠΎΠΊΠ° зависит ΠΎΡ‚ выполняСмой ΠΈΠΌΠΈ Ρ€Π°Π±ΠΎΡ‚Ρ‹. Когда ΠΎΠ½ΠΈ ΠΌΠ½ΠΎΠ³ΠΎ Ρ€Π°Π±ΠΎΡ‚Π°ΡŽΡ‚, ΠΎΠ½ΠΈ Π΄Π°ΡŽΡ‚ большой Ρ‚ΠΎΠΊ. Но, ΠΊΠ°ΠΊ ΠΌΡ‹ ΡƒΠ²ΠΈΠ΄ΠΈΠΌ, Ссли ΠΎΠ½ΠΈ Π²Ρ‹Π΄Π°ΡŽΡ‚ ΠΎΡ‡Π΅Π½ΡŒ большой Ρ‚ΠΎΠΊ, Ρ‚ΠΎ напряТСниС ΡƒΠΏΠ°Π΄Π΅Ρ‚.

МСньшС элСктроэнСргии ΠΎΠ·Π½Π°Ρ‡Π°Π΅Ρ‚ мСньшСС напряТСниС

Π’Ρ‹ ΠΌΠΎΠΆΠ΅Ρ‚Π΅ Π΄ΡƒΠΌΠ°Ρ‚ΡŒ ΠΎ напряТСнии ΠΊΠ°ΠΊ ΠΎΠ± энСргии, приходящСйся Π½Π° заряд.

Если доступно мСньшС элСктроэнСргии (ΠΏΠΎΡ‚ΠΎΠΌΡƒ Ρ‡Ρ‚ΠΎ Ρ‡Π°ΡΡ‚ΡŒ химичСской энСргии прСвращаСтся Π² Ρ‚Π΅ΠΏΠ»ΠΎ Π² Π±Π°Ρ‚Π°Ρ€Π΅Π΅), Ρ‚ΠΎ напряТСниС Π½Π° ΠΊΠ»Π΅ΠΌΠΌΠ°Ρ… Π±Π°Ρ‚Π°Ρ€Π΅ΠΈ ΡƒΠΏΠ°Π΄Π΅Ρ‚. Π­Ρ‚ΠΎ ΠΎΠ·Π½Π°Ρ‡Π°Π΅Ρ‚, Ρ‡Ρ‚ΠΎ напряТСниС Π² Ρ†Π΅ΠΏΠΈ Ρ‚Π°ΠΊΠΆΠ΅ ΠΏΠ°Π΄Π°Π΅Ρ‚.

ΠŸΡ€Π°ΠΊΡ‚ΠΈΡ‡Π΅ΡΠΊΠΈΠΉ Ρ€Π΅Π·ΡƒΠ»ΡŒΡ‚Π°Ρ‚ состоит Π² Ρ‚ΠΎΠΌ, Ρ‡Ρ‚ΠΎ Ссли Π²Ρ‹ сдСлаСтС Π°ΠΊΠΊΡƒΠΌΡƒΠ»ΡΡ‚ΠΎΡ€Π½ΡƒΡŽ Π±Π°Ρ‚Π°Ρ€Π΅ΡŽ большим Ρ‚ΠΎΠΊΠΎΠΌ ΠΊ

  • заставляСт Π΅Π³ΠΎ Π·Π°ΠΏΡƒΡΠΊΠ°Ρ‚ΡŒ Ρ‡Ρ‚ΠΎ-Ρ‚ΠΎ, Ρ‡Ρ‚ΠΎ Ρ‚Ρ€Π΅Π±ΡƒΠ΅Ρ‚ большого Ρ‚ΠΎΠΊΠ°, Π½Π°ΠΏΡ€ΠΈΠΌΠ΅Ρ€ ΠΎΡ‡Π΅Π½ΡŒ ΡΡ€ΠΊΡƒΡŽ (с Π½ΠΈΠ·ΠΊΠΈΠΌ сопротивлСниСм) Π»Π°ΠΌΠΏΠΎΡ‡ΠΊΡƒ
  • , ΠΏΠΎΠ·Π²ΠΎΠ»ΡΡŽΡ‰ΠΈΠΉ Π·Π°ΠΏΡƒΡΠΊΠ°Ρ‚ΡŒ мноТСство ΠΎΠΏΠ΅Ρ€Π°Ρ†ΠΈΠΉ ΠΎΠ΄Π½ΠΎΠ²Ρ€Π΅ΠΌΠ΅Π½Π½ΠΎ (ΠΏΠΎΠ΄ΠΊΠ»ΡŽΡ‡Π΅Π½Ρ‹ ΠΏΠ°Ρ€Π°Π»Π»Π΅Π»ΡŒΠ½ΠΎ)
  • Π·Π°ΠΊΠΎΡ€Π°Ρ‡ΠΈΠ²Π°Π΅Ρ‚ аккумулятор (соСдинив ΠΎΠ΄ΠΈΠ½ Π²Ρ‹Π²ΠΎΠ΄ с Π΄Ρ€ΡƒΠ³ΠΈΠΌ ΠΏΡ€ΠΎΠ²ΠΎΠ΄ΠΎΠΌ)

напряТСниС ΡƒΠΏΠ°Π΄Π΅Ρ‚.

Если элСктричСская энСргия прСобразуСтся Π² Ρ‚Π΅ΠΏΠ»ΠΎΠ²ΡƒΡŽ, это Π΄ΠΎΠ»ΠΆΠ΅Π½ Π±Ρ‹Ρ‚ΡŒ рСзистор

Π‘Π°Ρ‚Π°Ρ€Π΅ΠΈ Π½Π΅ ΠΈΠΌΠ΅ΡŽΡ‚ Π²Π½ΡƒΡ‚Ρ€ΠΈ рСзистора, ΠΊΠΎΡ‚ΠΎΡ€Ρ‹ΠΉ ΠΌΠΎΠΆΠ½ΠΎ Π²Ρ‹Π½ΡƒΡ‚ΡŒ ΠΈ ΠΏΠΎΡΠΌΠΎΡ‚Ρ€Π΅Ρ‚ΡŒ. Но Ρƒ Π½ΠΈΡ… Π΄Π΅ΠΉΡΡ‚Π²ΠΈΡ‚Π΅Π»ΡŒΠ½ΠΎ Π΅ΡΡ‚ΡŒ источники сопротивлСния, Π½Π°ΠΏΡ€ΠΈΠΌΠ΅Ρ€ ΠΏΡ€ΠΎΠ΄ΡƒΠΊΡ‚Ρ‹ химичСских Ρ€Π΅Π°ΠΊΡ†ΠΈΠΉ ΠΈ мСталличСскиС части со всСми ΠΈΡ… соСдинСниями.

Π’Π°ΠΊΠΈΠΌ ΠΎΠ±Ρ€Π°Π·ΠΎΠΌ, Π±Π°Ρ‚Π°Ρ€Π΅ΠΈ часто ΠΌΠΎΠ΄Π΅Π»ΠΈΡ€ΡƒΡŽΡ‚ ΠΊΠ°ΠΊ ΠΈΠ΄Π΅Π°Π»ΡŒΠ½Ρ‹ΠΉ источник питания (напряТСниС ΠΊΠΎΡ‚ΠΎΡ€ΠΎΠ³ΠΎ Π½ΠΈΠΊΠΎΠ³Π΄Π° Π½Π΅ ΠΏΠ°Π΄Π°Π΅Ρ‚), Π²ΠΊΠ»ΡŽΡ‡Π΅Π½Π½Ρ‹ΠΉ ΠΏΠΎΡΠ»Π΅Π΄ΠΎΠ²Π°Ρ‚Π΅Π»ΡŒΠ½ΠΎ с Π²ΠΎΠΎΠ±Ρ€Π°ΠΆΠ°Π΅ΠΌΡ‹ΠΌ рСзистором.

ΠžΡ‡Π΅Π²ΠΈΠ΄Π½ΠΎ, Π±Π°Ρ‚Π°Ρ€Π΅ΠΉΠΊΠΈ ΠΈΡΠΏΠΎΠ»ΡŒΠ·ΡƒΡŽΡ‚ΡΡ для Ρ€Π°Π±ΠΎΡ‚Ρ‹ Ρ‡Π΅Π³ΠΎ-Ρ‚ΠΎ Π²Ρ€ΠΎΠ΄Π΅ Π»Π°ΠΌΠΏΠΎΡ‡ΠΊΠΈ.Π‘ΠΎΠΏΡ€ΠΎΡ‚ΠΈΠ²Π»Π΅Π½ΠΈΠ΅ Ρ†Π΅ΠΏΠΈ, Π² ΠΊΠΎΡ‚ΠΎΡ€ΠΎΠΉ ΠΎΠ½ΠΈ Ρ€Π°Π±ΠΎΡ‚Π°ΡŽΡ‚, называСтся сопротивлСниСм Π½Π°Π³Ρ€ΡƒΠ·ΠΊΠΈ R L .

Π’Π½ΡƒΡ‚Ρ€Π΅Π½Π½Π΅Π΅ сопротивлСниС ΠΎΠ±Ρ‹Ρ‡Π½ΠΎ обозначаСтся символом r. Π­Ρ‚ΠΎ Π½Π΅ Π·Π½Π°Ρ‡ΠΈΡ‚, Ρ‡Ρ‚ΠΎ ΠΎΠ½ всСгда малСнький. Π‘ΠΎΠΏΡ€ΠΎΡ‚ΠΈΠ²Π»Π΅Π½ΠΈΠ΅ Π±Π°Ρ‚Π°Ρ€Π΅ΠΉΠΊΠΈ для ΠΌΠΈΠ½ΠΈΠ°Ρ‚ΡŽΡ€Π½Ρ‹Ρ… часов ΠΌΠΎΠΆΠ΅Ρ‚ ΡΠΎΡΡ‚Π°Π²Π»ΡΡ‚ΡŒ 100 Ом ΠΈΠ»ΠΈ ΠΎΠΊΠΎΠ»ΠΎ Ρ‚ΠΎΠ³ΠΎ. БатарСя фонаря ΠΈΠΌΠ΅Π΅Ρ‚ Π²Π½ΡƒΡ‚Ρ€Π΅Π½Π½Π΅Π΅ сопротивлСниС ΠΎΠΊΠΎΠ»ΠΎ 0,1 Ом, Π° Π°Π²Ρ‚ΠΎΠΌΠΎΠ±ΠΈΠ»ΡŒΠ½Ρ‹ΠΉ аккумулятор — ΠΎΠΊΠΎΠ»ΠΎ 0,001 Ом.

Наш Π²ΠΎΠΎΠ±Ρ€Π°ΠΆΠ°Π΅ΠΌΡ‹ΠΉ Π²Π½ΡƒΡ‚Ρ€Π΅Π½Π½ΠΈΠΉ рСзистор подчиняСтся Π·Π°ΠΊΠΎΠ½Ρƒ Ома, ΠΊΠ°ΠΊ ΠΈ любой Π΄Ρ€ΡƒΠ³ΠΎΠΉ рСзистор. ЕдинствСнноС ΠΎΡ‚Π»ΠΈΡ‡ΠΈΠ΅ Π² Ρ‚ΠΎΠΌ, Ρ‡Ρ‚ΠΎ ΠΎΠ½ спрятан Π²Π½ΡƒΡ‚Ρ€ΠΈ аккумулятора.

ΠŸΠΎΡ‡Π΅ΠΌΡƒ Π²Π°ΠΆΠ½ΠΎ Π²Π½ΡƒΡ‚Ρ€Π΅Π½Π½Π΅Π΅ сопротивлСниС?

ΠΠ²Ρ‚ΠΎΠΌΠΎΠ±ΠΈΠ»ΡŒΠ½Ρ‹ΠΉ аккумулятор Π½Π° 12 Π’ ΠΈΠΌΠ΅Π΅Ρ‚ Ρ‚Π°ΠΊΠΎΠ΅ ΠΆΠ΅ напряТСниС, ΠΊΠ°ΠΊ ΠΈ восСмь аккумуляторов 1,5 Π’. AA. Могли Π±Ρ‹ Π²Ρ‹ ΠΈΡΠΏΠΎΠ»ΡŒΠ·ΠΎΠ²Π°Ρ‚ΡŒ эти Π±Π°Ρ‚Π°Ρ€Π΅ΠΈ, Ρ‡Ρ‚ΠΎΠ±Ρ‹ завСсти ΠΌΠ°ΡˆΠΈΠ½Ρƒ?

ΠžΡ‚Π²Π΅Ρ‚ — Ρ€Π΅ΡˆΠΈΡ‚Π΅Π»ΡŒΠ½ΠΎΠ΅ Π½Π΅Ρ‚. Π•ΡΡ‚ΡŒ Π΄Π²Π° взгляда Π½Π° это.

  1. Π‘Π°Ρ‚Π°Ρ€Π΅ΠΉΠΊΠΈ Ρ‚ΠΈΠΏΠ° AA Π½Π΅ ΠΌΠΎΠ³ΡƒΡ‚ ΠΎΠ±Π΅ΡΠΏΠ΅Ρ‡ΠΈΠ²Π°Ρ‚ΡŒ ΡΠ½Π΅Ρ€Π³ΠΈΡŽ ΠΎΡ‡Π΅Π½ΡŒ быстро, Π² Ρ‚ΠΎ врСмя ΠΊΠ°ΠΊ Π²Π°ΡˆΠ΅ΠΌΡƒ стартСру трСбуСтся энСргия ΠΎΡ‡Π΅Π½ΡŒ быстро.
  2. Π’Π½ΡƒΡ‚Ρ€Π΅Π½Π½Π΅Π΅ сопротивлСниС Π²Π°ΡˆΠΈΡ… Π±Π°Ρ‚Π°Ρ€Π΅Π΅ΠΊ AA слишком Π²Π΅Π»ΠΈΠΊΠΎ, поэтому напряТСниС ΠΏΠ°Π΄Π°Π΅Ρ‚ с 12 Π’ Π΄ΠΎ ΠΏΠΎΡ‡Ρ‚ΠΈ 0 Π’, ΠΊΠ°ΠΊ Ρ‚ΠΎΠ»ΡŒΠΊΠΎ Π²Ρ‹ ΠΏΡ‹Ρ‚Π°Π΅Ρ‚Π΅ΡΡŒ Π·Π°ΠΏΡƒΡΡ‚ΠΈΡ‚ΡŒ Π΄Π²ΠΈΠ³Π°Ρ‚Π΅Π»ΡŒ.

Если Π²Ρ‹ Π·Π½Π°Π΅Ρ‚Π΅, Ρ‡Ρ‚ΠΎ Π²Π°ΠΌ Π½ΡƒΠΆΠ½ΠΎ ΠΎΡ‡Π΅Π½ΡŒ быстро ΠΏΠΎΠ΄Π°Π²Π°Ρ‚ΡŒ ΡΠ½Π΅Ρ€Π³ΠΈΡŽ, Π²Π°ΠΌ Π½ΡƒΠΆΠ½ΠΎ ΠΎΡ‡Π΅Π½ΡŒ Π½ΠΈΠ·ΠΊΠΎΠ΅ Π²Π½ΡƒΡ‚Ρ€Π΅Π½Π½Π΅Π΅ сопротивлСниС. Как ΠΈ толстый кусок ΠΏΡ€ΠΎΠ²ΠΎΠ΄Π°, большая батарСя ΠΈΠΌΠ΅Π΅Ρ‚ мСньшСС сопротивлСниС. ΠŸΠΎΡΡ‚ΠΎΠΌΡƒ аккумуляторныС Π±Π°Ρ‚Π°Ρ€Π΅ΠΈ большой мощности Π΄ΠΎΠ»ΠΆΠ½Ρ‹ Π±Ρ‹Ρ‚ΡŒ большими, ΠΊΠ°ΠΊ Π°Π²Ρ‚ΠΎΠΌΠΎΠ±ΠΈΠ»ΡŒΠ½Ρ‹Π΅.

Если Π²Π°ΠΌ Π½Π΅ Π½ΡƒΠΆΠ½Π° энСргия ΠΎΡ‡Π΅Π½ΡŒ быстро, Π΄Ρ€ΡƒΠ³ΠΈΠΌΠΈ словами, вашС устройство потрСбляСт Ρ‚ΠΎΠ»ΡŒΠΊΠΎ ΠΊΡ€ΠΎΡˆΠ΅Ρ‡Π½Ρ‹ΠΉ Ρ‚ΠΎΠΊ, ΠΊΠ°ΠΊ Ρ†ΠΈΡ„Ρ€ΠΎΠ²Ρ‹Π΅ часы, Ρ‚ΠΎΠ³Π΄Π° Π²Π½ΡƒΡ‚Ρ€Π΅Π½Π½Π΅Π΅ сопротивлСниС ΠΌΠ΅Π½Π΅Π΅ Π²Π°ΠΆΠ½ΠΎ, поэтому Π²Ρ‹ ΠΌΠΎΠΆΠ΅Ρ‚Π΅ ΠΏΠΎΠ·Π²ΠΎΠ»ΠΈΡ‚ΡŒ сСбС ΡƒΠΌΠ΅Π½ΡŒΡˆΠΈΡ‚ΡŒ Ρ€Π°Π·ΠΌΠ΅Ρ€ Π±Π°Ρ‚Π°Ρ€Π΅ΠΈ. Π­Ρ‚ΠΎ ΠΏΠΎΠ»Π΅Π·Π½ΠΎ, Ссли Π²Ρ‹ Ρ…ΠΎΡ‚ΠΈΡ‚Π΅ ΠΏΠΎΠΌΠ΅ΡΡ‚ΠΈΡ‚ΡŒ Π΅Π³ΠΎ Π² часы!

Π§Ρ‚ΠΎ Ρ‚Π°ΠΊΠΎΠ΅ Β«Π½Π΅Π·Π½Π°Ρ‡ΠΈΡ‚Π΅Π»ΡŒΠ½ΠΎΠ΅ Π²Π½ΡƒΡ‚Ρ€Π΅Π½Π½Π΅Π΅ сопротивлСниС»?

Π­Ρ‚ΠΎ Π²Ρ‹Ρ€Π°ΠΆΠ΅Π½ΠΈΠ΅ часто встрСчаСтся Π² экзамСнационных вопросах.

Π­Ρ‚ΠΎ Π½Π΅ ΠΎΠ·Π½Π°Ρ‡Π°Π΅Ρ‚, Ρ‡Ρ‚ΠΎ сопротивлСниС Π΄ΠΎΠ»ΠΆΠ½ΠΎ Π±Ρ‹Ρ‚ΡŒ малСньким. ΠΊΠ°ΠΊ Ρ‚Π°ΠΊΠΎΠ²ΠΎΠ΅ . Π­Ρ‚ΠΎ просто ΠΎΠ·Π½Π°Ρ‡Π°Π΅Ρ‚, Ρ‡Ρ‚ΠΎ батарСя Π½Π΅ Ρ€Π°Π±ΠΎΡ‚Π°Π΅Ρ‚ достаточно усСрдно, Ρ‡Ρ‚ΠΎΠ±Ρ‹ Π΅Π΅ напряТСниС сильно ΡƒΠΏΠ°Π»ΠΎ.

ЭлСктродвиТущая сила — это напряТСниС аккумулятора, ΠΊΠΎΠ³Π΄Π° Π½Π° Π½Π΅ΠΌ Π½ΠΈΡ‡Π΅Π³ΠΎ Π½Π΅ Ρ€Π°Π±ΠΎΡ‚Π°Π΅Ρ‚

МаксимальноС напряТСниС, ΠΊΠΎΡ‚ΠΎΡ€ΠΎΠ΅ Π²Ρ‹ ΠΌΠΎΠΆΠ΅Ρ‚Π΅ ΠΏΠΎΠ»ΡƒΡ‡ΠΈΡ‚ΡŒ ΠΎΡ‚ Π±Π°Ρ‚Π°Ρ€Π΅ΠΈ, называСтся элСктродвиТущСй силой ΠΈΠ»ΠΈ Π­Π”Π‘. Π­Ρ‚ΠΎ называСтся Ρ‚Π°ΠΊ ΠΏΠΎ историчСским ΠΏΡ€ΠΈΡ‡ΠΈΠ½Π°ΠΌ, Π½ΠΎ Π² этом Π½Π΅Ρ‚ Π½ΠΈΡ‡Π΅Π³ΠΎ особСнного. Π­Ρ‚ΠΎ просто напряТСниС. ΠžΠ±Ρ‹Ρ‡Π½ΠΎ Π΅ΠΌΡƒ присваиваСтся символ Ξ΅.

Если Π²Π½ΡƒΡ‚Ρ€Π΅Π½Π½ΠΈΠΌ сопротивлСниСм нСльзя ΠΏΡ€Π΅Π½Π΅Π±Ρ€Π΅Ρ‡ΡŒ ΠΈ аккумулятор Ρ€Π°Π±ΠΎΡ‚Π°Π΅Ρ‚, Π½Π°ΠΏΡ€ΠΈΠΌΠ΅Ρ€, ΠΎΡ‚ Π»Π°ΠΌΠΏΠΎΡ‡ΠΊΠΈ, Ρ‚ΠΎ фактичСски ΠΈΠ·ΠΌΠ΅Ρ€Π΅Π½Π½ΠΎΠ΅ напряТСниС Π½Π° Π²Ρ‹Π²ΠΎΠ΄Π°Ρ… аккумулятора (Π° Ρ‚Π°ΠΊΠΆΠ΅ Π½Π° Π»Π°ΠΌΠΏΠ΅) Π±ΡƒΠ΄Π΅Ρ‚ Π½ΠΈΠΆΠ΅, Ρ‡Π΅ΠΌ Π­Π”Π‘. ΠœΡ‹ Π½Π°Π·Ρ‹Π²Π°Π΅ΠΌ это Π½ΠΈΠΆΠ½Π΅Π΅ напряТСниС напряТСниСм Π½Π°Π³Ρ€ΡƒΠ·ΠΊΠΈ, V L .

ΠžΠΏΡ€Π΅Π΄Π΅Π»Π΅Π½ΠΈΠ΅ Π²Π½ΡƒΡ‚Ρ€Π΅Π½Π½Π΅Π³ΠΎ сопротивлСния ΠΈ Π­Π”Π‘. Π±Π°Ρ‚Π°Ρ€Π΅ΠΈ

Π’Ρ‹ ΠΌΠΎΠΆΠ΅Ρ‚Π΅ ΠΈΠ·ΠΌΠ΅Ρ€ΠΈΡ‚ΡŒ Π­Π”Π‘. Π±Π°Ρ‚Π°Ρ€Π΅ΠΈ, просто ΠΈΠ·ΠΌΠ΅Ρ€ΠΈΠ² напряТСниС Π½Π° ΠΊΠ»Π΅ΠΌΠΌΠ°Ρ…, ΠΊΠΎΠ³Π΄Π° ΠΎΠ½Π° Π½ΠΈ ΠΊ Ρ‡Π΅ΠΌΡƒ Π½Π΅ ΠΏΠΎΠ΄ΠΊΠ»ΡŽΡ‡Π΅Π½Π°.Π­Ρ‚ΠΎ называСтся ΠΈΠ·ΠΌΠ΅Ρ€Π΅Π½ΠΈΠ΅ΠΌ напряТСния Π² Β«Ρ€Π°Π·ΠΎΠΌΠΊΠ½ΡƒΡ‚ΠΎΠΉ Ρ†Π΅ΠΏΠΈΒ».

Π’Ρ‹ Π½Π΅ ΠΌΠΎΠΆΠ΅Ρ‚Π΅ просто ΠΈΠ·ΠΌΠ΅Ρ€ΠΈΡ‚ΡŒ Π²Π½ΡƒΡ‚Ρ€Π΅Π½Π½Π΅Π΅ сопротивлСниС Π½Π°ΠΏΡ€ΡΠΌΡƒΡŽ, ΠΏΠΎΡ‚ΠΎΠΌΡƒ Ρ‡Ρ‚ΠΎ Π²Ρ‹ Π½Π΅ ΠΌΠΎΠΆΠ΅Ρ‚Π΅ ΠΏΠΎΠΏΠ°ΡΡ‚ΡŒ Π²Π½ΡƒΡ‚Ρ€ΡŒ Π±Π°Ρ‚Π°Ρ€Π΅ΠΈ. Π˜Ρ‚Π°ΠΊ, Π²Π°ΠΌ Π½ΡƒΠΆΠ½ΠΎ провСсти экспСримСнт, Π² ΠΊΠΎΡ‚ΠΎΡ€ΠΎΠΌ Π²Ρ‹ измСняСтС Ρ‚ΠΎΠΊ, потрСбляСмый ΠΎΡ‚ Π±Π°Ρ‚Π°Ρ€Π΅ΠΈ (измСняя сопротивлСниС Π½Π°Π³Ρ€ΡƒΠ·ΠΊΠΈ) ΠΈ измСряя p.d. Ρ‡Π΅Ρ€Π΅Π· Ρ‚Π΅Ρ€ΠΌΠΈΠ½Π°Π»Ρ‹.

Π—Π°ΠΊΠΎΠ½ ΠšΠΈΡ€Ρ…Π³ΠΎΡ„Π° ΠΏΠΎ Π½Π°ΠΏΡ€ΡΠΆΠ΅Π½ΠΈΡŽ гласит, Ρ‡Ρ‚ΠΎ Ссли ΡΠ»ΠΎΠΆΠΈΡ‚ΡŒ напряТСния Π½Π° всСх ΠΊΠΎΠΌΠΏΠΎΠ½Π΅Π½Ρ‚Π°Ρ… Π² ΠΏΠΎΡΠ»Π΅Π΄ΠΎΠ²Π°Ρ‚Π΅Π»ΡŒΠ½ΠΎΠΉ Ρ†Π΅ΠΏΠΈ, ΠΎΠ½ΠΈ Π΄ΠΎΠ»ΠΆΠ½Ρ‹ Ρ‚ΠΎΡ‡Π½ΠΎ Ρ€Π°Π²Π½ΡΡ‚ΡŒΡΡ Π½Π°ΠΏΡ€ΡΠΆΠ΅Π½ΠΈΡŽ Π±Π°Ρ‚Π°Ρ€Π΅ΠΈ.

e.ΠΌ.Ρ„. = напряТСниС Π½Π° Π²Π½ΡƒΡ‚Ρ€Π΅Π½Π½Π΅ΠΌ сопротивлСнии + напряТСниС Π½Π° Π½Π°Π³Ρ€ΡƒΠ·ΠΊΠ΅ (Π½Π°ΠΏΡ€ΠΈΠΌΠ΅Ρ€, Π»Π°ΠΌΠΏΠΎΡ‡ΠΊΠ΅)

Π’ символах это ΡƒΡ€Π°Π²Π½Π΅Π½ΠΈΠ΅

Ξ΅ = V Π²Π½ΡƒΡ‚Ρ€Π΅Π½Π½ΠΈΠΉ + V L

ΠœΡ‹ Π·Π½Π°Π΅ΠΌ, Ρ‡Ρ‚ΠΎ V = IR, ΠΈΠ»ΠΈ ΠΈΡΠΏΠΎΠ»ΡŒΠ·ΡƒΡ ΡΠΎΠΎΡ‚Π²Π΅Ρ‚ΡΡ‚Π²ΡƒΡŽΡ‰ΠΈΠ΅ Ρ‚Π΅Ρ€ΠΌΠΈΠ½Ρ‹ для Π²Π½ΡƒΡ‚Ρ€Π΅Π½Π½Π΅Π³ΠΎ сопротивлСния V Π²Π½ΡƒΡ‚Ρ€Π΅Π½Π½Π΅Π΅ = Ir, поэтому

Ξ΅ = Ir + V L

ΠœΡ‹ ΠΌΠΎΠΆΠ΅ΠΌ ΠΈΠ·ΠΌΠ΅Π½ΠΈΡ‚ΡŒ это ΡƒΡ€Π°Π²Π½Π΅Π½ΠΈΠ΅, Ρ‡Ρ‚ΠΎΠ±Ρ‹ ΠΏΠΎΠ»ΡƒΡ‡ΠΈΡ‚ΡŒ

Π’ L = -rI + Ξ΅

Если Π²Ρ‹ настроили схСму с ΠΏΠ΅Ρ€Π΅ΠΌΠ΅Π½Π½Ρ‹ΠΌ рСзистором для Π½Π°Π³Ρ€ΡƒΠ·ΠΊΠΈ, Π²Ρ‹ ΠΌΠΎΠΆΠ΅Ρ‚Π΅ ΠΈΠ·ΠΌΠ΅Π½ΠΈΡ‚ΡŒ Ρ‚ΠΎΠΊ I, потрСбляСмый ΠΎΡ‚ Π±Π°Ρ‚Π°Ρ€Π΅ΠΈ, ΠΈ ΠΈΠ·ΠΌΠ΅Ρ€ΠΈΡ‚ΡŒ напряТСниС Π½Π° Π²Ρ‹Π²ΠΎΠ΄Π°Ρ…, V L .

Π“Ρ€Π°Ρ„ΠΈΠΊ V L ΠΎΡ‚Π½ΠΎΡΠΈΡ‚Π΅Π»ΡŒΠ½ΠΎ I Π΄Π°Π΅Ρ‚ ΠΏΡ€ΡΠΌΡƒΡŽ линию с Π²Π΅Π»ΠΈΡ‡ΠΈΠ½ΠΎΠΉ Π³Ρ€Π°Π΄ΠΈΠ΅Π½Ρ‚Π°, Ρ€Π°Π²Π½ΠΎΠΉ Π²Π½ΡƒΡ‚Ρ€Π΅Π½Π½Π΅ΠΌΡƒ ΡΠΎΠΏΡ€ΠΎΡ‚ΠΈΠ²Π»Π΅Π½ΠΈΡŽ r. Π­.Π΄.с. это Ρ‚ΠΎΡ‡ΠΊΠ° пСрСсСчСния Π½Π° оси напряТСния, Π΄Ρ€ΡƒΠ³ΠΈΠΌΠΈ словами, напряТСниС ΠΏΡ€ΠΈ Π½ΡƒΠ»Π΅Π²ΠΎΠΌ Ρ‚ΠΎΠΊΠ΅.

Π½Π°Π·Π°Π΄ ΠΊ Π£Ρ€ΠΎΠΊΡƒ 7: Π‘ΠΎΠΏΡ€ΠΎΡ‚ΠΈΠ²Π»Π΅Π½ΠΈΠ΅ ΠΈ Π·Π°ΠΊΠΎΠ½ Ома

Π’Π½ΡƒΡ‚Ρ€Π΅Π½Π½Π΅Π΅ сопротивлСниС | Аккумуляторы ΠΈ Π΄Ρ€ΡƒΠ³ΠΈΠ΅ источники питания

Π’Π½ΡƒΡ‚Ρ€Π΅Π½Π½Π΅Π΅ сопротивлСниС | Аккумуляторы ΠΈ Π΄Ρ€ΡƒΠ³ΠΈΠ΅ источники питания

Π’ элСктричСских цСпях, ΠΊΠΎΡ‚ΠΎΡ€Ρ‹Π΅ ΠΌΡ‹ рассмотрСли Π΄ΠΎ сих ΠΏΠΎΡ€, ΠΌΡ‹ рассматривали источник напряТСния, ΠΏΠΈΡ‚Π°ΡŽΡ‰ΠΈΠΉ Ρ†Π΅ΠΏΡŒ, ΠΊΠ°ΠΊ ΠΈΠ΄Π΅Π°Π»ΡŒΠ½Ρ‹ΠΉ источник напряТСния.Ρ‚ΠΎ Π΅ΡΡ‚ΡŒ:

  • ΠœΡ‹ ΠΏΡ€Π΅Π΄ΠΏΠΎΠ»ΠΎΠΆΠΈΠ»ΠΈ, Ρ‡Ρ‚ΠΎ напряТСниС, ΠΏΠΎΠ΄Π°Π²Π°Π΅ΠΌΠΎΠ΅ Π² Ρ†Π΅ΠΏΡŒ, остаСтся постоянным, нСзависимо ΠΎΡ‚ Ρ‚ΠΎΠ³ΠΎ, ΠΊΠ°ΠΊΠΎΠΉ Ρ‚ΠΎΠΊ подаСтся. поставляСтся.
  • ΠœΡ‹ Ρ‚Π°ΠΊΠΆΠ΅ ΠΏΡ€Π΅Π΄ΠΏΠΎΠ»ΠΎΠΆΠΈΠ»ΠΈ, Ρ‡Ρ‚ΠΎ энСргия прСобразуСтся Π² Ρ‚Π΅ΠΏΠ»ΠΎ Ρ‚ΠΎΠ»ΡŒΠΊΠΎ Π·Π° счСт сопротивлСния Π²ΠΎ внСшнСй Ρ†Π΅ΠΏΠΈ.

На ΠΏΡ€Π°ΠΊΡ‚ΠΈΠΊΠ΅ это Π½Π΅ Ρ‚Π°ΠΊ, ΠΏΠΎΡΠΊΠΎΠ»ΡŒΠΊΡƒ:

  1. НапряТСниС Π½Π° ΠΊΠ»Π΅ΠΌΠΌΠ°Ρ… Π±Π°Ρ‚Π°Ρ€Π΅ΠΈ ΡƒΠΌΠ΅Π½ΡŒΡˆΠ°Π΅Ρ‚ΡΡ ΠΏΠΎ ΠΌΠ΅Ρ€Π΅ Ρ‚ΠΎΠ³ΠΎ, ΠΊΠ°ΠΊ Ρ‚ΠΎΠΊ, ΠΏΠΎΠ΄Π°Π²Π°Π΅ΠΌΡ‹ΠΉ Π² Ρ†Π΅ΠΏΡŒ, увСличиваСтся.
    Π­Ρ‚ΠΎ Ρ‚ΠΎ ΠΆΠ΅ самоС для всСх Ρ€Π΅Π°Π»ΡŒΠ½Ρ‹Ρ… источников напряТСния. (Однако Ρ€Π°Π·Ρ€Π°Π±ΠΎΡ‚Ρ‡ΠΈΠΊΠΈ Π±Π»ΠΎΠΊΠΎΠ² питания Π΄Π΅ΠΉΡΡ‚Π²ΠΈΡ‚Π΅Π»ΡŒΠ½ΠΎ производят стабилизированныС Π±Π»ΠΎΠΊΠΈ питания, Π² ΠΊΠΎΡ‚ΠΎΡ€Ρ‹Ρ… обратная связь схСмы ΠΈΡΠΏΠΎΠ»ΡŒΠ·ΡƒΡŽΡ‚ΡΡ для поддСрТания ΠΎΡ‚Π½ΠΎΡΠΈΡ‚Π΅Π»ΡŒΠ½ΠΎ постоянного Π²Ρ‹Ρ…ΠΎΠ΄Π½ΠΎΠ³ΠΎ напряТСния).
  2. ВсС Π±Π»ΠΎΠΊΠΈ питания Π½Π°Π³Ρ€Π΅Π²Π°ΡŽΡ‚ΡΡ Π²ΠΎ врСмя использования, Ρ‡Ρ‚ΠΎ ΡΠ²ΠΈΠ΄Π΅Ρ‚Π΅Π»ΡŒΡΡ‚Π²ΡƒΠ΅Ρ‚ ΠΎ Ρ‚ΠΎΠΌ, Ρ‡Ρ‚ΠΎ Ρ‡Π°ΡΡ‚ΡŒ выдСляСмой ΠΈΠΌΠΈ энСргии фактичСски расходуСтся. прСобразуСтся Π² Ρ‚Π΅ΠΏΠ»ΠΎ Π²Π½ΡƒΡ‚Ρ€ΠΈ самого Π±Π»ΠΎΠΊΠ° питания!

Π­Ρ‚ΠΎ ΠΏΠΎΠΊΠ°Π·Π°Π½ΠΎ Π½Π° Π°Π½ΠΈΠΌΠ°Ρ†ΠΈΠΈ Π½ΠΈΠΆΠ΅.

Π­Ρ‚ΠΎ ΠΈΠ·ΠΌΠ΅Π½Π΅Π½ΠΈΠ΅ напряТСния питания ΠΏΠΎΡ‚Π΅Π½Ρ†ΠΈΠ°Π»ΡŒΠ½ΠΎ Ρ‚Ρ€ΡƒΠ΄Π½ΠΎ ΠΎΠ±ΡŠΡΡΠ½ΠΈΡ‚ΡŒ, особСнно Ссли ΡƒΡ‡Π΅ΡΡ‚ΡŒ, Ρ‡Ρ‚ΠΎ ΠŸΡ€ΠΈΡ‡ΠΈΠ½Ρ‹ отклонСния Π±ΡƒΠ΄ΡƒΡ‚ Π·Π°Π²ΠΈΡΠ΅Ρ‚ΡŒ ΠΎΡ‚ Ρ‚ΠΈΠΏΠ° ΠΈΡΠΏΠΎΠ»ΡŒΠ·ΡƒΠ΅ΠΌΠΎΠ³ΠΎ источника питания.
НапримСр:

  • Для аккумулятора: напряТСниС ΠΏΠ°Π΄Π°Π΅Ρ‚ ΠΈΠ·-Π·Π° скорости химичСских Ρ€Π΅Π°ΠΊΡ†ΠΈΠΉ, ΠΏΠ΅Ρ€Π΅Π΄Π°ΡŽΡ‰ΠΈΡ… заряд Π½Π° ΠΊΠ»Π΅ΠΌΠΌΡ‹ аккумулятора, Π½Π΅ ΠΌΠΎΠΆΠ΅Ρ‚ ΡΠΎΠΎΡ‚Π²Π΅Ρ‚ΡΡ‚Π²ΠΎΠ²Π°Ρ‚ΡŒ скорости, с ΠΊΠΎΡ‚ΠΎΡ€ΠΎΠΉ заряд ΠΏΠΎΠΊΠΈΠ΄Π°Π΅Ρ‚ ΠΊΠ»Π΅ΠΌΠΌΡ‹, Ρ‡Ρ‚ΠΎΠ±Ρ‹ Ρ‚Π΅Ρ‡ΡŒ ΠΏΠΎ Ρ†Π΅ΠΏΠΈ.
  • Для Π³Π΅Π½Π΅Ρ€Π°Ρ‚ΠΎΡ€Π°: Ρ‚ΠΎΠΊ создаСт Π±ΠΎΠ»Π΅Π΅ ΡΠΈΠ»ΡŒΠ½Ρ‹Π΅ ΠΌΠ°Π³Π½ΠΈΡ‚Π½Ρ‹Π΅ поля Π²Π½ΡƒΡ‚Ρ€ΠΈ Π³Π΅Π½Π΅Ρ€Π°Ρ‚ΠΎΡ€Π°, ΠΊΠΎΡ‚ΠΎΡ€Ρ‹Π΅ Π·Π°ΠΌΠ΅Π΄Π»ΡΡŽΡ‚ Ρ€Π°Π±ΠΎΡ‚Ρƒ Π³Π΅Π½Π΅Ρ€Π°Ρ‚ΠΎΡ€Π° ΠΈ ΡƒΠΌΠ΅Π½ΡŒΡˆΠ°ΡŽΡ‚ напряТСниС питания.

К ΡΡ‡Π°ΡΡ‚ΡŒΡŽ, ΠΌΡ‹ ΠΌΠΎΠΆΠ΅ΠΌ ΠΈΠ·Π±Π΅ΠΆΠ°Ρ‚ΡŒ этих Π΄Π΅Ρ‚Π°Π»Π΅ΠΉ, ΠΈ, нСзависимо ΠΎΡ‚ фактичСского Ρ…Π°Ρ€Π°ΠΊΡ‚Π΅Ρ€Π° источника питания, ΠΌΡ‹ ΠΌΠΎΠΆΠ΅ΠΌ ΠΏΡ€Π΅Π΄ΡΡ‚Π°Π²ΠΈΡ‚ΡŒ Π΅Π³ΠΎ Π² Π²ΠΈΠ΄Π΅ модСль Π±Π»ΠΎΠΊΠ° питания, ΠΏΠΎΠ»Π½ΠΎΡΡ‚ΡŒΡŽ состоящая ΠΈΠ· простых элСктричСских ΠΊΠΎΠΌΠΏΠΎΠ½Π΅Π½Ρ‚ΠΎΠ². Один ΠΈΠ· способов ΡΠ΄Π΅Π»Π°Ρ‚ΡŒ это — ΠΏΡ€Π΅Π΄ΡΡ‚Π°Π²ΠΈΡ‚ΡŒ Π²Π»Π°ΡΡ‚ΡŒ ΠΊΠ°ΠΊ ΠΈΠ΄Π΅Π°Π»ΡŒΠ½Ρ‹ΠΉ источник напряТСния (Π½Π°ΠΏΡ€ΠΈΠΌΠ΅Ρ€,ΠΌ.Π΄.) ΠΏΠΎΡΠ»Π΅Π΄ΠΎΠ²Π°Ρ‚Π΅Π»ΡŒΠ½ΠΎ с Π²Π½ΡƒΡ‚Ρ€Π΅Π½Π½ΠΈΠΌ сопротивлСниСм. Когда эта модСль источника питания примСняСтся ΠΊ внСшнСй Ρ†Π΅ΠΏΠΈ, Ρ‚ΠΎΠΊ Ρ†Π΅ΠΏΠΈ Ρ‚Π°ΠΊΠΆΠ΅ Ρ‚Π΅Ρ‡Π΅Ρ‚ Ρ‡Π΅Ρ€Π΅Π· Π²Π½ΡƒΡ‚Ρ€Π΅Π½Π½Π΅Π΅ сопротивлСниС. Π­Ρ‚ΠΎ Π²Ρ‹Π·Ρ‹Π²Π°Π΅Ρ‚ Π²Π½ΡƒΡ‚Ρ€Π΅Π½Π½Π΅Π΅ ΠΏΠ°Π΄Π΅Π½ΠΈΠ΅ напряТСния Π²Π½ΡƒΡ‚Ρ€ΠΈ источника питания, Ρ‡Ρ‚ΠΎ, ΡΠ»Π΅Π΄ΠΎΠ²Π°Ρ‚Π΅Π»ΡŒΠ½ΠΎ, сниТаСт напряТСниС Π½Π° ΠΊΠ»Π΅ΠΌΠΌΠ°Ρ… источника питания. ΠœΠΎΡ‰Π½ΠΎΡΡ‚ΡŒ, рассСиваСмая Π²Π½ΡƒΡ‚Ρ€Π΅Π½Π½ΠΈΠΌ сопротивлСниСм, прСдставляСт собой Ρ‚Π΅ΠΏΠ»ΠΎ, выдСляСмоС Π² источникС питания. Π­Ρ‚ΠΎ ΠΏΠΎΠΊΠ°Π·Π°Π½ΠΎ Π½Π° Π°Π½ΠΈΠΌΠ°Ρ†ΠΈΠΈ Π½ΠΈΠΆΠ΅.

НапряТСниС Π½Π° ΠΊΠ»Π΅ΠΌΠΌΠ°Ρ… (Π’) Ρ€Π°Π²Π½ΠΎ эл.ΠΌ.Ρ„. напряТСниС (E) Π·Π° Π²Ρ‹Ρ‡Π΅Ρ‚ΠΎΠΌ Π²Π½ΡƒΡ‚Ρ€Π΅Π½Π½Π΅Π³ΠΎ падСния напряТСния (Ir).
(ΠΈΡΠΏΠΎΠ»ΡŒΠ·ΡƒΡ Π·Π°ΠΊΠΎΠ½ Ома: Π²Π½ΡƒΡ‚Ρ€Π΅Π½Π½Π΅Π΅ ΠΏΠ°Π΄Π΅Π½ΠΈΠ΅ напряТСния = Ρ‚ΠΎΠΊ (I) x Π²Π½ΡƒΡ‚Ρ€Π΅Π½Π½Π΅Π΅ сопротивлСниС (r)).

Π§Ρ‚ΠΎΠ±Ρ‹ ΡΠΌΠΎΠ΄Π΅Π»ΠΈΡ€ΠΎΠ²Π°Ρ‚ΡŒ любой Ρ€Π΅Π°Π»ΡŒΠ½Ρ‹ΠΉ источник питания, Π½Π°ΠΌ просто Π½ΡƒΠΆΠ½ΠΎ ΠΎΠΏΡ€Π΅Π΄Π΅Π»ΠΈΡ‚ΡŒ ΠΏΡ€Π°Π²ΠΈΠ»ΡŒΠ½Ρ‹Π΅ значСния E ΠΈ r для использования.

Когда источник питания Π½Π΅ ΠΏΠΎΠ΄ΠΊΠ»ΡŽΡ‡Π΅Π½ ΠΊ Ρ†Π΅ΠΏΠΈ, Ρ‚ΠΎΠΊ Π½Π΅ ΠΏΡ€ΠΎΡ‚Π΅ΠΊΠ°Π΅Ρ‚, поэтому:

  • V = E — 0 x r.
  • Π’ = E.
  • Ρ‚ΠΎ Π΅ΡΡ‚ΡŒ e.НапряТСниС m.f Ρ€Π°Π²Π½ΠΎ Π½Π°ΠΏΡ€ΡΠΆΠ΅Π½ΠΈΡŽ Π½Π° ΠΊΠ»Π΅ΠΌΠΌΠ°Ρ… холостого Ρ…ΠΎΠ΄Π° источника питания.

Π’Π½ΡƒΡ‚Ρ€Π΅Π½Π½Π΅Π΅ сопротивлСниС ΠΌΠΎΠΆΠ½ΠΎ ΠΎΠΏΡ€Π΅Π΄Π΅Π»ΠΈΡ‚ΡŒ, ΠΏΠΎΠ΄ΠΊΠ»ΡŽΡ‡ΠΈΠ² Ρ†Π΅ΠΏΡŒ с извСстным сопротивлСниСм ΠΈ ΠΈΠ·ΠΌΠ΅Ρ€ΠΈΠ² ΠΏΡ€ΠΎΡ‚Π΅ΠΊΠ°ΡŽΡ‰ΠΈΠΉ Ρ‚ΠΎΠΊ.

  • I = E / (R + r).
  • , ΡΠ»Π΅Π΄ΠΎΠ²Π°Ρ‚Π΅Π»ΡŒΠ½ΠΎ, r = (E / I) — R.

ΠœΠΎΡ‰Π½ΠΎΡΡ‚ΡŒ, подаваСмая э.Π΄.с., опрСдСляСтся ΠΊΠ°ΠΊ P = EI, Π° ΠΌΠΎΡ‰Π½ΠΎΡΡ‚ΡŒ, рассСиваСмая Π² источникС питания, опрСдСляСтся ΠΊΠ°ΠΊ P = I 2 r.
ЭнСргия, обСспСчиваСмая e.m.f. опрСдСляСтся ΠΊΠ°ΠΊ W = EIt, Π° энСргия, рассСиваСмая Π² источникС питания, опрСдСляСтся ΠΊΠ°ΠΊ W = I 2 rt.

.
Π Π°Π·Π½ΠΎΠ΅

Π”ΠΎΠ±Π°Π²ΠΈΡ‚ΡŒ ΠΊΠΎΠΌΠΌΠ΅Π½Ρ‚Π°Ρ€ΠΈΠΉ

Π’Π°Ρˆ адрСс email Π½Π΅ Π±ΡƒΠ΄Π΅Ρ‚ ΠΎΠΏΡƒΠ±Π»ΠΈΠΊΠΎΠ²Π°Π½. ΠžΠ±ΡΠ·Π°Ρ‚Π΅Π»ΡŒΠ½Ρ‹Π΅ поля ΠΏΠΎΠΌΠ΅Ρ‡Π΅Π½Ρ‹ *